Вы находитесь на странице: 1из 321

Study Guide

Pharmacology for
Nursing Care
Ninth Edition

RICHARD A. LEHNE, PhD

Prepared by
Jennifer J. Yeager, PhD, RN
Assistant Professor
Department of Nursing
Tarleton State University
Stephenville, Texas
3251 Riverport Lane
St. Louis, Missouri 63043

STUDY GUIDE FOR LEHNE’S PHARMACOLOGY FOR ISBN: 978-0-323-32259-1


NURSING CARE, NINTH EDITION

Copyright © 2016, 2013, 2010, 2007, 2004, 2000, 1999 by Elsevier

All rights reserved. No part of this publication may be reproduced or transmitted in any form or by any means,
electronic or mechanical, including photocopying, recording, or any information storage and retrieval system,
without permission in writing from the publisher. Details on how to seek permission, further information about
the Publisher’s permissions policies and our arrangements with organizations such as the Copyright Clearance
Center and the Copyright Licensing Agency, can be found at our website: www.elsevier.com/permissions.

This book and the individual contributions contained in it are protected under copyright by the Publisher (other
than as may be noted herein).

Notices
 

 
 




  
   


  



broaden our understanding, changes in research methods, professional practices, or medical treatment
may become necessary.
Practitioners and researchers must always rely on their own experience and knowledge in evaluating
and using any information, methods, compounds, or experiments described herein. In using such infor-
mation or methods they should be mindful of their own safety and the safety of others, including parties
for whom they have a professional responsibility.


 




   


 

 



 

current information provided (i) on procedures featured or (ii) by the manufacturer of each product to
be administered, to verify the recommended dose or formula, the method and duration of administra-
tion, and contraindications. It is the responsibility of practitioners, relying on their own experience and
knowledge of their patients, to make diagnoses, to determine dosages and the best treatment for each
individual patient, and to take all appropriate safety precautions.
To the fullest extent of the law, neither the Publisher nor the authors, contributors, or editors, as-
sume any liability for any injury and/or damage to persons or property as a matter of products liability,
negligence or otherwise, or from any use or operation of any methods, products, instructions, or ideas
contained in the material herein.

ISBN: 978-0-323-32259-1

Content Strategist: Jamie Randall


Content Development Specialist: Samantha Taylor
Publishing Services Manager: Hemamalini Rajendrababu
Project Manager: Saravanan Thavamani

Printed in the United States of America


Last digit is the print number: 9 8 7 6 5 4 3 2 1
Introduction

The critical thinking and study questions in this Study to safely administer pharmacotherapy. They are an excel-
Guide include review of knowledge, application of lent source of review when preparing for the NCLEX®
knowledge to nursing care, analysis of nursing situations examination.
that require clinical decision-making, and prioritization of A useful strategy when presented with the case studies
nursing actions. When this book is used as a study guide, 

 


 

 

 
-
the questions that do not have a ³ or 6 before them 

" 
 




 

are excellent tools to augment the initial reading of the of pertinent information in the case. When reading the
textbook before attending class and to use for review after rest of the information in the case, identify key words and
classroom activities. Knowledge of drug action, interac-   
#$
 


  
 

tions, administration directives, and adverse effects is that are normal or abnormal. Think about the situation
required before the nurse can engage in clinical decision- to identify if it is a normal part of nursing care or if it
making. requires collaboration with the prescriber.
The questions that have a ³ or 6 before them require Critical thinking by the nursing student requires assimi-
more than repetition of the information in the textbook. lating classroom learning and clinical experience. No two
Questions preceded by ³ require application and analy- experiences are the same. Be careful not to add informa-
sis of information; whereas those preceded by 6 require tion into the question based on your experience with
prioritization, including selecting the most important patients when answering questions in this Study Guide.
information or action. Identifying the correct answer to But do engage in discussion about similar patient experi-
these questions requires careful examination of the data ences with faculty and peers. Be prepared to explain your




 


 
 
!
- thinking. It will enrich your learning experience.
dent will have to integrate other nursing knowledge, such The author and Elsevier welcome any feedback you
as developmental considerations and laboratory values, have about the content of the Study Guide.
as well as timing and prioritization of actions. These ³ Answers to the Case Studies can be reviewed online
and 6
 

" 


 
 
at http://evolve.elsevier.com/Lehne/.
students and new graduate nurses must be able to perform

Copyright © 2016, Elsevier Inc. All Rights Reserved. iii


This page intentionally left blank
Contents

Chapter 1 Orientation to Pharmacology . . . . . . . . . . 1 Chapter 29 Pain Management in Patients


Chapter 2 Application of Pharmacology in with Cancer . . . . . . . . . . . . . . . . . . . . . . 56
Nursing Practice. . . . . . . . . . . . . . . . . . . . 2 Chapter 30 Drugs for Headache . . . . . . . . . . . . . . . . 58
Chapter 3 Drug Regulation, Development, Chapter 31 Antipsychotic Agents and Their
Names, and Information. . . . . . . . . . . . . . 4 Use in Schizophrenia . . . . . . . . . . . . . . . 60
Chapter 4 Pharmacokinetics . . . . . . . . . . . . . . . . . . . 5 Chapter 32 Antidepressants . . . . . . . . . . . . . . . . . . . 63
Chapter 5 Pharmacodynamics . . . . . . . . . . . . . . . . . 7 Chapter 33 Drugs for Bipolar Disorder . . . . . . . . . . 67
Chapter 6 Drug Interactions . . . . . . . . . . . . . . . . . . . 9 Chapter 34 Sedative-Hypnotic Drugs. . . . . . . . . . . . 69
Chapter 7 Adverse Drug Reactions and Chapter 35 Management of Anxiety Disorders . . . . 72
Medication Errors . . . . . . . . . . . . . . . . . 10 Chapter 36 Central Nervous System Stimulants
Chapter 8 Individual Variation in Drug 
%&'* 

Responses. . . . . . . . . . . . . . . . . . . . . . . . 12 Disorder . . . . . . . . . . . . . . . . . . . . . . . . . 74
Chapter 9 Drug Therapy During Pregnancy Chapter 37 Drug Abuse I: Basic Considerations . . . 77
and Breast-Feeding . . . . . . . . . . . . . . . . 13 Chapter 38 Drug Abuse II: Alcohol . . . . . . . . . . . . . 79
Chapter 10 Drug Therapy in Pediatric Patients . . . . 15 Chapter 39 Drug Abuse III: Nicotine and
Chapter 11 Drug Therapy in Geriatric Patients . . . . 16 Smoking . . . . . . . . . . . . . . . . . . . . . . . . . 82
Chapter 12 Basic Principles of Chapter 40 Drug Abuse IV: Major Drugs of
Neuropharmacology. . . . . . . . . . . . . . . . 18 Abuse Other Than Alcohol and
Chapter 13 Physiology of the Peripheral Nicotine . . . . . . . . . . . . . . . . . . . . . . . . . 83
Nervous System . . . . . . . . . . . . . . . . . . . 19 Chapter 41 Diuretics. . . . . . . . . . . . . . . . . . . . . . . . . 86
Chapter 14 Muscarinic Agonists and Chapter 42 Agents Affecting the Volume and
Antagonists . . . . . . . . . . . . . . . . . . . . . . 21 Ion Content of Body Fluids . . . . . . . . . . 90
Chapter 15 Cholinesterase Inhibitors and Chapter 43 Review of Hemodynamics. . . . . . . . . . . 92
Their Use in Myasthenia Gravis . . . . . . 24 Chapter 44 Drugs Acting on the Renin-
Chapter 16 Drugs that Block Nicotinic Angiotensin-Aldosterone System . . . . . 94
Cholinergic Transmission: Chapter 45 Calcium Channel Blockers . . . . . . . . . . 97
Neuromuscular Blocking Agents Chapter 46 Vasodilators . . . . . . . . . . . . . . . . . . . . . 100
and Ganglionic Blocking Agents . . . . . . 26 Chapter 47 Drugs for Hypertension . . . . . . . . . . . . 102
Chapter 17 Adrenergic Agonists. . . . . . . . . . . . . . . . 27 Chapter 48 Drugs for Heart Failure . . . . . . . . . . . . 105
Chapter 18 Adrenergic Antagonists . . . . . . . . . . . . . 29 Chapter 49 Antidysrhythmic Drugs . . . . . . . . . . . . 109
Chapter 19 Indirect-Acting Antiadrenergic Chapter 50 Prophylaxis of Coronary Heart
Agents . . . . . . . . . . . . . . . . . . . . . . . . . . 32 Disease: Drugs That Help
Chapter 20 Introduction to Central Nervous Normalize Cholesterol and
System Pharmacology . . . . . . . . . . . . . . 34 Triglyceride Levels . . . . . . . . . . . . . . . 112
Chapter 21 Drugs for Parkinson’s Disease . . . . . . . . 35 Chapter 51 Drugs for Angina Pectoris . . . . . . . . . . 115
Chapter 22 Alzheimer’s Disease . . . . . . . . . . . . . . . 37 Chapter 52 Anticoagulant, Antiplatelet, and
Chapter 23 Drugs for Multiple Sclerosis . . . . . . . . . 39 Thrombolytic Drugs. . . . . . . . . . . . . . . 118
Chapter 24 Drugs for Epilepsy . . . . . . . . . . . . . . . . . 42 Chapter 53 Management of ST-Elevation
Chapter 25 Drugs for Muscle Spasm and Myocardial Infarction . . . . . . . . . . . . . 122
Spasticity . . . . . . . . . . . . . . . . . . . . . . . . 46 Chapter 54 Drugs for Hemophilia . . . . . . . . . . . . . 124
Chapter 26 Local Anesthetics . . . . . . . . . . . . . . . . . . 48 Chapter 55 &
$
&
 . . . . . . . 127
Chapter 27 General Anesthetics . . . . . . . . . . . . . . . . 49 Chapter 56 Hematopoietic Agents . . . . . . . . . . . . . 130
Chapter 28 Opioid (Narcotic) Analgesics, Chapter 57 Drugs for Diabetes Mellitus. . . . . . . . . 133
Opioid Antagonists, and Chapter 58 Drugs for Thyroid Disorders . . . . . . . . 137
Nonopioid Centrally Acting Chapter 59 Drugs Related to Hypothalamic
Analgesics . . . . . . . . . . . . . . . . . . . . . . . 52 and Pituitary Function . . . . . . . . . . . . . 140

Copyright © 2016, Elsevier Inc. All Rights Reserved. v


vi Contents

Chapter 60 Drugs for Disorders of the Chapter 88 Sulfonamides and Trimethoprim . . . . . 221
Adrenal Cortex. . . . . . . . . . . . . . . . . . . 143 Chapter 89 Drug Therapy of Urinary Tract
Chapter 61 Estrogens and Progestins: Basic Infections . . . . . . . . . . . . . . . . . . . . . . . 224
Pharmacology and Chapter 90 Antimycobacterial Agents: Drugs
Noncontraceptive Applications . . . . . . 145 for Tuberculosis, Leprosy, and
Chapter 62 Birth Control . . . . . . . . . . . . . . . . . . . . 147 Mycobacterium avium
Chapter 63 Drug Therapy of Infertility . . . . . . . . . 150 Complex Infection . . . . . . . . . . . . . . . . 226
Chapter 64 Drugs that Affect Uterine Function . . . 152 Chapter 91 Miscellaneous Antibacterial Drugs:
Chapter 65 Androgens . . . . . . . . . . . . . . . . . . . . . . 154 Fluoroquinolones, Metronidazole,
Chapter 66 Drugs for Erectile Dysfunction Daptomycin, Rifampin, Rifaximin,
and Benign Prostatic Hyperplasia . . . . 156 Bacitracin, and Polymyxins . . . . . . . . . 230
Chapter 67 Review of the Immune System . . . . . . 159 Chapter 92 Antifungal Agents . . . . . . . . . . . . . . . . 232
Chapter 68 Childhood Immunization . . . . . . . . . . . 161 Chapter 93 Antiviral Agents I: Drugs for
Chapter 69 Immunosuppressants . . . . . . . . . . . . . . 165 Non-HIV Viral Infections . . . . . . . . . . 236
Chapter 70 Antihistamines . . . . . . . . . . . . . . . . . . . 168 Chapter 94 Antiviral Agents II: Drugs for
Chapter 71 Cyclooxygenase Inhibitors: HIV Infection and Related
+  
%  
Opportunistic Infections. . . . . . . . . . . . 239
Drugs and Acetaminophen. . . . . . . . . . 170 Chapter 95 Drug Therapy of Sexually
Chapter 72 Glucocorticoids in Nonendocrine Transmitted Diseases . . . . . . . . . . . . . . 242
Disorders . . . . . . . . . . . . . . . . . . . . . . . 173 Chapter 96 Antiseptics and Disinfectants. . . . . . . . 245
Chapter 73 Drug Therapy of Rheumatoid Chapter 97 Anthelmintics. . . . . . . . . . . . . . . . . . . . 247
Arthritis . . . . . . . . . . . . . . . . . . . . . . . . 175 Chapter 98 Antiprotozoal Drugs I:
Chapter 74 Drug Therapy of Gout . . . . . . . . . . . . . 178 Antimalarial Agents . . . . . . . . . . . . . . . 250
Chapter 75 Drugs Affecting Calcium Levels Chapter 99 Antiprotozoal Drugs II:
and Bone Mineralization . . . . . . . . . . . 180 Miscellaneous Agents . . . . . . . . . . . . . 252
Chapter 76 Drugs for Asthma and Chronic Chapter 100 Ectoparasiticides . . . . . . . . . . . . . . . . . 255
Obstructive Pulmonary Disease. . . . . . 184 Chapter 101 Basic Principles of Cancer
Chapter 77 Drugs for Allergic Rhinitis, Chemotherapy . . . . . . . . . . . . . . . . . . . 256
Cough, and Colds. . . . . . . . . . . . . . . . . 188 Chapter 102 Anticancer Drugs I: Cytotoxic
Chapter 78 Drugs for Peptic Ulcer Disease . . . . . . 191 Agents . . . . . . . . . . . . . . . . . . . . . . . . . 258
Chapter 79 Laxatives . . . . . . . . . . . . . . . . . . . . . . . 195 Chapter 103 Anticancer Drugs II: Hormonal
Chapter 80 Other Gastrointestinal Drugs . . . . . . . . 197 Agents, Targeted Drugs, and
Chapter 81 Vitamins . . . . . . . . . . . . . . . . . . . . . . . . 201 Other Noncytotoxic Anticancer
Chapter 82 Drugs for Weight Loss . . . . . . . . . . . . . 204 Drugs . . . . . . . . . . . . . . . . . . . . . . . . . . 262
Chapter 83 Basic Principles of Antimicrobial Chapter 104 Drugs for the Eye . . . . . . . . . . . . . . . . . 266
Therapy . . . . . . . . . . . . . . . . . . . . . . . . 206 Chapter 105 Drugs for the Skin . . . . . . . . . . . . . . . . 269
Chapter 84 Drugs That Weaken the Bacterial Chapter 106 Drugs for the Ear . . . . . . . . . . . . . . . . . 273
Cell Wall I: Penicillins . . . . . . . . . . . . . 209 Chapter 107 Miscellaneous Noteworthy Drugs . . . . 275
Chapter 85 Drugs that Weaken the Bacterial Chapter 108 Dietary Supplements . . . . . . . . . . . . . . 278
Cell Wall II: Cephalosporins, Chapter 109 Management of Poisoning . . . . . . . . . . 281
Carbapenems, Vancomycin, Chapter 110 Potential Weapons of Biologic,
Telavancin, Aztreonam, Radiologic, and Chemical
Teicoplanin, and Fosfomycin. . . . . . . . 211 Terrorism . . . . . . . . . . . . . . . . . . . . . . . 283
Chapter 86 Bacteriostatic Inhibitors of Protein
Synthesis: Tetracyclines, Answer Key . . . . . . . . . . . . . . . . . . . . . 287
Macrolides, and Others . . . . . . . . . . . . 215
Chapter 87 Aminoglycosides: Bactericidal
Inhibitors of Protein Synthesis. . . . . . . 218

Copyright © 2016, Elsevier Inc. All Rights Reserved.


CHAPTER 1 Q Orientation to Pharmacology 1

1
Orientation to Pharmacology

STUDY QUESTIONS 10. The nurse reassesses the patient 20-30 minutes
after administering an opiate analgesic.
Matching ________________________________________

     
11. During discharge teaching, the nurse assesses if
1. ___ Any chemical that can affect living pro- the patient will be able to take a prescribed drug 4
cesses. times a day as ordered.
2. ___ The medical use of drugs.
________________________________________
3. ___ The study of drugs and their interactions
with living systems.
12. When a patient is or could be pregnant, the nurse
4. ___ The study of drugs in humans.
researches the pregnancy category of every drug
a. Clinical pharmacology administered.
b. Drug ________________________________________
c. Pharmacology
d. Therapeutics 13. The nurse teaches patients that the medicine cabi-
net is a bad place to store medications because the
Completion heat and humidity can damage the drug.
        ________________________________________
        
14. The nurse is aware that African Americans
5. The nurse teaches a patient to avoid engaging in often do not respond as well as Caucasians to
hazardous activities when taking an antihistamine   %
@
 
=>Z
#?

for allergy symptoms. prescribed for hypertension.


________________________________________ ________________________________________
6. The nurse explains that a generic form of a newly
prescribed drug is available to a patient who has CRITICAL THINKING, PRIORITIZATION,
limited insurance coverage for drugs. AND DELEGATION QUESTIONS
________________________________________
15. Which patient would be the highest priority when
7. The nurse researches if an antidote is available providing nursing care to multiple patients?
when administering drugs that have the potential a. A patient who would like to know if a newly

  
  
 

  prescribed drug is available in generic form
b. A patient who has requested a laxative because
________________________________________
he has not had a bowel movement in 2 days

<
!
 
 
 
=>?
c. A patient who received oral drugs 1 hour ago
through a second intravenous line separate from and has complained of tingling around his
all other drugs. mouth
d. A patient who is prescribed a drug, the name of
________________________________________ which the nurse does not recognize
9. The nurse explains that quinapril and Accupril are
names for the same drug.
________________________________________

Copyright © 2016, Elsevier Inc. All Rights Reserved. 1


2 CHAPTER 2 Q Application of Pharmacology in Nursing Practice

16. Which   action would best meet the thera- CASE STUDY
peutic objective of drug therapy?
a. Assessing the patient for adverse effects of A patient returned to the nursing unit after an esophago-
drug therapy  
=Z\&?

]^``
{




b. Prescribing a drug with the fewest adverse ef- her 0900 oral drugs before the procedure because of an
fects 
$



=+|}?
c. Prescribing a drug with the fewest serious
adverse effects 1. What factors must the nurse consider when
d. Recommending that a patient avoid taking deciding which of the 0900 oral drugs should be
drugs because there are possible adverse effects administered at this time?

2
Application of Pharmacology in Nursing Practice

STUDY QUESTIONS CRITICAL THINKING, PRIORITIZATION,


AND DELEGATION QUESTIONS
Matching
         8. The nurse is preparing to administer an antihy-
   

= 
 




 ?
!
 
 

 ‚


1. ___ Knowing the major adverse reactions of a pressure at 110/70 mm Hg. What action should the
drug, when they are likely to occur, early nurse take at this time?
signs of development, and interventions to a. Administer the medication because the antihy-
minimize discomfort and harm pertensive medication is prescribed.
2. ___ The rights of drug administration coupled b. Assess the patient’s baseline blood pressure and
with the knowledge of pharmacology the blood pressure before and after the last dose
3. ___ Knowing the reason for drug use and being of this medication to determine if the medica-
able to assess the patient’s medication needs tion should be administered.
4. ___ Knowing the early signs of toxicity and the c. Call the prescriber, report the current blood
proper intervention when it occurs pressure, and ask if the medication should be
5. ___ Collecting baseline data, identifying high- administered.
risk patients, and determining the patient’s d. Withhold the medication because the patient’s
capacity for self-care blood pressure is too low to administer an anti-
6. ___ Taking a thorough drug history, advising the hypertensive drug.
patient to avoid drugs that may interact with
prescribed medication, and monitoring for 6 9. The nurse should withhold a drug and contact the
adverse interactions prescriber if the patient reported an allergy to the

~

!



$



- drug with which symptom occurring shortly after
cial or causes harm the last time the drug was taken?
a. Constipation
a. Preadministration assessment b. Dry mouth
b. Dosage and administration c. Vesicular rash

Z  


 
d. Wheezing
effects
d. Minimizing adverse effects
e. Minimizing adverse interactions
$
€ 
%
=|+?
 
g. Managing toxicity

Copyright © 2016, Elsevier Inc. All Rights Reserved.


CHAPTER 2 Q Application of Pharmacology in Nursing Practice 3

 10. Which postoperative patient assessment would CASE STUDY


warrant withholding an opiate analgesic that de-
 

 

 
=>+{?ƒ A 52-year-old man is admitted with uncontrolled hy-
a. BP 150/92 mm Hg pertension. He has been prescribed three medications:
b. Pulse 110 beats/min  @
  
=‰ ?
‹`



 Œ
-
c. Respirations 9/min  @


$


 Œ

  


! 
]`^^„
…
=†‡„
>? =>  ?
<`




 
!
 

 

he does not take the drugs as prescribed. The nurse identi-


 11. Which nursing diagnosis would be most appropri- 
 

 








ate for a patient who is receiving an opiate anal- because he cannot “feel” his high blood pressure and he
gesic that depresses the central nervous system does not have insurance for drugs.
=>+{?ƒ
a. Fatigue 1. What are possible interventions to overcome these

# 
  
  factors that are interfering with the patient’s ability
c. Risk for activity intolerance to perform self-care with this medication regimen?
d. Risk for injury

6 12. A patient who is admitted to the nursing unit from



    
 

=|>ˆ?

 


pain. The patient is due for another dose of pain


medication. What is the nursing priority at this 2. How does the nurse determine if administered
time? antihypertensive drugs are effective?
a. Administer prescribed pain-relieving drugs.
b. Assess the patient’s vital signs, tubes, and sur-
gical site.
c. Obtain the patient’s pain rating on a scale of
1-10.
d. Review the patient’s allergy history. 3. The nurse is preparing to administer hydrochlo-
rothiazide, a diuretic drug that lowers blood
 13. The nurse is preparing to administer insulin based pressure by increasing the excretion of water and
on the patient’s metered blood glucose level. electrolytes—sodium and potassium—via the
Which action could be delegated to a licensed kidneys. What laboratory tests should the nurse
  '  
 
=‰|+'‰Š+?ƒ review before administering this drug?
a. Documenting the insulin that the RN adminis-
tered
b. Drawing up the insulin in the syringe

#$

 
$
 
 -
tration
d. Obtaining the metered blood glucose level 4. What should be included when providing teaching
about a newly prescribed drug?

Copyright © 2016, Elsevier Inc. All Rights Reserved.


4 CHAPTER 3 Q
&
  
&
+  

#$ 

3
Drug Regulation, Development, Names, and Information

STUDY QUESTIONS CRITICAL THINKING, PRIORITIZATION,


AND DELEGATION QUESTIONS
Matching
    
 11. When preparing discharge instructions for a pa-
tient, what should the nurse do?
1. ___ acetaminophen a. Ask the patient which name he or she uses for

^

+%=‹%?
  the drug and use only that name in discharge
3. ___ Tylenol teaching.

Z 


 



 

a. Chemical name does not become confused.



\
  
Z 

 
 

  


 

c. Brand name for the patient to remember.


d. Review the prescriptions written for discharge
Completion and include the stated trade name and generic
name in discharge teaching.
    

12. A patient has been taking a brand-name drug
4. The same ___________________________ name for a chronic condition for several years. Recent
is never used for more than one medication. changes in his insurance plan require the use of
generic drugs whenever they are available. The pa-
5. The ___________________________ name tells tient asks the nurse if he should pay out of pocket
the nurse the active ingredients of the drug. to continue receiving the brand-name drug. The
 ‚
  





$ = ?ƒ

6. The ___________________________ name of a ={



 
?
drug will be the same, no matter which company a. That continuing to use the brand-name drug
produces the drug. will prevent confusion.
b. That drugs requiring monitoring of blood levels
7. The ___________________________ name may should have levels checked when changing
  


$




| ‰
from brand-name to generic drugs.
which lowers phosphorus. c. That generic drugs contain the same active
ingredients as their brand-name counterparts.
8. ___________________________ names of drugs d. That generic drugs usually are not absorbed at
in the same therapeutic class often have a similar the same rate as brand-name drugs.
$
 

 

$ e. That brand-name drugs contain different active
ingredients than generic drugs.
9. Nursing drug reference books are usually orga-
nized by ___________________________ names.

10. Fosinopril, lisinopril, moexipril,


quinapril, and ramipril are examples of
___________________________ names.

Copyright © 2016, Elsevier Inc. All Rights Reserved.


CHAPTER 4 Q Pharmacokinetics 5

13. The nurse is teaching a patient about tamsulosin CASE STUDY


=… ?



  
$
$

voiding due to an enlarged prostate. Because the The nurse is caring for a patient who has high blood
patient travels extensively and not all countries  
= ?
!
 
 
 


require prescriptions to get drugs, what it is impor- became worried about her high cholesterol when her sister
tant for the nurse to teach the patient?  

’ 
 “
{



 
$



a. That a drug labeled Flomax may have different see if they had any natural products that could help lower
active ingredients in different countries. 
 
{


 





b. That drugs purchased in countries other than for volunteers to enter a study. Volunteers are to submit

ˆ
{ 

>  

  
- a copy of their blood cholesterol results and again at the
safe. end of a year after taking the product as directed for a full
c. That Flomax is not available anywhere except  
Z 


 
$


 


-

ˆ
{ 

>    able based on these factors.
d. That doses of Flomax in countries that use the
metric system are different than doses of drugs 1. Control


ˆ
{  

]‹

 = ?
 

= ?


complete positive and negative information about



 ƒ
={

 
? 2. Randomization
a. Drug sales representatives
b. Drug Facts and Comparisons

#
 
d. Pharmacists
3. Blinding

4
Pharmacokinetics

STUDY QUESTIONS d. Movement into and out of body


e. Movement into the blood
Matching f. Movement out of the body


       


      


 




1. ___ Absorption 7. ___ May require energy or pores


2. ___ Distribution 8. ___ Requires small size

†

Z  9. ___ Requires lipid solubility

‹

Z
5. ___ Metabolism a. Passage through channels
6. ___ Pharmacokinetics b. Direct penetration of the membrane
c. Passage with the aid of a transport system
a. Change in drug structure
b. Change in drug structure and movement
out of the body
c. Movement from blood into tissue and
cells

Copyright © 2016, Elsevier Inc. All Rights Reserved.


6 CHAPTER 4 Q Pharmacokinetics

True or False CRITICAL THINKING, PRIORITIZATION,


   
  T   F AND DELEGATION QUESTIONS
  
6 27. The nurse is administering an intravenous push
10. ___ Cell membranes are composed of fat with  
$

 
   
=
 ?

phosphate. After administering half of the dose, the nurse


11. ___ Most drugs enter cells through channels or notes that patient’s respirations have decreased
pores. from 15/minute to 11/minute. What is the priority
12. ___ P-glycoprotein transports many drugs out of nursing action at this time?
cells. a. Assess the patient’s current pain.
13. ___ A transport mechanism is needed for a b. Call the prescriber.
water-soluble drug to enter a cell. 
{ 

 

]‹

#@ 



 





{
  
$

 
$

 
enter a cell.

]

#$

"  



6 28. The nurse inadvertently administers heparin 100
is injected into a vein it will produce effects,  '‰
   


  


but it will not if taken orally. 


 


 
$

 
 

16. ___ Polar drugs can enter fetal circulation and 
  
$

 
```
 '‰
 

breast milk. was prescribed to prevent postoperative deep vein

]~

 

= ?

 
  
=&Š!?
 



 

drugs, is primarily absorbed in the small concern?


intestine. a. Assessing the patient for DVT.

]<

Z




 
  
b. Consulting the prescriber for direction.
crushing these preparations can cause stom- c. Filling out an incident report.
ach distress or cause the acid in the stomach d. Preventing excessive bleeding.
to alter the drug.

]‡



   
=#€?
”
$

 29. The nurse realizes that when injecting insulin,
antibiotic to treat syphilis will be completely which lowers blood glucose, into the subcutaneous
effective within 24 hours after administra- fat above the deltoid muscle in a very thin child,
tion. the drug may have been inadvertently adminis-
20. ___ The protein-bound portion of a drug in cir- tered into the muscle. What is the patient at risk
culation is not able to exert its action. for?
21. ___    means most of the drug is a. Blood glucose dropping too rapidly
activated by the liver, so it must be adminis- b. Blood glucose not dropping rapidly enough
tered orally.

^^




 
 %
$$
 
30. Which laboratory test result suggests that a pa-
be given sublingually to allow the drug to be tient’s excretion of a drug may be impaired?
absorbed directly into the systemic circula- 
{!•
~^
  

tion. 
#+•
‹^
23. ___ Adding a drug to a patient’s drug regimen can 
\…•
†`
‰'
cause the other drugs to be metabolized more d. WBC: 13,000/mm3
slowly or more rapidly.

^‹

#  
@
 
 

$
6
†]
 



 


|>ˆ

-
bile in the duodenum, causing the drug to be mote excretion of most anesthetics?
reabsorbed. 
Z 
%   

^

!
 


 


 

€  

 


prescribed for bacterial meningitis would most c. Monitor urine output.
likely get to the site of infection if the drug is d. Prevent constipation.
water-soluble.
26. ___ Chemotherapy is administered through a
central intravenous line because chemother-
apy is caustic to the vein and a large central


 






moves the medication quickly.

Copyright © 2016, Elsevier Inc. All Rights Reserved.


CHAPTER 5 Q Pharmacodynamics 7

†^
& 
=‰ ?



 


 

†‹

 

 

={?

therapeutic range. When administering this drug, 3 days ago. This drug has a half-life of 7 days. He
 


 


ƒ
={

 
tells the nurse that the drug must not have been
? necessary because he does not feel any different.
a. Administer the medication only on an as- What is the basis of the nurse’s explanation of why

=|+?
  the patient has not noticed any change in how he
b. Carefully monitor the patient for therapeutic feels?
and toxic effects. a. The patient’s dose was probably too high, so
c. Be diligent about the timing of administering the drug is still working.
the medication. b. The patient could not have been taking the drug
d. Monitor blood levels of the drug to assess if it as prescribed.
is in the therapeutic range. c. The drug probably was not needed if the patient
e. Teach the patient that the drug takes several has not experienced any symptoms.
weeks to reach full effectiveness. d. The drug’s previous doses have not been com-
pletely eliminated from the body.

††
\ $
”
 


|‹`

CYP3A4 drug-metabolizing enzyme and inhib-


its P-glycoprotein in the intestines for days after CASE STUDY
ingestion. This especially affects calcium chan-
nel blockers, benzodiazepines, cyclosporine, and The nurse is caring for a hospitalized patient who has
-statin drugs. The nurse should teach a patient been prescribed the following medications: 0800 gly-
who is prescribed any of these drugs that drinking 
=& – ?
]`
 Œ
`‡``
" 
*>
=?

grapefruit juice can cause what problem? 20 mg. Breakfast is served at 0810. Hospital policy states

Z 
 %
$$ that medications may be administered 30 minutes before

Z

$



 or after the designated time. To save time, the nurse plans

‰ 
$
 
$$
$

 to administer both of these medications at 0830.
d. Rapid excretion of the drug
1. Research the onset of these medications and ex-
plain why this is not a good plan.

5
Pharmacodynamics

STUDY QUESTIONS 4. ___ !






 
$


needed to get the desired effect.


True or False 5. ___ Two drugs in the same therapeutic class with
$$

 
=^

 
^``

   


  T   F  ?
 
 
" 
$$ 
   6. ___ A drug that stimulates the transcription fac-
tor receptors may not reach a peak therapeu-
1. ___ Pharmacodynamics includes the study of tic effect until taken regularly for days to
how drugs work. weeks.
2. ___ Phase 2 of the dose-response relationship
~



 


$



starts at the point when the therapeutic effect will produce more unintended effects than a
does not increase with increasing the dose. nonselective drug.
3. ___ 





   

<

#$




$

 


effect that a drug can produce. is safe.

Copyright © 2016, Elsevier Inc. All Rights Reserved.


8 CHAPTER 5 Q Pharmacodynamics

9. ___ "  is the strength of attraction between


]<

 

  
 -
a drug and its receptor. sitivity of receptors when a patient suddenly
10. ___ Drugs with high intrinsic activity cause an 
 

=!?

 2 receptor
intense response. antagonist that slows the heart rate?
11. ___ An agonist drug blocks the stimulation of a a. BP 80/56 mm Hg
receptor. b. Pulse 118 beats/min

]^

#$



  
 

c. Respirations 26/minute
a drug, the cell can down-regulate and de- 
! 
]`‹„
…
=‹`„
>?

crease the response to the drug.


13. ___ A patient who suddenly stops taking an 6 19. An unresponsive patient who has overdosed on
antagonist drug may experience hypersensi-  
*>
=€ ?

 

 

tivity of the receptor and overstimulation. into the emergency department with severely

]‹

!
Z&50 is usually the recommended dose depressed respirations. The patient receives
for the drug.  
 
*>
=+  ?

 
-
tagonist, with a dramatic improvement in the level
of consciousness and respiratory rate and effort
CRITICAL THINKING, PRIORITIZATION, within minutes. Which is a priority nursing action
AND DELEGATION QUESTIONS 45-60 minutes after the naloxone HCl is adminis-
tered?
15. The nurse is reading research about a drug. The a. Assess for return of pain because the effect of
literature states that the drug is . What does the naloxone HCl should be peaking.
this mean? b. Assess for the need to administer another dose
a. The drug produces its effects at low doses. of naloxone HCl as the naloxone HCl effect is
b. The drug produces strong effects at any dose. ending.
c. The drug requires high doses to produce its ef- c. Prepare to administer the drug intramuscularly
fects. if the drug has not taken effect.
d. The drug is very likely to cause adverse effects. d. Prepare to counteract the effects of opiate with-
drawal.

]—
Z
$

 

 




 %
  ƒ
={

 
20. When the therapeutic index of a drug is narrow,
? what should the nurse expect?
a. Drugs can mimic the actions of endogenous a. Blood levels of the drug would be monitored
molecules. throughout therapy.
b. Receptors for drugs do not respond to hor- b. The drug would produce the desired effect at
mones and neurotransmitters produced by the low doses.
body. c. The drug would produce many adverse effects
c. The binding of a drug to its receptor is usually at low doses.
irreversible. d. The drug would only be used in an emergency.
d. Drugs can block the actions of endogenous
molecules.
e. Drugs can give the cell new functions.

 17. The nurse is administering morphine sulphate for


moderate to severe postoperative pain. Based on
the other regulating effects of morphine, what


 
ƒ
={

 
?
a. Assess respirations and hold the medication if
adventitious breath sounds are present.
b. Assess respiratory rate and hold the medication
if respirations are weak or fewer than 12/min.
c. Maintain the patient on complete bedrest with
all four side rails elevated to prevent injury.
d. Assist the patient when ambulating to prevent
injury.

Z 
  
$

  


promote bowel motility.

Copyright © 2016, Elsevier Inc. All Rights Reserved.


CHAPTER 6 Q
&
#  9

6
Drug Interactions

STUDY QUESTIONS  <


#


 ‚
   


 


decide what time a drug will be administered.


True or False >  
=˜  ?


  
$

a patient who takes many drugs for diabetes and


   
  T   F hypertension. The patient takes the other drugs
   at 0900 and 1700. At this institution, drugs can
be given up to 1 hour before or after the assigned
1. ___ A prescriber may prescribe a drug solely time. Based on proper timing of administration
because of its interaction with another drug. of this drug, when should the cholestyramine be
2. ___ Powdered drugs can be mixed once they are administered?
dissolved in a liquid. a. 0800
3. ___ Alcohol, bananas, cigarette smoking, chew- b. 0900
ing tobacco, chocolate candy, garlic, and c. 1000
grapefruit juice can interact with drugs and d. 1100
cause adverse effects.
4. ___ Antacids increase urinary excretion of alka- 9. Patients receiving certain drugs that are metabo-
line drugs. lized by the CYP3A4 enzyme should not drink
5. ___ Adding potassium clavulanate to amoxi- grapefruit juice because grapefruit juice inhibits

= ?


‚
the CYP3A4 enzyme and the drug transporter P-
beta-lactamase enzyme from destroying the glycoprotein. The patient asks what might happen
amoxicillin. if she drinks grapefruit juice and takes the medica-
6. ___ The nurse should consult with the prescriber tion. The nurse’s response is based on knowledge
regarding adequate blood levels of oral that the effect could be what?
drugs when a patient has persistent diarrhea. 
‰ 
$
  
  






absorbed
CRITICAL THINKING, PRIORITIZATION, 
‰ 
$
  
  







site of action
AND DELEGATION QUESTIONS 
# 
  


$



‰
  
  

>™|†‹
@

6 7. The nurse is providing discharge teaching for a


destroys the liver
 



 
 @
=… ?

after discharge. The drug information states to


6
]`

 
 

  
 
=| ?

caution the patient that a reaction similar to the


to prevent blood clots after experiencing a

$
  
=  ?
 

#


  
$ 
=€#?
*
$%  

priority to teach the patient to avoid which food or


 @
=| 
}!>?
$
  
–  

drink?
 @
 
 

$ 
$
 

a. Beer
=| ?




$

 

 


b. Chocolate
patient to report which effect?
c. High-fat food
a. Dizziness

{ 
b. Nausea and vomiting
c. Joint pain

{

$
 

 
 

Copyright © 2016, Elsevier Inc. All Rights Reserved.


10 CHAPTER 7 Q
 
&
 

€ 
Z

š
]]

 


 

=& ?
14. A patient who drinks grapefruit juice and does not
a drug with a narrow therapeutic index, to con- inform the prescriber may experience muscle pain

@ 
#$
$  
=!?

  
from the adverse effect of rhabdomyolysis, which
to lower triglycerides, drug interaction puts the is possible if the patient is prescribed which drug?
patient at risk for which effect? 
‰  
=€ ?
$
 
 

Z 

  
+$
=|  ?
$
 

{@ 
! @ 
=* ?
$
 

!
$

$  
=!? 
Š  
=>  ?
$
 
  

!
$


=& ?

12. Tobacco induces the CYP1A2 liver enzyme. CASE STUDY


Methadone is metabolized by the CYP1A2 liver
@
#$

 % 
 

 

The nurse is caring for a 92-year-old patient who was
methadone and smoked cigarettes, what would this admitted yesterday with pneumonia. The patient’s history
tobacco use be expected to do? includes hypertension and heart failure. The patient is
a. Diminish the effect of methadone exhibiting signs of experiencing hallucinations.
b. Make the effect of methadone more intense

{

 
$
  1. What information does the nurse need to provide

{

 
$
  to the prescriber to help determine if this behavior
is related to a drug interaction?
š
]†

 


  
 
=Š  ?

for erectile dysfunction. The nurse teaches the


patient that metabolism of this drug may be
increased, possibly decreasing the effect, if the
patient uses which dietary supplement?
a. Fish oil

#

\ 
d. Multivitamin

7
Adverse Drug Reactions and Medication Errors

STUDY QUESTIONS a. Anaphylaxis


b. Hemolytic anemia
Matching c. Hepatotoxicity

˜!
 

       e. Nephrotoxicity
f. Neutropenia
1. ___ Clay-colored stool with nausea and vomiting

^

\…
`
‰'

 

~`
Completion
‰' Œ

 
]``
‰' 
3. ___ Fatigue and hemoglobin/hematocrit 9.2%/28 7. Withdrawal syndrome occurs when a drug is
 '‰ stopped when a person is ____________________
4. ___ Frequent infections or infection with rare ____________________ on the drug.
microbe and WBC count fewer than 5,000/
mm3
<
=?

 



5. ___ Numbness and tingling around the mouth immune response.


and respiratory distress
6. ___ Pulse 52 beats/min and unexplained fainting

Copyright © 2016, Elsevier Inc. All Rights Reserved.


CHAPTER 7 Q
 
&
 

€ 
Z 11

9. _________________________ means cancer- 18. Which symptom suggests an ototoxic reaction to a


causing. drug?
a. Dizziness
10. Drugs that are _____________________________ b. Headache
can harm a fetus if the patient takes the drug while c. Nausea
she is pregnant. d. Tinnitus

]]
Z$$
 

 
  
-

]‡
–$
 
  
=€ ?

ondary drug effects at therapeutic doses -statin drug known to be toxic to the liver, the
are called ___________________ nurse reviews the patient’s laboratory tests. Test
______________________.  

 
  $ 
=‰!?

]†‹
  
 '‰

  
-

]^
\
$$
 
  


  $ 
={!?
‡~
 '‰




responses. These are called __________________ most appropriate at this time?


reactions. a. Administer the drug.
b. Assess the vital signs, and if stable, administer
13. ____________________ disease is when a drug the drug.
causes symptoms closely resembling a disease. c. Hold the drug and report the results to the pre-
 
{|
14. ____________________ is the detrimental physi- d. Hold the drug.
ologic effects caused by excessive drug dosing.
20. The nurse is mentoring a nursing student. The
student complains that it is unfair that the nurs-
CRITICAL THINKING, PRIORITIZATION, 
  
"
]``œ


 

AND DELEGATION QUESTIONS calculation on a dose calculation examination


during the last semester because it is stressful for

]




 
$$
$




students. What is the basis of the nurse’s response



* 
} @ 
=*}?ƒ to the student?
a. Constipation from codeine for pain 
Z 
 

 



  

b. Nausea and vomiting from chemotherapy drug doses under stress and dosing errors can


  
$
 
 
be fatal.
blocker vasodilators to lower blood pressure 
{ 





 
 

d. Respiratory depression from overdose of ben- when the nurse was in school.
zodiazepine sleeping pills 
{

‚
$$




policy because it is unrealistic.


16. When do adverse drug reactions occur more often? 
{



  
]``œ

={

 
? 

 
  

"

a. When patients take multiple drugs 



 
 
+  
>
‰ 

b. When patients take potent drugs Z  


=+>‰Z?
  
c. When a drug error occurs
d. When patients are younger than 1 year old  21. The nurse is caring for a patient who is experienc-
e. When patients have multiple chronic illnesses ing severe pain. Morphine sulfate 10 mg intramus-
 
=#€?



‹



!

6
]~
!
 

 
" 
=?
patient asks the nurse to administer the medication
One hour after administration, the patient reports a 

 
 
=#Š?
 
  


tingling sensation of the lips. The nurse notes that  


 

 
$


”
 

the patient has perioral edema. Which action is of should the nurse do in this situation?
greatest priority? 
 

 

  
 



&

  the way that it is ordered.
b. Hold the next dose of the medication. b. Administer the medication via the intravenous
c. Notify the prescriber. cap.
d. Withhold all food and water. 
&
$

 

$
$

  -


 




 
$


$

Z 


 
 

 
 

change the route of a medication without a


prescriber’s order and that paging the prescriber
may take some time.

Copyright © 2016, Elsevier Inc. All Rights Reserved.


12 CHAPTER 8 Q
# 
Š  

&
  

6 22. A nurse has incorrectly administered an exces- 24. What is not required on FDA-approved
sive dose of a prescribed drug to a patient. Which €\
$
  ƒ



 



 
a. The name of drug
in this situation? b. What the drug is supposed to do
a. Assess the patient for effects of the drug. c. The safe dose range of the drug
b. Place the nurse on unpaid leave. d. What to do if a dose is missed
c. Report the error to the institution’s safety coor- e. Who should not take the drug
dinator.
d. Require the nurse to identify possible strategies 25. Which is the criterion for a black box warning in
to prevent this type of error in the future. the drug literature?
a. There is potential for serious adverse effects.
 23. When should the nurse report possible adverse 

€\


"
effects of new drugs to www.fda.gov/medwatch? 
–
$




 
$


Z


 


 
adverse effects.
effect d. The drug is provided as a sample from the

+Œ



  ‚
” prescriber.
c. When the nurse notes an adverse symptom
when administering a newly released drug
^—
 


 
$

|‰Z&\Z


d. When the nurse is sure that a drug is causing an $



 
Z  

€ 
{  

adverse effect =Z€{?


$
 ƒ
a. The drug can cause dysrhythmias.
b. The drug must be taken as directed to be effec-
tive.
c. The drug is sedating and could lead to injury if
taken when driving.
d. The drug can cause fetal harm if taken during
pregnancy.

8
Individual Variation in Drug Responses

STUDY QUESTIONS 4. _______________________ is a reduction in drug


responsiveness brought on by repeated dosing
Completion over a short time.

1. Dose requirements for drugs with a narrow



#$

 

 
 $ 


 -
therapeutic range are most accurately calculated nourished patient with a serum protein level of 2.9
based on the patient’s _______________________ '‰

 



 




_______________________ increased risk for _______________________.


_______________________.
6. Based on differences in metabolism of alco-
2. When a patient takes a drug for a long time and hol, if both a man and woman consume the
becomes tolerant, the nurse would expect the 

$

=

 % ” 

prescriber to _______________________ the dose  ?



 


 @
=… -
to achieve the desired effect. ?

  %
 

-
cohol and the drug in the woman should be
3. _______________________ is the ability of a more _______________________ and last
drug to reach the systemic circulation from its site _______________________ than for the man.
of administration.

Copyright © 2016, Elsevier Inc. All Rights Reserved.


CHAPTER 9 Q Drug Therapy During Pregnancy and Breast-Feeding 13

CRITICAL THINKING, PRIORITIZATION, 6 9. The nurse is caring for a 57-year-old African-


AND DELEGATION QUESTIONS American man who has been prescribed sulfa-
 @


={
#Š?


7. A terminal cancer patient has been receiving  


 


\%—%|&

#

narcotic analgesics for severe pain for more than 6 would be a priority to report which laboratory test
months. The prescriber has increased the dose of result for this patient to the prescriber?
the long-acting opiate and added an “as-needed” 
‰!
^~<
#ˆ'‰
opiate for breakthrough pain. What should the 
–ˆ+
^^
 '‰
nurse do? 
*  
<
 '‰

˜ 

 
 
$
 
  

#+
]^
the patient is at risk for narcotic addiction.

˜ 


 


- 10. A patient is completely dependent on a respirator
scribed on a regular and an as-needed basis. to initiate respirations. Recent laboratory values
c. Recognize that the higher dose is needed be- include pH 7.48, pCO2 32 mm Hg, and HCO3 20
cause the patient would have undergone down- Z"'‰
 
  
   
 

regulation of opiate receptors. would be the effect of this acid-base imbalance on


d. Recognize that the additional medication is acidic drugs?
needed because cancer causes an up-regulation 
#@ 
$





 

of opiate receptors. the blood level of the drug


b. Trapping of the drug in the cells, decreasing the
 8. A nitroglycerin transdermal patch is ordered to be blood level of the drug
applied at 0900 and removed at 2100. The patch
from the previous day is still in place when the 11. Which symptom of an electrolyte imbalance
nurse is preparing to administer the 0900 patch. would be of most concern to the nurse when a
What is the best action by the nurse?  

  
 
=‰ ?ƒ

‰ 


 






a. Fatigue
in a different site. b. Perioral tingling
b. Remove the old patch and apply the new one in c. Arrhythmia
a different site. d. Muscle weakness and spasms
c. Remove the old patch and do not apply a new
patch until tomorrow.
d. Remove the old patch and consult with the
prescriber for further directions.

9
Drug Therapy During Pregnancy and Breast-Feeding

STUDY QUESTIONS 3. ___ The health of the fetus depends on the health
of the pregnant patient.
True or False 4. ___ Pregnant women should not take any drugs.



!
 
$

€ 
Z 


   


  T   F |  
 
Z  
|  



   identify drugs that have adverse effects on


birth outcomes.
1. ___ Drugs are not a common cause of birth 6. ___ Hepatic metabolism decreases during preg-
defects. nancy.
2. ___ The health of the fetus supersedes the health
~

\ 
 
 

 -
of the mother when drugs are prescribed to nancy.
pregnant patients.

Copyright © 2016, Elsevier Inc. All Rights Reserved.


14 CHAPTER 9 Q Drug Therapy During Pregnancy and Breast-Feeding

8. ___ All drugs can cross the placenta to some 21. A pregnant patient with a history of controlled
extent. hypertension asks the nurse why the prescriber
9. ___ Drugs are most likely to pass into fetal cir- changed her high blood pressure medication from
culation if they are water-soluble. " 
=?


=?

10. ___ Angiotensin-converting enzyme inhibitor The nurse’s response should be based on what
drugs for hypertension are prohibited during knowledge?
the second and third trimesters of pregnancy. a. Methyldopa is more effective for treating hy-
pertension during pregnancy.
Matching b. Methyldopa is pregnancy category B and
" 

  
  
>


 

       # $


trimester and category D in the second and
third trimesters.
11. ___ Alcohol
c. The dose of quinapril is more potent than the

]^

# $
dose of methyldopa.

]†

# 

˜ 
 

  



]‹

‰ 
does not.
15. ___ Methotrexate
16. ___ Phenytoin
22. What is an important role of the nurse when a
17. ___ Tetracycline
pregnant patient has a known exposure to a known
18. ___ Valproic acid
teratogen during week 4 of the pregnancy?
a. Ordering an ultrasound

>+{
 $  

   

b. Providing the diagnosis from the ultrasound to


defects
the parents

>+{


 $ 
c. Providing information and emotional support

\
  

>+{
$
d. Recommending termination of pregnancy if a

‰

 

 
  
severe malformation is detected
e. Neural tube defects
f. Premature closure of the ductus arteriosus
23. Which instruction should be included in patient
g. Renal failure and malformed skull
teaching regarding breast-feeding and drug
h. Tooth and bone anomalies
therapy?
a. Avoid taking any drugs.
CRITICAL THINKING, PRIORITIZATION, b. Avoid drugs that have a long half-life.
AND DELEGATION QUESTIONS c. The best time to take a needed drug to mini-
mize transfer to the infant is just after breast-
19. Based on pharmacokinetics, drugs are most likely feeding.
to pass into fetal circulation if they are 
ˆ 
  % 
$
$
 
a. highly polar.
b. ionized. CASE STUDY
c. protein bound.
d. lipid-soluble. A patient with a history of heroin addiction has delivered
a full-term infant.
20. A patient with a history of heroin addiction has de-
livered a full-term infant. Which signs and symp- 1. What should be included in the nursing care to ad-
toms suggest the infant is experiencing withdrawal dress possible drug withdrawal in the neonate?
symptoms?

{


 
b. Respiratory depression and lethargy
c. Peripheral cyanosis and hypotension
d. Apgar score of 3 at 1 minute and 4 at 5 minutes

^
Z 

  


 
-
riod can produce gross malformation in the fetus.
What nursing actions can help decrease teratogen-
esis during this period?

Copyright © 2016, Elsevier Inc. All Rights Reserved.


CHAPTER 10 Q Drug Therapy in Pediatric Patients 15

10
Drug Therapy in Pediatric Patients

STUDY QUESTIONS  9. A pregnant patient received morphine sulfate late


in labor. The neonate was born 22 minutes later.
True or False Because the drug crosses the placenta and the
characteristics of the blood-brain barrier of the
   
  T   F  

  
 



   be of most concern to the nurse?


a. Abdominal breathing
1. ___ All drugs that are considered safe to admin- b. Pulse 160 beats/min
ister to adults are safe for administration c. Respirations 22/min
to pediatric patients if the dose is adjusted 
! 
‡<]„
…
=†—~„
>?
appropriately for size.
2. ___ For drug purposes, 




]`
!
 

 

 
‹

$

from the end of 4 weeks to 1 year of age. medication to a shy 6-year-old child. Which nurs-
3. ___ The majority of drugs used in pediatrics ing intervention is most likely to gain cooperation
have never been tested on children. from the child with taking the drug?

‹

#   

$




ˆ 

 

 
 


in the infant than in the adult. medication and ask the parents if they have any
5. ___ Toddlers often need higher doses per body special technique for administering medication
weight than preschool children. that has been effective with this child.
6. ___ Approximating safe doses of drugs for chil- b. Mix the medication in a 6-ounce glass of juice
dren is most accurate when calculated based to mask the taste.
on weight. c. Place the medication in a large empty glass so
that the child can see that there is only a small
CRITICAL THINKING, PRIORITIZATION, amount of medication.

ˆ 

 

 
 

-
AND DELEGATION QUESTIONS cation and squirt the medication into the child’s
mouth.
7. Premature infants are at risk for what kind of
response to drugs?
11. Which drugs are recommended by the American

# " 

%
  
 
$
| 
=|?

$

$$


# 

 
  
for relieving cold symptoms in a 4-year-old child?

# " 

 
  
={

 
?

# 

%
  

 
=!?

& 
=’&€“
 
  ?
8. An infant is diagnosed with scurvy caused by

# $
=€?
 
>

– 



$


# $

 
=>‚

pH-dependent ionization and ion trapping and the


€
>?
differences in gastrointestinal physiology in the

|
=| > 
   ?
infant, the nurse would expect that the prescribed
 
$
 
>
=  
?
” 
$

weight would be what?



‰
 


 

Z" 



 
c. More than an adult dose

+Œ
 
>





$  

Copyright © 2016, Elsevier Inc. All Rights Reserved.


16 CHAPTER 11 Q
&
! 

\ 
| 

12. Which is the recommendation by the Centers for DOSE CALCULATION QUESTIONS
&  
>

|
=>&>?
$
%
%
=}!>?
 



 

14. The prescriber has prescribed 225 mg of a drug
children younger than 2 years? twice a day for a 7-year-old child who is 43 inches
a. Do not give any OTC drugs to a child younger tall and weighs 45 lb. The recommended adult
than 2 years. dose of this drug is 500 mg twice a day. Based on
b. Do not give any OTC cough and cold prepara- body surface area, is this dose safe?
tions to a child younger than 2 years.
c. OTC cough and cold drugs are safe and effec-

$

  

 
 
$

]
!


  



$
^`
 '
‰

children and is properly measured. How much medication will the nurse administer?

ˆ 
 
 

 

designed for pediatric use when administering


OTC cough and cold drugs to children younger
than 2 years. CASE STUDY
 13. The safe dose of a liquid drug for a 6-month-old is New parents tell the nurse that it is impossible to get their
`~
‰



 
 

 
infant to take an oral suspension of an antibiotic. They tell
administer the drug? the nurse that they have been putting the medicine into the

]%‰
 


 baby’s formula.

†%‰
 




%‰
 


 1. What can the nurse do to increase adherence to the
d. Calibrated plastic medicine cup drug regimen?

11
Drug Therapy in Geriatric Patients

STUDY QUESTIONS 6. ___ Changes in body fat and lean body mass
that occur with aging can cause lipid-soluble
True or False drugs to have a decrease in effect and water-
soluble drugs to have a more intense effect.
   
  T   F
~

‰
@

$

 


   the older adult.


8. ___ Drug accumulation secondary to decreased
1. ___ The goal of therapy for the older adult is to renal excretion is the most common cause of
cure the disease. adverse reactions in older adults.
2. ___ There is a wider individual variation in drug 9. ___ A reduction in the number of receptors and/
response in the older adult. 
 
$
$




3. ___ Older adults are less sensitive to drugs. older adult may decrease the response to
4. ___ Older adults absorb less of the dose of medi- drugs that work by receptor interactions.
cation than young adults.
5. ___ Absorption of many drugs slows with aging.

Copyright © 2016, Elsevier Inc. All Rights Reserved.


CHAPTER 11 Q
&
! 

\ 
|  17

CRITICAL THINKING, PRIORITIZATION, 15. The nurse is aware that an older adult patient is at
risk for which problem when prescribed toltero-
AND DELEGATION QUESTIONS

=&?ƒ
a. Diarrhea
6 10. When evaluating kidney function in the older
b. Disturbed sensory perception
debilitated adult, it is a priority for the nurse to

…


review the results if which test?

# 

 

{


 
=–ˆ+?

{
 
c. Creatinine clearance CASE STUDIES
d. Renal ultrasound
Case Study 1
11. An older adult with liver disease is receiving sev-
eral drugs that are normally highly protein bound. The nurse knows that adverse drug reactions and drug-
!
 ‚

 

^<
 '‰
= 
drug interactions in the older adult patient can have many
†


 '‰?
!
 




different causes. How can the nurse decrease the inci-
patient for symptoms of dence of adverse drug reactions and drug-drug interac-
a. excessive action of the drugs. tions in the older adult based on the following factors?
b. inadequate action of the drugs.
1. Altered pharmacokinetics
š
]^
!
 

 

 

+{#&

 
=! ?


  
<†% %

woman who has joint pain. Which symptom would


warrant withholding the drug and contacting the
prescriber?
a. Constipation 2. Multiple severe illnesses and multiple-drug

&$
 



 therapy
c. Dizziness when changing from a prone to up-
right position
d. Tarry stools

6 13. Which assessment is a priority before administer-




=?


 
 ƒ 3. Poor adherence
a. Alertness and orientation
b. BP and pulse
c. Bowel and bladder elimination

# 



6 14. The nurse is aware that anticholinergic adverse Case Study 2


effects of drugs commonly cause more problems A 76-year-old patient who has mild arthritis and a cataract
in older adults than in younger adults. Which of the left eye has been prescribed four medications with
anticholinergic effect would be a priority to report varied time intervals.
to the prescriber?
a. Blurred vision
‹
#$


 

 

 


b. Dry mouth overcome obstacles and promote adherence to the


c. Has not voided in 16 hours prescribed drug regimen.
d. No bowel movement for 48 hours

Copyright © 2016, Elsevier Inc. All Rights Reserved.


18 CHAPTER 12 Q Basic Principles of Neuropharmacology

12
Basic Principles of Neuropharmacology

STUDY QUESTIONS 13. ___ Drugs can promote but cannot prevent trans-
mitter release.
Matching
]‹

{

 
 

increase the release of serotonin from the



      vesicles into the synapse.

]

{


$

 
  

1. ___ Areas on the axon where neurotransmitters qualities that a drug can have.
are stored.
2. ___ Areas on the postsynaptic cells that can be
stimulated or blocked by drugs. CRITICAL THINKING, PRIORITIZATION,
3. ___ Drugs that activate receptor activity. AND DELEGATION QUESTIONS
4. ___ Drugs that prevent receptor activity.
5. ___ Moving the axon potential down the neuron. š
]—
!
 


  
 

-
6. ___ Arrival of this at an axon terminal triggers ministering a nonselective blocker of alpha1, beta1,
release of a transmitter. and beta2 receptors to patients with which chronic
7. ___ Molecules from the axon terminal that bind  = ?ƒ
={

 
?
to receptors on the postsynaptic cell. a. Asthma
8. ___ The process by which information is carried b. Benign prostatic hyperplasia
across the synaptic gap. c. Diabetes mellitus
9. ___ The process by which the parts of neu- 
*#Š
$
rotransmitters are recycled back to the e. Rheumatoid arthritis
neuron from which they were released.

]~
{

 
 




a. Action potential of antidepressant drugs. What is their mechanism


b. Agonist of action?
c. Antagonist a. Decrease the synthesis of serotonin in the axon
d. Axonal conduction b. Decrease the pumping of serotonin back into

+  
=  ? the axon from which it was released
f. Receptor 
# 

 
$

$


g. Reuptake vesicles into the synapse



{ 
    
{ 

$

i. Vesicle
18. A cholinergic drug that mimics the action of
True or False acetylcholine would cause the heart rate to slow
because the drug does what?
   
  T   F
a. Blocks receptors for acetylcholine
  
b. Causes an increased release of acetylcholine
c. Causes a decreased release of acetylcholine
10. ___ The impact of a drug on a neuronally regu-

{ 
  

$

lated process is dependent on the ability of
 





11. ___ Activation of a receptor always results in
speeding up a physiologic process.
12. ___ Drugs can cause an increase in the formation
of neurotransmitters.

Copyright © 2016, Elsevier Inc. All Rights Reserved.


CHAPTER 13 Q
|  
$

| 
+
{  19

]‡
 
   

 
=+  ?

20. Drugs that block transmitter reuptake have what
take the place of an opiate at opiate receptors and effect on receptor activation?
reverse respiratory depression caused by an opiate a. They decrease receptor activation.
= ?
   ƒ
={

 
? b. They increase receptor activation.
a. Binding of naloxone and opiates to opiate c. They have no effect on receptor activation.
receptors is reversible. d. They have an unknown effect on receptor acti-
b. Naloxone has a longer half-life than opiates. vation.

+ 


 
$
$

-
tor.
d. Naloxone is selective for opiate receptors.
e. The dose of naloxone is larger than the dose of
the opiate.

13
Physiology of the Peripheral Nervous System

STUDY QUESTIONS 9. Parasympathetic stimulation of the heart causes


_________________ of the heart rate.
Matching

]`
{ 
 
$

 -
       
   tinal tract causes slowing of gastrointestinal
 %   _________________ and _________________.

1. ___ Regulation of smooth muscle 11. Parasympathetic stimulation of the urinary bladder
2. ___ Thinking, emotion, and processing data causes urinary _________________.
3. ___ The somatic and autonomic nervous systems
4. ___ Heart, secretory glands, and smooth muscle
]^
{ 
 


$
 

5. ___ Performs “housekeeping” chores of the body to _________________ _________________.

a. Autonomic
b. Central CRITICAL THINKING, PRIORITIZATION,
c. Parasympathetic AND DELEGATION QUESTIONS
d. Peripheral

{  š
]†

 



 
=–|?
$
]<<']`‹

mm Hg takes a potent vasodilating drug to lower


Completion the blood pressure. One hour after administration,
the BP has dropped to 135/78 mm Hg in response

—
{ 
$

 

 



 

 
 


causes contraction of _________________


is an expected response to this rapid change in
_________________.
BP?

>  

$
^

7. Parasympathetic stimulation of the eye causes
b. Rubor when legs dangling
_________________ _________________ and
c. Pulse increase of 20-30 beats/min
_________________ _________________.

ˆ

 
$

 
‰'

<
{ 
 
$



causes ________________ and increased


________________ ________________.

Copyright © 2016, Elsevier Inc. All Rights Reserved.


20 CHAPTER 13 Q
|  
$

| 
+
{ 

14. Why do most drugs that affect muscarinic re-


]‡
!   
=… ?



 


ceptors of the parasympathetic nervous system blocks alpha1A receptors in the bladder neck and
produce adverse effects?  
#








 

a. High doses are needed to get therapeutic ef- use of this drug to a surgeon who is planning to do
fects. which type of surgery?
b. Most drugs that affect muscarinic receptors are a. Cataract removal
nonselective. b. Colon resection
c. Muscarinic receptors are stimulated by epi- c. Herniorrhaphy
nephrine, which is secreted by the adrenal d. Valvuloplasty
gland.
d. Muscarinic receptors are present on all post- 20. A decrease in inactivation of norepinephrine oc-
ganglionic neurons. curs with administration of monoamine oxidase
=€}?
 
 
 



 -
6 15. A military nurse receives a soldier from a combat ment when administering these drugs?
zone who may have been exposed to nerve gas. a. Alertness and orientation
Because nerve gas inhibits the enzyme cholin- b. BP and pulse
esterase, which normally breaks down the neu- c. Bowel sounds and abdominal distention
rotransmitter acetylcholine, it is a priority for the 
€ 
 


nurse to assess for what?
a. Bradycardia
b. Fever CASE STUDY
c. Headache
d. Yellowing of skin A patient receives a nonselective drug administered to
slow the heart rate and acts by blocking stimulation of
š
]—
!
 

 


  
$

beta1 and beta2 sympathetic nervous system receptors.
anticholinergic drug was prescribed for a patient =!
$

 
$
’ 

 “?
 


with which health issue? be the possible related adverse effects on the following
a. Diarrhea organs or processes? What are the appropriate nursing ac-

\    

  
=\Z&? tions for these adverse effects?
c. Hypotension

ˆ 

]
Z

6
]~
& @ 
=>  ?

 1 adrenergic block-
ing drug, is prescribed for a patient with benign 2. Respiratory rate
  
 
=–|*?






  
 


 

concern for this patient is assessing for what?


3. Airway
a. Cardiac output—bradycardia

{ $ž@@

 
  

{ž

 

{$% ž 
 
 
‹
\   


18. Which is the only neurotransmitter that activates


beta2 adrenergic receptors? 5. Production of glucose by the liver and release into
a. Acetylcholine the blood
b. Dopamine

Z
d. Norepinephrine
6. Heart

Copyright © 2016, Elsevier Inc. All Rights Reserved.


CHAPTER 14 Q Muscarinic Agonists and Antagonists 21

14
Muscarinic Agonists and Antagonists

STUDY QUESTIONS 6 3. The emergency department nurse receives a


patient brought in by an ambulance with suspected
Administration and Consultation poisoning from exposure to muscarinic insecti-
cide. The priority nursing action is to prepare
1. The nurse is preparing to administer a muscarinic a. to relieve the pain of muscle spasms.
 
#$

$
   
 - b. for possible respiratory arrest.



 
 
$



c. to place the patient in a side-lying position

$

 


=  ?

$
because the patient is likely to vomit.
consultation with the prescriber would be indi- 

 
 
 
 
= ?
a. Pulse 110 beats/min ________________  4. The nurse has administered atropine as a preopera-
tive medication. The patient complains of a sudden
b. BP 100/60 mm Hg ________________ feeling of warmth. Her skin is warm and dry, and
c. Wheezing ________________ 
$ 

 
 


 
ƒ

+$

 
{!!
d. Drooling ________________ b. Assess the patient’s vital signs and notify the
e. Postoperative abdominal distention surgeon.
________________ c. Assess the patient’s vital signs and mental sta-
tus and notify the operating room that surgery
f. Recent bowel resection ________________ must be postponed.
g. Recent vaginal delivery of a 7-lb neonate d. Assess the patient’s vital signs and mental sta-
________________ tus and, if within normal limits, document the




 


| 
  
$
†`

$



obstruction ________________ 5. What form of antidote would the nurse expect to


i. Positive hemoccult of stool ________________ be prescribed for a person who is at risk of expo-
sure to toxic levels of insecticide or nerve gas?
”
!{*
`^
 '‰Œ
!4
]<
 '‰

Z% 
 
________________ 
Z% 
 

‰"
CRITICAL THINKING, PRIORITIZATION, 
{$%” 

AND DELEGATION QUESTIONS

^


 



 


eats them. Which symptoms suggest to the camp


nurse that the camper is experiencing muscarinic
  ƒ
={

 
?
a. Tachycardia
b. Hypertension
c. Profuse salivation and lacrimation
d. Dilated pupils that do not respond to light
e. Wheezing

Copyright © 2016, Elsevier Inc. All Rights Reserved.


22 CHAPTER 14 Q Muscarinic Agonists and Antagonists

—
} 
=& ?


  
- 9. The nurse should review the patient’s history for
ergic drug that is available in four formulations.  
$ 

 %˜!

$

A patient has experienced many adverse effects  



 ƒ
={

 

when taking oxybutynin immediate-release tablets ?


=& 
#?
$
 
 
=}–?
{

& $ 
=Z ?
asks the nurse how the transdermal patch could 
… 
=! @?

cause fewer adverse effects if it is essentially the 


{$ 
=ŠZ{# ?
same drug. The nurse’s response should be based 
!
=&?
on what knowledge? 
! 
={  ?
a. Transdermal absorption bypasses metabolism
in the intestinal wall.
]`
 
  


=&?

$ -
b. The transdermal form is water-soluble and 
=! @?


 

$

 

poorly absorbed. lacks the P450 cytochrome CYP2D6 isoenzyme?


c. The transdermal form is a different drug than ={

 
?
oral oxybutynin. a. More adverse effects
d. The transdermal form is less effective than the 
‰ 
$
 
  
extended-release oral form. c. Prolonged drug action
d. Need to take the drug on an empty stomach
6
~
!
 

 
 
={ 

?


~<% %
 



 
$

]]
! 
={  ?
^`



 

-
OAB, diabetes, osteoarthritis, and osteoporosis. As scribed for an older adult patient with OAB. The
the nurse explains the drug action, the patient asks nurse instructs the patient to take the medication at




$$
$
 
=&
what time of day?
‰?



 
!
 

a. One hour before breakfast
explain that the priority reason for the prescriber’s b. One hour after breakfast
choosing trospium instead of oxybutynin, based on 


 

$
 $ 
age and chronic conditions, is prevention of what d. Any time during the day as long as it is at the
effect? same time each day
a. Blood sugar spikes
b. Constipation 12. The nurse is caring for a patient who has taken an
c. Dry mouth overdose of a tricyclic antidepressant drug that has

>+{

$$ pronounced antimuscarinic properties. The nurse

 


 ƒ
={

 
?

<
Z 
 
  ‚

$

{
  

 

 
*#Š



b. Administer atropine.
mycobacterium infection include clarithromycin c. Administer drugs for hypotension.
=– ?
``

|}


 



d. Administer phenothiazine antipsychotic drugs
home medications. The patient’s regimen includes to treat delirium.
 $ 
=Z ?





 
 - e. Administer physostigmine.
virdine 400 mg three times daily, nevirapine 200
mg twice a day, and ritonavir 600 mg twice a day. 13. Which intervention for dry mouth caused by anti-
Based on possible drug interactions with darifena- cholinergic drugs would be least likely to produce
cin, the nurse should do what? adverse effects?

\

 

$


\#
a. Cough drops
distress. 
‰ 
  
 
b. Administer the medications so that there is at c. Water
least 1 hour between each medication. 
{ %$
 
 

 
c. Review the home medications and the medica-
tion order with the prescriber.
d. Hold all of the medications.

Copyright © 2016, Elsevier Inc. All Rights Reserved.


CHAPTER 14 Q Muscarinic Agonists and Antagonists 23

6
]‹

 


  
 
=\"?
$

 
{

 

 Œ







#



membranes are dry. Bowel sounds are hypoactive, and
for the nurse to assess for which action that can there are multiple areas of dullness when percussing
possibly cause inadvertent excessive dosing of this the abdomen. The bladder is palpable above the pubic
drug?   

 
 
 
—`
‰
 

a. Not drinking a full glass of water with the drug   


 
 

–ˆ+
†^
 '‰Œ
 

b. Not washing hands after administering the drug ^†


 '‰Œ
 
]~<
 '‰Œ
–>
]^‹``'3. Cre-
c. Taking the drug at the same time as other drugs atinine clearance has been ordered, and the collection was
d. Taking on an empty stomach started at 0600. The nurse is preparing the patient’s 0900
 


=&?

DOSE CALCULATION QUESTIONS 1. What do laboratory test results suggest?

15. A patient is prescribed 5 mg of oxybutynin syrup


every 12 hours. Oxybutynin syrup is available as

 '
‰
*




 

administer per dose? 2. What concern does the nurse have about adminis-
tering tolterodine?

16. The recommended subcutaneous dose of atropine


for a child is 0.01 mg/kg, not to exceed 0.4 mg.
a. What is the recommended dose for a child who
is 32 inches long and weighs 24 lb? 3. What symptoms suggest antimuscarinic toxicity?

b. Based on the recommended dose, what amount


of drug should the nurse draw into the syringe
$


  

`
 '‰ƒ
4. What should the nurse do?

CASE STUDY
A 66-year-old woman with a history of type 2 diabetes
mellitus, hypertension, depression, and OAB is admit- 5. The prescriber discontinues the tolterodine. The
ted to a medical unit with the diagnosis of altered mental patient’s condition improves. The patient is con-
  
  


–|
]<^']]`

* 
cerned about bladder control. What teaching can
P 118, R 14, and T 102.4° F. The patient is becoming the nurse provide to assist a patient with OAB to
confused and is exhibiting symptoms that suggest she is attain bladder control?
hallucinating. The patient resists efforts to open her eyes

Copyright © 2016, Elsevier Inc. All Rights Reserved.


24 CHAPTER 15 Q
>  
# 

!
ˆ 

€ 
\ 

15
Cholinesterase Inhibitors and Their Use in Myasthenia Gravis

STUDY QUESTIONS 10. Therapeutic doses of cholinesterase inhibitors


 

$$= ?ƒ
={

 
?
Matching a. Constrict pupils
b. Constipate

      c. Relieve wheezing

{
  

]

ˆ 

  
 
  
{ 
 
 
 
2. ___ Antidote for poisoning by organophosphate
insecticides 11. Toxic levels of cholinesterase inhibitors can do

†

ˆ 

 
 %
  
 ƒ
={

 
?
blockade a. Blur vision

‹

#  
  
 
 

b. Cause respiratory depression
treat glaucoma c. Cause tetany



#  
  
 

d. Constipate
gas that can be used in bioterrorism 
# 
 
 
6. ___ Antidote for cholinergic crisis

~

{% 
  
  
 - 6 12. What is a priority nursing concern when neostig-
tor that does not cross blood-brain barrier mine is used to reverse neuromuscular blockade in
8. ___ Organophosphate insecticide postoperative patients?
a. Aspiration of secretions
a. Atropine b. Diaphoresis

Z  c. Flatus

Z 
ˆ 

d. Malathion
e. Neostigmine  13. A 66-year-old man has been taking ambenonium
f. Physostigmine =€ ?
$
 
 
$
‹
  
*

g. Pralidoxime is admitted to the hospital for a prostate biopsy.


h. Tabun The ambenonium is scheduled to be administered.
What should the nurse do?
a. Administer the drug.
CRITICAL THINKING, PRIORITIZATION,
b. Hold the drug.
AND DELEGATION QUESTIONS c. Assess for urine retention before and after
administering the drug.
9. What can cholinesterase inhibitors do? d. Consult with the urologist before administering

# $
   

  
”- the drug.
tions

# $
   

  
 -
onic, and neuromuscular junctions
c. Prevent transmission at neuromuscular junc-
tions
d. Prevent transmission at muscarinic, ganglionic,
and neuromuscular junctions

Copyright © 2016, Elsevier Inc. All Rights Reserved.


CHAPTER 15 Q
>  
# 

!
ˆ 

€ 
\  25

 14. The nurse is caring for a male patient who is pre- 3. The prescriber is unsure, by history, if the patient
 
  
=€ ?
^‹`
 

is experiencing myasthenic crisis or cholinergic
times daily for myasthenia gravis. The patient   
{

  
$


states that he is experiencing an extreme increase to differentiate. What should the nurse do, along
in muscle weakness and that he needs the nurse to with preparing to administer the edrophonium?
administer 300-mg doses. What should the nurse
do?
a. Administer 240 mg because that is the dose
ordered by the prescriber.
4. The nurse is doing discharge teaching for this
b. Administer 300-mg doses because myasthenia
patient. What teaching should the nurse do to help
gravis patients often need to adjust the dose of
the patient monitor the response to medication for
medication according to symptoms.
his myasthenia gravis?
c. Call the prescriber and request an order for the
increased dose.

# 


 
$

-
toms including excessive muscarinic stimu-
lation, and contact prescriber with patient’s
Case Study 2
" 

 
 
The nurse has been asked to speak to a 4H group in a
farming community on preventing poisoning by organo-
DOSE CALCULATION QUESTIONS phosphate insecticides. The nurse has stressed the impor-
tance of following all directions provided and seeking

]
 
=€ ?





  
$



 
mouth 3 times a day. The pharmacy stocks 10-mg
tablets. How many tablets should the nurse admin- 5. What information should the nurse include to
ister for one dose? prevent
a. exposure through the skin or eyes when using
insecticides?

]—
+  
=|  ?



 

test for myasthenia gravis as a single dose of 0.02


mg/kg. The patient weighs 154 lb. Neostigmine b. exposure through the respiratory tract when
=|  ?

  


” 

using insecticides?
$
`
 '‰
*

  



administered to this patient?


c. oral exposure?

CASE STUDIES
6. What would the nurse include in an explanation of
Case Study 1 why insecticides have such potential for poisoning
A patient who is receiving a reversible cholinester- humans?
ase inhibitor for myasthenia gravis is brought into the
emergency department by his family because of extreme
 
 

$
  
7. What information would the nurse include when

]
#

 



 


explaining which symptoms warrant seeking im-
if the cause of the weakness is myasthenic crisis mediate medical attention when using pesticides?
versus cholinergic crisis?

8. What information should be provided regarding


the treatment for pesticide poisoning?
2. What assessments should the nurse perform? What
questions should the nurse ask the patient and
family? Why?

Copyright © 2016, Elsevier Inc. All Rights Reserved.


26 CHAPTER 16 Q Drugs that Block Nicotinic Cholinergic Transmission

16
Drugs that Block Nicotinic Cholinergic Transmission: Neuromuscular Blocking Agents
and Ganglionic Blocking Agents

STUDY QUESTIONS 8. The patient is at greater risk for enhanced neu-


romuscular blockade when which antibiotics are
Matching  ƒ
={

 
?
a. Penicillins

      b. Tetracyclines

˜
1. ___ Muscle twitching d. Aminoglycosides
2. ___ Process that leads to muscle contraction
3. ___ Pumping positively charged ions from inside 9. Which drug would decrease the risk of hypoten-
to outside the cell membrane sion when atracurium is administered?

‹

ˆ
  
$
 
  

 
=!?
across the inner and outer cell membrane 
 

= ?
5. ___ Positive charges move inward, making 
& 
=– ?
the inside of a membrane more positively 
# $
=€?
charged than the outside of the membrane
10. The nurse should assess for and plan interventions
a. Depolarization to relieve muscle pain 12-24 hours after surgery

Z % 
 for patients who have received which drug?
c. Fasciculations a. Atracurium
d. Polarization b. Pancuronium
e. Repolarization 
{
d. Tubocurarine
CRITICAL THINKING, PRIORITIZATION,
 11. An anesthesia resident is supervising the reversal of
AND DELEGATION QUESTIONS the neuromuscular blockade for a patient who has
received succinylcholine. The resident directs the
6. A patient who is 8 weeks pregnant must have sur-  

 
`

$
  
=|  -
gery. The anesthesiologist administers vecuronium ?
  
 


 
ƒ

=+?


 
  
–  
={

 
?
this is a quaternary ammonium compound, what a. Administer the medication slowly and monitor
is the most likely effect of this medication on the respirations.
developing fetus? 
˜ 

 
  
  
-

ˆ  

  
 
  


b. Respiratory depression succinylcholine, a depolarizing neuromuscular


c. Bradycardia blocker.
d. Teratogenesis c. Administer the medication only if mechanical
assistance for ventilation is available.
6 7. Which preoperative laboratory test result would be 
˜ 

 





a priority to report to the anesthesiologist when a should also be administered.


patient is scheduled to have a competitive neuro-
muscular blocking agent because this condition
increases the risk of respiratory arrest when turbo-
caine is administered?

‰!
†`
#ˆ'‰

–ˆ+

 '‰

… 

 
]~
 '‰

| 
†]
Z"'‰

> 
Ÿ
^`]—
Z 
#

 
 
CHAPTER 17 Q Adrenergic Agonists 27

6 12. A patient will be receiving succinylcholine before 2. The nasogastric tube is draining a large amount of
electroconvulsive therapy. What is the nursing bile-colored liquid. How does this affect potas-
priority? sium levels and nursing care?
a. Administering the succinylcholine
b. Administering atropine if toxicity occurs
c. Performing the electroconvulsive therapy
d. Preparing for possible respiratory arrest
3. As the patient regains neuromuscular functioning,
the nurse instructs the patient to take deep breaths.
DOSE CALCULATION QUESTION How does deep-breathing counteract the adverse
effects of histamine release stimulated by the com-

]†
& 
=& ?
^
 '

  
petitive neuromuscular blocking agent?
{!!

 

$ 
$

^^`%
 

who is experiencing malignant hyperthermia after


anesthesia with succinylcholine. How many mg
should the nurse administer?
Case Study 2
A patient is receiving a neuromuscular blocker for pro-
CASE STUDIES longed paralysis during mechanical ventilation.

Case Study 1 4. Describe measures that should be included in nurs-


!
    
 

=|>ˆ?
 

 
$

ing care and their rationale.
patient who received a competitive neuromuscular block-
ing agent during surgery to remove a cancerous section
$


!
 




|>ˆ


  
 


  
=+\?
 


low-intermittent suction. 5. The patient spikes a temperature of 102° F within


an hour after the neuromuscular agent infusion is

]
 



 
 

|>ˆ
begun. The infusion of the neuromuscular block-
nurse must monitor while the patient is still under ing agent is stopped, the patient receives a dose of
the effects of the competitive neuromuscular block-  
=& ?


 ‚
 -
ing agent? ture begins to drop. Why was dantrolene adminis-
tered to lower this patient’s temperature instead of
an antipyretic such as acetaminophen?

17
Adrenergic Agonists

STUDY QUESTIONS a. Alpha1


b. Alpha2
Matching c. Beta1
d. Beta2
   
       
-
   

1. ___ Vasoconstriction

^

# 
$
$
  
 
3. ___ Relief of severe pain
4. ___ Promotes bronchodilation

Copyright © 2016, Elsevier Inc. All Rights Reserved.


28 CHAPTER 17 Q Adrenergic Agonists

True or False  14. The nurse is working in the emergency depart-


ment. A local anesthetic combined with epineph-
   
  T   F
rine is often used when suturing wounds that need
  
a small area of anesthesia and are likely to bleed.
  

 

  
  -
5. ___ Catecholamines are inactivated before
tion with the prescriber before administering a
reaching systemic circulation if administered

 


 ƒ
={

orally.

 
?
6. ___ Catecholamines cross the blood-brain bar-
a. Blood pressure 86/50 mm Hg
rier, activating the central nervous system.
b. History of AV heart block
7. ___ Catecholamines include the drugs epineph-
c. History of angina pectoris
rine, norepinephrine, isoproterenol, dopa-

|
  

mine, and dobutamine.
e. Pulse 50 beats/min
8. ___ Catecholamines are effective when adminis-
tered by any parenteral route.
6
]
!
 

 




9. ___ Catecholamines are polar molecules.


patient with a history of asthma and type 1 diabe-
10. ___ Catecholamines are destroyed by mono-


=!]&€?
#





$


 
=€}?

 %}%
which possible effect of this drug?
  $ 
=>}€!?
@




*

 

liver and intestinal wall.
b. Dizziness with position changes

{ 

 
CRITICAL THINKING, PRIORITIZATION, d. Tremor
AND DELEGATION QUESTIONS
16. A patient has a history of depression. Treatment
11. Direct-acting adrenergic drugs mimic the action of with which antidepressant is most likely to de-
 ƒ
={

 
? crease the inactivation of dopamine and increase


=>? 
 
$
ƒ
={

 
?
b. Dopamine a. Bupropion

Z
=? b. Nortriptyline

+
=+Z? 
{ 
d. Tranylcypromine
12. The nurse is caring for a patient who is receiving

 
 
=#Š?

!
 


]~
|
=+%{?

  


been assessing her patient’s vital signs every 15 



=}!>?


 
   

 -
minutes. The nurse notes that since the last assess- tient states that she understands why the label says



 




it should not be used if she has high blood pres-
What should the nurse do? sure, but she does not understand why it should
a. {


$ 


 
$
not be used by diabetics. The nurse’s response is
adverse effects, and notify the prescriber. based on knowledge that phenylephrine can cause
b. {top the infusion, notify the pharmacy for an  ƒ
={

 
?
immediate replacement, and assess the patient. a. Anorexia
c. Continue to assess the patient and infuse the b. Diaphoresis
solution as prescribed. c. Hypoglycemia in the diabetic patient
d. Continue to assess the patient and infuse the d. Hyperglycemia in the diabetic patient
solution as prescribed for up to 24 hours after e. Tremor, which the patient may interpret as
hanging. hypoglycemia

13. Patients with which chronic diseases would be


at most risk for adverse effects from adrenergic
  ƒ
={

 
?

>
 
 
  
=>}|&?

\    

  
=\Z&?
c. Hypertension
d. Diabetes
e. Osteoarthritis

Copyright © 2016, Elsevier Inc. All Rights Reserved.


CHAPTER 18 Q Adrenergic Antagonists 29

 ]<
!
 

 


 
$- CASE STUDY
sion. The nurse assesses the large antecubital
vein and notes that the site is swollen, cold, and A 4-year-old child comes to the emergency department
extremely pale. What is the basis of the nurse’s with angioedema, wheezing, and hypotension after eating
next action?
 


Z

 
a. A clot has formed and can break off and be-
come an embolus. 1. Describe how epinephrine treats the symptoms of

Z  
$
 
 
 
  
anaphylactic shock.
tissue necrosis.
c. The nurse should infuse dopamine only through
a central vein.

!

$ 
 





2. The child is prescribed an epinephrine auto-



 $
  ”
=Z|
 ?
 


 
 

the parents and the child about administering the


medication?
DOSE CALCULATION QUESTIONS

]‡

 

  





infusion of dopamine at a rate of 300 mcg/min.


3. How can the parents ensure that their young child
The patient weighs 154 lb. The dopamine infusion


Z|
 
  


 ƒ

  



$
]``
 '^`
‰
!

$ 


  

‰'
 




 
 

 

  


$-
sion pump?

^`
&  
*>
=& ?

  


 

$
^`
 '^`
‰
!
  
 
$

 -
tient weighing 110 lb is 250 mcg/min. What is the
recommended dilution and administration rate?

18
Adrenergic Antagonists

STUDY QUESTIONS 
– 
  
 
=–|*?
b. Pheochromocytoma
Matching c. Raynaud’s disease

         


   
   
    

]

–
 
=–|?
‡`'—`

*

]^`'<`

mm Hg

^

– 
  


†

| 
  

 

Copyright © 2016, Elsevier Inc. All Rights Reserved.


30 CHAPTER 18 Q Adrenergic Antagonists

True or False 6 15. A patient with coronary artery disease is receiving


 @ 
$
 
#





   


  T   F
for the nurse to report to the prescriber which ef-
  
fect of the drug?
The nurse would consult the prescriber before adminis- a. Drop of 15 mm Hg in systolic BP with position
tering the drug if a patient was just prescribed alfuzosin changes
=ˆ  ?


 
” 
 
 

 

b. Headache
a history of 
# 

 
$
^`
  '
d. Nasal congestion
4. ___ Angina pectoris



 
*#Š
$ 6 16. A patient who is prescribed an alpha1-adrenergic
6. ___ Diabetes mellitus type 2 antagonist for high blood pressure is brought into

~

Z
 $ 
 
 
$
 
 

8. ___ Frequent urinary tract infections =Š  ?





$
!

$


9. ___ Hepatitis B nurse is to assess for and follow orders to prevent


10. ___ Hypertension what effects?
11. ___ Ventricular dysrhythmia 
 
  

  
b. Migraine headache and photophobia
c. Prolonged erection and discomfort
CRITICAL THINKING, PRIORITIZATION, 
{
 

  
  
AND DELEGATION QUESTIONS

]~

 
   
=… ?


12. The generic names of alpha1 receptor antagonists assessment would indicate that therapy has
 
 

$ƒ achieved the desired effect?
a. -azole 
Š
^`


^%†


 
b. -lol b. Position changes without dizziness
c. -osin c. Absence of dysuria when voiding
d. -sartan 
| 
  

 
†``

6
]†
#



$

 

  

 -
]<
*

   
=… ?

 -
ratory test result before administering alfuzosin tered?
=ˆ  ?


 ƒ a. With food at any time of day

{!'‰! b. 1 hour before or 2 hours after the same meal

–ˆ+'  each day

…–{'] c. 30 minutes after the same meal each day
d. Na+/K+ d. With a full glass of water and remain upright
for 30 minutes

]‹

 

” 

  
 @ 
=€-
 ?

 
$

 

19. A patient who is receiving phenoxybenzamine
indicate a need for further teaching? =& @?

 

$  


’#





 @ 
BP is 75/40 mm Hg, P 135 beats/min. The nurse

$
]^%^‹

$
#
 
 

contacts the prescriber and prepares to administer
medication or have a dose increase.” what?

’#

 

 
 




Z
morning.” b. Norepinephrine

’#




 
$


$

$

# 


$
 


#




d. Metoprolol
morning.”

’#





†`%—`

$
20. The generic names of beta1 and beta2 receptor
#
 

 
 
$

 “   
 
 

$ƒ
a. -azole
b. -lol
c. -osin
d. -sartan

Copyright © 2016, Elsevier Inc. All Rights Reserved.


CHAPTER 18 Q Adrenergic Antagonists 31


^]
 
 


  
- 27. Which adrenergic antagonists block alpha1, beta1,
ing a beta blocker and consulting the prescriber? and beta2 receptors?
a. Apical pulse 48 beats/min a. Atenolol and bisoprolol
b. BP 110/70 mm Hg b. Acebutolol and pindolol
c. 2+ ankle edema c. Carvedilol and labetalol

>  

 
‡`
 '‰ d. Propranolol and timolol

22. The nurse is caring for a full-term neonate whose 6


^<
#



$

 


$
 -
mother took the drug betaxolol throughout preg- sive cardiosuppression when a patient is pre-
 

 




- scribed metoprolol and what other drug?
ity to report to the pediatrician? 
  
=‰?
a. Apical pulse 80 beats/min 
* @
=*&#ˆ#‰?
b. BP 65/45 mm Hg 
! @ 
=*?

| 
 
‹<
 '‰ 
Š  
=>  ?
d. Respirations 35/min

! 
†~^„
>
=‡‡„
…?
DOSE CALCULATION QUESTIONS
6
^†

$
 

 
 
$


]

$
 


^‡
| 

 


 '‰
# -
=‰ ?








venous push directions state to administer over at
prescriber? least 1 minute. The nurse will administer each 0.1
a. Drop in apical pulse from 80 to 65 beats/min ‰

 




 ƒ

 
 



> 
 


d. Headache
†`
‰  

  


 

^‹

$
 
 
$



infusion at 2 mg/min, to be adjusted by patient re-
 
$

 



 


 
#




 
$
^
 '†

 %  






‰
!
 


   

‰'



 ƒ
={

 
? What rate will the nurse initially program into the
a. AV heart block pump?
b. Chronic obstructive pulmonary disease
c. Depression
d. Diabetes mellitus
CASE STUDIES

{
 
 



6
^
#


 




$
 
Case Study 1
  
 
  
$



 
A 58-year-old man has just been prescribed prazosin


  

=‰ ?


=€ ?
$
 
prescriber?

> 
]^
 '‰
]
Z 


 
$$

 -

Z”
$ 

   
$
^`œ ing teaching needed to provide both comfort and
c. Hemoglobin A1c
=   
  ?
safety for this patient.
5.5%

{


   
=Z>\?

š
^—

$
 



$
 

2. What assessments should be performed before the
to the nurse if a patient is prescribed propranolol nurse administers an alpha1-adrenergic antagonist
=# ?ƒ $
 

 
 


a. Apical pulse 94 beats/min would warrant not administering the medication


b. BP 157/88 mm Hg and notifying the prescriber?

ˆ 
 
d. Wheezing

Copyright © 2016, Elsevier Inc. All Rights Reserved.


32 CHAPTER 19 Q
#%
  
 

3. The patient’s BP is not controlled by his pre- absolute contraindication for the use of propranolol. The
scribed alpha1 antagonist. The prescriber has prescriber can appeal to the insurance company and jus-
added a loop diuretic and a beta1 blocker. The tify the use of the more expensive decision. The prescriber
 
 
’

#
 

 

$$- 

$
 

 

  
 
$

 
$


 ƒ
> ‚
#

just have a higher dose of one medication?” How 5. Describe and explain the adverse effects that this
should the nurse explain the rationale for this drug patient might experience relating to his diabetes
regimen? diagnosis and propranolol.

4. Why is it very important to address ejaculation 6. The insurance appeal is approved, and the patient
problems and other adverse effects with this pa- receives metoprolol, a beta1-selective adrenergic
tient?   
!
 
  
’#‚
 
#



 

 
 
#
 
 
$"

 
$
 
  


”
#

 
 
 

#
“
 


Case Study 2 the nurse do and why?


A 45-year-old man is being treated for hypertension.
{ 

$
 
 
 


tried without success. He also has been taking glargine


7. The nurse discovers that this patient has a history
 
=‰  ?
^^




 
 

of poor adherence to medication regimens. Why


=+‰ ?



 



 
$

are beta-adrenergic blockers poor choices for this


diabetes. His prescriber orders metoprolol 50 mg once a
patient?
 


 




 
 


  

   


 

  

covers only the less expensive beta blocker, propranolol,


unless the patient experiences adverse effects or has an

19
Indirect-Acting Antiadrenergic Agents

STUDY QUESTIONS 3. The effect of centrally acting alpha2 ago-


nists is like that of ________________
Completion ________________ ________________
________________ ________________.

]
#% 
  
 ‚

$$

is to reduce activation of ________________ 4. The most common adverse effects of cloni-


________________ receptors. dine are __________________________,
___________________________, and
2. ________________ is the primary use for cen- ___________________________.
trally acting alpha2 agonists.
5. The principal mechanism of blood pres-
sure reduction with methyldopa is
__________________________.

Copyright © 2016, Elsevier Inc. All Rights Reserved.


CHAPTER 19 Q
#%
  
  33

CRITICAL THINKING, PRIORITIZATION, 11. A patient is receiving methyldopa. Which labora-


tory tests should be monitored throughout ther-
AND DELEGATION QUESTIONS
ƒ
={

 
?
a. Bilirubin and Coombs’ test
6. The nurse is teaching a 56-year-old truck driver
b. Hematocrit and hemoglobin

 

#


 


c. Potassium and sodium


explain that the patient should take the medication

{$%

 

]
in what way?
e. Troponin T and electrocardiogram
a. As two doses 12 hours apart
b. With a larger dose at bedtime
12. A patient is receiving methyldopa 250 mg twice
c. With food
a day. Vital signs are BP 170/90 mm Hg, P 92,
d. On an empty stomach
and R 20. The nurse is reviewing new laboratory
 
  


‰!
†
  

6
~
#









 '‰
 
`<
 '‰
–ˆ+
^`
 '‰

report which laboratory test result?


> ‚
 
 

]‹
Z"'‰



‰!

#ˆ'‰
 
‹<
Z"'‰
!
 


 ƒ

> 
]‹
 '‰
a. Administer the medication and continue to as-

>\
‡``
#ˆ'‰
sess the patient.
d. WBC 10,500/mm3
b. Administer the medication and notify the pre-
scriber of the vital signs and laboratory results.
8. The nurse is teaching a patient who has been
c. Hold the medication and notify the prescriber
prescribed clonidine for hypertension. Which
of the vital signs and laboratory results.
statement suggests a need for additional teaching?
d. Hold the medication and page the prescriber

’Z$$

 



$

{!!
#
 


$
 
 “

’#


 


$
#


13. A patient was started on reserpine 0.5 mg once a


weird dreams.”
day 4 days ago for hypertension unresponsive to

’#

$

$

 
 


other antihypertensive agents. The nurse assesses


 
#

 

 “
the patient’s BP, which is 155/72 mm Hg. The

’# 
 





 
-
patient is discouraged. What is the nurse’s best
crease the risk of constipation.”
response?

’
 
 



6 9. Depletion of the neurotransmitters norepinephrine


release, which can elevate BP temporarily.”
and serotonin by reserpine makes it a priority for
b. “The medication must be taken on an empty
the nurse to assess the patient’s
stomach.”
a. blood count.

’#
 
]%^

$

-
b. bowel sounds.
tion to occur.”
c. mental status.
d. “Reserpine primarily affects systolic blood
d. stool for blood.
pressure.”
š
]`
!
 

 
$

 




reserpine for hypertension. The patient is passive, DOSE CALCULATION QUESTIONS





 




$

to engage in conversation. The nurse is concerned 14. Reserpine, 500 mcg once a day, is prescribed.
that the patient is experiencing a drug-induced Available are 0.25-mg tablets. How many tablets
depression. The nurse should share her concern should be administered?
with the
a. patient.
b. patient and the prescriber.
]
€ 
$
 
$ 
]`
 '‰

c. prescriber and the patient’s family. is prepared in 5% dextrose in water. The 100-
d. patient, the prescriber, and the patient’s family. ‰




$ 

†`
 
 
 

should the nurse program into the intravenous



$


   

‰'ƒ

Copyright © 2016, Elsevier Inc. All Rights Reserved.


34 CHAPTER 20 Q
#

> 
+
{ 
|   

CASE STUDY 2. What should the nurse teach about drug-herb/


supplement interactions and drug-food interac-
A 68-year-old woman who plays tennis regularly has been tions?
prescribed guanfacine 10 mg once a day for hypertension
not controlled by other agents. The prescriber asks the of-

 


 
 

 

1. What should the nurse teach about side effects?


3. What serious reactions can occur if this drug is
stopped abruptly?

20
Introduction to Central Nervous System Pharmacology

CRITICAL THINKING, PRIORITIZATION, 4. A patient who has been taking an opiate analge-
sic for chronic pain due to terminal cancer needs
AND DELEGATION QUESTIONS
higher doses to produce the same pain relief as
when originally prescribed. This may be an ex-
1. The evidence supporting which of the following
ample of what?

  


>+{



a. Addiction
 ƒ
={

 
?
b. Physical dependence
a. Dopamine
c. Tolerance
b. Adenosine
d. Withdrawal syndrome

\  

{
CASE STUDY
2. Which of the following neurotransmitters are
 



>+{

|+{ƒ
={

 
A patient has recently been diagnosed with major depres-
? 
€ 

” 

  
 

>+{


\– neurotransmission. The patient verbalizes concerns to the
b. Norepinephrine  
  

 

’
 “
{


c. Acetylcholine also experiencing some daytime sedation.


d. Oxytocin
1. How should the nurse respond?
3. Which are characteristics of drugs that are able to


%  
ƒ
={

 
-
?

‰%  
b. Highly ionized
c. Move via transport systems
d. Protein-bound
e. Water-soluble

Copyright © 2016, Elsevier Inc. All Rights Reserved.


CHAPTER 21 Q Drugs for Parkinson’s Disease 35

2. The patient asks why drug companies have not 3. There are many psychotherapeutic drugs that are
been able to develop new psychotherapeutic drugs effective in treating patients’ symptoms. What fac-
that do not have adverse effects. What would be tors decrease patient adherence with these medica-
the basis of the nurse’s response? tions?

21
Drugs for Parkinson’s Disease


<
*

 
=Z
¢  ?

 -
STUDY QUESTIONS 
=@?


|&
 ƒ

–

 
$
\–

Completion
release
&
     b. Prevent dopamine breakdown

–

$$
$
\–

 
 
1. ________________ ________________ activate 
# 
 

dopamine receptors.
9. A PD patient who is receiving levodopa displays
2. _________________________________ enhances slow, involuntary, writhing movements of the
effect of dopamine by blocking degradation. extremities. The nurse holds the medications and


  
 
 


3. ________________ ________________ block the nurse use when documenting this movement?
muscarinic receptors in the striatum. a. Ballismus
b. Choreoathetosis
4. ________________ ________________ c. Fasciculation
________________ ________________ prevent d. Tremor
dopamine breakdown.
6
]`




$
 

$


5. ________________ promotes dopamine synthesis. nurse is preparing to administer levodopa?


a. Ataxia
6. ________________ promotes dopamine release b. Dark-colored urine
and may block dopamine reuptake. c. Dizziness with position changes
d. Tics

CRITICAL THINKING, PRIORITIZATION, 11. Dietary teaching that may help the “on-off” phe-
AND DELEGATION QUESTIONS nomenon of PD includes avoiding which foods?
a. Foods with a high glycemic index
 7. What is a realistic outcome for a patient receiving b. High-fat meals

 
$
|  ‚
  
=|&?ƒ c. High-protein foods
a. Absence of tremor d. Processed foods
b. A normal gait

#


$

$
 
12. Which drug would not be a concern if prescribed
living for the psychological effects of PD and the adverse
d. Reversal of neurodegeneration effects of treatment with levodopa/carbidopa?

> @
=+%> @?

>@ 
=>@ ?

* 
=* ?

! 
=|  ?

Copyright © 2016, Elsevier Inc. All Rights Reserved.


36 CHAPTER 21 Q Drugs for Parkinson’s Disease

6 13. Which is the most appropriate nursing action when  ]<


#


$
  


 


the nurse notes an asymmetric, irregular, darkly   


$







pigmented mole on the back of a patient who is assessing a patient who is prescribed bromocrip-
receiving levodopa for PD? 
=| ?ƒ
a. Notify the prescriber of the mole characteris- a. Abnormal movements
tics. b. Confusion
b. Continue to assess the patient. c. Red, swollen, hot hands
c. Tell the patient that he should consult a derma- d. Heart murmur
tologist.

{
 

   ]‡

 



 
=!  ?

complains of nausea and abdominal pain. His

]‹
{

| 

 %  
urine is dark amber. Which laboratory test results
combinations prescribed. What is the effect of the 

 
  


 



$
  


 ƒ
={

these symptoms to the prescriber?
 
? 
‰!
]~
  
 '‰Œ
{!
]‡
 -

#



 
$$
  
 -  
 '‰
bidopa more readily crosses the blood-brain 
–ˆ+
^^
 '‰Œ
 
]]
 '‰
barrier than levodopa. 
*
]^
'‰Œ
*
†‡œ

#

$

 


 
d. Na+
]‹^
Z"'‰Œ
+
‹<
Z"'‰

  
 
$$ 

#
 
 
$$


  

 20. The nurse notes an orange color to the urine of a
vomiting. PD patient who is scheduled to receive a dose of

#
 

 
$
 
$$
$
-   
=> ?
 


 


mal movements and psychiatric symptoms. in this situation?



#
 

 
$


 - a. Assess for symptoms of liver failure.
mine in the intestines and in tissue outside the b. Consult with the prescriber.
>+{ c. Continue nursing care, including administration
of the medication.

]
#

 
$

 

 

 
d. Hold the medication and contact the prescriber


  
 
=€ ?


immediately.
use of which OTC medication for heartburn?

+@ 
=?
^]

 = ?



  



… 
=|?  

  
 
=Z?ƒ
={


>
=! ? 
 
?

 
=¢  ? 
– 
={ ?

…
=|@ ?
 ]—
#







  

*
=& ?
 



 




€
=&?
="?ƒ 
{ 
=¢$?

‰!
^<
  
 '‰

–ˆ+
^^
 '‰

\…

‰' DOSE CALCULATION QUESTIONS

*
]]<
'‰

^^
| 
=€ ?
]




-

]~
!
 
  

=€?
$

scribed. The hospital pharmacy stocks pramipex-
 

|&
 

=?
]

ole 0.25 mg. How many tablets should the nurse
  

 @ 
=! ?
†``
administer?
mg, up to three doses per day as needed. Which
situation warrants the nurse administering this
medication?
^†
} 
   
 
=¢  ?
^


a. A sleep attack once a day is prescribed. Available is selegiline


b. An “off” episode =¢  ?
]^
 
*
 
 



c. Nausea nurse administer?


d. Psychotic symptoms

Copyright © 2016, Elsevier Inc. All Rights Reserved.


CHAPTER 22 Q Alzheimer’s Disease 37

CASE STUDY 4. The patient’s symptoms progress, and his medica-


tion regimen is changed to carbidopa/levodopa
A 54-year-old male patient who works as a welder has ={?
!
 



  

recently been diagnosed with PD. He is married, and has drug has been prescribed. What information can
two adult children and one college-age child. the nurse provide?

1. What assessments does the nurse need to make


to aid the prescriber in determining the needed
therapy for this patient? 5. The patient is admitted to a medical unit for a 10-
 
’
 “
  
$
  
 

$$ 
#

 
 

 

 

severe return of symptoms, including rigidity and


2. The patient is prescribed selegiline. Why is it im- postural instability. What are probable nursing is-
portant to assess the patient’s mood and teach the sues, and what interventions will the nurse employ
patient to use the same pharmacy for all prescrip- to address these problems?
tions?

3. What teaching should the nurse provide about the


possible adverse effects of selegiline?

22
Alzheimer’s Disease

STUDY QUESTIONS
<



= Z‹?


$

AD.
True or False 9. ___ Research suggests that many AD patients
experience more intense symptoms on aris-
   
  T   F ing in the morning.
   10. ___ Research suggests cholinesterase inhibitors
enhance transmission by central cholinergic

]



 
 


  
$
neurons.
@‚
  
=&? 11. ___ The American College of Physicians recom-
2. ___ An early symptom of AD is loss of appetite. mends trying a cholinesterase inhibitor in all
3. ___ Current drug therapy for AD is not highly patients with mild to moderate AD.
effective for relieving symptoms. 12. ___ The neuronal damage occurring with AD is

‹

Z$$
$

$
&

 %   irreversible.
5. ___ High levels of the neurotransmitter acetyl- 13. ___ Drugs can be helpful in treating secondary
choline are found in patients with AD. effects of AD such as depression and incon-
6. ___ Neuritic plaques are found in the hippocam- tinence.
pus and cerebral cortex of patients with AD.
7. ___ Production of an abnormal form of a protein
= ?
 

  
  
$
&

Copyright © 2016, Elsevier Inc. All Rights Reserved.


38 CHAPTER 22 Q Alzheimer’s Disease

CRITICAL THINKING, PRIORITIZATION,  19. Which result would warrant withholding adminis-
AND DELEGATION QUESTIONS  
$
 
=+  ?

$


prescriber?
 14. High levels of homocysteine are associated with 
‰!
]]^
  
 '‰
an increased risk of AD and other disorders. The 
–ˆ+
]<
 '‰
nurse can teach patients to lower their homocyste- 
\…
‹
‰'
ine levels by eating a diet high in vitamins B6 and 
| 
‹~
Z"'‰
folate found in what foods?
a. Citrus fruits and meat
^`
 



$
 
$
 
=  ?

b. Fruits and vegetables 


 @ 
=¢ ?
$
&
  ƒ
c. Dairy products and eggs 

$$
$



\
 $
  

 
b. Decreased neuropsychiatric symptoms

#

6 15. The nurse is caring for a patient who is receiving 
#

  
=Z?
$
&
 


 

priority relating to the most common adverse ef- 21. Research suggests that which treatment decreases
fects of the drug? 
 
$




$
&ƒ
={


Z 
" 


 
 
?
b. Monitoring liver function tests a. Omega-3 fatty acids
c. Preventing unusual bruising or bleeding 
\ 

d. Reviewing other prescribed drugs for drug c. Vitamin C
interactions d. Postmenopausal hormone therapy
e. None of the above
 16. Adverse effects of cholinesterase inhibitors come
from parasympathetic stimulation. What is a DOSE CALCULATION QUESTIONS
nursing intervention to prevent a common adverse
effect?
^^
\   
= @ ?

<

 


a. Assess for symptoms of hyperglycemia.  



  
  

  
= @ -
b. Assist with activities of daily living because of ?

<
 
*
 
  



tremors. nurse administer?



#
" 
 





dehydration from diarrhea.


d. Provide mouth moisturizers for dry mouth.
^†
€ 
=+  ?

  









=`<``?

]`



 17. The nurse should be particularly cautious when 




=]<``?
!
 

 

administering a cholinesterase inhibitor drug to to administer the 0800 dose. Available is meman-
any patient with a history of which condition? 
=+  ?
^
 '‰
*

 

={

 
? =+  ?


 
 



a. Asthma should the nurse measure this dose?



>
 
 
  
=>}|&?

* 
$ 
=*…?
d. Hypertension
e. Peptic ulcer disease CASE STUDY
18. Which treatment for dyspepsia could potentially A 77-year-old man who has a history of AD, diabetes
 

 

$
 
=+ - 
=&€?
 

  
 


 ?

  
ƒ been admitted with a hip fracture that occurred when

> 
   
=!ˆ€{? wandering around the house at night. He is scheduled for

‰   @
=| ? 



 
 
$

$ 



 
=¢  ? next morning. On admission the nurse discovers that the

{
   
= 
 ? patient’s spouse had been administering ginkgo biloba
for the past 8 months in hope that it would improve his
memory.

1. Why is it important for the nurse to report this to


the orthopedic surgeon?

Copyright © 2016, Elsevier Inc. All Rights Reserved.


CHAPTER 23 Q
&
$
€
{  39

2. The patient’s spouse is concerned that the patient’s


‹
!
 ‚
‰!

]~<
  
 '‰


confusion, memory loss, and wandering have not 


 

†^
 '‰
!
  
 

improved much and that he seems to have gotten the patient’s spouse and the patient to discontinue
worse since the weather got warm. The patient’s 
  
= @ ?

 

}!>

spouse has provided a list of all of the patient’s    



 
 
  
=Z-
prescribed medications, which include hydrochlo- ?
!
 ‚
 


 



 @
=*&#ˆ#‰?
$
=\- prescriber changed the medication to one in the
 ?

  
= @ ?
– 


same class if the galantamine is not working well
patient’s history, what class of over-the-counter and is hurting the patient’s liver.
=}!>?
 


 





the prescriber because it could be contributing to


the patient’s sudden decline in functioning?
5. The nurse should teach the patient’s spouse what
information regarding adverse effects of rivastig-

=Z?ƒ

†
 
 
=

  

  ?


 
$
$
 -


  
= @ ?

 

6. The patient’s spouse heard from friends of some-
would warrant consulting the prescriber? thing sold in the health food store to cure AD.
The friend said that the store provided results of
research that states that the supplement is effec-
tive in improving memory in AD patients. The
patient’s spouse is on a limited budget and asks
the opinion of the nurse. How should the nurse
respond?

23
Drugs for Multiple Sclerosis

STUDY QUESTIONS


>

 
$
€{
 
 

symptoms during a relapse.


True or False
—

>

 
$
€{
 


mobility and safety.


   
  T   F
~

>

 
$
€{
 


    





$
€{

<

>

 
$
€{
 



1. ___ Research suggests initiation of the disease disease process.


process is linked to genetic, environmental,
‡


 

   %
€{

and microbial factors. should receive an immunomodulator.


2. ___ Current drug therapy repairs the myelin
]`

#$
 
  

 

sheath on peripheral nerves. are more effective than natalizumab.

†

>

 
$
€{
 


11. ___ Current drug therapy is most effective if the
disease.  

 
  
€{

‹

>

 
$
€{
 
 

the formation of lesions.

Copyright © 2016, Elsevier Inc. All Rights Reserved.


40 CHAPTER 23 Q
&
$
€
{ 

CRITICAL THINKING, PRIORITIZATION,  17. A patient is scheduled to receive mitoxantrone


AND DELEGATION QUESTIONS =+ ?
 



 




 

 

  ƒ
={

12. A 26-year-old patient seeks medical care for 


 
?
blurred vision and severe muscle weakness. Which a. Amenorrhea
information from her history suggests possible risk b. Blue-green colored urine
$ 
$

 
=€{?ƒ
={

 

‰ŠZ…
—`œ
? 
>\
†`
  
 '‰

{


 e. Neutrophils 1000 cells/mm3

{

 
 
 

{

 
 



-  ]<

 



 
=+ ?

lege. 



@ = ?ƒ
={


d. Her father has had a cerebrovascular accident  


?
=£>Š¤
? a. Hepatitis B vaccine
e. Her maternal and paternal grandparents were 
#@
 
Norwegian immigrants. 
€  
 

 
=€€?
d. Tetanus, diphtheria toxoids, and acellular per-
13. Which of the following best describes treatment of  
=! ?
   %
€{

  ƒ e. Varicella virus vaccine
a. They should always be administered orally.
b. They should be used only if attacks last more than  ]‡

 

€{

   
 

1 week. tea-colored urine, clay-colored stools, and right



!







"  
=ˆ˜?
 
‰!
 


totally ineffective or toxic. ^``


  
 '‰
!
 

 

d. They should only be used during periods of with the prescriber if the patient was prescribed
relapse. 
 ƒ
={

 
?
a. Betaseron
 14. The nurse is administering high-dose intravenous b. Copaxone
=#Š?
 


 


- c. Mitoxantrone



  
$
€{


- d. Dimethyl fumerate
dition, if also present, could be adversely affected by
this treatment?  20. Which symptom suggests a complication of
a. Asthma  % 
  


€{
 ƒ

& 

=&€? a. Dysuria
c. Hypertension 
&$
 
 
d. Rheumatoid arthritis c. Nocturia

ˆ 
 
6
]

 

€{


  @

-
monia is receiving the immunomodulator inter- 21. Which teaching is most effective in preventing
$
 %]
=?

†`




#
constipation when bulk-forming products are used
would be of greatest priority for the nurse to report for fecal incontinence?
which laboratory test result to the prescriber? a. Administer the product after meals.

‰!
‹
  
 '‰ b. Administer the product before meals.
b. Hemoglobin A1c 5.2% c. Administer the product in the morning and a
c. RBC 4.9 million cells per microliter stimulant laxative at night.
d. WBC 12,000/mm3 
 




 
^‹`

$

 ]—
!
 

 
 
=+ - 
?


 


  



 

$
€{
!
 


hold the medication and contact the prescriber if the


patient exhibits which symptoms?
a. Diaphoresis and low blood sugar
b. Dizziness and orthostatic hypotension
c. Headache and elevated blood pressure
d. Weight gain of 3 lb in 24 hours and shortness of
breath

Copyright © 2016, Elsevier Inc. All Rights Reserved.


CHAPTER 23 Q
&
$
€
{  41

^^

€{
 
 
 



- 3. What are some possible nursing diagnoses for this
pressed and fatigued despite being prescribed patient throughout the disease process?

=|@ ?
$
†
 
 



the nurse would be most appropriate at this point?



Z 

 

 

 
$

caffeine.
4. When the nurse is administering medications, the

Z 
 

$$
$
 
 

patient refuses the drug, stating, “What’s the use?


many months to occur.
€

 
€{



‚

{

c. Plan for the patient to discuss this problem with




]‡‡`
{


~
 
“
 

the prescriber.
should be the basis of the nurse’s response?

{ 
]•]

  

DOSE CALCULATION QUESTIONS




!
 

€{

 


-
23. A patient is prescribed an initial subcutaneous  
$
 %]
=?

†`


 
$
<<

$
$
]
= $?
$

once a week. What should the nurse include in
 
^

$
 
!




teaching regarding this drug?
^^
 '`
‰
*




 -
tient administer with each dose?

6. The patient and the prescriber have tried treat-

^‹
!
 
 
  @
=! ?
†``

ment with the immunomodulators without


 ']


]``

$
 


$

success. They have agreed to try the immunosup-



$ 
!




 


 
 
=+ ?
 
 -
1 hour. How much solution should infuse during
ing should the nurse provide this patient?
each 10 minutes of the hour?

CASE STUDY 7. What steps does the nurse need to take before
 

 
=+ ?


A 36-year-old college professor has been diagnosed with #Š


$ ƒ
   %
€{
$


$

 


1. What assessments are important for the nurse to


include when caring for this patient?
8. The patient is experiencing urinary incontinence.
What pharmacologic and nonpharmacologic mea-
sures can be employed to prevent this problem?

^
*

 
  
 
 

are the primary factors of the pathophysiologic



$
€{
 
 



 

abate but the disease is still present?

Copyright © 2016, Elsevier Inc. All Rights Reserved.


42 CHAPTER 24 Q
&
$
Z 

24
Drugs for Epilepsy

STUDY QUESTIONS 
‰%\  

=‰\{?
i. Myoclonic seizure
Matching ”
{ 
 @
@

{@

      
{
  
@
1. ___ Abnormal motor phenomena associated with 
{ 

epilepsy 
!%
=  
 ?
@
2. ___ Any seizure activity that lasts longer than 30
minutes CRITICAL THINKING, PRIORITIZATION,
3. ___ Brief loss of consciousness with or without
mild, symmetric motor activity
AND DELEGATION QUESTIONS

‹

\ 

 

 

-
15. What is a mechanism of action of an antiepileptic
sciousness, motor, sensory, autonomic, and

=Z&?ƒ
={

 
?
psychoillusionary symptoms that occur with
a. Bind to sodium channels when they are in the
epilepsy
inactive state



\
$
 

 
 

b. Block the actions of glutamate at NMDA and


seizure activity
AMPA receptors
6. ___ Muscle rigidity followed by muscle jerks

# 

$
 


 


  
 
¥
‡`


$


prevent transmitter release





 

$

 

# 


$

  

7. ___ Partial seizure that transitions to generalized


\–
seizure activity
8. ___ Repetitive movements that lack purpose
16. Developmentally, what would be the most likely
such as lip smacking
reason an 18-year-old male patient may refuse to

‡

{@




 

seek treatment for possible seizure activity?


with an elevated temperature
a. Concern that he may lose his driver’s license if

]`

{
$
$
 

 

diagnosed with epilepsy


delay and a mixture of partial and general-
b. Drugs are not likely to be helpful
ized seizures
c. Fear of injury when performing hazardous

]]

{

$
 
 generalized
activities
muscle contraction
d. Refusal to maintain a seizure frequency chart

]^

{

$
 


]†

{



$

 


17. What should be included in teaching when a


stimulated, last a minute or less, and patient
 



 
 

=&-
does not become unconscious at any point
 ?

   @
=! ?


 

during the seizure


attempting drug withdrawal?
14. ___ Trance-like state for 45-90 seconds followed
a. Dose of both drugs should be gradually de-
by automatism
creased.

 
@
=
 ? b. Dose of one drug should be gradually de-
b. Atonic seizure creased while maintaining the usual dose of the
c. Automatism second drug.
d. Complex partial seizure c. Carbamazepine should be stopped and the
e. Convulsions valproic acid continued for at least a week.
f. Febrile seizure d. Valproic acid should be stopped and the carba-
g. Focus mazepine continued for at least a week.

Copyright © 2016, Elsevier Inc. All Rights Reserved.


CHAPTER 24 Q
&
$
Z  43

18. Research suggests that there is a statistically sig- 23. Parents have received instructions regarding
 
 
 
$
 
$


  
$




 
$



 

  

Z&ƒ
={

 
=& 
  ?




 -
? ment made by the parents suggests a need for

>   @
=! ? further teaching?

& 
=& ? a. “We must be sure to shake the medication thor-

‰  
=‰  ? oughly before measuring the dose.”

|
=& ? b. “We should give him a teaspoonful of the drug

!  
=!  ? at each dose.”
c. “We should give him the medication with break-

]‡

 


 

Z&
fast and with a snack before bedtime.”
 
  
Z& ƒ 
’#

 
 

 


 

a. Birth defects are more common when a preg- we have been instructed.”
 
 
 

Z& 

+
$


Z&

 % 24. Which problem is most likely to occur with car-
metabolizing enzymes.  @
=! ?
 





  
  

Z&


$$- =& ?ƒ
tive. 
{ 
d. There is less clinical experience with newer b. Reduced osmolarity of blood
Z&  
{

  
$

 


arms
6 20. Which action is of greatest priority to address 
{

‚
=& ‚ ?
 
 
ƒ
a. Check plasma levels of the drug as prescribed. 6 25. Which nursing action would be of greatest priority

# 
 
$
 
  when the patient who is prescribed carbamazepine
c. Have regular dental checkups. =! ?





=+>?


ˆ 

 
$
$

 of 500/mm3?
a. Handwashing
21. Which laboratory test result would be of most 
‰
 
concern to the nurse when a 27-year-old female c. No fresh fruit or vegetables
patient admitted after injuries sustained in an MVA d. Protective isolation

  

=& ?

=! -
?
 @ 
=Š ?

 $ 
=> - 6 26. What is the priority nursing action when a patient
?ƒ 

  
   @
=! ?
-

‰!
†
#ˆ'‰ plains of vertigo?

>\
†^~`
#ˆ'‰ 
Z 
 
$

#+
^] 
| 

  
{!!

|

 '‰ c. Reviewing most recent lab results of CBC,
–ˆ+


6 22. When caring for a patient who is receiving phenytoin d. Withholding the medication
=& ?

 






report to the prescriber?  27. The nurse is teaching a patient about adverse ef-
a. Morbilliform-like rash $
 


   @
=! ?

b. Continuous back-and-forth movements of the Which instruction should not be included in this
eyes teaching?

Z 

$

  a. Avoid situations where you are likely to be

# " 
 
$
$


 - exposed to people with contagious infections.
cium and vitamin D b. Plan your day to allow for rest periods.

{
 


$


 

vision.

ˆ 


 @
  
$

  
 @

because you may bleed easily.

Copyright © 2016, Elsevier Inc. All Rights Reserved.


44 CHAPTER 24 Q
&
$
Z 

6 28. To prevent a life-threatening skin reaction, it 34. A patient with a history of hypertension and
would be a priority to review which laboratory test complex partial seizures has been prescribed
 
$
 
   @
=! ?
  @
=! ?
]``



 


to a patient of Asian descent?  @


=€@?
#

 


{!
for the nurse to assess for which condition that

–ˆ+   
  
  ƒ
={

 

c. Creatinine ?

*‰%–¦]`^

&

  

 

b. Hyperactive bowel sounds and tall tented T
29. The nurse instructs patients who have just received  

   
=Z>\?

 
$
   @
=! ?

c. Muscle spasms and weakness
avoid consuming grapefruit juice for which time d. Headache and nausea
period?
a. Four hours after taking the medication  †
#$

 



  
=‰ -
b. No more than twice a week when taking the  ?


 
 


 
ƒ
medication a. Assess vital signs.
c. Not at all when taking the medication b. Continue nursing care.
d. Two hours before taking the medication 
# 
 


  
d. Review laboratory test results.
6
†`
#


$
  

$

 


report which adverse effect of valproic acid? 6


†—
| 
=‰ ?
 
  
  
 

a. Abdominal pain should the nurse teach the patient to do if he or she


b. Belching notes symptoms of angioedema?
c. Hair loss 
\


$
 


”



d. Weight gain seizure.



Z 

 
31. A woman who is of childbearing age must take 
{
 
 
 
valproic acid to control seizures. Which statement, d. Take the drug with food.
if made by the patient, suggests a need for further
teaching?
†~
 



  
= ?ƒ
a. “Folic acid supplements are a good idea in case 
#
 
  
 
  
#
 

  “ 
#
 
 

$$
$
 $ 


’
#


 


$
#


 
=> ?
#

  “ 
#
  
$


 
$



’#


$

$

 
 
- channels.

 
#

 
 
“ 
#


 

$

 


’!

$
#


 

 
$
$
gastrointestinal adverse effects.
birth control.”
 38. A ketogenic diet is especially dangerous when a
32. Which would be the best goal for a child who is patient with epilepsy is prescribed which drug?
prescribed ethosuximide for absence seizures? 
‰  
a. Control of seizures allows for normal participa- b. Pregabalin
tion in activities c. Tiagabine
b. No adverse effects from drug d. Topiramate
c. Plasma levels within therapeutic range

{ 
$



   39. Which laboratory test result would be of most
concern to the nurse when a patient is prescribed
33. A patient is admitted with phenobarbital overdose.   
=!  ?ƒ
Which prescribed action would be of greatest 
> 
]
 '‰
priority? b. HCO3
^`
Z"'‰
a. Assess respiratory status. c. Hct 32%

# 

  d. WBC 4600/mm3
c. Obtain drug level.
d. Perform seizure precautions.

Copyright © 2016, Elsevier Inc. All Rights Reserved.


CHAPTER 24 Q
&
$
Z  45

40. Which symptom suggests a critical adverse effect 1. What nursing precautions are needed when admin-


 

  
  
=!  ?
istering phenytoin and cimetidine intravenously?



   @
=! ?
$
%
clonic seizures?
a. Ataxia

&$
 

^
|

  
$

$ 


-

{

 
 
$
`
 '‰
*



 

d. Weight loss
take when administering the 800-mg loading dose?
 41. What is the most appropriate action if the nurse

 
 
=\ ?






Z&

  ƒ
a. Administer the medication. 3. What symptoms should the nurse be alert for be-
b. Administer the medication and ask the patient cause of the possible interaction of phenytoin and
why he or she taking the medication. cimetidine?
c. Withhold the medication.
d. Withhold the medication and consult the pre-
scriber.

‹
!

 





 '‰

42. The nurse would consult the prescriber before


Why is it critical for the nurse to closely monitor
 
@ 
=¢  ?
$

 

plasma drug levels of phenytoin?


notes that the patient had a known anaphylactic
reaction to which drug?

   
=| ?

# 
 
=+‰ ?

\@
=\? 5. One week later, the patient goes into labor. Based

€$
=\ ? on initiation of phenytoin near the end of the preg-
nancy, what fetal effects are most likely to occur?
DOSE CALCULATION QUESTIONS
43. The 5-year-old patient who weighs 35 lb is newly
prescribed phenytoin 20 mg every 12 hours. The 6. What precautions might be taken to protect the fe-
suggested initial dose is 2.5 mg/kg/day in divided tus during drug therapy, during delivery, and after
  
#


$

$$
 
$

delivery?
child?

44. Phenytoin mg is available as a suspension of


]^
 '‰
*




 - 7. Two weeks later, the patient is stabilized and has
tered if the prescribed dose is 20 mg twice a day? been transferred to the neurologic progressive unit.
}
 

 



   

rash on the patient’s trunk. What should the nurse


do and why?
CASE STUDY
A 26-year-old patient who is 8 months pregnant is admit-
ted to the neurologic intensive care unit with the diagnosis
of head injury. An automobile accident has left the patient
<
|
=& ?

 
>  -
  
}
 

 
=#Š?

$
@
=! ?

 


 ‚

D51§2
+{

`
‰'

 


 


antiseizure drug. The patient is concerned about
on the unit, the patient has several tonic-clonic seizures. taking seizure medication because she wants to
{ 






 

 -  


{


 



 @ 
=Š ?



 
!
  

  
 
$$


$ 
#




$

@




 
 
$
referring concerns to the prescriber, what informa-

=& ?
<``

$

]``


—



 


@ 
Z& 

 





 



and pregnancy?
300 mg twice a day.

Copyright © 2016, Elsevier Inc. All Rights Reserved.


46 CHAPTER 25 Q
&
$
€ 
{ 

{ 

25
Drugs for Muscle Spasm and Spasticity

STUDY QUESTIONS
]^

 



  

 -
 
  
 
$$


 


True or False   


  
={ ?ƒ
a. Nausea and vomiting
   
  T   F b. Right upper quadrant pain
   
{ 

 
d. Tea-colored urine
1. ___ Most drugs used to treat spasticity do not
relieve acute muscle spasm. 13. The nurse should teach a patient who is prescribed
2. ___ Muscle relaxant drugs shorten time of reha- a centrally acting muscle relaxant to avoid which
bilitation from traumatic injuries that cause drug?
muscle spasm. 
& 
= 
– ?
3. ___ The therapeutic effects of muscle relaxants 
# $
= 
€?
are similar to the effect of taking aspirin. c. Multiple vitamins with iron

‹

& @ 
=Š ?


 
$$

|

= 
€ ‰ ?
muscle relaxant.
5. ___ Most centrally acting muscle relaxants are 14. A resident is writing orders for hospitalized
not effective in treating spasticity associated patients. The nurse would consult the prescriber
with cerebral palsy. $
$
=‰ ?

  



6. ___ The nurse should assess all patients who are muscle spasms for a patient with which condition?
prescribed a centrally acting muscle relaxant a. Cerebral palsy
for feelings associated with depression. b. Cerebrovascular accident
7. ___ Dark brown or black urine should be re- c. Multiple sclerosis
ported to the prescriber of methocarbamol 
{ 

”
= ?

<

#

 
$

 




–ˆ+

 


 

  
DOSE CALCULATION QUESTIONS
tizanidine, metaxalone, or dantrolene.

‡

–

  
\–
-
]
& @ 
=Š ?
^

 

<



duces spasticity. prescribed for muscle spasms. The drug is avail-


10. ___ Baclofen decreases spasticity. 


 '‰
*



 

administer at each dose?

CRITICAL THINKING, PRIORITIZATION,


AND DELEGATION QUESTIONS
]—
& 
^`


 

$ 

-
scribed. The recommended dose to treat malignant
6 11. The nurse knows that the priority nursing inter- hyperthermia is initially 2 mg/kg. The patient
ventions and teaching for a patient receiving a  
]<
 
#


$
 ƒ
centrally acting muscle relaxant are related to
a. elimination.
b. emotional support.
c. nutrition.
d. safety.

Copyright © 2016, Elsevier Inc. All Rights Reserved.


CHAPTER 25 Q
&
$
€ 
{ 

{  47

CASE STUDIES Case Study 2


A patient with multiple sclerosis is admitted with pneu-
Case Study 1  
{


 @ 
=…?


A 19-year-old man is admitted to the neurologic intensive relieve muscle spasms.


care unit after a motorcycle accident. He is paraplegic
with severe lower extremity muscle spasms. He is receiv- 5. What are common anticholinergic adverse effects
ing intrathecal baclofen. A pump is being used to infuse associated with this drug and nursing interventions
the drug because abrupt discontinuation can cause rhabdo- relating to these effects?
myolysis and multiple organ system failure.

1. What is rhabdomyolysis, what organ is particu-


larly sensitive to its effects, and what assessment


  
 


 ƒ

6. What nonpharmacologic interventions and teach-


ing could the nurse provide to help relieve the
patient’s discomfort and prevent complications?
2. The patient is transferred to a rehabilitation unit.
Baclofen 20 mg is prescribed 4 times a day. What
assessments should be included in the nurse’s plan
of care for this patient relating to drug therapy?

Case Study 3
A patient returns to the postanesthesia care unit after
general anesthesia. The vital signs have increased from
3. Developmentally, this patient is at risk for non- normal to a temperature of 103.6° F, pulse 116 beats/min,
adherence to therapy. Why should the patient respirations 22 breaths/min, and BP 145/99 mm Hg. The
be discouraged from abrupt discontinuation of patient is developing muscular rigidity.
baclofen?
7. Why are antipyretics not appropriate to treat this
fever?

‹
& 
=& ?





for this patient because it only needs to be taken


once a day. Why is it not a good choice for this
<
!
 

  
 
=& ?

patient? ]`


 



 

 

the patient on his or her side with the side rails up?

Copyright © 2016, Elsevier Inc. All Rights Reserved.


48 CHAPTER 26 Q
‰ 
 

26
Local Anesthetics

STUDY QUESTIONS 6
]‹

 




  


]`

AM
=]```?
$

 
 
 

True or False performed with spinal anesthetic. During the as-


sessment at 2 PM
=]‹``?

 

 


   


  T   F patient has not voided. The patient denies a need
   to void. What is the priority action by the nurse at
this time?
ˆ 

  





 

a. To administer bethanechol 5 mg subcutane-


anesthetic
ously ordered as needed for urinary retention
b. To tell the patient that she needs to go to the
1. ___ allows for the use of less anesthetic.
bathroom to attempt to void
2. ___ causes local vasoconstriction.
c. To sit the patient upright on a bedpan to attempt
3. ___ can cause adverse effects from systemic
to void
absorption of the vasoconstrictor.
d. To perform a bladder scan
4. ___ causes local vasodilation.
5. ___ delays onset of anesthesia. 15. Topical local anesthetic should be applied in what
6. ___ delays systemic absorption.  ƒ
={

 
?

~






$$
  a. Followed by heat to increase absorption
8. ___ increases the risk of toxicity. 
ˆ 

 
$$
 

‡






$$
  
\


$

 

 
10. ___ reduces the risk of toxicity. 







$$

11. ___ requires the use of a larger dose of anes- e. Only to the affected area
thetic.
6 16. Which symptoms, if occurring after a patient has
received a local anesthetic with epinephrine, is a
CRITICAL THINKING, PRIORITIZATION, priority to report to the prescriber?
AND DELEGATION QUESTIONS a. Pulse 120 beats/min
b. BP 100/60 mm Hg
š
]^
!
 

 

 
 
c. Respirations 24/min
provider with anesthetizing and suturing multiple 
! 
†<„
>
=]``‹„
…?
wounds on the extremities of a patient who was
attacked by a dog. Which of these new develop-
]~
#

 
 
$

 

$


ments would be of most concern to the nurse? physician of allergy concerns if the patient has a
a. Numbness and tingling around the mouth 
 

 
=+  ?


b. Rash is to receive which medication?


c. Respirations 24/min a. Bupivacaine
d. Restlessness 
‰ 
c. Mepivacaine
š
]†

  
 


 
 - d. Tetracaine
thetic 10 minutes before delivering at 39 weeks of
gestation. The neonate is brought to the nursery 18. Cocaine differs from other ester-type local anes-
$








thetics in which way?
concern to the nursery nurse? 
#
 

  
  
$



a. Apical pulse 160 beats/min patient with heart failure.


b. Positive Babinski’s sign 
#
  
 
  
c. Respirations 22/min 
#



  
  


! 
†—<„
>
=‡<^„
…? 
#

>+{


 

Copyright © 2016, Elsevier Inc. All Rights Reserved.


CHAPTER 27 Q
\ 
  49

19. A patient received intravenous regional anesthetic Case Study 2


containing lidocaine without epinephrine when

 

 
#





$
A patient received lidocaine for the removal of an in-
bradycardia, hypotension, and respiratory depres- 
 
{

 

$

  @

sion from the lidocaine at which time? 



 
Z


  
 



# 
$
”
$

  often combined with lidocaine to delay systemic absorp-
b. When the surgeon makes the incision tion of the lidocaine. As a vasoconstrictor, it would have
c. When the patient is in the postanesthesia care decreased the blood seepage.
unit
d. When the patient is in her hospital room on the 2. Why was it not used in this situation?
medical-surgical unit

CASE STUDIES 3. The patient has had this procedure before and is


 
{
 @

 


Case Study 1
use ice immediately to prevent pain. What precau-
An 8-year-old child, accompanied by her parents, comes tions should the nurse provide regarding the use of
to the emergency department with a scalp laceration sus- cold?
tained when she fell off of her bicycle. The plan includes
administration of lidocaine with epinephrine to close the
wound with 6-8 interrupted sutures. The child is tearful
but cooperative.
4. The patient asks if she can use over-the-counter
topical lidocaine anesthetic on her toe when sensa-
1. What are the nursing responsibilities for this pro-
tion returns to relieve discomfort. What teaching
cedure?
should the nurse provide?

27
General Anesthetics

STUDY QUESTIONS 
&  

Z 
Matching c. Droperidol

#  
     e. Ketamine
f. Propofol
1. ___ High risk of bacterial infection 
{ 
2. ___ Hypotension can occur from vasodilation
3. ___ Can induce seizures
4. ___ Delirium and psychotic symptoms can oc- CRITICAL THINKING, PRIORITIZATION,
cur postoperatively or days or weeks after AND DELEGATION QUESTIONS
surgery



> 

 



  

<
\ 
  



$
apparatus a. sensibility to pain.

—

€ 

˜!
  b. consciousness and sensibility to pain.
7. ___ Tachycardia and hypertension can occur if c. consciousness and sensibility to pain and tem-
blood levels drop suddenly perature.
d. consciousness and sensibility to pain, tempera-
ture, and taste.

Copyright © 2016, Elsevier Inc. All Rights Reserved.


50 CHAPTER 27 Q
\ 
 

9. The primary goal of using multiple agents to š


]‹

 
 



 @ 

achieve anesthesia is to do what? =Š ?




@ @

” 

a. Decrease the pain after the procedure been admitted to the postcolonoscopy recovery
b. Make the patient unable to feel pain during the area. He informs the nurse that he needs to urinate.
procedure The nurse should initially do what?
c. Permit full anesthesia with fewer adverse ef- a. Assist the patient to the bathroom, staying with
fects the patient at all times.
d. Prevent the patient from remembering the expe- b. Provide the patient a urinal and ask him to try
rience to use it while still in bed.
c. Provide the patient with a urinal and assist him
10. Nitrous oxide has an extremely high minimum with standing.
 
 
=€>?
–  
$
 
d. #nsert a Foley catheter.
which statement is true?
a. The drug can be administered at low doses and š
]
 


 

 

  

achieve adequate anesthesia. the nurse withholding morphine from a postopera-



{  
  
 

  

tive patient?
nitrous oxide alone. a. Blood pressure 160/92 mm Hg
c. The drug can achieve adequate, safe anesthesia b. Pain 9 on a scale of 1 to 10
if a high enough dose is administered slowly. c. Respirations 8/min
d. The drug will make the patient unresponsive to d. Temperature 38.2° C
painful stimuli.
6
]—

]‹%%
 

  


11. The nurse is caring for a patient in the postanes- succinylcholine during surgery. Which immedi-
 
 

=|>ˆ?
 


  
- 
  
 




ity assessment by the nurse? priority concern?


a. Breathing a. Blood pressure 86/54 mm Hg

‰
$
   b. Pulse 68 beats/min
c. Pain c. Respirations 30/min

ˆ 
 
! 
]`†—„
…
=†‡<„
>?

12. The nurse should assess all postoperative patients 6 17. The emergency department has sent a patient to
who have received inhaled anesthesia for what the OR for emergency surgery including general
condition?  

  
!
$ 

” 

a. Diarrhea arrived, and the nurse has learned that the patient

Z 
  has been taking the calcium channel blocker am-

ˆ 
  
=+ ?
!
$ 




d. Wheezing  






|>ˆ
–  
$


 

$$
$
  

-
š
]†
!
 



 
- ine, what is the priority assessment?
list for a same-day surgery patient who has been a. Blood pressure for hypotension
 


 

=}?
!
 
b. Pulse for bradycardia
makes a comment to the nurse that he hopes they c. Respirations for hypopnea or apnea
“really knock me out” because he needs a lot of a d. Temperature for malignant hyperthermia
drug to get a good effect. On further discussion,
the patient reveals that he has been using oxyco- 18. Why is nitrous oxide widely used in surgery?

=}>?
 
$
^
 

 

a. Balanced anesthesia can be achieved with
last dose was 6 hours ago. What should the nurse nitrous oxide alone.
do? b. #

  
   
$$

 -
a. Call the OR and cancel the surgery.  
  

  
 
b. Note the medication on the patient’s chart and c. #t can produce a state of unconsciousness at
send the patient to the OR. very low doses.
c. Notify anesthesiologist and the surgeon of the d. Postoperative nausea and vomiting are uncom-
  mon.
d. Nothing, the medication should be out of the
patient’s system at this time.

Copyright © 2016, Elsevier Inc. All Rights Reserved.


CHAPTER 27 Q
\ 
  51

19. What is the purpose of administering methohexital


^
 




$

|>ˆ

in the OR? nurse for this patient relating to general anes-


a. Decrease respiratory secretions thetic?
b. Prevent muscle contraction
c. Prevent sensation of pain
d. Produce unconsciousness

20. A cataract surgery patient receives midazolam and


†
 



|>ˆ
 


fentanyl. The nurse would expect these medica- relating to this concern?




$$ ƒ
={

 

?
a. Absence of anxiety
b. Analgesia
c. Flaccid paralysis Case Study 2

{ 
The nurse is admitting a patient with a history of type

ˆ  
2 diabetes mellitus and hypertension to the same-day
surgery unit. The patient is scheduled for an inguinal
21. What is required when propofol is being used?
herniorrhaphy.

{
"

{ 


$@

 



4. What type of agents would be included if this


before use
patient is to receive balanced anesthesia?

ˆ 
$

^%^—
 

 
$
 -
tration

ˆ 

†`

$
  

22. What is the priority assessment when a patient has


5. What data are important for the nurse to collect
received propofol?
relating to scheduled general anesthesia?
a. Blood pressure
b. Breathing
c. Heart rate

# 



—
!
 



|>ˆ


” 
 

CASE STUDIES but is very drowsy. Describe nursing interventions


relating to the effects of inhalation anesthetic that
Case Study 1 



 
 
  
A 65-year-old man with a history of lung cancer has been
admitted the evening before surgery and is scheduled for
a thoracotomy at 8 AM under balanced general anesthetic.
He is 5 feet 6 inches tall and weighs 210 lb. The patient 7. The postoperative patient reports having awakened
has a productive cough and admits to tobacco use for the during surgery and being in pain, but being unable
last 40 years, smoking one pack of cigarettes per day. He to tell the surgeon. What should the nurse do?
has hypertension and takes hydrochlorothiazide, which
has been effective in keeping his blood pressure under
control.

1. Describe the ideal anesthetic for this patient.

Copyright © 2016, Elsevier Inc. All Rights Reserved.


52 CHAPTER 28 Q
}
=+ ?
    
}
   

+
> 

   

28
Opioid (Narcotic) Analgesics, Opioid Antagonists, and
Nonopioid Centrally Acting Analgesics

STUDY QUESTIONS CRITICAL THINKING, PRIORITIZATION,


AND DELEGATION QUESTIONS
True or False
   
  T   F 8. Four patients return to the nursing unit from the
       
 

=|>ˆ?

]—``


of these patients would be most likely to experi-


1. ___ An opiate is a drug that contains compounds ence respiratory depression during the 0700-1530
found in opium, while an opioid may be a shift the next day?
laboratory-created compound. a. 45-year-old female patient, postop open cho-
2. ___ The term    refers only to synthetic lecystectomy, who is reluctant to cough and
compounds that relieve pain. deep-breathe and who had the last dose of 2 mg

†

Z  
 


$



]ܠ`
are body substances with opioid properties. b. 84-year-old female patient who had an open
4. ___ The respiratory depression, physical depen- 

 
 
$

 


dence, and euphoria that occur with opioid $ 




 



|>ˆ

analgesics are related to activation of mu but received a subcutaneous dose of 5 mg of


opioid receptors. morphine at 1930



–
=–?


 - c. 5-year-old male patient who is postop appen-
pation and sedative effects than morphine. dectomy and received 0.1 mg/kg of morphine
6. ___ An agonist-antagonist blocks pain when #Š

^`‹
taken alone but improves the pain-relieving d. 63-year-old male patient post–total knee arthro-
effect of another opioid if in the blood at the plasty who received 5 mg of morphine epidural
same time. at 1125 during surgery
7. ___ The dose of oral morphine is higher than the
 
=#Š?
 
  
 
 
 9. The nurse is ambulating a postoperative patient
pass metabolizes some of the drug before it in the hall who is receiving an opioid analgesic
reaches the central nervous system. for pain. The patient complains of severe nausea.
What is the priority nursing action at this time?
a. Administer the prescribed antiemetic.
b. Assist the patient to sit down.

\

 

d. Walk the patient back to his room.

š
]`

 






 
 
#$

 
$$



to the morphine, which intervention by the nurse


might aid in reversing it?
a. Assist the patient to ambulate.

# 

 

  
 


# 

 

 


  
d. Keep the room well-lit.

Copyright © 2016, Elsevier Inc. All Rights Reserved.


CHAPTER 28 Q
}
=+ ?
    
}
   

+
> 

    53

6 11. A neonate is born to a known heroin addict. The 16. What would be most likely to cause a life-
infant is exhibiting symptoms of opioid with- threatening reaction?
drawal. Which of these nursing issues is of the a. The patient chews the pellets in a morphine/
highest priority as the nurse cares for the neonate naltrexone capsule.
in the nursery? b. The patient crushes and snorts the pellets in a
a. Altered nutrition morphine/naltrexone capsule.
b. Disturbed sleep c. The patient consumes a shot and a beer while

…
 taking morphine/naltrexone as prescribed.
d. Parenting d. The patient does not take the prescribed mor-
phine/naltrexone.
12. A cancer patient who is receiving oxycodone
=}>?
$
 
$


 
$
17. What is a reason to withhold a newly prescribed
being prescribed an antibiotic. The prescriber fentanyl transdermal system patch?
discontinues the antibiotic and prescribes diphen- a. The patient has not taken any opioids in the
 
=– ?
!
 


past 30 days.
in the plan of care assessment and interventions b. The patient is also prescribed a short-acting
for which possible effects of the combination of opioid for breakthrough pain.
oxycodone and diphenhydramine? c. The patient rates pain as 5/10.
a. Delirium d. The patient weighs more than 200 lb.
b. Fever
c. Hypotension 6
]<
!
 
 

 


Z&
$

 


ˆ 
 who is prescribed fentanyl lozenge on a stick
="?
{
 
 

$

^% %

6 13. A patient who has overdosed on oxycodone   




 



{

=}>?

 

 


Z&
took it away from him, and he is standing next
unresponsive with severely depressed respirations. to her crying. Which action would be of greatest
!
 


 
*>
=+  ?

priority?
narcotic antagonist, with an improvement in level a. Call 911.
of consciousness and respiratory rate and effort b. Assess how much drug was consumed.
within minutes. The nurse is scheduled to go to c. Contact the poison hotline.
lunch in 15 minutes. What is the nursing priority d. Rinse out the child’s mouth.
that guides information that must be communi-
cated before the nurse leaves for lunch? 19. Which is the appropriate method of administering
a. Prevention of abstinence syndrome $ 
 

=}  ?ƒ
b. The short half-life of naloxone a. Apply the patch to dry skin that is not hairy to
c. The short half-life of oxycodone ensure adhesion.
d. History of substance abuse b. Place the drug between the cheek and the lower
gum and actively suck on it.

]‹

$ 
=&  ?
 


  

| 





$


-
for a patient after multiple musculoskeletal in- rectly under the tongue, and allow it to dissolve
juries. The most important reason why the nurse completely. Do not chew, suck, or swallow the
should teach the patient not to apply heat in the drug.
area of the patch is because it might 
|



$


 

 

a. inactivate the drug. area of inside of the cheek for 5 seconds, and
b. loosen the patch from the skin. then leave it there.
c. prolong the drug’s effect.
d. cause respiratory depression.

15. The nurse should assess for respiratory depression


for 24 hours after a patient has discontinued use of
which medication?

#  


#   

=&?

} 
 
=&?

!   
$ 
="?

Copyright © 2016, Elsevier Inc. All Rights Reserved.


54 CHAPTER 28 Q
}
=+ ?
    
}
   

+
> 

   

20. An aphasic patient, who has been receiving me- 6


^

 




Z&
$

  

peridine 75 mg every 3-4 hours for pain for the injury and is in severe pain from compound
past 36 hours, has suddenly become restless and fractures. Which chronic condition would be of
irritable. The last dose of meperidine was 4 hours greatest concern when administering an opioid?
ago. Why is it important for the nurse to withhold a. Asthma
the meperidine, do a complete assessment on the b. Diabetes
 

 

  


- c. Head injury
ings? d. Hypertension
a. Respiratory depression is imminent.
b. The patient is becoming physiologically depen- 26. The nurse would consult the prescriber if the nurse
dent on the meperidine. 
 

 
 

 
$


c. Tolerance to the meperidine has developed. chronic condition and was prescribed tramadol
d. Toxicity from a metabolite may be occurring. =ˆ ?
$
 
$ƒ

* 
$ 
=*…?
21. Which ordered diagnostic test would be a priority 
>
 
 
  
=>}|&?
to review when a patient is prescribed methadone? c. Dementia

‰! 
{@
 

Z>\

\… š
^~

 
 

  

=& -
d. Potassium ?
$

 
 




to the prescriber before administering the drug?


š
^^

 
 


 
 
 
={

 
?
$

=}>?
$
 
$

a. Blood pressure 86/60 mm Hg
teaching can prevent the adverse effect that is most 
> 
]]
 '‰

likely to persist with long-term use of this drug? 


#+
^<

> 
" 


  d. Pulse 88 beats/min
b. Change positions slowly. e. Temperature 99.8° F
c. Do not operate dangerous machinery.
d. Take deep breaths every hour. 6 28. What is the most common nursing concern when a
 


  
@
=| ?ƒ
23. A cancer patient has received instructions regard- a. Breathing

  
$

=}>?
b. Circulation
Which of these statements made by the patient 
Z 
would indicate that the patient needs further teach- 
{ $
ing?

’#


$


$
#


episodes of pain between doses.” DOSE CALCULATION QUESTIONS



’#$
#
 
$
 


#

should crush it and take it with a small amount 29. A patient is prescribed 3 mg of morphine by
of applesauce.” intravenous route every 3 hours as needed for

’#



$

 
 

postoperative pain. Available are vials containing
 
#

 


  “ 
 '‰
*
 

$





’#


 


$
#

 

nurse administer at each dose? The drug handbook
|



}
$
#


 


 






 
=!?“ of sterile water and administered slowly over 5
 
!
]`%‰
 

 

`^%‰

š
^‹
 


 
  
$

 

increments. How many seconds should elapse
agitation, rhinorrhea, and yawning after adminis- 
 
 
$
`^

$


-
tration of pentazocine, nalbuphine, butorphanol, or phine?
buprenorphine?
a. The patient has an opioid agonist in her system.
b. The patient is allergic to the drug. 30. Naloxone 10 mcg/kg is prescribed to treat respi-
c. The patient is experiencing a common adverse ratory depression for a 7-lb neonate. The drug is
effect of the drug.   

`‹
 '‰
 



$

d. The patient is unable to excrete the drug. naloxone that should be administered?

Copyright © 2016, Elsevier Inc. All Rights Reserved.


CHAPTER 28 Q
}
=+ ?
    
}
   

+
> 

    55

CASE STUDIES Case Study 2


A patient continues to require morphine long after the
Case Study 1 usual time frame for opioids after the type of surgery that
The nurse is providing preoperative teaching to a 35-year- 
$
{


 
 



-
old female patient who has been admitted for an abdomi- ance for pain, but the prescriber has decided that she must
 
 
| %
   
=|>?

stop the morphine. The nurse is concerned that the patient
anticipated as the mechanism of postoperative pain relief. may have been dependent on drugs before coming to the
hospital and that withdrawing the morphine may lead to
1. What preoperative teaching should the nurse pro- abstinence syndrome.
vide this patient to enhance pain relief and prevent
adverse effects, including respiratory depression? 5. How should the nurse address these concerns?

2. Postoperatively, the patient is ordered morphine 6. What are the symptoms of abstinence syndrome


|>

$

  
 
!
that the nurse should include in patient assess-
pump is set to deliver 1 mg of morphine per injec- ment?
tion and up to a maximum of 5 mg/hr. What are
the advantages and disadvantages of PCA?


~
#

  
 

 


 
-
controlled pain, not be a substance abuser, and not
3. The patient appears to be sleeping. Her skin is have any other medical problems?
pale, and her respiratory rate is 8/min and shallow.
What should the nurse do at this time in order of
priority?

8. How does physical dependence on opioid analge-


sics differ from abuse and addiction?

4. What measures, other than drug therapy, can the


nurse employ to reduce this patient’s pain?

9. Describe a situation where physical dependence


on opioid analgesics is rarely a complication.

Copyright © 2016, Elsevier Inc. All Rights Reserved.


56 CHAPTER 29 Q Pain Management in Patients with Cancer

29
Pain Management in Patients with Cancer

STUDY QUESTIONS 6 8. Which of these laboratory test results would be a


priority to report to the prescriber when a cancer
Matching patient treated with chemotherapy is taking a pain
reliever that contains ibuprofen?
      
‰!
‹`
  
 '‰

]

#”


”
  
–ˆ+
^^
 '‰
2. ___ Peripheral nerve injury c. Platelet count 60,000/mm3

†

#”

  d. WBC 5500/mm3
4. ___ Tissue injury pain
9. A postoperative cancer patient was receiving
a. Neuropathic 
$ 

]`


‹

$
 

b. Nociceptive !
  

 





{  and ordered morphine sulfate 20-mg tablet every
d. Visceral 4 hours. How does the oral dose compare with the

 ƒ
CRITICAL THINKING, PRIORITIZATION, 
!
 
 
 

 



dose.
AND DELEGATION QUESTIONS b. These are equianalgesic doses.
c. The oral dose produces less analgesia than the
5. What is the most reliable indicator of the need for

 
pain relief in the oncology patient?
d. The oral dose produces more analgesia than the
a. Changes in vital signs that can occur with pain

 

Z 
$

 

c. Patient’s expressions and reluctance to move
š
]`
!
 

 
  
 


d. Patient’s report of a need for pain relief


most likely to occur with which pain regimen?
a. A loading dose of 10 mg of morphine before
6 6. When developing a plan of care for pain man-
starting PCA
agement for the cancer patient, achievement of

# 
 %

 
$

$

____________ is the priority outcome.


5 mg to 7 mg
a. a rating of pain less than 3 on a scale of 0 to 10

| %
   
=|>?



b. comfort that allows the patient to complete all


1 mg of morphine per injection up to a maxi-
activities of daily living
mum of 5 mg/hr
c. pain relief that is acceptable to the patient

—`

$






 

d. total pain relief


who has been receiving steadily increasing
doses of morphine
7. A cancer patient whose history includes type 2
diabetes mellitus informs the nurse that she is ex-
11. A patient has been taking oral opioids for moder-
periencing constant burning pain in her feet. Based
ate pain associated with prostate cancer that has
on the type of foot pain commonly experienced by
  @



{ 

$

diabetics, the nurse would consult the prescriber


and the patient is experiencing moderate to severe
regarding possibly ordering which drug?
postoperative pain. Which analgesic is most ap-


   


 
=!?
propriate in this case?
b. An antiseizure adjunct drug, such as gabapentin

–

=+?
b. Fentanyl transdermal patch


  
%  



#

 
$

=+{#&?


 
=?
d. Morphine sulfate via PCA pump
d. An opioid-agonist analgesic, such as morphine

Copyright © 2016, Elsevier Inc. All Rights Reserved.


CHAPTER 29 Q Pain Management in Patients with Cancer 57

6 12. What is of most concern to the nurse when a DOSE CALCULATION QUESTION
 

  

+{#&
   


regular basis?
]<
*
=& ?
]`


  


‰ 
$
" 
 
$ 

 
!



-
b. Heartburn ministering at a rate of 2-5 mg/min. The drug
c. Rigid abdomen 
  

]`
 '‰
!
 
 


{
 to administer the drug using a syringe marked

 
#$

 
 
``^

13. Celecoxib 200 mg twice a day is prescribed for every 5 seconds, will the rate be safe?
a patient with cancer that has metastasized to the
bone. Which adverse effect is more likely to occur


 


+{#& ƒ
a. BP elevation CASE STUDY
b. Cerebrovascular accident

\#
 A cancer patient is being discharged on oxycodone and
d. Physical dependence  
=|?

6 14. Which laboratory test result would be of greatest 1. What teaching should the nurse provide regarding





  
$

+{#&
the opioid and nonopioid components of this drug?
drug to a cancer patient?

–ˆ+

 '‰

…–\
^††
 '‰
c. Platelets 40,000/mm3 !
 ‚
 





|
=-
d. WBC 4700/mm3 
]`


 
†^
 ?
 
 

every 4 hours. The prescriber has discontinued the


6 15. What is of greatest priority when naloxone is |




=}>?
‹`


administered to combat respiratory depression as- 



 





—




sociated with opioid use for cancer patients? and acetaminophen 650 mg every 6 hours. The patient is
a. Hydration overwhelmed by the multiple medications and asks why

‰
$
   he cannot just take three Percocet tablets every 4 hours.

| 
$


d. Return of pain 2. How should the nurse respond?

16. The nurse is preparing to administer metoclo-


 
=  ?


 
 


-
riencing nausea associated with opioid use. How
should the nurse respond when the patient asks how
3. What interventions can the nurse offer to help the
this drug works?
patient understand his analgesic regimen?
a. “The drug counteracts gastric slowing caused by
opioid use.”
b. “The drug decreases the release of histamine.”
c. “The drug relaxes smooth muscle.”
d. “The drug works on the nausea center in the
brain.” 4. What teaching should the nurse provide regarding
the adverse effect of constipation?
6 17. What is the priority concern when a patient com-
bines prescribed opioids with alcohol?
a. Nausea and vomiting
b. Orthostatic hypotension
c. Physical dependence
d. Respiratory depression 5. What teaching can the nurse provide to enable
the patient to assist the prescriber with decisions
regarding analgesic prescription?

Copyright © 2016, Elsevier Inc. All Rights Reserved.


58 CHAPTER 30 Q Drugs for Headache

30
Drugs for Headache

STUDY QUESTIONS 9. What is the primary reason why abortive medica-


tion for migraine headaches should not be used
Matching more than 1 or 2 days a week?
a. Abuse potential
     
    b. Development of chest pain
         c. Development of tolerance so medication is not
    effective
d. Medication-overuse headache
1. ___ Abdominal pain and amylase 500 interna-
 
 '‰ 10. Which over-the-counter medication has not dem-
2. ___ Apical pulse 48 beats/min onstrated effectiveness in relieving mild to moder-

†

*
~†^Œ
*>}3

Z"'‰Œ
CO2 35 mm Hg ate migraine headache pain?
4. ___ Positive Homans’ sign and swelling in left a. Aspirin
calf b. Acetaminophen



| 
 
 
—``
‰ c. Acetaminophen, aspirin, and caffeine

—

| 
—
Z"'‰ 
# $

~

{ 
–|

†`

*

  

from sitting to standing 11. Which nursing concern would be of greatest


priority when a patient is prescribed meperidine


=
 ? =&?
$
  
   ƒ

>   
=–? a. Abuse potential

& 
=  @? b. Potential for respiratory distress

Z  
=? c. Constipation

| 
= 
? d. Relief of headache
$
!  
=  @?

Š  
=>>–? 6
]^

 



$
  

priority to report to the prescriber if the nurse was


preparing to administer ergotamine to a patient
CRITICAL THINKING, PRIORITIZATION,
who is experiencing the start of a migraine head-
AND DELEGATION QUESTIONS ache?

‰!
†`
  
 '‰
8. A nurse knows the following about migraine head- b. Blood pressure 90/58 mm Hg
 
={

 
? 
>  

$
]^

a. They cannot be treated effectively after the pain 
… 
$
 

  

starts.

!

  
$
   
13. The rationale for administering both ergotamine
blood vessels. 
 
=  ?


 



c. They involve vasoconstriction of intracranial a migraine headache includes that the metoclo-
blood vessels. pramide
d. Their therapy includes regular schedule for eat- a. decreases nausea and vomiting.
ing, sleep, and exercise. b. minimizes sensitivity to light.
e. Their therapy should start at the earliest sign of c. prevents adverse effects of weakness and myal-
attack. gia from ergotamine.
d. provides additional vasoconstriction to relieve
cranial vessel vasodilation.

Copyright © 2016, Elsevier Inc. All Rights Reserved.


CHAPTER 30 Q Drugs for Headache 59

 14. The nurse would consult the prescriber be- 6 19. A patient is prescribed propranolol for migraine
fore administering ergotamine or triptan drugs 

 




for migraine headache to a patient with which priority to report to the prescriber?
= ?ƒ
={

 
? a. BP 130/80 mm Hg
a. Cardiovascular disease 
Z 
@
b. Diabetes mellitus c. Hunger and thirst
c. Hypertension d. Pulse 98 beats/min

#$
e. Peptic ulcer disease 20. What would be a contraindication for estrogen use
to prevent menstrual migraines?
15. What is true about triptan therapy for migraine a. Asthma
headaches? b. Dysuria
a. Administering an oral dose of the drug after c. Menorrhagia
subcutaneous administration will prevent recur- d. Thrombophlebitis
rence of the headache.
b. Combination with ergotamine provides more 21. A patient is receiving lithium carbonate for
relief than triptan alone. prevention of cluster headaches. This drug has a
c. Peripheral vasoconstriction is the primary cause narrow therapeutic index. Which drug level would
of adverse effects. fall into the safe and effective range?
d. Therapy relieves headache, nausea, and sensi- 
`]
Z"'‰
tivity to sound and light. 
`^
Z"'‰

`
Z"'‰
6]—
!
 

 
  
=#? 
`<
Z"'‰
š

   
$
  
  
$


^]``

PM?


†% %

}

$-
ter administration, the patient complains of heavy DOSE CALCULATION QUESTIONS
arms and chest pressure. What is the priority ac-
tion at this point?
^^
&  
  
=&*Z
‹?

 -
a. Activate the medical emergency team. 


=]
 '‰?
|  

]```

b. Assess vital signs and evaluate the chest pain.   


#€
*




 


>
 Œ



 
 
$$ administer?
d. Notify the prescriber.

17. A patient does not inform her primary care


^†
+   
= ?



  
 -

=|>|?
 


  

able are 2.5-mg tablets. How many should the
=>  ?


   
!
|>|
- nurse administer?
scribes a triptan for migraines. What symptoms of
serotonin syndrome are possible with this combi-
nation? CASE STUDY
a. Confusion and incoordination

Z


 

 A 49-year-old patient is seen in the emergency department
c. Hypotension and fainting =Z&?


$
 
$

$"
  

d. Nausea and vomiting that begin mid-morning and grow progressively worse
 

 
{


  
  


18. The nurse would plan for which potential nursing vomiting. Advil has been minimally successful in treating
 

 
 
=* ?
the headaches. Her headaches are pulsatile and usually
for migraine headache?   

$

 ‚
  



a. Altered tissue perfusion 


 
 
{  
=#?
—

  -

…

 neously is prescribed to be administered now, along with

{ $
 
$
#
{!!
 
|
—



 


{
  as directed.

1. Describe what the nurse should include in teaching


this patient about administering the self-injecting
pen.

Copyright © 2016, Elsevier Inc. All Rights Reserved.


60 CHAPTER 31 Q
 
 

!
ˆ 

{@

^
!
 




 


4. The patient tells the nurse that a friend takes er-
hospital outpatient pharmacy, but her insurance gotamine. Her friend “got hooked” on ergotamine
does not cover sumatriptan. Dihydroergotamine is and had to go to the hospital to “get off of it.”
prescribed in place of the sumatriptan. What teach- What would the nurse include when comparing







Z&
 
 - ergotamine and dihydroergotamine?
ing the administration of dihydroergotamine?

5. The patient is instructed to seek follow-up with


3. What teaching regarding preventing toxicity from her PCP. The PCP discusses the use of prophylac-
dihydroergotamine should be provided to this tic drugs to prevent the migraine headaches. What
patient? drugs might be considered?

31
Antipsychotic Agents and Their Use in Schizophrenia

STUDY QUESTIONS CRITICAL THINKING, PRIORITIZATION,


AND DELEGATION QUESTIONS
True or False
   
  T   F 9. A patient who is receiving chemotherapy for breast
    

  
 @
=! @?

There is no mention of other medical or psychiatric


1. ___ High-potency antipsychotic drugs are more disorders in the patient’s history. What is the most
likely to be effective than low-potency likely reason why this patient is receiving the drug?
drugs. a. History of delusions
2. ___ High-potency antipsychotic drugs produce b. History of hallucinations
desired effects at lower doses than low- c. History of nausea
potency drugs. d. History of tics
3. ___ High-potency antipsychotic drugs treat both
positive and negative symptoms more than 10. Which extrapyramidal adverse effect of antipsy-
low-potency drugs. chotic drugs would the nurse expect to occur only

‹

{%  
 
={\?
after an extended period of drug therapy?
drugs are often thought to be more effective. a. Repetitive, slow, twisting movements of the



{\

 

 
 %  
tongue and face
 
=…\ ? 
{
 
$


$ 



—

{\

 
$$
  back

~

{\


  

 
$$  
{



 
8. ___ Antipsychotic drugs can cure schizophrenia 
ˆ 





if taken as prescribed.

Copyright © 2016, Elsevier Inc. All Rights Reserved.


CHAPTER 31 Q
 
 

!
ˆ 

{@ 61

6 11. What is the greatest priority when a patient is 16. Which symptom would be of greatest concern
experiencing acute laryngeal dystonia? 
 

…\
 
  
˜!
-
a. Airway clearance longation?
b. Anxiety treatment 
Z 
$ 

 
c. Pain therapy b. Fatigue

Z 
 
$ 
{
$ 
d. Thirst
6
]^

$
 
 


 




receiving a neuroleptic antipsychotic, is of highest 17. Which statements suggest that a family member
priority and should be reported to the prescriber needs additional teaching regarding administration
immediately? $

 
"
…\ƒ
={

 
?
a. Agitation 
’#
 



$


$



&$
  in juice so it tastes better.”

# 

$

  
’#






 
”



{
% 
 
 
 

 
}





#



that he takes all of it.”


6 13. What would be the priority action to reduce a pa- 
’#


 



 
$
#



‚

 
$
]`‹„
…
=‹`„
>?
  
drug.”
by neuroleptic malignant syndrome? 
’#







 



 
 
=!?
- comes in and put it in a dark cabinet.”
scribed every 4-6 hours as needed 
’#



 


 $

 


 
 
=& ?
- 


 
 
#

 
“
ately as prescribed

# 

 

]``
‰'  
]<


$

 
$$
$


d. Tepid bath 

 



 @
=€-
 ?

 
=* ?


 

š
]‹
!
 

 

  

 - report to the prescriber immediately?
sure and pulse of a patient who is prescribed a a. Fainting not associated with sudden position
neuroleptic antipsychotic drug. Which assessment changes



 





b. Milk secretion from a male patient
consult the prescriber? 
{
 

$


a. BP lying 110/66 mm Hg and pulse 76 beats/ 
{ 
Œ
^

 
–|
 
]`<'~`


Hg and pulse 80 beats/min 19. What is another name for second-generation anti-
b. BP lying 120/84 mm Hg and pulse 70 beats/  
={\ ?ƒ
Œ
^

 
–|
 
]]`'~—

a. Atypical antipsychotics
* Œ
 
~^
  ' b. High-potency antipsychotics
c. BP lying 116/70 mm Hg and pulse 72 beats/ 
‰%
 
Œ
^

 
–|
 
]]`'—<

d. Typical antipsychotics
Hg and pulse 122 beats/min
d. BP lying 146/90 mm Hg and pulse 73 beats/ 
^`
#

 
$

 

 
 


Œ
^

 
–|
 
]†<'<<

are prescribed any neuroleptic antipsychotic drug
* Œ
 
~<
  ' to avoid taking which over-the-counter medica-
tion?

]

—<% %
 


  

…\

 
=!?
is diagnosed with benign prostatic hyperplasia. 
 

= ?
Which assessment would be of greatest priority 
& 
=– ?
related to the drug and this condition? 
| 
={ $?
a. Abdominal pain
b. Blood pressure 21. A patient is prescribed haloperidol decanoate
c. Dizziness =* ?
^



‹
 
 





# 

  

$
   
=#€?
”-
tion?
a. Deltoid
b. Dorsogluteal
c. Vastus lateralis
d. Ventrogluteal

Copyright © 2016, Elsevier Inc. All Rights Reserved.


62 CHAPTER 31 Q
 
 

!
ˆ 

{@

6 22. Which laboratory test result would be of greatest


^~
# 
@
=> ?
]



priority to report to the prescriber of haloperidol   


{!!
$

 
 

=* ?ƒ 

…\
–@

  

]
 '‰


–ˆ+
]<
 '‰ and can be administered undiluted over at least 1

…–{
^<—
 '‰ minute. The nurse decides to be cautious and ad-

| 
†^
 '‰ minister the drug over 2 minutes. How much drug

! 
‡~
 '‰ should the nurse administer every 10 seconds?

^†
#

 
$

 


$
 

hunger, thirst, and urination if a patient is pre- CASE STUDY


 

 = ?ƒ
={

 
?

> @
=! @? An 82-year-old patient who resides in a nursing facility

>@ 
=>@ ? 



  

 
{



* 
=* ? agitated and is not oriented to place. The on-call hospital-

} @ 
=¢ ? ist is consulted.

6
^‹
#







  1. Why is it important to inform the hospitalist about
š




 



@ 
the presence or absence of a history of dementia?
=>@ ?ƒ
a. Blood pressure drop of 10 mm Hg systolic
when changing from lying to standing position

  
$
]`

$

$


! 
]`‹„
…
=‹`„
>? Haloperidol 0.5 mg every 8 hours is prescribed as needed
d. Weight gain of 2 lb in 1 month when nonpharmacologic measures cannot control agita-
tion. Two hours after administering the drug orally, the
š
^
 
  
 

 
$

 
nurse notes that the patient is moaning. Her eyes are
to monitor when caring for a patient who is pre- rolled upward, and her back is arched.
 
 
=  ?ƒ
={

 

?
^
 



 
 
 ƒ

‰!
b. Cholesterol
c. Creatinine
d. Hemoglobin A1c

#+ 3. What other assessment data are important for the
nurse to collect and communicate to the pre-
scriber?
DOSE CALCULATION QUESTIONS

^—
> @
]``





 


ordered. The drug will be administered in the ven-


trogluteal muscle. The drug is available as 25 mg/
‰
*
 



”


 ƒ

Copyright © 2016, Elsevier Inc. All Rights Reserved.


CHAPTER 32 Q Antidepressants 63

32
Antidepressants

STUDY QUESTIONS 11. Patients who are prescribed MAO in-


hibitors should not eat foods containing
Completion ___________________.

1. The most common symptom of depression in addi-


tion to depressed mood is ___________________ CRITICAL THINKING, PRIORITIZATION,
_________________ or _____________________ AND DELEGATION QUESTIONS
in usual activities and pastimes.
6 12. When assessing a patient who is receiving an an-
2. Due to the risk of suicide, close observation is tidepressant, which question would be of greatest
 


 
$

$
- priority for the nurse to ask?
depressant therapy and ___________________ a. “Are you concerned about weight gain when
___________________. you take medications?”
b. “Are you having thoughts about doing anything
3. Maximum response to antidepressant drugs occurs
that could harm you?”
in _____ to _____ weeks.
c. “Do you experience dizziness when you stand
4. Relapse is more likely if a patient discontinues up?”
pharmacotherapy sooner than _____ to _____ 
’* 

 

$
 ƒ“
months after symptoms resolve.
13. Patient compliance with antidepressant drug


!
 
=!> ?

 
therapy is most likely to be increased if the nurse
$
 

 

 

 %
explains the
drugs because they have ___________________ a. possible drug interactions.
___________________ ___________________. b. reason for limiting the number of units of the
drug supplied with each prescription.

—
{

 
 
={{# ?
c. seriousness of the adverse effects.
should be administered in the morning because d. expected timing of the therapeutic response.
they are ___________________ and can cause
___________________. 6 14. When a patient who is receiving an antidepressant
verbalizes suicidal ideation, what is the priority
7. Overdose of TCAs can cause death because of ef-
nursing action?
fects on the ___________________.
a. Administer the antidepressant.
8. When taken at doses and times as prescribed, b. Ask the patient why he or she feels this way.
___________________ antidepressants can still c. Notify the prescriber.
cause hypertensive crisis and death. d. Provide a safe environment.

9. Patients who are prescribed amitriptyline hydro- 15. Which antihypertensive drugs could increase the

=Z ?
$
 

 
risk of hyponatremia if prescribed in addition to
the prescriber or pharmacist before taking any 
=|@ ?ƒ
over-the-counter ___________________ or a. Candesartan
___________________ ___________________. 
Z  
c. Hydrochlorothiazide
10. When a drug has the possible adverse effect of 
{ 
bruxism, the nurse should assess the patient for
___________________, ___________________
___________________ and __________________
__________________.

Copyright © 2016, Elsevier Inc. All Rights Reserved.


64 CHAPTER 32 Q Antidepressants

6
]—

  
 
 
$



 22. A patient complains to the nurse that he has been
 





 $ 
experiencing erectile dysfunction since he was
would be a priority to report to the prescriber?   
 
=| ?
 





{!
†`
  
 '‰ nurse’s best response?

> 
`<
 '‰ 
Z 

 

 




#+
‹ with the prescriber.
d. Platelets 250,000/mm3 
# 

 


 

 -
sant on weekends.
17. The nurse should consult the prescriber regarding 
{  
 

 


  


which order?   

 


=-

! 

=|@ ?
^`




?
a day at bedtime. d. Withhold the drug until the issue is resolved.

! 
 @ 
=?





once a day at bedtime. 6 23. What is the priority assessment when a patient is

! 

={  $?
^`



switched from a TCA antidepressant to an MAO
days 14-28 of menstrual cycle. inhibitor?

& 

=|@ ?
 

a. Blood glucose
 
% 

=|@ 
?
b. Blood pressure
90 mg by mouth once a week in 7 days. 
{


{
 
18. The nurse would be especially vigilant to assess
the patient for unexplained bleeding when a pa-
^‹
 



$
!>

{{# ƒ
={




  

  
?
a. an antihistamine. 
Z$$

  

 
b. lithium. 
Z$$

  


c. low-dose aspirin therapy. 
{
 
$$
 

$
 

d. a thiazide diuretic. with MAO inhibitor antidepressants.


d. Therapeutic effects occur rapidly.
19. The nursery nurse sees that a neonate, whose e. Therapeutic effects occur slowly.



 
 $ 
=Z$$
?

for major depression throughout pregnancy, is 25. Which change, when assessing the BP of a patient
tremulous and very irritable. Which assessment 

  

!>
  
  

 



 

 

orthostatic BP change and risk of fainting?
be of most concern to the nurse? 
–|
]†—'~—

|

 Œ
–|
]]^'—`

|
‡<


&$
  standing
b. Flexed extremities 
–|
]~^'‡—

|

 Œ
–|
]——'‡`

|
~<

c. Pulse 186 beats/min standing


d. Respirations 20/min 
–|
]]‹'~—

|

 Œ
–|
]`^'—`

|
~<

standing
6 20. The nurse is reviewing laboratory test results for 
–|
]^`'<^

|

 Œ
–|
]`<'~—

|
<`

a patient whose drug regimen includes desvenla- standing


$ 
=| "?

 


$
  

priority to report to the prescriber? 26. Which data would be of greatest concern when a

\…

‰'  

  
 
= $ ?ƒ

>\
‡``
%  
 '‰ a. Patient reports experiencing “yawngasm.”
c. Osteoarthritic changes in the spine b. Patient reports frequent palpitations and a rac-

{
]†
Z"'‰ ing heart.
c. Patient reports that he burns easily when in the

^]

$
 
 
$



 - sun.

$

 
  

=>  ?
d. Patient reports use of polyethylene glycol
would be of concern to the nurse? =€ ‰?



  
a. Back pain
b. Cirrhosis
c. Diabetic neuropathy

{
 


Copyright © 2016, Elsevier Inc. All Rights Reserved.


CHAPTER 32 Q Antidepressants 65

6 27. The nurse has received a start of shift report


††
#

 
 
$

 

 

 

on patients who have recently been prescribed 




  
 @
=}?



antidepressant medications and are experiencing what?


the following adverse effects. Which patient is of 
&

 
^``

$

 
 
highest priority for the nurse to assess? 
# 


 




 -
a. Amitriptyline—sleepiness and constipation pation.
b. Fluoxetine—nausea and diarrhea c. Report any symptoms of a urinary tract infec-

! 
=|  ?ž  

tion.
vomiting 
{
 
 
$



 

d. Venlafaxine—anorexia and sweating after four hours.

28. Which foods can cause a hypertensive crisis when 34. What should be included in teaching the patient

 

  

€}#ƒ
={

 

 
 



$% 

{

? John’s wort?


a. Apples, pears 
#$

 
 

 


 -
b. Baked goods that contain yeast ing this drug.
c. Bologna, pepperoni, salami b. Research has proven the product is as effective
d. Milk, yogurt, cottage cheese, cream cheese as antidepressant drugs for mild depression.

{
 c. The substance has not been proven to be effec-
tive.
29. Which instruction should be included when teach- d. The substance is completely safe because it is


 


  

€}#

natural and not a drug.
orthostatic BP?
a. Avoid certain foods to prevent orthostatic hypo-
tension. DOSE CALCULATION QUESTIONS
b. Change positions from lying to sitting and
standing slowly.
†
| 
=| ?

`


  


c. Drink at least two servings of caffeinated bev- day. Available are 25-mg and 62.5-mg CR tablets.
erages each day to decrease the risk of ortho- What should the nurse administer?
static hypotension.

{
 


$


@@-
ness when standing from a sitting position.
†—
Š $ 
=Z$$
?

]`


-
 
!
  
  

~%

30. The nurse assesses for shivering and fever when capsules. How many capsules should the nurse

 


  

€}#


administer?
which drug for postoperative pain?

– 
={ ?

*'  
=Š? CASE STUDIES

*
=& ?

€
=&? Case Study 1
6
†]
#


$
  




A 28-year-old patient, who is a high school science
new condition to the prescriber of bupropion teacher with two children, comes to the community men-
= ?ƒ tal health center at her family’s urging because of lack of
a. Anemia  

  
 
$
  
 -
b. Diabetes mellitus sive sleepiness, and feeling “down” every day for the past

Z  †
 
&  
=+ ?
`






\  prescribed.

632. Which laboratory result would be of greatest 1. Developmentally, what are the advantages of this
concern to the nurse when a patient is receiving particular TCA for this patient?
nefazodone?

{!
†—
  
 '‰

–ˆ+
^^
 '‰

+
]‹—
Z"'‰
d. WBC 11,000/mm3

Copyright © 2016, Elsevier Inc. All Rights Reserved.


66 CHAPTER 32 Q Antidepressants

2. The patient is hopeful that the medication will Case Study 2



{

’#$





#
 

to meet with the doctor every week?” What is the An adolescent patient who has been irritable, refusing to
basis of the response to this question? go to school, or eat meals for several months is admitted
to an adolescent psychiatric unit. Cognitive behavioral
 

 


   
  


=|@ ?

8. What precautions teaching would the nurse pro-


3. What symptoms should the patient, family mem- vide because of the risk of serotonin syndrome?
bers, and/or caregivers be told to report?

The patient’s insurance allows only 10 days of inpatient


hospitalization, and the patient is to be discharged to
4. The patient asks why the drug is supposed to be the care of his parents. All antidepressants have a boxed
taken at bedtime. How should the nurse respond? warning for suicide risk, especially in children and adoles-
cents.

9. The nurse would caution the patient’s parents that


suicide risk is greatest at what point in antidepres-
sant therapy?
5. What teaching should the nurse provide regarding
 

 
$$
$
!> ƒ
=]?
 -
Œ
=^?
  
 Œ
=†?
-
gic effects
10. What actions should be taken because of this risk?

6. The patient is concerned about cost because the


prescriber has prescribed only 1 week’s supply at a
time. What would be the rationale for such a small 11. What rationale would support the use of antide-
prescription amount? pressants in this case?

7. The patient is admitted for an inguinal hernior-


rhaphy. When providing history information, the
patient states that she has been taking desipramine
=+ ?
^``



$
†
 

{


 
 

 

$ 


take the medication, but does not feel bad the next
 
{
 
 



 



the medication any more. How should the nurse


respond?

Copyright © 2016, Elsevier Inc. All Rights Reserved.


CHAPTER 33 Q Drugs for Bipolar Disorder 67

33
Drugs for Bipolar Disorder

STUDY QUESTIONS š
]]

 


  
 
$

 

receiving lithium carbonate for an acute episode of


True or False BPD. Which result should the nurse report to the
prescriber immediately?
   
  T   F 
‰!
†`
  
 '‰
   
> 
^~
 '‰

{
]†^
Z"'‰

]


 

 
 
=–|&?

| 
‹
Z"'‰
alternate between mania and depression, but
length of episodes vary. š
]^
!
 

 
$



 


2. ___ BPD is a chronic condition. is prescribed lithium carbonate for BPD. Which
3. ___ BPD requires treatment for the rest of the  = ?

 




patient’s life.   
ƒ
={

 
?
4. ___ Manic episodes are always distressing to the a. Colonic cleansing preparation for colonoscopy
patient with BPD. b. Frequent dressing change on a diabetic wound
5. ___ The cause of BPD is an unstable personality. c. Hemodialysis
6. ___ The pathophysiology of BPD involves atro- d. Repeated hypoglycemic episodes with diapho-
phy of brain regions involved with emotion. resis

~

!

 

=  ?
 
e. Respiratory tract suctioning
promote neuronal growth and survival in the
subgenual prefrontal cortex. 6
]†
#





 

  
$

8. ___ Antipsychotic drugs should not be used in lithium carbonate about monitoring lithium levels
BPD unless the patient has symptoms of if the BPD patient is diagnosed with hypertension
psychosis. and prescribed what drug?
a. Amlodipine
b. Atenolol
CRITICAL THINKING, PRIORITIZATION,
c. Hydrochlorothiazide
AND DELEGATION QUESTIONS d. Valsartan
6 9. A patient is admitted with an acute mixed epi- š
]‹

 
 



sode of BPD. The patient is started on divalproex   = ?





 





=  ?


 




 

  ƒ
={

 

priority nursing concern? ?



Z
   a. Constant midepigastric pain radiating to the
b. Hydration back

{ $ b. Nausea, vomiting, and very dark-colored urine
d. Toxicity c. Pale conjunctiva
d. Petechial rash
š
]`

 

   

e. Warm or hot skin and fruity breath
would be the most appropriate nursing outcome?

&

 @
 
$


Z 
 


{

 

 

 
  

{
<


 

Copyright © 2016, Elsevier Inc. All Rights Reserved.


68 CHAPTER 33 Q Drugs for Bipolar Disorder

6 15. A patient with BPD is prescribed carbamazepine business venture and accumulated almost $10,000 worth
200 mg twice a day. Trough levels drawn 30 min- of bills before she was caught for writing bad checks. Her

$


  
 

—
 '‰
husband was called when she tried to purchase a car, and
What is the priority nursing action at this time? 


 

 
 $
$



a. Administer the drug as prescribed. 



{

 


  




b. Assess for seizure activity. manic state. On admission she moves about restlessly,
c. Consult the prescriber. waving her arms in a threatening manner while loudly

Z 


 
$


$
$  

 


  
 $$
{
 

to be released from “this jail” and curses the nurse who


š
]—
!
>–>


 





interviews her.

 
 
$
   @

 -

]‹
'‰
 
]~````'3, reticulocyte 1. Why might this patient have stopped taking her
count 2%, and WBC 6600/mm3. What should the medication?
nurse do?
a. Administer the drug as ordered.
b. Hold the medication and assess the patient.
c. Hold the medication and consult the prescriber.
d. Hold the medication and consult the prescriber
^
‰
   
†``


  
‹


{!! day. Why must lithium be taken in divided daily


doses?
6
]~

 

–|&

 
  
=‰ -
 ?


 


 


 

in the patient’s oral cavity. What is the priority


nursing action at this time?
a. Administer the drug as ordered. 3. The patient complains that she cannot play cards
b. Hold the medication. or write a letter. What should the nurse do?
c. Hold the medication and consult the prescriber
during rounds.
d. Hold the medication and consult the prescriber
immediately.
4. The nurse has explained to the patient and fam-
ily the importance of monitoring plasma lithium
DOSE CALCULATION QUESTIONS levels every 2-3 days until therapeutic levels
have been reached and maintained and every 1-3

]<
‰
   
`—




 

- months once maintenance level dose has been es-
scribed. How many capsules should be adminis- tablished. What should the nurse teach about other
tered if 300-mg capsules are available? times that the patient should consult the prescriber
about the need for additional monitoring?

19. Divalproex sodium is available as a syrup contain-



^`
 '
‰
*
 



-
istered if the prescribed dose is 375 mg three times
a day? 5. What measures can the nurse teach the patient to

 


 ƒ

CASE STUDY
A 35-year-old patient who has a history of BPD has 6. What is the therapeutic range of plasma lithium

 

  
 

= - for maintenance therapy, and at what point are
'& ?
{

 


  
  

lithium levels critical?
has become increasingly hyperexcitable over the past 5
  
{

 



 
 

  






 

  
{

has called friends and relatives all over the country at all

$

 




 
{



spending spree to buy new clothes and equipment for her

Copyright © 2016, Elsevier Inc. All Rights Reserved.


CHAPTER 34 Q
{ %*
& 69

7. Adherence to drug therapy is often an obstacle to 9. What nonpharmacologic measures can the nurse
managing BPD. How can the nurse increase the teach the patient and family that may help modu-
likelihood of this patient taking her medication late mood?
and participating in therapy as prescribed?

8. What teaching does the nurse need to provide to


the patient and family relating to prescribed and
over-the-counter drugs that relate to the patient’s
medical history?

34
Sedative-Hypnotic Drugs

STUDY QUESTIONS
]^


Z€

13. ___ require reduced doses for people with liver
Matching impairment.

]‹

 
  
  
 
 
$

' '     suddenly discontinued.


  

1. ___ Cerebral cortex and hippocampus CRITICAL THINKING, PRIORITIZATION,


2. ___ Cortical areas AND DELEGATION QUESTIONS

†

‰ 
 

‹

{  

 15. What does the lipid-solubility of benzodiazepines
ƒ
={

 
?
a. Anterograde amnesia and confusion a. Causes induction of hepatic metabolizing en-
b. Muscle relaxation zymes
c. Promote sleep b. Decreases the risk that the drug will cause con-
d. Reduce anxiety genital defects

Z
 
 $%$
True or False 
# 


#
  


>+{
   
  T   F
  
š
]—
 


 


 
@ @-
Benzodiazepine-like drugs epines?

!

 
$

\#
 


5. ___ can intensify the effects of alcohol. slowly.


6. ___ are metabolized into an inactive compound. 
!
 $

$$
$
\–
7. ___ produce moderate muscle relaxation. c. Their metabolites are not pharmacologically
8. ___ are used to prevent seizures. active.
9. ___ prolong periods of uninterrupted sleep. d. They are short-acting.
10. ___ promote falling asleep.
11. ___ have a rapid onset.

Copyright © 2016, Elsevier Inc. All Rights Reserved.


70 CHAPTER 34 Q
{ %*
&

š
]~

 

$




^^``
{
23. A child with a severe head injury is in a
states that she can fall asleep but awakens during barbiturate-induced coma. The child’s parents

 


$




ask the nurse why he is being kept unconscious.
!
 
  

=€?
 
They want the drug to be stopped so they can see
 @ 
=* ?
`]^



$

= ?
if he will respond. The best explanation is that this
as needed. What should the nurse do? therapy
a. Administer the triazolam. a. decreases the brain’s need for oxygen and glu-
b. Administer the triazolam and communicate the cose.
need to assess for effectiveness during shift b. produces amnesia so the child will not remem-
report. ber the accident.
c. Contact the prescriber and discuss the need for c. relaxes skeletal muscle so the patient is not
a slower-onset hypnotic. uncomfortable.
d. Hold the triazolam because it is not effective in d. stabilizes the patient’s blood pressure and pulse.
this situation.
24. The nurse should assess for which symptoms in a
6 18. The nurse in the emergency department adminis- patient with a suspected barbiturate overdose?

  
 
$
 @
= @?
a. Apnea and hyperthermia
to a patient who has overdosed on a benzodiaze- b. Respiratory depression and constricted pupils
pine and alcohol. What is a nursing priority in this c. Hypertension and hypothermia
situation? d. Hypotension and dilated pupils

Z 
" 

b. Monitoring breathing 6 25. A nursing priority for a patient during immediate
c. Reducing anxiety care after overdose on a barbiturate is to monitor
d. Monitoring renal function what?
a. Bowel sounds
6 19. What is the nursing priority when administering a 
&


long-acting benzodiazepine as prescribed? c. Peripheral pulses

\ 
  d. Oxygen saturation
b. Potential for abuse
c. Respiratory depression 26. Patients with depression sometimes experience

{ $ insomnia. The nurse recognizes that in addition to
being a symptom of the disorder, insomnia is an
20. When caring in the morning for a patient who has adverse effect of some antidepressants. Which an-

@
= ?
$


 
tidepressants are effective in treating depression-
before, the nurse would monitor for symptoms of  
  ƒ
={

 
?
which common adverse effect? 
–
= ?
a. Dizziness 
&
={?
b. Hypertension 
…
=|@ ?

{  
… 
=‰?
d. Tremors 
! @
=& ?

^]

 
$ 


$


27. The lowest prescribed dose of doxepin for depres-


frequently awakens during the night. The nurse
sion is approximately how many times greater
would consult the prescriber if which drug was
 

  
 
$

={?
-
  
$
ƒ
={

 
?
scribed for insomnia?

Z @
=‰  ?
a. 2

 
=@?
b. 3

! @ 
= ?
c. 12

¢ 
={  ?
d. 20

¢
= 
>?

22. Tolerance to barbiturates does not produce cross-


tolerance to which substance?
a. Alcohol
b. Benzodiazepines

\ 
 
d. Opiates

Copyright © 2016, Elsevier Inc. All Rights Reserved.


CHAPTER 34 Q
{ %*
& 71

28. An older adult patient occasionally uses diphen- CASE STUDIES


 
={?
$
  
 


the nurse include in patient education to prevent a Case Study 1


very common adverse effect?

# 
 


 An 18-year-old college student comes to the outpatient

‰

   
$

 
 
 

{
 


Z 

%$ 
 worries about upcoming exams and states that she cannot
d. Take the medication with food. sleep, cannot concentrate on her studies, has felt her heart
pound, is dizzy, and has trouble catching her breath. These
29. The nurse is caring for a hospitalized patient who symptoms are unusual for her and began several weeks
 
 


$
$ 


ago.
away from home. Which intervention should the
 

 ƒ 1. What additional data should the nurse collect
a. Assess for a possible reason for insomnia. about the patient’s symptoms?

& 


$
 
$
  
c. Medicate with prescribed as-needed zolpidem
= ?
d. Offer a backrub.

†`

 = ?


 
 
$
 - 2. Why is it imperative that patients on benzodiaz-

$
  ƒ
={

 
? epines be cautioned against combining them with
a. Research suggests melatonin is effective but alcohol?
 
  
$
  
b. Melatonin is a dietary supplement and is not
closely regulated.
c. Melatonin relieved insomnia in a blinded study
of insomniacs.

{
 

 
 

$$
3. What information should be included in the
in preventing jet lag. nurse’s health teaching plan for this patient?
e. There is no guarantee that the product contains
the amount of melatonin as listed on the label.

DOSE CALCULATION QUESTIONS


Case Study 2

†]
¢
]`






  

Available are 5-mg immediate-release tablets. A patient who has been receiving benzodiazepine therapy
How many tablets would be administered at one $






 

  



dose? longer cover the medication.

4. What are possible effects of sudden discontinua-

†^
… @
= @?
`^



$

tion of the drug?
patient with a benzodiazepine overdose, followed
by a second dose of 0.3 mg 30 seconds later by
intravenous push. The drug is available in 0.1 mg/
‰
*




 
 

for each dose? 5. What can the nurse do to help this patient if she
can no longer afford a drug to treat insomnia?

Copyright © 2016, Elsevier Inc. All Rights Reserved.


72 CHAPTER 35 Q Management of Anxiety Disorders

35
Management of Anxiety Disorders

STUDY QUESTIONS
]†
# 
  
$ 
$
  •

____________ ____________ ____________


True or False
14. Persistent uncontrollable thinking and re-
   
  T   F petitive actions: ____________ ____________
   ____________

]

{ 

 @

  - 15. Reexperiencing, avoidance, and/or emotional,
sive-compulsive, panic, and posttraumatic numbing, and hyperarousal: ____________
stress disorders are all primary anxiety ____________ ____________ ____________
disorders.
2. ___ Benzodiazepines are approved for use for
]—
{

 
 
 

$ 
$
-
three major anxiety disorders—generalized ing or going crazy: ____________ ____________
anxiety disorder, obsessive-compulsive
disorder, and phobias.
3. ___ Cognitive behavioral therapy combined with CRITICAL THINKING, PRIORITIZATION,
drug therapy is effective for panic disorder. AND DELEGATION QUESTIONS
4. ___ Depression often coexists with anxiety dis-
orders. 6 17. A nurse is admitting a patient who is scheduled for



\ @

 
=\&?


outpatient surgery. The patient was instructed to
acute condition. take her levothyroxine with a sip of water in the
6. ___ Onset of relief from anxiety with lorazepam morning before coming to the hospital. The patient
= ?

 
=–{ ?

  reveals that she was extremely anxious, so she
7. ___ Principal adverse effects of buspirone also took a lorazepam, but only with the same sip
=–{ ?

 

 
of water “about 1 hour ago.” Which nursing action
slowing. would be the greatest priority?

<

{

 
 
a. Assessing for abuse of benzodiazepines
={{# ?

$$
 


b. Determining if the patient signed the needed
when depression is not present. consents before coming to the hospital

‡

{
 
  
'
- 
Z 
 

 
  

-
laxation therapy is usually all that is needed erative teaching
in mild to moderate anxiety disorders. d. Notifying anesthesia of the medications taken

]`

{
$
  

 

this morning
intense, but they are temporary.
18. The nurse is preparing to administer buspirone
Completion =–{ ?


 



 



for 2 months. What do nursing considerations


         
include?

# 
$
 



-
11. Anxiety when patient thinks he or she cannot
ach
leave a room or situation: ____________

{ $
 
  
 


c. Teaching to avoid drinking grapefruit juice
12. Chronic uncontrollable worrying: ____________
d. Teaching to not discontinue this drug suddenly
____________ ____________

Copyright © 2016, Elsevier Inc. All Rights Reserved.


CHAPTER 35 Q Management of Anxiety Disorders 73

]‡

 


 
 
=| ?
$
24. The nurse notes that a patient becomes very upset
anxiety for 3 months. The nurse expects that the when the nurse rearranges any object in the room.
drug has been most effective in reducing which !
 





  


 ƒ
={

 
? the patient’s history?

’– “


  a. Obsessive-compulsive disorder
b. Palpitations b. Panic disorder
c. Poor concentration 
{ 

 
d. Tension headache 
{  

e. Worrying
š
^


 
 
 
=¢$?


6
^`

 


 
$
\&



patient for social anxiety disorder, which outcome

  
>}|&
#

$
  

would indicate that therapy has achieved the de-
to consult with the prescriber regarding continua- sired effect?
tion of which drug, if the patient has been taking a. Patient goes on errands without experiencing
the drug at home? palpitations, chest pain, dizziness, or fear of
a. Alprazolam losing control.
b. Buspirone b. Patient presents teaching to nursing students.
c. Duloxetine c. Patient rides in elevators without experiencing
d. Paroxetine an anxiety attack.
d. Patient touches people without fear of contami-
21. The nurse is caring for a patient who has been nation.
diagnosed with panic disorder. Teaching should
include which information?
^—
Z



 


-
a. Avoid strenuous exercise because it increases  
   

 
=|!{&?ƒ
anxiety. 
Z " 
b. Drug therapy helps the patient be more com- b. Flood
fortable with situations and places he or she has c. Rape
been avoiding. d. Tornado

#

 

  
" 
  

sleep habits.

{
  

 
]%^
  DOSE CALCULATION QUESTIONS

^^
#




$

 


 

^~
– 
=–{ ?
~

|}


 


a patient with panic attacks and depression has ordered. Available are 5-mg tablets. How many
admitted to attempting suicide in the past if the tablets should the nurse administer per dose?
patient were currently prescribed which drug?
a. Clomipramine
b. Clonazepam
^<
 


$
 
  
$
 @ 
= ?

c. Fluoxetine intravenous push for a 121-lb adult patient when



#  the recommended dose is 0.02-0.06 mg/kg?

6 23. The priority outcome when teaching a patient


about phenelzine prescribed for panic disorder CASE STUDY
with depression is that the patient will state the
a. actions needed to prevent constipation. A 58-year-old woman with a history of 70 pack-years
b. importance of avoiding aged cheese, meats, and of tobacco use is admitted with exacerbated chronic
   
 
  
=>}|&?
!
 


c. importance of exercise. very demanding. During care, the patient states that she
d. possible adverse sexual effects.   



 
 

 
{







$

 

tasks because of weakness and fatigue.


]
 


 

 ƒ

Copyright © 2016, Elsevier Inc. All Rights Reserved.


74 CHAPTER 36 Q
> 
+
{ 
{ 

%&$'* 
& 

2. The nurse tries nonpharmacologic interventions to 5. The patient complains that the drug is not working
relieve the patient’s anxiety without success. The 

  
{

$

 
 



  

 @ 
=  ?
`

†
 
$
 
=–{ ?
*


 

times a day. What should the nurse include in her respond?


teaching about this benzodiazepine drug?

6. The patient is referred for cognitive behavioral


3. The patient’s daughter comes to visit and ex- therapy to augment drug therapy. What are the
presses concern about her mother receiving a ben- principles of this therapy?
@ @
 
{
 
 




a history of alcohol abuse but has not had a drink


for several years. How should the nurse respond to
this information?

4. A consult is ordered, and the patient is diagnosed



\&

 
$

 


 @ 
=  ?

 

 

=–{ ?
~



 





this drug a good choice for this patient?

36

          

STUDY QUESTIONS a. Found in coffee, tea, soda, and energy


drinks
Matching b. The drug is not effective if injected or
inhaled
        c. A 50:50 mixture of dextroamphetamine
and levamphetamine
1. ___ Amphetamine d. Has stimulant effects but is used for bron-
2. ___ Dexmethylphenidate chodilation

†

‰  $  e. Approved for obesity, although not pre-
4. ___ Methamphetamine ferred treatment for the condition
5. ___ Methylphenidate f. The dosage is one-half the dosage of
6. ___ Methylxanthine methylphenidate
7. ___ Theophylline 
{ 
  
$

-
amines but the pharmacologic actions are
essentially the same

Copyright © 2016, Elsevier Inc. All Rights Reserved.


CHAPTER 36 Q
> 
+
{ 
{ 

%&$'* 
&  75

True or False š
]<

 


 
$
  
 


&
{   

 
 
  

(     


   
  after a right knee replacement. He has become
#)  $     agitated and is refusing all treatments, including
pain medications. He accuses the staff of trying to

<

#


$
 
$
%' hurt him. What should the nurse do?
 
 
=&*&?

  a. Consult the attending physician.

‡

#


 
 
$
  b. Administer doxapram.

]`

#


 
$

” 
>
 
 Œ





]]

#
 
^%†

$
 
$$
within hours.
develop. d. Restrain the patient so that he does not injure
12. ___ Patients doing well on stimulant drugs the surgical site.




{ 
  



safer and more effective. 19. A child who has been prescribed Metadate CD for
13. ___ Patients with an atypical form of the &*&

 
$
 

-
CYP2D6 metabolizing enzyme of cyto- cation. What is a logical nursing action?
chrome P450 need higher doses of the drug a. Crush the medication and mix it in a small
to be effective. amount of applesauce.

]‹

{ 
 $

 


b. Notify the prescriber because the medication
possible adverse effects in adults. must be taken whole.
c. Open the capsule and sprinkle the beads in a
CRITICAL THINKING, PRIORITIZATION, small amount of soft food, telling the child to
be careful not to chew the beads.
AND DELEGATION QUESTIONS d. Open the capsule and dissolve the contents in
liquid.

]
> 

 
=>+{?
 

-
 
 
$

 ƒ
={



^`

$%  


&*&


-
 
?
scribed Daytrana transdermal methylphenidate
a. ADHD
patch. He normally awakens at 7:30 AM, attends

>+{
 
 
school from 8:30 AM until 2:45 PM, and does his
c. Depression
homework as soon as he gets home from school.
d. Narcolepsy
He goes to bed at 9:00 PM. The nurse will teach
e. Diabetes
the parents that which is the best time to apply the
patch?
š
]—
!
 

 
 
$

% %
a. 5:30 AM





>+{
 

$

b. 8:00 AM
&*&
!




$
 -
c. 4:00 PM

$

$


 


d. 9:00 PM
would be a concern?
a. BP 120/84 mm Hg
21. The nurse teaches a parent to apply Daytrana
b. Heart rate 100 beats/min
transdermal methylphenidate patch to unbroken
c. Respirations 25/min
skin on which part of the child’s body?
d. Weight gain of 2 lb since last year
a. Arm
b. Back
17. A patient asks the nurse why her physician will not
c. Buttock
prescribe amphetamines to help her lose weight.
d. Hip
{
 
 





 

weight. The nurse’s response should be based on


22. A nurse’s aide is experiencing a headache aura.
what knowledge about amphetamines?
Caffeine often helps prevent a full-blown mi-
a. They are ineffective for weight loss.
 
{


 





b. They can cause hypotension and bradycardia.


most caffeine. What is the best response by the
c. They can cause physical and psychological
nurse?
dependence.
a. Cola, 12 oz
d. They can unmask latent bipolar disorder.

#
 
]^
@
c. Chocolate bar, 1.5 oz
d. Orange soda, 12 oz

Copyright © 2016, Elsevier Inc. All Rights Reserved.


76 CHAPTER 36 Q
> 
+
{ 
{ 

%&$'* 
& 

23. Research suggests that caffeine consumption dur- CASE STUDIES


ing human pregnancy is associated with
a. birth defects. Case Study 1
b. low birth weight.

 
 
 
 The school nurse cares for a seventh-grade child who
d. preterm birth. recently was diagnosed with ADHD, combined type. The
parents rejected the diagnosis previously because they do

^‹
€ 
=| ?


  
$

not like the idea of their child using medication. The child
22-year-old woman who works rotating shifts. 
  
 
= ?
]`

†


When teaching about this drug, the nurse should day. The parents speak with the nurse when they deliver
include which instruction? the medication and medication administration forms to the
a. Not taking with food because absorption of the school. They verbalize concern that they will be “drug-
drug will decrease ging” their son and ask why behavioral therapy would not
b. Orthostatic blood pressure precautions 
$
c. Taking immediately after waking because the
drug can cause insomnia 1. Based on current research, how will the nurse

ˆ 


$
$


$
 
explain the rationale for drug therapy?
contraceptives are used

6 25. When a child is prescribed guanfacine or clonidine


for ADHD, it is a priority to teach parents to report
to the prescriber if the child experiences what?

&$

 2. The nurse has been administering the second dose
b. Dizziness with position changes at 11:00 AM. The nurse is planning for the individ-
c. Weight gain of greater than 2 lb in 1 year  
 
 
=#Z|?


  

d. Pulse less than 70 beats/min  



  
{

 




not been gaining weight. The child admits that he





 
#

 

 
 

DOSE CALCULATION QUESTIONS that the parents give the child methylphenidate
at 6:30 AM. The child eats his breakfast at school.
26. The recommended initial dose of caffeine citrate The last dose is administered at 5:00 PM with din-
=> $?
$
  


^`
 '

- ner. What changes can be made to help improve
 

†`
 
> $$
 
=> $?
the appetite of this child?
60 mg has been prescribed for a neonate who
 
—

]`
@
#

 
$ƒ

^~
> $$
 
=> $?



 

$
^`
 '‰
 


$ 
 



†
€ 
 


#Z|

 
 


per minute if 60 mg is to be administered over 30 child’s behavior has improved since starting




$ 

—`
 '‰ƒ medication. However, his organization and study
skills are still very weak. Based on the expected
therapeutic response to methylphenidate, what
information can the nurse provide?


‹
!
$%
=]`•]%]]•``
AM?
 
 

that the child is very sleepy in his class. What


could be happening to cause this sleepiness?

Copyright © 2016, Elsevier Inc. All Rights Reserved.


CHAPTER 37 Q
&
  

– 
>   77


{ 

 

‚
 


Case Study 2
see the nurse, stating that the pediatrician has rec-
ommended that their son be switched to Metadate A 25-year-old woman is seeking medical help for amphet-
CD 20 mg once a day. The parents are concerned 

{

  
 
$

because “This is a newer drug, and they don’t  



 
 


 
]``
 
{



know enough about it.” How should the nurse 


$
—

 

 
]†`
 
{

 -
respond? ing amphetamines after she reached her goal weight and
now wants to stop taking the drug, but she says she cannot
do it alone. When she stops taking the amphetamines, she
experiences withdrawal symptoms and becomes fright-
ened, so she continues to use the drug.

6. The child does well at school on Metadate CD but 8. What signs and symptoms would you expect

$





the patient to exhibit when she is using amphet-
done and socially at after-school activities. The amines?
prescriber puts the child on Concerta. How does
the nurse explain the difference between Concerta
and Metadate CD?

‡
#

 

 

 


since the patient has previously experienced


withdrawal symptoms. What withdrawal symp-
toms should the nurse be alert for when a patient
is withdrawing from amphetamine use, and what
7. What teaching should be provided about adminis- nursing interventions can help the patient cope
tration of Concerta? with these symptoms?

37
Drug Abuse I: Basic Considerations

STUDY QUESTIONS 5. Higher doses of a drug are needed in


___________________ because the person has
Completion abused another drug, usually in the same class.

1. ___________________ is when a particular dose 6. ___________________ is when one drug can


elicits a less intense response. prevent withdrawal from another drug.

2. When a person is ______________ 7. ___________________ is the neurotransmitter


______________ on a drug, withdrawal syndrome involved in the reward circuit of the brain.
will develop if the drug is stopped.

<
&


=?


3. ______________ ______________ is the intense ___________________ illness.


subjective need for a drug.

4. When a person experiences ______________


______________, symptoms are often opposite of
the normal effects of the drug.

Copyright © 2016, Elsevier Inc. All Rights Reserved.


78 CHAPTER 37 Q
&
  

– 
>  

CRITICAL THINKING, PRIORITIZATION, 13. A patient has been prescribed hydrocodone 5 mg



 
``

=Š?



AND DELEGATION QUESTIONS



 
 

$ 
  
 
 -


$

 ƒ
9. Which statements are true regarding addictive

#
 


 ƒ
={

 
?

#
 





†
 

#

  
   

 


#
 





—
 
harm.

#
 



 



-

#

$
$
 

-
 


]^
 
tion in intensity of an unpleasant experience.

#

 
$

 ‚


14. When there are different laws on the state and


reward circuit.
federal levels regarding prescribing controlled

#


  
  
substances, which law takes precedence?

#

 
$

 

  
-
a. The most restrictive law
dent on a psychoactive substance.
b. The least restrictive law
c. The federal law
10. Psychological factors associated with tendencies
d. The state law
toward drug abuse would make it important for the
nurse to provide drug abuse prevention education
to children who have been diagnosed with which CASE STUDY
ƒ
={

 
?

%' 
 
!
 







 

=&*&? unit. Describe how the nurse’s expertise can be applied to


b. Anxiety address the following issues of drug abuse.
c. Cerebral palsy
d. Depression 1. Diagnosis and treatment of toxicity
e. Developmental delay

11. An example of addiction is when an individual ex-


periences withdrawal symptoms if he or she stops
taking which substance?
a. Amphetamines used to reduce symptoms of 2. Diagnosis and treatment of secondary medical
ADHD complications
b. OxyContin used for cancer pain
c. Phenobarbital used to prevent seizures
d. Tobacco used to reduce stress

12. Drugs that the nurse can administer and that have
the highest potential for abuse and dependence 3. Facilitating withdrawal

 


>
{  


which schedule?

{
#

{
##

{
###

{

‹
Z 

 

 


$


{
Š maintaining long-term abstinence

Copyright © 2016, Elsevier Inc. All Rights Reserved.


CHAPTER 38 Q
&
  
##•
 79

38
Drug Abuse II: Alcohol

STUDY QUESTIONS
]~

|
  

 Œ
 


death.
True or False
a. Acetaminophen
   
  T   F b. Antihypertensives
   c. Benzodiazepines

&  
1. ___ Pattern of drinking alcohol is more impor- 
+  
%  


tant than type of alcohol when evaluating =+{#&?


cardiovascular effects.
2. ___ Moderate alcohol consumption may de-
crease the risk of type 1 diabetes mellitus CRITICAL THINKING, PRIORITIZATION,
=!]&€? AND DELEGATION QUESTIONS
3. ___ Alcohol consumption speeds the develop-
ment of osteoporosis. 18. The nurse is caring for a patient with a history of
4. ___ A person with COPD should never drink chronic alcohol abuse. The patient is prescribed
alcohol because even 1-2 drinks can slow thiamin to prevent Wernicke’s encephalopathy.
respirations. The nurse should assess for which symptoms of



#


$


 
 

 ƒ
={

 
?
on a regular basis. a. Abnormal ocular movements
6. ___ Chronic alcohol drinkers develop hepatitis b. Confabulation
more often than cirrhosis. c. Confusion
7. ___ Diuresis that occurs with alcohol consump- 
# 


%


 %
tion occurs because of increased release of term memory


=&*? e. Nystagmus
8. ___ Alcohol enhances sexual desire and perfor-
mance. 19. Research on older adults suggests that consump-
9. ___ Moderate alcohol intake is a risk factor for tion of one alcoholic drink per day has been as-
several common cancers. sociated with what?
10. ___ Valid research suggests drinking a maximum a. Atrophy of the cerebrum
of 1-2 drinks a week is safe during preg- 
‰

$
 % 


nancy. =*&‰ ?
11. ___ Alcohol is an effective treatment for insom- 
#

" 
$

nia. d. Preservation of cognitive functioning
12. ___ Drinking one alcoholic beverage with a meal
can promote gastric functioning. 20. Research suggests that alcohol consumption has
positive effects in which situations?
a. A woman who eats nutritious foods, exercises
Matching regularly, and has a drink in the evening 1-2
times a week
    
  
b. A man who has a drink with dinner 3-4 times a

week
c. A person who drinks 5-6 drinks 1-2 times a
13. ___ High alcohol intake counteracts effects.
month

]‹

# 
 
$
”

\#
 
d. A person who drinks 1-2 drinks to prevent

]

# 
 
$

  
hypothermia when outside in the cold

]—

# 
>+{
 

Copyright © 2016, Elsevier Inc. All Rights Reserved.


80 CHAPTER 38 Q
&
  
##•


š
^]

 




 

$
†`
27. A patient is admitted with a fractured hip and
years is admitted with constant, severe midepi- scheduled for surgery. Why is it important to in-

 
  


 
#



form the surgeon of the patient’s history of alcohol
priority for the nurse to report which laboratory  

 
$
 
=Š ?

 


test result to the prescriber? abstinence?



 
``
  
 '‰ 
!


   

 



–+|
††
 '‰ patient should not receive.

– 
]
 '‰ b. The patient is at risk for addiction to opioids

|
—
'‰ used for postoperative pain.
c. The patient should not receive any opioid anal-
22. For the average person with normal liver func- gesics for postoperative pain.
tioning, alcohol levels in the blood will begin to 
ˆ  
 
$

   
$
  -
increase if a person consumes which drink in 1 tive pain may be ineffective.
ƒ
={

 
?

}
‰
#  


^<


 
$

”




b. One 1.5-oz shot of whiskey administered only once a month?


c. One 8-oz glass of wine 
#



 $%$
d. One 24-oz glass of beer 
#

%  
e. One 8-oz wine cooler 
#


 
$

 
d. This prevents adverse effects.
23. Chronic alcohol consumption does not produce
tolerance to which condition? 6 29. Which teaching would be of greatest priority when
a. Activation of the reward circuit
 

  
  
=>  ?ƒ
b. Decreased alertness a. The importance of seeking help if thoughts of

& 
 self-harm occur
d. Respiratory depression b. That the full effects do not occur for about a
week
24. Which benzodiazepine regimen has been shown

! 

 
 
 


 


to protect against seizures and breakthrough


the diet if diarrhea occurs
symptoms of alcohol withdrawal while promoting
d. The importance of including psychosocial sup-
speedier withdrawal?
port in therapy
a. Administered as needed in response to symp-
toms
30. Dietary counseling for a patient with alcohol abuse
b. Administered as needed in response to symp-
should include adequate servings of what type of
toms in combination with another drug to
$


 
ƒ
reduce withdrawal symptoms
a. Citrus fruits

 








\
 $
  
schedule

‰%$ 

d. Administered around the clock in declining
d. Pork products
 





25. Which symptoms suggest the most serious effect DOSE CALCULATION QUESTIONS


 


$
 
  

=  ?



 

$

†]
&  
=  ?
]^

 




beer? prescribed for a patient motivated to abstain from


a. Dizziness and palpitations alcohol. Available are 250-mg tablets. The nurse
b. Flushing and sweating will teach the patient to self-administer how many
c. Nausea and vomiting tablets per dose?
d. Thirst and headache

†^
& @ 
=Š ?
]`

 
 


^—
&  
=  ?
 
 

$

prescribed for a patient in alcohol withdrawal.


patient is exposed to alcohol and takes which
  


 '‰


 

†

antimicrobial medication?
minutes. To make it easier to inject the drug over

@
=¢ ?
this period of time, the nurse decides to dilute the

\ 
=\  ?



 
$
†
‰
*





€ @
=… ?
the nurse add to the drug solution in the syringe to

>$ @
=$?


 
$
†
‰ƒ

Copyright © 2016, Elsevier Inc. All Rights Reserved.


CHAPTER 38 Q
&
  
##•
 81

CASE STUDIES Case Study 2


{%

 
 
” 
 
 

Case Study 1 




 
  
 
 
=Z&?

The nurse is caring for a 45-year-old man with a history and are having a postconference. They are discussing a
of alcohol abuse and cirrhosis of the liver who is newly 21-year-old male patient who was brought in during their
admitted with gastrointestinal bleeding from ulcers. $


   


Z&
$

 

of alcohol. A fellow college student accompanying the pa-


1. What would be appropriate nursing diagnoses and tient reported that he had drunk four shots in 15 minutes
related assessments for this patient? while playing a drinking game. The students had not eaten
anything for more than 3 hours before starting the game.

6. The nursing students identify which priority nurs-


ing problem for this patient at this time?
2. The patient’s blood alcohol level is 0.320%. a. Airway clearance
> @
=‰ ?



]``
b. Anxiety


{!!
$

 
 


 

 

route with the dose decreasing every 3 days. What 
{ 
 
 
is the rationale for this treatment regimen?

~
!
 
$

 



Z&
!

patient’s father asked how this could happen. He


 
 


 

 










 
$ 


3. What would be an appropriate nursing outcome the nurse identify as having contributed to the
for this patient relating to the reason for adminis- patient’s extreme intoxication?

 @
=‰ ?ƒ

8. Why would discussion about alcohol’s effects on


the heart, liver, stomach, and kidneys not be likely
4. The patient has been admitted once before for to motivate this patient to abstain from alcohol?
 
*

   

 



  
 
$
 @
=‰ ?

Based on the believed effects of alcohol on the


>+{



 



-
hol abuse? 9. Developmentally, discussion of what adverse ef-
fects of alcohol might motivate this young man to
drink responsibly?


$
 

 
 


-
 

"
*

$
  
=  ?

to help him refrain from drinking. What teaching


should the nurse provide to the patient and family
about this drug?

Copyright © 2016, Elsevier Inc. All Rights Reserved.


82 CHAPTER 39 Q
&
  
###•
+

{

39
Drug Abuse III: Nicotine and Smoking

STUDY QUESTIONS
]—

Z% 

$$




smoking cessation.
True or False 17. ___ The most troubling side effects of Vareni-

£> 
> ¤


   

   


  T   F erratic behavior, and suicidality.
  

1. ___ Tobacco is a product that, when used exactly CRITICAL THINKING, PRIORITIZATION,
as directed, kills adults and children. AND DELEGATION QUESTIONS
2. ___ Medical costs from smoking are greater than
nonmedical costs such as lost time at work 18. A 9-month-old child is admitted to the emergency
and disability. department after eating a cigarette. Which as-

†

!
 
$



ˆ{

 



$
 




the 21st century is approximately 30%. nurse?


4. ___ Tobacco smoke contains nicotine, carbon a. Apical pulse 145 beats/min
monoxide, hydrogen cyanide, ammonia, b. Blood pressure 66/48 mm Hg
nitrosamines, and tar. c. Respirations 15/min
5. ___ Nicotine in cigar smoke is absorbed primar- d. Temperature 37° C
ily from the mouth.

—

#
 
 


$

19. Which nicotine product produces the most similar
nicotine from inhaled smoke to reach the effect on the pleasure system as tobacco use?
brain of a fetus. a. Chewing gum
7. ___ Nicotine elevates BP and heart rate in new 
‰@ 
and chronic smokers. c. Nasal spray
8. ___ Nicotine slows gastric motility, which is d. Transdermal patch
why new smokers often vomit.
9. ___ The effects of nicotine on the pleasure 20. Which statement, if made by a patient who is plan-
system are mild compared to the effects of ning to use nicotine gum for smoking cessation,
cocaine and amphetamines. suggests the need for more teaching?
10. ___ Nicotine replacement is safer than tobacco a. “Chewing releases the nicotine.”
smoke during pregnancy. 
’#
 


$

 

“
11. ___ Research suggests that gradual reduction in 
’#



$
 


 

tobacco use prolongs withdrawal symptoms. stomach.”


12. ___ Nicotine is the active ingredient in some 
’#




“
insecticides.
13. ___ The risk of COPD and death from myocar- 21. When using a transdermal patch, what should the
dial infarction equals that of those who have person attempting smoking cessation be instructed
never smoked 20 years after quitting. to do?
14. ___ Health care providers can predict which a. Change the patch daily following brand instruc-
smoking cessation product will be best for a tions.
particular patient. b. Change the site of application once a week.
15. ___ Counseling increases the chance that smok- c. Report mild redness and itching at the patch
ing cessation drugs will be effective. site if present when the patch is removed.

{ 


$


 

Copyright © 2016, Elsevier Inc. All Rights Reserved.


CHAPTER 40 Q
&
  
#Š•
€ ”
&
$
  
}
! 


+ 83

22. The nurse is teaching a patient how to use a nico- 25. Why should e-cigarettes not be used for smoking
tine nasal spray. The nurse teaches the patient to  ƒ
={

 
?

 ƒ
={

 
? a. The doses of nicotine vary.
a. Administer 2 sprays per dose. b. Products of known safety are available.
b. Administer up to 5 doses per hour. c. Their vapor contains contaminants.
c. Administer up to 60 doses per day. d. They are not regulated by the FDA.
d. Direct the spray away from the nasal septum. e. They are proven to be ineffective.

ˆ 

 
 

 

 

23. The nurse will withhold which smoking cessa- DOSE CALCULATION QUESTIONS
tion drug and contact the prescriber if the patient
exhibits seizure activity?
^—
–
=¢ ?
‹`



  


–
=¢ 
 ? –
=¢ ?
]`



  
*


+
 
=+
+
>˜? many tablets should the nurse administer?

+
@ 
=>?

+
=
| ?

^~
Š 
=> ?
``


  
$

6 24. Which is the greatest teaching priority for a patient 


 
†
 
$

 
 




  
 
=> ?ƒ Available are 0.5-mg tablets. How many tablets
a. How to manage dry-mouth symptoms should the patient self-administer?

!
 
 





 
$

constipation
c. That the drug will decrease the pleasurable ef-
fect of nicotine from tobacco use
d. To report thoughts of self-harm

40
Drug Abuse IV: Major Drugs of Abuse Other Than Alcohol and Nicotine

STUDY QUESTIONS
‹


 



$

 



at greater risk for respiratory depression than


True or False a person who has used the heroin for years.
5. ___ Research suggests that new OxyContin OP
   
  T   F tablets are less likely to be abused than the
   older OC formulation.
6. ___ Meperidine causes less miosis than most
1. ___ Nurses should teach hospitalized patients other abused opioids.
who are prescribed opioids for pain relief to 7. ___ Opioid drug abusers must be hospitalized
not take the drug unless the pain is unbear- during withdrawal because opioid with-
able, to prevent the risk of addiction to the drawal syndrome can be life-threatening.
opioid. 8. ___ A patient who is stable after receiving a dose
2. ___ Nurses are at greater risk for opioid abuse $
 
=+  ?
$

$% 

than teachers, engineers, or architects. overdose of heroin is still at risk for fatal
3. ___ Heroin is the most commonly abused opioid respiratory depression.
among street users. 9. ___ Buprenorphine is only available through


 
   

Copyright © 2016, Elsevier Inc. All Rights Reserved.


84 CHAPTER 40 Q
&
  
#Š•
€ ”
&
$
  
}
! 


+

10. ___ Naltrexone is a drug that blocks the desired 6 26. A patient who has a history of heroin abuse is
effects of opioids that can be administered as  


 
 
=Z&?
-
a depot injection once a month. sponsive with pinpoint pupils. The nurse assesses
11. ___ Methadone therapy should not be offered  

 


  
#


to a patient who admits that he or she is not to supporting respirations, the patient is prescribed
 

 


  
=#Š?
 
*>
=+  ?
!

]^

{
  
$




nurse notes an improvement in level of conscious-
to cause severe symptoms and possible ness, respiratory rate, and effort within minutes of
death than sudden withdrawal of barbitu- administration of naloxone. What is the priority
rates. action by the nurse 30-40 minutes after adminis-
13. ___ A person who has required increased doses tration of the drug?
of barbiturates will also need higher doses of a. Assessing the patient for withdrawal symptoms
morphine in order to achieve adequate pain 
#$

 

relief after surgery. c. Monitoring of pulse and blood pressure
14. ___ During surgery, a person who has developed d. Reassessing for respiratory depression
tolerance to barbiturates might be awake
and experiencing pain despite anesthesia but š
^~




 
 
 


be unable to speak or move due to muscle the nurse that the dose of methadone prescribed to
paralysis. minimize withdrawal syndrome is inadequate?
15. ___ Naloxone can reverse the physical effects of 
!
 

$
 


-
opioids, barbiturates, cocaine, and metham- ing.
phetamines. b. The patient is experiencing daily vomiting and
16. ___ Research suggests that psychological depen- diarrhea.
dence on methamphetamines is greater than c. The patient is exhibiting fatigue throughout the
physical dependence. day.
17. ___ Marijuana has several approved medical d. The patient states that the dose is inadequate.
uses.
18. ___ Marijuana increases the incidence of psy- š
^<

 
$
 


 



chotic episodes in people diagnosed  not been prescribed clonidine to assist with with-
diagnosed with schizophrenia.   
$



=}>?

]‡

>
 
$
‰{&

 

would indicate a need for further teaching?
physical dependence. 
’#‚
 
 
#
‚
$

#


 “

^`

‰{&
 
  
 
  
 - 
’#



 $


”


$


bances. too quickly.”

^]

& 
 

‰{&

 

’#

 

 
 
$
#
 


related to the effect on the heart. drug.”


22. ___ Dissociative drugs were developed for use in d. “The medication should help prevent vomiting
surgery. and diarrhea.”

^†

|
=|>|?
 

 

contraction and cause hypothermia. 29. A patient has been prescribed sublingual
24. ___ There is no known effective treatment for % 
={ ?
$
 %

PCP overdose. management of heroin addiction. What would oc-


cur if the patient crushed, dissolved, and injected


#Šƒ
a. An overdose
CRITICAL THINKING, PRIORITIZATION, b. Heroin-induced euphoria
AND DELEGATION QUESTIONS c. Respiratory depression
d. Withdrawal symptoms
6 25. What is the priority concern when caring for an
opioid addict who is experiencing withdrawal 30. What has been associated with maintenance
syndrome? therapy with methadone?

> 

 
=>+{?
  
Z 
 
b. Preventing respiratory arrest 
# 

$
  



c. Relapse to opioid use 
‰ 
$




!
$
 
$
 $
=? 
+
  
 

 

Copyright © 2016, Elsevier Inc. All Rights Reserved.


CHAPTER 40 Q
&
  
#Š•
€ ”
&
$
  
}
! 


+ 85

6 31. A patient with a suspected barbiturate overdose is š


†~

 


 


 -
 


Z&
  


tient has taken excessive amounts of amyl nitrate?
pupils. What is the nursing intervention of greatest a. Confusion
priority? b. Constipation
a. Administering phenobarbital 
…


b. Maintaining respirations 
{ $
c. Preventing withdrawal symptoms

{ 

 

†<

 
 

  
 

]‹% %

 
 
 
 
=$$?
 
 

š
†^

 


 
$
 
remover?
which outcome would best indicate that therapy a. BP 135/82 mm Hg
has been successful? b. P 48 beats/min
a. Blood pressure 110/70-145/90 mm Hg c. R 16/min
b. No adventitious lung sounds 
!
]`^^„
…
=†‡„
>?

{ 

 
d. Temperature 98° F to 99° F
DOSE CALCULATION QUESTIONS
š
††
!
 

 
$

 
 
 -



!
‡‡‹„
…
–|
]~<']`^

39. A patient with known opioid addiction is pre-
Hg, P 110 beats/min, respirations 25/min, rhinor- scribed buprenorphine/naloxone, 4 mg/1 mg
rhea, and coryza. The patient admits to a history sublingual. Available are buprenorphine/naloxone
of amphetamine abuse. Which drug should not be 2 mg/0.5 mg sublingual tablets. How many tablets
administered to this patient? should be administered?

> 
=>%!?

& 
=– ?

\ $ 
=* ?
‹`
& 
=& ?
]
 '

  
$


| 
={ $? an 80-kg patient experiencing malignant hyper-
thermia associated with MDMA use. The drug is
34. A patient has been admitted with adverse effects   

^`
 '‰
!


  


$
 
€&€
=  ?

 

 


 

^
 
*



  
 

  
 



 
 ƒ
regarding possible administration of dantrolene?

Z


$ 
b. Hallucinations and delusions CASE STUDY

{ 
”

 
 

{ 
 

  At an outpatient substance abuse support group, a new
 
 


$

 
{
 
-
35. Which condition would increase the risk of serious ing in high school and by tenth grade was getting drunk
adverse effects if the patient used ecstasy?  



{
 



 

a. Anxiety 

$
 
  
{

 
 
 @ 



' 
  =Š ?



{


 
  


c. Anorexia the mornings, so she would have a beer or two and some
d. Obsessive-compulsive disorder  

’ 
 $
 “
{

 

take a few aspirins for the headache and go to work feel-


36. Which are potential effects of high doses of ing sick. As a rule, she would leave work at 5:00 PM, stop
dextromethorphan found in cough medications? by the liquor store, and then go home to watch TV, drink,
={

 
? and take a few diazepam to calm down. After a few years
a. Disorientation $
 

 


 
 
{

 


Z to do her job because she was either high or was in need
c. Hallucinations $






{
 


 


d. Paranoia $


 
$

 
{



{  get into violent arguments with her husband. He physi-
 
 



 
$


 




‚

{



 

 

times over the years, but could only stop for a few days

Copyright © 2016, Elsevier Inc. All Rights Reserved.


86 CHAPTER 41 Q Diuretics

$

 

$

{
 
 

- 2. What assessments should the nurse perform to as-
port her habit. After being arrested for stealing, she was sess for drug withdrawal?
required to enter a treatment program.

1. How has tolerance and physical dependence con-


tributed to this person’s drug abuse? 3. What interventions might the nurse employ to
decrease the adverse effects of withdrawal?

41
Diuretics

STUDY QUESTIONS Completion

Matching
]
!


 



    $
 
 
 

]

€ 
  
  
$

 - 16. The normal amount of urine excreted in 24 hours
tion 

‰
2. ___ Moves passively following osmotic gradient
3. ___ Filtered then reabsorbed by active transport 17. Most diuretics work by blocking
4. ___ Nonselective process that does not regulate ____________________ and
urine composition ____________________ reabsorption.



ˆ 


$
 



organic bases for active transport 18. Diuretics that produce the most sig-
 
 
$$
 



a. Filtration ____________________ ____________________


b. Reabsorption of the ____________________
c. Active tubular secretion ____________________ of the

{ ____________________ ____________________
e. Water ____________________.

True or False 19. Diuretics can affect blood levels of


___________________, ___________________,
   
  T   F ___________________, ___________________,
   ___________________, ___________________,
This food is a good source of potassium when a patient is ___________________, ___________________,
receiving a potassium-excreting diuretic. ___________________, ___________________,
and ___________________.
6. ___ Bananas
7. ___ Beans
8. ___ Cheese
9. ___ Red meat
10. ___ Oranges
11. ___ Pork
12. ___ Raisins

]†

{ 
14. ___ Yogurt

Copyright © 2016, Elsevier Inc. All Rights Reserved.


CHAPTER 41 Q Diuretics 87

CRITICAL THINKING, PRIORITIZATION, 6 26. The nurse takes orthostatic BP readings before
AND DELEGATION QUESTIONS  




$
  
#

would be a priority to teach orthostatic BP precau-


20. Antidiuretic hormone affects kidney reabsorption tions to a patient with which orthostatic blood
of which substance? pressure readings?

\  
]`'‡`

*
 Œ
]^'~`

*

b. Potassium 
]‹`'<^

*
 Œ
]^'~^

*


{ 
]†`'‡`

*
 Œ
]]<'~<

*

d. Water 
]]—'~`

*
 Œ
]]`'—<

*


š
^]


$
 
 
 
]
 

27. Which laboratory test results should the nurse
 

 
$ 
=*…?

 
]~—

monitor when a patient is prescribed a high-ceiling
has been prescribed a high-ceiling diuretic with ƒ
={

 
?

 
$

$
]```

$
 


Z
output in the next 24 hours. The next morning, 
\ 
if this goal is met, the nurse would expect the 
‰&‰'*&‰
patient’s weight to be what? 
ˆ

a. 170 lb e. WBC
b. 172 lb
c. 174 lb 6
^<

 


  
$ 
=‰ ?

d. 175 lb Which symptom is a priority for the nurse to


monitor that suggests possible excessive loss of
22. A patient in chronic renal failure has been retain- potassium?


 
   



a. Hunger and fatigue
the nurse expect to administer? 
‹«



a. Bumetanide c. Muscle weakness and cramping

* @
=}
*&#ˆ#‰? 
! 

!
 

Z>\

€ @
=¢ ?

{ 
= ? 29. Which over-the-counter medication can counteract
the effects of diuretics?
23. Which food is a good source of potassium when a a. Acetaminophen
patient is prescribed a potassium-wasting diuretic? 
# $
={

 
? 
#
a. Baked potato d. Multiple vitamins
b. Cantaloupe
c. Pork 30. What is the recommended diuretic for initial
d. Raisins therapy of essential hypertension?

{  a. High-ceiling diuretic
b. Osmotic
š
^‹

 

$% 



  

c. Potassium-sparing diuretic
high-ceiling diuretic. Which assessment most ac- d. Thiazide
 


 
$$
$

 ƒ
a. Drop in systolic blood pressure of 10 mm Hg 31. Which symptoms suggest ototoxicity when a
b. Normal heart sounds patient has been prescribed a high-ceiling diuretic?
c. Clear lung sounds a. Photophobia
d. Pulse 80 and regular b. Telangiectasia
c. Tinnitus
š
^

 



  






^
 

=!^&€?


-
 
$ 
=‰ ?
^`




š
†^
!
 




 
$

 

a day. Based on the effect of furosemide on the who has been prescribed lithium carbonate for
patient’s T2DM, it is important for the nurse to bipolar disorder and furosemide for heart failure
assess this patient for which symptoms? =*…?

 



 
$
ƒ

# 
    

 
  
>
]``
Z"'‰
b. Diaphoresis, shakiness, and tachycardia 
€  
]<
Z"'‰

# 
 Œ
$ Œ



 
| 
‹]
Z"'‰
d. Nausea, vomiting, and diarrhea 
{
]^<
Z"'‰

Copyright © 2016, Elsevier Inc. All Rights Reserved.


88 CHAPTER 41 Q Diuretics

33. Which new symptom suggests hypomagnesemia 6


†‡
#



$

 

" 

 

when a patient is prescribed a thiazide or high- about which home feature when a patient has been
ceiling diuretic? prescribed a diuretic?
a. Confusion a. Central air
b. Muscle weakness and tremor b. Forced air furnace

{ 
–|

$
^`

* c. Water softener

{

 $

 d. Well or city water

†‹

 


  
   
=& ?

does not mention that she is taking Yasmin, an oral DOSE CALCULATION QUESTIONS
contraceptive that contains the potassium-saving
 
 
{




Z&

‹`
… 
=‰ ?
‹`

 


 



 
 

Z>\

  
 
  
  

$ 
=‰ ?
<`%

the combination of drugs is causing hyperkalemia? tablets. How many tablets will the nurse adminis-

| 
{!
  ter?
b. Tall, tented T waves

& 
{!
 

Z
|
  41. The drug handbook states that 40 mg of furose-

=‰ ?
 

 



š
†

  
 





]%^
 Œ



‹
 '

 

prescriber if a diabetic patient is prescribed hydro- with renal impairment. The nurse is unsure of the
chlorothiazide 50 mg once a day and metformin/ patient’s renal status and plans to administer the
 
=\ ?
``
 '^





]`


 '?
!
$ 

day? =‰ ?
 
 
‹`
 '‹
‰
!
%‰
 


…–{
‡`
 '‰ 
 




`^
‰
!
 


b. A1c 8.2%  


`^

=
?
$
$ 
=‰ ?


| 
‹
Z"'‰ over what time period?

{
]†—
 '‰

6 36. A 20-year-old patient has been prescribed spirono-


 
= ?
$
 
 
 
CASE STUDY

=|}{?
& 
 



a priority nursing diagnosis? An 80-year-old nonsmoking woman has been taking


a. Activity intolerance related to loss of sodium  
= ?
]``
 ' 
$

—

b. Altered body image related to hirsutism and years to control her moderate hypertension and mild HF.
deepening of the voice {



 
 

  


&  

 

 
  
=  ?





 




d. Fluid volume excess related to excessive salt  


$
]‡`']^`

* 
{


$
 


intake  
  
=*
]†‹
  '?

 -
retic. Her family assures you that she has been taking her
37. The nurse is preparing to administer mannitol medication. The family tells you she has been getting
=} ?


 
!
 

  

worse over the past 2 weeks since having friends bring

 

 


 

 ƒ her favorite lunch of hot dogs and potato chips every day.
a. Administer the solution. | 


  
}

{!!

b. Discard the solution.  


 
$ 
=‰ ?
‹`





ˆ 


 $ 
^
 
†
‰
$
 

 
  
-
d. Warm the solution to dissolve the crystals. lytes, and complete blood count, and an indwelling Foley
catheter.
38. Because of the risk of hyperkalemia, the nurse
consults the prescriber and monitors potassium
]
*

$ 
=‰ ?




levels if a patient is prescribed spironolactone and patient’s symptoms?






$ƒ
={

 
?

% 
b. -olol
c. -pril
d. -sartan
e. -statin

Copyright © 2016, Elsevier Inc. All Rights Reserved.


CHAPTER 41 Q Diuretics 89

2. What information must the nurse know about furo- 7. What nursing assessments would be made on at

=‰ ?
  
$
 - least a daily basis while she is in the hospital?
ing the drug via intravenous push?

8. The patient continues to improve and is being


3. Why was the Foley catheter ordered?    


$ 
=‰ ?
^`




 

 
= ?
`


daily. What teaching should be provided regarding


dietary patterns, activity, and signs and symptoms
of further problems?

‹
#


‹



 



$ 
=‰ ?
*
…
 



 
{

 



 
 
-
sessments should be made at this point?
9. What information should the nurse provide to
prevent problems with drug interactions?


‰
  
 
$



-
 
 

*


]‹`
= •

]†%]‹
Z"'‰?


]]`
= •
‡<% 10. The patient will need to be started on digoxin.
]`—
Z"'‰?

 

†
= •
†%
What must the nurse remember now that the pa-
Z"'‰?

 



$ - 


 

$ 
=‰ ?ƒ
 


=‰ ?
$
 

 

 
laboratory values need to be monitored?
what should be included in the nurse’s plan of
action?

—
!
 

  


 

6 hours. Her blood pressure is 138/90 mm Hg,


pulse 102, respirations 20, and rales are heard only



  
{





could have happened since she had been taking


her medicine as prescribed. What should the nurse
discuss with the patient as possible reasons for her
condition?

Copyright © 2016, Elsevier Inc. All Rights Reserved.


90 CHAPTER 42 Q
 
$$

Š

#
>
$
–
…

42
Agents Affecting the Volume and Ion Content of Body Fluids

STUDY QUESTIONS CRITICAL THINKING, PRIORITIZATION,


AND DELEGATION QUESTIONS
Completion
1. Cirrhosis of the liver can cause ______________ 6
]†

$ 

 
=#Š?



______________. cases of isotonic contraction, it is a priority for the


nurse to do what?
2. Diuretics and chronic kidney disease can cause 
 
$
 



 
______________ ______________. b. Calculate the infusion rate in drops per minute
and hang by gravity.

†
Z 

 
  


#$ 


"
______________. 
#$ 
`‹œ



4. Vomiting and diarrhea can cause ______________ 14. Why are intravenous solutions of 5% dextrose
______________. =>6H12O6?

 
 
 



but hypotonic in the body?


5. Total osmolality of plasma is equal to approxi- a. The body uses the dextrose for energy.
mately ______________ ______________ b. The kidneys rapidly excrete the dextrose.
______________ ______________. c. Dextrose is a nonabsorbable sugar.
d. The dextrose is metabolized in the blood to
6. NaCl plus KCl to increase excretion of bicar- H2O and CO2.
bonate is used to treat ______________
______________. š
]

]% %
 



 

laxatives is admitted to the hospital after experi-


7. ______________ ______________treatment encing extreme leg weakness. Potassium chloride
includes rebreathing CO2. ‹`
Z"




 


  

The nurse reviews the most recent laboratory tests

<
{
   

 

 
before administering the drug. Results include
_____________ _______________. +
]†~
Z"'‰

†
Z"'‰

>
]``
Z"'‰

What should the nurse do?


Matching 
| 

  
{!!
b. Administer the medication.
          c. Hold the medication and contact the prescriber.
d. Administer the medication and contact the
9. ___ Metabolic acidosis prescriber.
10. ___ Respiratory acidosis
11. ___ Metabolic alkalosis
12. ___ Respiratory alkalosis

a. Can occur with salicylate toxicity.



‰
  
$



 
$

this condition.
c. May be associated with prolonged gastric
suctioning.

{


 
 

Copyright © 2016, Elsevier Inc. All Rights Reserved.


CHAPTER 42 Q
 
$$

Š

#
>
$
–
… 91

š
]—
!
 

 
$ 
=‰ ?

6 21. Which action is of greatest priority when a patient
  % 
 

=%&?

is diagnosed with hyperkalemia?
a patient with HF. The nurse notes that the patient a. Administer sodium polystyrene sulfonate.
is chewing the potassium chloride. The patient 
# 
  
 
states that she cannot swallow the large pill. What 
˜ 

 

 
 
should the nurse do? d. Teach foods that should be avoided because of
a. Crush the pill and mix it with applesauce or high potassium content.
pudding.

# 

 



 
<

$
22. Which condition might prevent oral magnesium
water after chewing the potassium pill. supplementation from being effective?
c. Contact the prescriber. a. The patient has impaired renal functioning.

| 

  
{!! b. The patient is a diabetic.
c. The patient has hypertension.

]~

 






d. The patient experiences diarrhea.
report to the prescriber when a patient is taking
KCl tablets? 6 23. The nurse reviews laboratory test results that
a. Black stool 

  

$
]
Z"'‰




b. Constipation 
 
#

$
  

$


c. Muscle weakness nurse to inform the prescriber of this result if the


d. Nausea patient has a history of which disorder?
a. Asthma
6 18. Which laboratory result would be a priority to b. AV heart block
report to the prescriber when a patient is receiving 
\    

  
=\Z&?
 

 
 ƒ d. Rheumatoid arthritis

–

 
=–ˆ+?
^<
 '‰

> 
‹^
 '‰ 24. Because of the risk of neuromuscular blockade

| 
†
Z"'‰ when administering magnesium, the nurse must

{
 
^^`
 '‰ 


  


 ƒ
a. Calcium gluconate
 19. A diabetic patient is admitted with hyperosmo- b. Potassium chloride
 
 

=**{?
*


{

 


~`
 '‰



*


{
 
$ 
~‹]
# 
  
 

 


=`‡œ
+ >?

  



 ‚


 

 


 
DOSE CALCULATION QUESTIONS
the nurse needs to assess the patient for which
symptoms of a possible electrolyte disorder that A 165-lb patient is admitted to the burn unit with burns
can be caused by this treatment? 
†—œ
$

 

$ 

=!–{?
–  


{ 
 
 



$
  

  

 


]œ


=  ? $
!–{
"

 

 
 



&  
   

@
=- cases of hypertonic contraction. This facility uses the
   ? |  
=– ?
$
$

 
>  -
c. Muscle weakness, hypotension, and sedation 
 

  
 ‚

‹
‰'
¬

=   ? œ!–{'^‹



d. Confusion, anxiety, heavy legs, and tall T
 

Z>\
=  ? 25. What is the total amount of lactated Ringer’s solu-

 



$ 


 

 ƒ
20. Which medication used to treat hypertension can
increase the risk of hyperkalemia if prescribed

 


 
ƒ
={

 

^—
* $
$





 



?  
<
 
 
 

‰'





=+ ?  
  



ƒ


=!?

… 
=‰ ?

 
= ?

{ 
= ?

Copyright © 2016, Elsevier Inc. All Rights Reserved.


92 CHAPTER 43 Q Review of Hemodynamics

CASE STUDIES Case Study 2


A postoperative patient has been NPO for 4 days with a
Case Study 1 nasogastric tube set to low intermittent suction. The pa-

 



 
  


- 

 
 

  
{


$
`‹œ



‹`
Z"
$
 
€

]`
Z"'‰
!
  




$



  

]`
‰' ]`œ
  
$ 

$ 

]``
‰'

1. What laboratory tests does the nurse need to moni-



 


 

$
 


tor when administering potassium supplements? therapy?

2. What assessments does the nurse need to perform


—
!
  
  

$ 
 

‡`
‰'
to ensure safe infusion of the potassium solution? hr. What critical assessment should the nurse per-
form while administering magnesium?

3. What measures should the nurse take to ensure ac-


 
$ 
$

  
 
$



~
 
 


  

 

electrolytes? stopping the infusion and consulting the pre-


scriber?

4. What assessments should the nurse perform to as-



$
  

 ƒ

43
Review of Hemodynamics

4. ___ Conditions that decrease chest expansion


STUDY QUESTIONS with breathing decrease blood return to the
heart.
True or False
5. ___ Conditions that cause an inability of skeletal
   
  T   F  

 
=

  ?
 

   cause peripheral edema.



—

+ 




<
‰
1. ___ Arteries readily stretch in response to pres-
~

{ 
$

 



 -
sure changes. ing down to defecate, speeds the heart rate.

^






  8. ___ The average amount of blood ejected from
3. ___ Cardiac output is determined by the rate of the heart at each beat is slightly over 2
the heart contraction times the amount of ounces.
blood ejected from the heart with each beat. 9. ___ The heart of a normal adult pumps the entire
blood volume in approximately 1 minute.

Copyright © 2016, Elsevier Inc. All Rights Reserved.


CHAPTER 43 Q Review of Hemodynamics 93

10. ___ The majority of the blood in the body is in 25. Drugs that can be used to lower BP decrease
the arteries and the heart. 
   


 ƒ
={

 

11. ___ Vasodilation increases resistance to blood ?


 a. Dilating veins
b. Reducing right atrial pressure
Matching c. Reducing volume of blood

{ 
 
 


   
12. ___ Afterload
^—

 

 
 
$ 
=*…?



]†

 
=%   %  
average heart rate of 90 beats/min and a stroke
 ? 
$

‰
 




 ‚

14. ___ Baroreceptors per-minute cardiac output?


15. ___ Beta2-adrenergic receptors 
#



16. ___ Cardiac output 
#


17. ___ C-natriuretic peptide 
#

 
18. ___ Preload 
#


 

19. ___ Muscarinic receptors
20. ___ Natriuretic peptides ANP and BNP š
^~
!
 

  

 
!
  

^]

{  
 states that it decreases afterload. Before the nurse

^^

{
 administers this medication to a patient, which as-

^†

{ 

  sessment would be most critical?
a. BP for hypotension
a. Amount of blood pumped out of the heart
b. Pulse for tachycardia
in 1 minute
c. Respirations for tachypnea
b. Constricts vessels and increases kidney
d. Temperature for fever
water retention
c. Force that returns blood to the heart
6 28. The nurse is assessing a patient who is receiving
d. Force with which the ventricles of the
medication for acute HF. Which assessment would
heart contract
be a priority to report to the prescriber?
e. The amount of stretch in the ventricle
a. Cough with frothy sputum
before it contracts

Z 
@
$



f. Pressure in the aorta that the heart must
c. Pulse 100 beats/min
overcome to eject blood out of the heart
d. Respirations 25/min
g. Pressure sensors in the aortic arch and
carotid sinus
š
^‡
!
 

  

 
]

$

h. Primarily promotes vasodilation


administering a vasodilator to treat hypertension.

{$

$
  

   

  


–|
]]—'~`

*

compartment, increase diuresis, dilate


=
$
]‹'<<

* ?
 
<<
=
$
<`

arterioles and veins


  '?

  
^^
=
$
^`?
 

j. Where stimulation of heartbeat originates




 
ƒ
={

 
?

{ 
$
 

 

a. Continue collecting additional assessment data.


heart rate

&

 

{ 
$
 

 


Z 
  
–|
  
heart rate

| 

  
{!!
e. Raise all four bed side rails for safety.
CRITICAL THINKING, PRIORITIZATION, f. Reassess the vital signs within 1 hour.
AND DELEGATION QUESTIONS

^‹
# 

 
 "




experience an increase in vessel resistance. The


nurse should assess for which reaction that is the
body’s attempt to compensate?
a. Peripheral edema

 


 
=–|?

{$
$

$
  
 
=|€#?

{
$
 
 

Copyright © 2016, Elsevier Inc. All Rights Reserved.


94 CHAPTER 44 Q
&



%  % 
{ 

š
†`

 



  
 

2. The patient verbalizes understanding of nitroglyc-
a high incidence of orthostatic hypotension. erin, but states that she is afraid to take off the
Before administering the medication and after patch because she might have a heart attack. What
the patient has rested supine for 10 minutes, the is the rationale for not using nitroglycerin patches
nurse assesses the BP and pulse. Results are BP continuously?
145/80 mm Hg, P 68 beats/min. The nurse assists a. Absorption is decreased if patches are left on
the patient to stand, ensuring safety, and after 1 too long.
minute reassesses the BP and pulse. Which read- 
{
 

$
 

$



ing would be of most concern to the nurse? long.


a. BP 110/70 mm Hg, pulse 92 beats/min c. Tolerance to the drug’s effect develops if
b. BP 126/70 mm Hg, pulse 78 beats/min patches are left on too long.
c. BP 134/70 mm Hg, pulse 72 beats/min d. Toxicity develops if patches are left on too
d. BP 140/82 mm Hg, pulse 70 beats/min long.

Case Study 2
CASE STUDIES
The student nurse is caring for a patient with HF. The
Case Study 1  

  

 


 
 


stretched beyond the point of effective recoil. The student


A patient newly diagnosed with coronary artery disease nurse must do a teaching project.
=>&?


  

 
 
]^


on and 12 hours off. The nurse is explaining the medica- 3. Describe how the student nurse could use a bal-
tion to the patient. Nitroglycerin is a drug that causes loon to demonstrate the pathophysiology of HF to
extensive venous vasodilation. The drug handbook states the patient.
that the therapeutic effect is to decrease the workload of
the heart. The patient wants to know how this helps her
heart.

1. How should the nurse explain this effect?

44
Drugs Acting on the Renin-Angiotensin-Aldosterone System

STUDY QUESTIONS 3. ___ Drug that acts on renin to inhibit the


conversion of angiotensinogen into angio-
Matching  
#
 

%   %
 
 
={?

]

>  @

 
$
  
#
4. ___ Drug that blocks the action of angiotensin
= ?

  
##
= 
? ##
  
 
$

 


2. ___ Drug that dilates blood vessels by decreas- potassium-sparing diuresis, but does not


$
  
##
$ 
$
cause cough or prevent myocardial remodel-
  
#
 
 

ing.
of sodium and water, slightly decreasing 5. ___ Drug that produces selective blockade of
heart remodeling, and causing dry cough by aldosterone receptors causing potassium-
increasing bradykinin levels.  
 

  


of myocardial remodeling.

Copyright © 2016, Elsevier Inc. All Rights Reserved.


CHAPTER 44 Q
&



%  % 
{  95

6. ___ Hormone that causes retention of sodium 13. What would not stimulate renin release by the
and water and retention of potassium and kidneys?
hydrogen by the kidneys to maintain ad- a. Dehydration
" 

 


  b. Hemorrhage

~

{


  
  
##
c. Hypernatremia
formation. 
{ 
$
 1-adrenergic receptors

<

{ 



 
$
-
neys, resulting in aldosterone release from
]‹

 
$$

 
$

adrenals. bradykinin, may a patient may experience from


9. ___ Vasoconstricts and stimulates aldosterone  

>Z
 ƒ
release. a. Dehydration
b. Dry cough
a. Aldosterone c. Hyperkalemia
b. Aldosterone receptor antagonist d. Hyponatremia

  
##

  %
@
= 
š
]


 



 
 -
##? pril to a patient, which laboratory values should be

  %
@
=>Z?
ƒ
={

 
?
inhibitor a. Creatinine
$
  
##


=–? b. Fasting blood sugar

&

 
=&#? 
{

{ d. Potassium
i. Renin 
ˆ


6
]—
#





 
  
–|

CRITICAL THINKING, PRIORITIZATION,  




 
$

 
 
$


AND DELEGATION QUESTIONS >Z


 
$
  
 



result?
10. The nurse recognizes that the generic names of 
‰!
†
#ˆ'‰
–




$ƒ 
–ˆ+

 '‰
a. -mycin 
| 
‹]
Z"'‰
b. -olol 
{
]†^
 '‰
c. -pril
d. -sartan 17. The nurse receives laboratory test results, includ-

{!
^^
  
 '‰
‰!
††
-
11. The nurse recognizes that the generic names of   
 '‰
–ˆ+
†^
 '‰

 

>Z
 




$ƒ ^^
 '‰
$

 




-
a. -mycin 

>Z
 
 

>Z
 

b. -olol could be administered without concern of toxicity?


c. -pril a. Captopril
d. -sartan 
Z  
c. Fosinopril
12. A nurse understands how drugs that block the ef- 
‰ 
$
$
  
##
 

 
$
 

failure patients. What information would not be 6


]<

 


  

>Z
 

correct about ARBs? complains of tongue swelling and is experiencing


a. The drug improves cardiac function primarily obvious dyspnea. What is the priority action by the
by decreasing electrical conduction through nurse?
cardiac tissue. a. Administer prescribed PRN epinephrine
b. The drug can help heart function by increasing b. Assess BP
excretion of sodium and water by the kidneys. c. Assess lung sounds
c. The drug can decrease the formation of patho- d. Consult the prescriber
logic changes in cardiac structure.
d. The drug can help heart function by preventing
pathologic changes to the structure and function
of blood vessels.

Copyright © 2016, Elsevier Inc. All Rights Reserved.


96 CHAPTER 44 Q
&



%  % 
{ 

š
]‡

$
 
Z>\


  
- 25. The nurse assesses a hypertensive, diabetic patient
 


 


  

>Z


  

=#  ?
$
 

inhibitor and who was using a salt substitute? effects, which may be related to electrolyte imbal-
a. Flat T waves ance. These symptoms include

| 
˜!
  
 



{
˜!
  b. diarrhea.
d. Tall, peaked T waves c. irritability.
d. weak hand grasps.

^`

 



 

>Z
 

for hypertension is concerned because she has just 6 26. When taking a history from a patient who is
discovered that she is 7 weeks pregnant. The nurse   

>Z
 
–



knows that research suggests that if the patient =#  ?



" 


 
 


 

>Z
 


 
$
for the nurse to ask?
adverse effects on the fetus is a. “Are you bothered by a frequent nonproductive
a. unknown. cough?”
b. low. b. “Do you eat 4-5 servings of fruits and vegeta-
c. medium. bles each day?”
d. high. c. “Have you ever been diagnosed with iron-

 ƒ“
6
^]


 

  

>Z
 

d. “What do you use to season your food?”
is a priority to teach the patient not to take which
OTC drug without consulting the prescriber? š
^~

 



  
" 
=-
a. Antacids ?
$
 


 



b. Cough suppressant reduction in blood pressure. Regular use of which



‰   over-the-counter medications may be contributing

+{#& 
 
$
 
$$
$

>Z
 ƒ
a. Acetaminophen

^^
–


 

>Z
 

b. Calcium carbonate
that they are also associated with a decreased 
# $
potential for which adverse effect? d. Milk of magnesia
a. Cancer
b. Cough
c. Fetal harm DOSE CALCULATION QUESTIONS
d. Renal failure

^<
 
=! ?

  

]`%

23. What is an advantage of the direct renin inhibitor tablets. How many tablets are administered for a
 
=! ?

– ƒ 300-mg dose?
a. Reduced incidence of cough

…
  
=\#?
 
$$
c. No drug interactions
^‡
Z  
=Š ?
]^


  

 


ˆ 
  


 
#

  


$
]^

 '‰
!
 
 
 
 




^‹
{ 
= ?



 
than the prescribed infusion time. The nurse

=#  ?
!
 

 


 
`]



 
 ƒ
means what about spironolactone?

#
  

 
$$ 

#

$

  

#


$$

#

 
$$

 

Copyright © 2016, Elsevier Inc. All Rights Reserved.


CHAPTER 45 Q Calcium Channel Blockers 97

CASE STUDY 3. What assessments and actions should the nurse


$

 

 
 
$
 -
A 68-year-old man was admitted to the medical unit 
=> ?ƒ
with complaints of a 10-lb weight gain over the past
week, swollen ankles, and increasing shortness of breath.
 
$ 
=‰ ?

  
$$


  
 



>Z
 
!
 


  
 
=> ?
^





times a day. His blood pressure is currently 140/74 mm 4. What teaching should the nurse provide to the
* 
{  
 



  
patient and family about reasons to contact the
infarction 12 months ago and renal artery stenosis involv- prescriber?
ing the right kidney.

1. What question should the nurse ask when con-


sulting the prescriber about this new medication
order?
5. The patient’s spouse has a friend who used to take
 
=> ?


$‚
  

 

>Z
 

  




   
=& ?
!
 ‚

spouse would like the patient to be prescribed


2. What teaching should the nurse provide about the valsartan because it is “newer and better.” How

$

 
$
 
=> ?ƒ should the nurse respond?

45
Calcium Channel Blockers

STUDY QUESTIONS

!
$
$


 
$


CCBs is _______________________.
Completion

]
> 
 

=>>– ?


CRITICAL THINKING, PRIORITIZATION,
pressure by _______________________. AND DELEGATION QUESTIONS
2. CCBs differ from nitrates in that they do not cause 6
—
#


 

 
 



vasodilation of _______________________. priority assessment before administering diltiazem


=> @?ƒ
3. Dihydropyridine CCBs primarily affect a. Ankle edema
_______________________. b. Apical pulse
c. Peripheral pulses
4. The three net effects of the CCBs verapamil and d. Respiratory rate
diltiazem on the heart are __________________,
__________________ __________________
__________________, and __________________
__________________ __________________.

Copyright © 2016, Elsevier Inc. All Rights Reserved.


98 CHAPTER 45 Q Calcium Channel Blockers

7. The lowering of blood pressure and improvement š


]^

  






$
 




 
 

would warrant consultation with the prescriber
negative effects of reduced heart rate and reduced  
  
$
 @
=> @?ƒ
$
$
 
  

  
=>  ? a. Burrows and erythematous papules between the
a. True  
b. False 
Z  
@
 





inside of elbows
8. The nurse is caring for a patient who is scheduled 
{
 
 "


 


  
=>  ?


=!- d. White, cheesy plaque on the tongue
?

 1%  


`‡``
#


important for the nurse to assess for what critical 13. The nurse is administering the Verelan PM form
adverse effects based on the combination of these of verapamil. The patient states she takes all of her


$
  ƒ
={

 
medications in the morning when she awakens.
? The nurse’s response should be based on what
a. Crackles or pulmonary edema knowledge?
b. Muscle pain a. Verelan PM produces maximal effects in the

{
$
  morning when most cardiac events occur.

ˆ 

$

 
‹
‰' b. Verelan PM is given at night so it peaks during
e. Weight gain of 3 lb in 24 hours sleep to prevent orthostatic hypotension.
c. Verelan PM should not be administered with

‡
!



 
=–|?


any other medication.
the CCB activates baroreceptors and stimulates d. Verelan PM should be taken at night because it
the sympathetic nervous system. Which of these causes sedation.
CCBs is most likely to have the adverse effect of

   ƒ 14. A patient on a surgical unit has been prescribed

& @
=> @?  
=#Š?
 @
=> @?
$
 


+$
=|  ?   
 


 
ƒ

Š  
=>  ? a. Move the resuscitation cart into the patient’s
room.
š
]`
!
 

 
$

 


- b. Arrange for the patient to be transferred to a



  
=>  
{?
]<`


unit where constant monitoring of BP and elec-

=!?
`


`‡``
!
 
trocardiogram is possible.

 


 
=\% ?
 


c. Continuously monitor the patient’s oxygen
are to be administered via the tube. Which action saturation and pulse during infusion.
should the nurse take before administering these d. Have another nurse check the calculated drip
two drugs? rate before hanging the medication by gravity
a. Change the time of the drug administration so feed.
 


 


\% 


least 2 hours apart.


]

 
 

  
=>  ?


b. Consult the prescriber.  


=‰ ?


 
 
$
  

c. Crush both tablets separately. toxicity. The nurse should assess and monitor the

… 

\% 

%]

$
 
- patient for early symptoms of digitalis toxicity,
tween administering the drugs. which include
a. nausea and vomiting.
6
]]

 

  
 @
=> @?
#
b. slurred speech.
would be a priority to withhold the drug and con- c. tachycardia.
 

  
$

 


d. photophobia.
present?
a. BP 150/85 mm Hg
b. Constipation
c. Dizziness with position changes
d. Pulse 50 beats/min

Copyright © 2016, Elsevier Inc. All Rights Reserved.


CHAPTER 45 Q Calcium Channel Blockers 99

š
]—
!
 

 
 


DOSE CALCULATION QUESTIONS
medical-surgical unit in a hospital. The medication
administration record states that the patient should
^]
& @

  
$

$ 
$


> @
‰
]^`



 
!
 

 
  

 
$

 '

 ‚



> @
{
 
has been prescribed. The pharmacy has supplied
should the nurse do? 



 
$
`‹
 '‰
*

a. Administer the medication.  


‰'


 
  



b. Administer the medication and notify the pre- infusion pump?


scriber of the change.
c. Contact the pharmacy.
d. Hold the medication and contact the prescriber.
^^
!

 
 
$
 @
=> -
@?

 

`^
 '
$

  

17. Which drug might be prescribed to counteract the What is the recommended safe dose for a patient

  
 

  
who weighs 178 lb?
of dihydropyridine CCB?

& 
=‰ ?

Z  
=Š ?

… 
=‰ ? CASE STUDY

€
=‰ ?
A 75-year-old female patient who is a nonsmoker with a

]<

 



  
=
history of hypertension, esophageal cancer, and type 2 di-
   ?

 
$
=|  ?
abetes mellitus comes to the emergency department with
20 mg three times a day has had his prescription $  



$
 
{

  

  

$
=| 
‰?
—`



 
  
!
  

  

a day. The patient asks the nurse how taking the =>  ?



 
same medication once a day instead of three times
a day will help prevent palpitations. What should
]
!
 

 

  
=>  ?

the nurse’s response include? #Š


 

 

 


a. Rapid-acting formulas of nifedipine are more nurse notes a sudden reduction in heart rate and
potent than sustained-release formulas. prolongation of the PR interval on the cardiac
b. Blood levels of rapid-acting formulas of nife- monitor. What should the nurse do?
dipine rise more rapidly than with sustained-
release formulas.

{  % 
$
$
$
  

a more gradual drop in BP.



{  % 
$
$
$
-
^
!

$
”


  

press the automaticity of the heart like vera- the patient requests assistance with getting up and
pamil. using the toilet to void. What should the nurse do?

19. A patient has been prescribed extended-release


$
=|  ?
! 
 
-
istration should include which instruction?
a. Administer 1 hour before or 2 hours after 3. The patient is stabilized, and the prescriber has
meals.   
  
=>  ?
]^`




b. Do not crush or chew tablet. day. The nurse is reviewing the patient’s labora-
c. Take with grapefruit juice to improve absorp- tory tests. Which laboratory value would require
tion. consultation with the prescriber regarding admin-
d. Take on an empty stomach.   
$
  ƒ
={

 



 
?
6
^`
#






$
  
$$

 
  $ 
=‰!?
^`
 '‰
and toxicity in a patient who is receiving nifedip- 
–

 
=–ˆ+?
^<
 '‰

=|  ?
$
  
 
 
 
> 
†^
 '‰

‰!
†`
#ˆ'‰ 
… 

 
=…–{?
^`
 '‰

–+|
]``
   '‰ 
*  
=* ?
]`
 '‰

–ˆ+
^^
 '‰ $
#  
 @
 
=#+?
]<

\…
†^
‰' 
| 
=+?
††
Z"'‰

{
=+ +?
]‹—
 '‰

Copyright © 2016, Elsevier Inc. All Rights Reserved.


100 CHAPTER 46 Q Vasodilators

4. The nurse is reviewing the patient’s prescribed 5. What teaching should the nurse provide that can
  
#


  
<`


prevent problems with common adverse effects of


 

 

 
  
={ - verapamil?
?
]^`
 

 
 
 


of insulin by the pancreas and lowers the blood


sugar to treat the patient’s diabetes mellitus. Both
verapamil and nateglinide are metabolized by the
CYP3A4 hepatic enzyme. Verapamil inhibits the
action of this enzyme. Because of this interaction,
the nurse knows the patient is at increased risk for
hypoglycemia. What symptoms suggest that the
patient’s blood glucose has dropped below normal?

46
Vasodilators

STUDY QUESTIONS CRITICAL THINKING, PRIORITIZATION,


AND DELEGATION QUESTIONS
True or False
   
  T   F 6 10. Which would be a priority to report to the pre-
    
$
  @
= ?ƒ
a. Ankle edema

]

*  @
= ?



b. BP 150/80 mm Hg
of the heart by reducing afterload. c. Pulse 120 beats/min

^

*  @
= ?


 - d. 10 mm Hg drop in BP when sitting up from a
sure by causing vasodilation and diuresis. reclining position
3. ___ An adverse effect of minoxidil is impair-
ment in tissue perfusion. 11. Which drug is commonly prescribed for patients

‹

!
$$
$


$ 
$
 
with heart failure when prescribed hydralazine
end within minutes of stopping the infusion. = ?
  
$

 
$$
$

5. ___ Nitroglycerin is used to treat angina because sodium retention?


it dilates arterioles in the heart, allowing a. Atenolol
more oxygen to get to heart muscle cells. b. Furosemide
6. ___ Orthostatic hypotension is more likely to 
#   
 
occur when a patient is receiving a drug that d. Minoxidil
dilates veins than one that dilates arterioles.

~


  




6 12. When caring for a patient who is prescribed min-
that dilate both arterioles and veins. oxidil, it is most important for the nurse to assess

<

&


$
%

 


for what?

 


  
  a. Abnormal hair growth
9. ___ When a patient is receiving nitroprusside, the b. Bradycardia
nurse would report a thiocyanate level greater 
…


 
`]
 '‰ 
{
$
 

Copyright © 2016, Elsevier Inc. All Rights Reserved.


CHAPTER 46 Q Vasodilators 101

š
]†

 


 

  
17. The nurse is preparing to administer intravenous

=‰?
!
 

 

=#Š?
 

=+?
!
 

  
{!!
$
 





$
œ
 

^`
Z"
$
 -
what? 



‰'
 



 


& 

 
$
 
 
action by the nurse?
and narrow pulse pressure 
>

 

=+?



Z 
  


 

- the Y-port of the primary line above the intrave-
ety 





  
c. Headache, dizziness, and blurred vision b. Disconnect the primary intravenous line and

{$
$

$
  
 
=|€#?
^

 

 

=+?

to left and bruits over carotid arteries then reconnect the primary line.

{ 


 


 

14. The nurse is caring for a nonambulatory patient 


 

=+?
 




  @
= ?

separate intravenous pump.
monitoring for adverse effects, the nurse would 
{

 

$ 





 


 
 
ƒ  

=+?





a. Around the medial malleolus of the ankle the intravenous pump.


b. Fingers

{  
 18. A patient who weighs 165 lb is receiving in-
d. Dorsal aspect of the feet  
=#Š?
 

=+?

300 mcg/min. An important nursing assessment at


6
]
‰  
 
 


 

this dose is for what?
  @
= ?

 
- 
" 

 

 

 
 
=+?

$

b. Anorexia, nausea, and vomiting
prescriber of these lab results, it is a priority to 
…
  




 
 ƒ d. Palpitations, disorientation, and respiratory
a. Fatigue depression
b. Headache
c. Respirations 22/min 19. The nurse is preparing to administer nitroprus-

! 
]`]„
…
=†<†„
>? 

=+?
!


” 


prepared in the pharmacy, but has a faint brown


š
]—
!
 

 
 
=+- discoloration. What should the nurse do?
 ?


  
 

!
  
 
a. Administer the medication.
orders administration of oral antihypertensives. b. Consult the prescriber before administering the
The patient’s blood pressure is still elevated, but solution.
dropping slowly. The nurse should do what? c. Discard the solution and get a new bag from the
a. Administer both medications. pharmacy.
b. Hold both the nitroprusside and the oral 
 




 


$


antihypertensives until the prescriber can be brown color clears.


consulted.
c. Hold the nitroprusside and administer the oral
antihypertensives until the prescriber can be DOSE CALCULATION QUESTIONS
consulted.
d. Hold the oral antihypertensives and administer
^`
*  @
= ?
‹`

 


the nitroprusside until the prescriber can be dose is prescribed. The drug comes in a concentra-
consulted. 
$
^`
 '‰
!
 



 

undiluted over 1 minute. How many tenths of a





 

]`
 ƒ

21. What rate of infusion of nitroprusside would the


 
  




$
 

†
 ' '


 

 
]~—

=<`

 ?
$




`


^`

$
œ

dextrose in water?

Copyright © 2016, Elsevier Inc. All Rights Reserved.


102 CHAPTER 47 Q Drugs for Hypertension

CASE STUDY 3. The patient suddenly becomes diaphoretic and


complains of nausea, palpitations, and headache.
A 78-year-old patient is seen in the emergency depart- What should the nurse do?


 
  
{


$
^

 

and weighs 125 lb. Her BP is 230/130 mm Hg, and she




  
{


 
$

 


 

=+?

`†
 ' '


prescribed.
‹
!
 


  
!

prescriber orders hydralazine, metoprolol, and hy-


1. What assessments of the solution and precautions drochlorothiazide on a long-term basis to control
should the nurse take when administering this 

 
Z 

  
$


medication? combination of medications.

^

 
 

=+?
- 5. The nurse checks the drug handbook, and the


 
  
=
]

^
 ?
 
dose recommendation for adults for hydralazine
and how often should the nurse assess to follow is listed as 10-50 mg 4 times a day or 100 mg 3
the results of the drugs and the patient’s response? times a day. What are possible reasons for the low
dose being extended to every 8 hours?

47
Drugs for Hypertension

STUDY QUESTIONS 4. ___ Blood pressure reduction results from dila-



$


 Œ

$

Matching  
 

 Œ

 

of renin release.

 

# $    


‰
–|


 

$

      



 Œ

 
 

aldosterone receptor blockade.


1. ___ Because of their ability to conserve potas- 6. ___ Act directly on renin to inhibit conversion of
sium, these drugs can play an important role    

  
#
in an antihypertensive regimen. 7. ___ Produce much greater diuresis than the
2. ___ Promote dilation of arterioles and have di-  @ Œ


–|




rect suppressant effects on the heart, reduc- volume and promoting vasodilation.


    8. ___ Reduce BP by two mechanisms: reduction

†

|
 
$
 Œ

  
of blood volume and reduction of arterial
 
$
 Œ
 
$
  
- resistance.



‰
$
–|
 

$-



   

 





Copyright © 2016, Elsevier Inc. All Rights Reserved.


CHAPTER 47 Q Drugs for Hypertension 103

‡

& 
 
 

 Œ
- CRITICAL THINKING, PRIORITIZATION,


  
  

  -
AND DELEGATION QUESTIONS
 Œ



”  

cells of the kidney and reduce release of



]—
* 


 


 

Œ
 %
 

 

primary hypertension?
vascular resistance.
a. Diabetic in end-stage renal failure
10. ___ Act within the brainstem to suppress sympa-
b. Hyperthyroid adult




 


  
c. Postmenopausal African-American woman
11. ___ Depletes norepinephrine from postgangli-
d. Young adult male with an adrenal tumor
onic sympathetic nerve terminals reducing
sympathetic stimulation of the heart and
17. When possible, the nurse should assess the pa-

  Œ
 
  

 

tient’s BP with the patient in which position?


depression.

‰
 



]^

|
 
$
 Œ
 
  


‰




 
 

 
‹


   
degrees
13. ___ Prevent stimulation of receptors on arte-

{

$
  
rioles and veins, thereby preventing sym-

{

$



  
 
  Œ




 %
 
$

š
]<
#

 
$

 

 
 

hypertension.
effects of the antihypertensive drugs that are pre-

]‹

|
  
##­ 
 -
scribed because the nurse knows what about the
striction and release of aldosterone by block-
drugs’ adverse effects?



$
  
##
a. They affect adherence.

]

‰
–|


$ 
$

b. They are life-threatening.


  
##
 

 -
c. They are necessary if the antihypertensive is at
 
##­ 
  


a dose that is effective.


aldosterone-mediated volume expansion.

!


  
$
 
a. Adrenergic neuron blockers
19. The teaching plan for a patient taking antihyper-
b. Aldosterone antagonists
tensive drugs should include which information?
c. Alpha1
=¯1?
 



={

 
?
=  1
 ?
a. Cigarette smoking and consuming more than
d. Alpha1
=¯1?' 
 


two alcoholic drinks per day can reduce the ef-



=  1' 
 ?
fectiveness of antihypertensive drugs.
e. Angiotensin-converting enzyme inhibi-

Z 


$ 

 
   


=>Z#?
and tomatoes can help the patient lower his or
$
  


=– ?
her blood pressure.

–
=±?
 



c. Certain foods, such as milk, have substantial


= 
 ?
amounts of sodium even though they do not
h. Centrally acting alpha2 agonists
have a salty taste.
i. Dihydropyridine calcium channel block-
d. The patient’s symptoms of hypertension are the

=>>–?
best indicators of the need for antihypertensive
j. Direct-acting vasodilators
drugs.
k. Direct renin inhibitors
l. High-ceiling diuretics
20. When administering an antihypertensive drug to a
m. Nondihydropyridine calcium channel
patient who has a history of hypertension and type

=>>–?
2 diabetes mellitus, what would be an appropriate
n. Potassium-sparing diuretics
outcome for therapy?
o. Thiazide diuretics
a. BP 100/60 to 110/70 mm Hg
b. BP 110/70 to 130/80 mm Hg
c. BP 120/80 to 130/85 mm Hg
d. BP 128/80 to 138/90 mm Hg

Copyright © 2016, Elsevier Inc. All Rights Reserved.


104 CHAPTER 47 Q Drugs for Hypertension

š
^]

  
 




27. Which teaching would most likely prevent the
the prescriber if a patient is receiving a thiazide or most serious effect when a patient is prescribed
loop diuretic? clonidine for hypertension?

–

 
=–ˆ+?
^`
 '‰ a. Change positions slowly.
b. Hemoglobin A1c 5.5% b. Consult the prescriber before stopping the drug.

| 
=+?
†^
Z"'‰ 
Z 
  

ˆ

^`
 '‰ 
ˆ 
 %$
 

22. The nurse would teach a patient who is on which 28. Which intervention is most likely to decrease ad-
diuretic to avoid use of potassium-containing salt verse effects of hypertension in African-American
substitutes and excessive consumption of bananas adults?
and orange juice? 
Z 
 

 

 
$-

Z 

=Z? fects of antihypertensive drugs

… 
=‰ ? 
#$
 
$$
$
 


* @
=*&#ˆ#‰? drugs

! 
=&? c. Prescribing of the appropriate drug for hyper-
tension

^†
 
={ ?
 
=‰ ?

d. Promoting regular screening of BP
pindolol are beta blockers that have intrinsic
sympathomimetic activity. This decreases the
^‡






 %

incidence of what adverse effect? treatment for an African-American patient with


a. Bradycardia at rest hypertension?
b. Bronchoconstriction 
>Z
 
c. Heart block b. Calcium channel blocker
d. Hypoglycemia c. Diuretic
d. Vasodilator
24. Which nursing concern is of greatest priority when
a patient starts therapy with an alpha1 blocker? 30. What is the recommended time goal for lower-
a. Adequate exercise ing BP when a patient is in acute congestive heart
b. Hydration failure?

{ $ a. 30 minutes

ˆ 
 b. 60 minutes
c. 90 minutes

^

 

 



d. 2 hours
tachycardia if receiving which antihypertensive
drug?
†]

 
>\


$
 

$



>Z
  patient is prescribed
b. Beta blockers 
 
=> ?
c. Direct-acting vasodilators 
 @
=*&#ˆ#‰?
d. Nondihydropyridine calcium channel blockers 
 @ 
=€ ?

 
=# ?
26. Which action is of greatest priority when caring
for a patient who has just been prescribed a drug 32. What is the best treatment for severe preeclampsia?
for hypertension? a. Delivery of the fetus

#$
  



  
‰  
=+?
b. Teaching how the prescribed drug works 
‰ 
=¢ ?
c. Teaching the patient how to assess his BP sit- 
€
=?
ting and standing
d. Telling the patient his target BP 33. Which magnesium level is within the target range
for a patient being treated for eclampsia?

]
Z"'‰

^
Z"'‰


Z"'‰

<
Z"'‰

Copyright © 2016, Elsevier Inc. All Rights Reserved.


CHAPTER 48 Q Drugs for Heart Failure 105

†‹
#


$
  



 - 2. When is it important to lower this patient’s BP
lary blood sugar rather than rely on symptoms that within 1 hour rather than slowly as for most hyper-
suggest hypoglycemia if a patient is prescribed tensive patients?
which class of antihypertensive drugs?

>Z
 
b. Beta blocker

* %
=?

d. Thiazide diuretic 3. The resident writes an order for intravenous nitro-
prusside 80 mcg/min. What precautions should the
nurse take while administering this medication?
CASE STUDY
A 48-year-old African-American man with a history of
type 2 diabetes mellitus comes to the emergency de-
 
=Z&?


$ 
  

 @
4. The patient is stabilized and discharged on hydro-
 
$
’
$
“
ˆ
  

 @

 @
=> @?
}


BP is 210/120 mm Hg, his pulse is 98, and his respirations follow-up visit, the patient’s BP is 160/90 mm Hg.
are 24 and labored. The patient asks why the prescriber does not just
give him a high enough dose to bring his BP down
1. What risk factors does this patient have for a car- to normal. How should the nurse respond?
diovascular event?

48
Drugs for Heart Failure

STUDY QUESTIONS        

Matching
—

#
‰Š
”
$ 


symptoms, increase exercise tolerance,


  

   #*$  decrease hospitalization, enhance quality of

       life, and reduce mortality. However, they do

 

$
 Œ
$


1. ___ Faint peripheral pulses, decreased urine have no effect on cardiac remodeling.
output
~

|
$
  Œ
 


^

‰ 
$
 
 %$ 
 



\…



†

} 
”  

 
= Š&?
<

# 
  
 
$

4. ___ Palpitations, pallor thereby increasing cardiac output. Alter the



{
$
 

 

- electrical activity of the heart, and favorably
room affect neurohormonal systems.

‡

–

$
  
##
 

a. Activity intolerance release of aldosterone, and suppress degra-


b. Cardiac output, decreased dation of kinins, thereby improving hemo-
c. Fluid volume excess dynamics and favorably altering cardiac
d. Health maintenance, ineffective remodeling.
e. Perfusion, ineffective peripheral

Copyright © 2016, Elsevier Inc. All Rights Reserved.


106 CHAPTER 48 Q Drugs for Heart Failure

10. ___ Block aldosterone receptors in the heart and 17. The nurse cannot hear the apical pulse of a patient
blood vessels. with the diagnosis of HF when assessing at the

]]

|
 
  Œ
 

$
  
 


  


edema is not too great. Are ineffective when Which direction should the nurse move the stetho-
\…

Œ
 

 
$
  

scope diaphragm/bell to be more likely to hear the
is greatly reduced. pulse?

]^

> 

‰Š
”
$ 
 
a. Toward the clavicle
exercise tolerance, slow progression of HF, b. Toward the left side
reduce the need for hospitalization, and c. Toward the sternum
prolong survival. d. Toward the waist

]†

{
  
 
  
-
tive activation of beta1%  
 Œ
6 18. A patient with HF is prescribed spironolactone
can increase myocardial contractility, and = ?

  
=Š ?
#



can thereby improve cardiac performance. priority to consult the prescriber regarding evalua-
tion of the patient’s potassium levels if the patient
a. Aldosterone receptor antagonist exhibited

  %
@
=>Z?
a. confusion.
inhibitors b. constipation.

  


=– ? c. weakness.

– %  


=  % d. shallow respirations.
 ?
e. Cardiac glycosides 6 19. What is a priority assessment when administering
$
* %
=?
 dopamine?

{ 
 a. Bowel sounds
h. Thiazide diuretics 
>  

c. Temperature

ˆ

CRITICAL THINKING, PRIORITIZATION,
AND DELEGATION QUESTIONS š
^`

  
 
=#Š?
$ 
$

inamrinone is prescribed for a patient who has a


14. Which statement, if made by a patient with HF, current infusion of 5% dextrose and 0.45% NaCl
would indicate a need for further teaching? 

‹
‰'
*


 
-

’#$

 

 

 

ister the inamrinone?

$


 

 
$



$
 

 

breathe.” 
ˆ 




b. “My heart is stretching so much it is losing the c. As a secondary infusion through the primary
ability to squeeze out the blood.” line

’!
  


 
 

#
 
d. Over 2-3 minutes via the primary line
to feel tired and short of breath.”

’

 
 

$ 

 

21. Nitrates, including nitroglycerin and isosorbide,
properly.” reduce the workload of the myocardium by
a. decreasing the amount of blood returning to the

]
Z 
$
–% 

=–+|?
  
heart.
HF because BNP is released when the b. decreasing conduction of impulses through the
a. heart beats faster. heart.
b. heart’s atria stretch. c. decreasing the volume of blood in the vascular
c. heart does not get enough oxygen. system.
d. heart muscle thickens. d. increasing myocardial remodeling.

6 16. The nurse would be most concerned if a patient




 

ƒ
={

 

?
a. Peripheral edema

ˆ

$
<^




c. Distention of the jugular veins
d. Weight gain

Copyright © 2016, Elsevier Inc. All Rights Reserved.


CHAPTER 48 Q Drugs for Heart Failure 107

6 22. The nurse is preparing to administer hydralazine


^<


 
$
  @

   
=–&?

= ?
$
*…
–  




has been approved to treat HF in people of which
associated with drug-induced lupus-like syndrome, self-reported race?
it would be a priority to report what symptom to 
 
# 

  ƒ
={

 
? b. Asian
a. Facial rash c. Caucasian
b. Heartburn d. Black
c. Joint pain
d. Fever
^‡
#





$
  

$

 


 
=‰ ?

 

23. Before administering nitroglycerin to a newly 



]†
 '‰


 




admitted patient, it would be most important for laboratory result?


the nurse to identify if the patient has recently self- 
–+|
]<]†
   '‰
medicated with a drug for what condition? 
#+
]

Z
 $ 
| 
=?
†`
Z"'‰
b. Fever 
{
]`
Z"'‰

#
d. Pain
DOSE CALCULATION QUESTIONS
24. A possible explanation for why digoxin has not
demonstrated an improvement in mortality is that 30. Dobutamine HCl is prescribed at a rate of 2.5
the drug does not  ' '
!
 
 
]—

=
 ?

a. decrease heart rate. !




  

^
 '‰
$
œ
 

b. improve heart remodeling. !





   

‰'
 
 

c. increase cardiac output. 



 
  



ƒ
d. increase urine production.

š
^


 



  
 


†]



$
`

$
 

  

furosemide, the nurse should carefully assess the $



 

 
]‹
 
#

 
$
$

patient for which symptoms of digoxin toxicity? the recommended dose is 0.75 mg/kg?
a. Anorexia and nausea
b. Anxiety and abdominal cramps
c. Bone pain and constipation CASE STUDIES
d. Muscle spasms and convulsions
Case Study 1
š
^—
 



 



  

that the nurse not administer digoxin and notify An 81-year-old male patient with chronic HF who resides
the prescriber? in a long-term care facility has been treated with digoxin
a. Blood pressure 100/76 mm Hg =‰ ?
`]^
 
$ 
<`
 

 


& 
^‹
 '‰ 
=%&?
^`
Z"


 


$



c. Heart rate 100 beats/min 




  


=#  ?


| 
†
Z"'‰ 50 mg orally once a day to the medication regimen.

27. The nurse would consult the prescriber if a patient 1. When consulting the prescriber about this patient,
with HF who is experiencing pain is prescribed what questions might the nurse ask?
which drug?
a. Acetaminophen

# 

# $
d. Morphine 2. Why is it important for the nurse to monitor elec-
trolyte and digoxin levels on this patient?

Copyright © 2016, Elsevier Inc. All Rights Reserved.


108 CHAPTER 48 Q Drugs for Heart Failure

3. The long-term care nurse notes during the report 7. The patient is transferred to the coronary care unit.
from the night shift that the patient has become Oxygen is administered along with furosemide
increasingly restless, “coughs all night,” and seems =‰ ?
 

  
=Š ?

very fatigued when the staff is providing care. His What is the rationale for administration of these
appetite has been poor, even for things he normally drugs?
” 
  


–|
‡`'~`


* Œ
 
 


 


 
$

]]<
  'Œ
{3
 
Œ
 
 -


 Œ


%
 



 

week. What action should the nurse take at this


time?
8. The patient is stabilized and is transferred to the
nursing unit. Prescribed drugs include carvedilol,
enalapril, and furosemide. What laboratory values
does the nurse need to monitor when a patient is
4. The patient is admitted to an acute care facil- receiving these drugs?
ity. Based on the knowledge of digoxin and its
relationship to potassium, how could these blood
levels have contributed to the patient’s decompen-
sation?

9. What teaching should the nurse provide to prepare


this patient for discharge?
Case Study 2
A 64-year-old male patient is a direct admission to the
nursing unit with the complaint of abdominal pain, nau-
sea, and extreme weakness. The patient has a history of
diabetes, chronic renal failure, and sleep apnea. Admis-

 
 


 

10. The patient lives in an apartment building with


‡‡†„
…
–|
]~‹']]`

* 
 
‡—

  
|€#

12 stairs to climb to his apartment. After climbing


$


$
 Š&
‹



 
$
‰ 

the stairs, he needs to rest because he is short of


3+ pitting edema in the lower extremities, and tenderness
breath. He can, however, perform most activities


ˆ˜

 
  
of daily living. According to the New York Heart
  
=+™*?
 


 ‚

5. What nursing diagnosis is most appropriate for


  ƒ
this patient?

6. The nurse obtains laboratory results from tests


drawn on admission including BNP 1275 pg/
‰Œ
+
‹
Z"'‰
 
^~
 '‰
–ˆ+

‹^
 '‰
!
 
 

  




{!!
   



{%!
   
 



 
 

at this point?

Copyright © 2016, Elsevier Inc. All Rights Reserved.


CHAPTER 49 Q Antidysrhythmic Drugs 109

49
Antidysrhythmic Drugs

STUDY QUESTIONS CRITICAL THINKING, PRIORITIZATION,


AND DELEGATION QUESTIONS
Completion
1. " 
 is not as accurate a term as  - š
]^
 



 
  
$

 


 because arrhythmia literally means with a dysrhythmia?
_____________________ of a heart rhythm. a. Alteration in cardiac output

#  
$



2. + 
 means _____________________ 
# " 
 
 
$ 
heart rhythm. 
#$$
 
 

3. ,  
 is the term for a heart rate that š
]†
 


|
 
$

˜{
-
is _____________________ than normal. 


Z>\
 ƒ
a. Atria are depolarizing faster than the ventricles.
4. . 
 is the term for a heart rate that b. Atrial impulses to the ventricles are being
is _____________________ than normal. slowed.


€
 
 @ 
$
|”
 
c. Ventricles are depolarizing faster than the atria.
results in rapid _____________________ contrac- d. Ventricular impulses to the atria are being
tion. slowed.

Matching
]‹
Z
$
 
 

 
$

 
$

the discharges of the ectopic stimuli are


      a. atrial in origin.
   
$ 
 

{

c. from the ventricles.
6. ___ Blocking this ion slows impulse conduction. 

 

{

7. ___ Blocking this ion reduces myocardial con-
tractility. š
]

 


  

 

8. ___ Blocking this ion delays repolarization. 


 


˜!
 

Z>\
!

nurse should teach the patient the importance of


a. Calcium reporting which symptom that suggests that a
b. Potassium dangerous adverse effect, torsades de pointes, may

{ be occurring?
a. Ankle edema
  
      b. Feeling faint
c. Headache
9. ___ Delay repolarization of fast potentials by 
{
$
 

 
=+?
 

]`


 
=> ++?


$ 

š
]—
# 

 


Š  
 


slow depolarization and depress phase 4 asking the patient to do what?


repolarization of slow potentials a. Bear down as if having a bowel movement

]]

{
 




- b. Breathe through pursed lips




=+ +?
  c. Pant like a dog

! 



 

>
#

>
##



>
###

Copyright © 2016, Elsevier Inc. All Rights Reserved.


110 CHAPTER 49 Q Antidysrhythmic Drugs

š
]~

$ 
 
 
$

 

š
^†
!
 



Z>\
$

 


 
  

 

 


 
 
$
"
‹


of changes in what? 
!
 

 

˜{



a. Appetite 
^œ
$
%
 
 

b. Bowel pattern What should the nurse do?



&

 a. Administer a beta-blocker.
d. Mental status b. Continue nursing care.

& 


Z>\
š
]<

  
 
$
  
 - d. Notify the prescriber immediately.
@
 
=#+?
$



 


 
  



 $ 
=>- 24. The nurse should assess patients receiving long-
 ?

 
 
 $ 
 

term therapy with procainamide for which adverse
achieved the desired effect? $$ ƒ
={

 
?

#+

`‡%]] a. Abdominal pain and nausea

#+

^%† b. Chest pain and dyspnea

#+

%‡ 
Z 
 
 
=+?



#+
 
 
]` painful joint swelling

Z 
 


 


19. The nurse responds quickly when a ventricular thrombi formation


  


  


$
e. Fever and chills
dysrhythmia impairs the ability of the heart
a. to contract rapidly enough to meet body needs. š
^
!
  






 

b. to eject an adequate amount of blood. $


 
=#Š?

 
  
!

c. valves to close properly. #Š




 

]]•†`
AM. What is the
d. valves to open properly.   
 

 

 

 

dose of oral procainamide?


20. A patient with a dysrhythmia, type 2 diabetes mel- 
# 

=!^&€?

 
$ 
=*…?

  
b. 1:30 PM
quinidine, metformin, furosemide, and digoxin. c. 2:30 PM
The nurse should be vigilant in monitoring for d. 4:30 PM
which early adverse effect of digitalis toxicity that
may occur with this combination of drugs? 26. Which anticholinergic effect of disopyramide
a. Anorexia and nausea =+ ?


$
  





b. Cool, clammy skin the prescriber?



Z 
 

  a. Blurred vision
d. Yellow-tinted blurred vision b. Dry mouth and thirst

+
–€
¬
^
 
š
^]

$
 
 
 
$



| 
  
$
†`

a patient who is receiving quinidine, is of greatest
priority to report to the prescriber?
^~
–  
$
 
 %
$$

 

a. BP 150/88 mm Hg expects to administer lidocaine in which way?


b. Tinnitus 
{  % 
$

{
  
#  
d. Three soft stools in 8 hours c. Orally on an empty stomach
d. With a full glass of water
22. A patient asks the nurse why he was instructed to
take quinidine with food. An appropriate response 28. An appropriate nursing intervention to prevent
by the nurse is that taking the drug with food does adverse effects of gingival hyperplasia associ-
what? 

 %
 
$

=& ?



& 


$
\#
 teaching the importance of regular care with which

# 
 health care professional?
c. Prevents cinchonism a. Dentist
d. Prevents ventricular tachycardia b. Ophthalmologist
c. Podiatrist
d. Pulmonologist

Copyright © 2016, Elsevier Inc. All Rights Reserved.


CHAPTER 49 Q Antidysrhythmic Drugs 111

6 29. Which adverse effect of propafenone, if occur- š


†
–  
  
=>  ?
 
  
 

ring for 3-4 days, would be of most concern to the of calcium channels in vascular smooth muscle, a
nurse? priority is for the nurse to teach a patient who is
a. Abdominal pain prescribed this drug to do what?
b. Anorexia 
&
^``

$

 
 
  

c. Dizziness water.
d. Vomiting 
# 
 



c. Take orthostatic BP precautions.
30. The nurse is preparing to administer the cardiose- d. Report bruising.
lective beta-blocker acebutolol. Which assessment
would indicate the need for immediate consulta-  36. Which action is most appropriate when adminis-
tion with the prescriber? 
 
#Šƒ
a. Apical pulse 105 beats/min a. Administer oxygen.
b. Bronchial wheezes b. Monitor vital signs.

>  

 
^^`
 '‰ c. Have the patient perform the Valsalva maneu-
d. 1+ pitting edema of the dorsal aspect of the foot ver.
d. Administer dipyridamole concurrently.
31. The patient must be informed of possible damage
to the lungs when prescribed oral therapy with
†~
#
 
  


$
 


which drug? exceeds the risks of adverse effects when



 
=> ? a. the dysrhythmia is prolonged.

&$
=! ? 

˜!
 

 
 

| $
=? c. ventricular pumping is ineffective.

| 
=# ? 

{

   


 

]—`

beats/min.

†^
#




 
 


rhythm, what assessment must the nurse continu-


 


 

 
DOSE CALCULATION QUESTIONS
=> ?ƒ
a. Blood pressure
†<

 

 
]—


  


‰
$
   diltiazem. The recommended initial bolus dose is

# 
 0.25 mg/kg. What is the recommended dose for
d. Respiratory effort this patient?

š
††
 


 
  
$$


amiodarone and dronedarone?


†‡
}
 
$
 

^`

$
œ
 

a. Amiodarone does not increase thyroid hormone in water at a rate of 2 mg/min is prescribed. The
secretion. #Š


   




 
 

b. Amiodarone has a different effectiveness. should be programmed into the pump?


c. Amiodarone has a different risk of death.

 



 
$
 -
cant adverse effects. CASE STUDY
34. Which laboratory results would increase the risk A 75-year-old man underwent open-heart surgery 5 days
of torsades de pointes when a patient is prescribed ago and postoperatively developed a cerebrovascular ac-
dofetilide? 
=>Š?
*
 

 
 
-

| 
†^
 '‰ $

 


^% %% 



| 
†
 '‰ habit. He was progressing well postoperatively without

{
]†^
 '‰ any further complications. A physical therapist and nurse

{
]‹<
 '‰ assisted the patient to transfer to a recliner. Following
transfer, the patient complained to the physical therapist
that his heart felt “like it was racing.”

1. What assessments should the nurse perform at this


time?

Copyright © 2016, Elsevier Inc. All Rights Reserved.


112 CHAPTER 50 Q Prophylaxis of Coronary Heart Disease

The monitor nurse walks in to check on the patient be-


‹
}
 
$
 

^`

$
œ
 

cause the monitor revealed a heart rate of 185 beats/min. in water at a rate of 2 mg/min is prescribed. The
The nurse asks the patient to bear down to stimulate the #Š


   




–$

vagus nerve and slow the heart rate. After obtaining a full $ 

 

 

 


set of vital signs, the nurse places a call to the attending solution and not administer the drug if the solution
  
!
  


{!!
]^% 
Z>\
contains what additives?
The attending physician arrives on the unit, examines the
]^% 
Z>\

  
 
  
=%
 ?


 
$
 
]``




2. What precautions must the nurse take before and 5. What should be included when the nurse reports to

  
$



 ƒ the prescriber the patient’s response to lidocaine?

3. The bolus dose is followed by an infusion of a


solution of lidocaine. What patient data can the
nurse provide to assist the prescriber with deter-



$






the lidocaine?

50
Prophylaxis of Coronary Heart Disease: Drugs That Help Normalize
Cholesterol and Triglyceride Levels

STUDY QUESTIONS
—
} 
‰&‰
 




!|
###



 

 '‰
Completion

~
} 
*&‰
 




!|

]
Z  
  
  

- ###


 

 '‰
nal corticosteroids are ________________ that
require cholesterol for formation. 8. The recommendation for regular physical activity
is _______ to _______ minutes on most days.
2. The ________________ is the primary source of
endogenous cholesterol in the body.
CRITICAL THINKING, PRIORITIZATION,
3. When trying to lower blood cholesterol, it is im- AND DELEGATION QUESTIONS
portant to reduce intake of _______________ fats.
9. A 45-year-old female smoker with a family his-
4. ________________ serve as carriers for transport- 
$
 
 
  
=>*&?



ing lipids in blood. prescribed a lipid-lowering medication. Which


outcome would be most important for this patient?


‰ 
    means ________________ 
> 
 

 
^``
 '‰
___________________ and    
*&‰
 
 
‹`
 '‰
means ______________ _________________ 
‰&‰

 
]``
 '‰
______________. 
! 

 
]`
 '‰

> 
Ÿ
^`]—
Z 
#

 
 
CHAPTER 50 Q Prophylaxis of Coronary Heart Disease 113

10. A patient has received instructions regarding cho-  15. A patient who is receiving nicotinic acid to elevate
lesterol and the body. Which statement made by *&‰


 
 



the patient would indicate the patient needs further 


 
Œ
 Œ
 Œ

$ 

teaching? What is the appropriate next action?


a. “Drugs that decrease the liver’s ability to pro- a. Assess for chest pain.
duce cholesterol should be taken so that they b. Assess for abdominal pain.
peak during the night.” c. Continue nursing care.
b. “Drugs that prevent the absorption of choles- d. Notify the prescriber.
terol from food are more effective than drugs
that limit the making of cholesterol by the 16. Which OTC drug is recommended to prevent the
liver.”  
$$
$
 
 
 




’#


 




$
niacin?
saturated fat in my diet than the total amount of 
 
=!?
cholesterol.” 
 

= ?

’€
 
 

 

$
#
 

% 
& 
=– ?
fat diet and exercise regularly.” 
 
=¢  ?

š
]]

  
 
$

 

17. The nurse has been caring for a patient admitted
*&‰

 '‰
‰&‰
]‹
 '‰

 - with dehydration. The patient has been taking

—†`
 '‰
!
 



colesevelam and complains of nausea. Which as-
patient for what?  



$
 





ˆ
 
 

   nurse?
b. Chest pain and shortness of breath a. Bloating after medication administration
c. Rash and pruritus b. Flatulence
d. Tachycardia and diaphoresis c. Hyperactive bowel sounds with rushes

#  
š
]^
!
 



  
 
$

 





  
=€- 18. A patient has been prescribed cholestyramine in
 ?
‹`
 
 

{!
]^^
 '‰
addition to hydrochlorothiazide. When should the
‰!
]
 '‰
>
^`
 '‰
*&‰
‹
 '‰
cholestyramine be administered?

‰&‰
^‹
 '‰
 


 
ƒ a. 1 hour after other drugs
a. Administer the drug and continue nursing care. b. 1 hour before other drugs
b. Hold the drug and assess for chest pain. c. 4 hours after other drugs
c. Hold the drug and assess for abdominal pain. d. 4 hours before other drugs
d. Hold the drug and notify the prescriber of the
laboratory results. 19. The nurse would be most concerned about which
patient?
š
]†

†<% %
$ 
 


  
a. A patient receiving lovastatin and experiencing
with hypercholesterolemia. The prescriber has a rash

  
=¢?
^`
 


 

b. A patient receiving nicotinic acid and experi-
hour of sleep, to be started after laboratory results 

 

$  
 
are obtained. Which laboratory result would be of c. A patient receiving colesevelam and experienc-
greatest priority to report to the prescriber? ing nausea

‰!
~
  
 '‰ 

 

 @

-

> 
]‹
 '  
 '‰ ing right upper quadrant pain

>|
†
 '‰

>\
^<~
  
 '‰ š
^`

  
 
 


$
  

concern if noted in a patient who is prescribed


6 14. A patient who is receiving lovastatin experiences
@ 
=¢ ?ƒ
 
 
>


<`
 '‰
=>%€€


‰!
†
#ˆ'‰
‡‡œ?


!
``^
'‰
#

$
  


–ˆ+
^`
 '‰
priority for the nurse to monitor the functioning of
c. Platelets 75,000/mm3
which organ?
d. WBC 10,000/mm3
a. Brain
b. Heart
c. Kidneys

‰

> 
Ÿ
^`]—
Z 
#

 
 
114 CHAPTER 50 Q Prophylaxis of Coronary Heart Disease

^]

 

  
 @
=‰?

CASE STUDY
warfarin. Which lab result suggests that the warfa-
rin dose is therapeutic? A 46-year-old female Type 2 diabetes patient has been

#+
] prescribed simvastatin because, despite good nutrition

#+
† and exercise resulting in A1c within acceptable levels, her

#+
— ‰&‰
 




]`
 '‰
!


#+
]` patient asks why she has to have additional bloodwork
=‰…!
>

>\?
$



  


22. The nurse would teach the patient strategies to drug.


prevent constipation if the patient was prescribed
which drug for dyslipidemia? 1. How should the nurse respond?
a. Atorvastatin
b. Colestipol

\ @
d. Niacin

23. Which statement suggests that the patient who has 2. What teaching should the nurse provide about
been prescribed cholestyramine needs additional birth control?
teaching?

’#
 




 “

’#


 







other drugs.”

’#
 
 


“

’#






 
$
 

3. The patient asks how she would know if liver or
drug to be sure that all of it went down.” muscle cell damage was occurring. What should
be included in the explanation?
DOSE CALCULATION QUESTIONS

^‹
>  
=?



—^%

tablets. How many tablets should the nurse admin-


ister if the prescribed dose is 1.9 g?
4. Why is it important to teach the patient to report
chest pain and to describe the characteristics and

^
\ @
=‰  ?
]†‹



  
whether it is worsened by coughing or laughing?
Available are 67-mg capsules. How many capsules
should be administered per dose?

5. The patient calls after getting the prescription



{
 
 

 



 


drug in the evening but the pharmacy information


states that it can be taken at any time. How should
the telephone triage nurse respond?

> 
Ÿ
^`]—
Z 
#

 
 
CHAPTER 51 Q Drugs for Angina Pectoris 115

51
Drugs for Angina Pectoris

STUDY QUESTIONS 7. A patient asks the nurse why nitroglycerin can be


administered in so many ways. What is the basis
Completion of the nurse’s response?

+ 


 
 %
$$

1. Angina pectoris occurs when the in the liver.


____________________ supply to the b. Nitroglycerin has few adverse effects, so vary-
____________________ is inadequate. ing doses can be administered via different
routes.
2. Cardiac oxygen demand is determined by heart c. Nitroglycerin is an inactive compound, so it
rate, contractility, ____________________, and does not matter which route it is administered
____________________. by.
d. Nitroglycerin is lipid-soluble, so it is readily

†
 



  


absorbed via different routes.
during the ____________________ phase of a
heartbeat. 8. Nitroglycerin has myocardial vasodilation effects

  
=|@ ‚ ?
 





4. The most common drug used to decrease platelet stable angina, because in chronic stable angina
activity is ____________________. myocardial arterioles are
a. atherosclerotic.
b. experiencing spastic vasoconstriction.
CRITICAL THINKING, PRIORITIZATION,
c. occluded with thrombi.
AND DELEGATION QUESTIONS d. unable to diffuse oxygen into the cell.
6 5. What is the priority nursing concern for a patient 9. A patient who has been using a nitroglycerin patch
with angina pectoris? for angina has recently been prescribed diltiazem
a. Decreased cardiac output => @?

 
$
 




# 

   patient, would suggest that the patient understood

#$$
 
$ •
   teaching about the new drug?

#$$
 
  
’#


 


  
 

causes my blood pressure to go up.”


6. What is the goal of drug therapy for chronic stable 
’#$
#
 

$
 
 
#
 
 
-
angina?  
=Š  ?
$
#


 $-
a. Constrict coronary arteries to increase blood tion.”
pressure during stress c. “This drug prevents my heart from racing,
b. Decrease myocardial need for oxygen during which can happen when people take nitroglyc-
stress erin.”

# 
  



  d. “This drug will make the nitroglycerin work
d. Prevent coronary artery spasms better.”

Copyright © 2016, Elsevier Inc. All Rights Reserved.


116 CHAPTER 51 Q Drugs for Angina Pectoris

š
]`

 

  

 
 
 
š
]
*

 

  
 







`‡``
=‡•``
AM?



disease?
^]``
=‡•``
PM?

 

 

a. Damaging the heart valves
0900 patch, the nurse notes that a nitroglycerin 
# 

 
 
patch is still in place. What should the nurse do? c. Promotes atherosclerosis
a. Apply the new patch to a different site, but 
{


leave the old patch on until the nursing supervi-
sor is contacted. 16. The nurse recognizes that a drug is a beta blocker
b. Consult with the prescriber regarding applica- if the generic name of the drug ends in which suf-
tion of a new patch. ƒ
c. Remove the old patch and apply the new patch a. -cillin
to a different site. b. -olol
d. Remove the old patch and change the timing of c. -pril
the medication so that the patch is removed at d. -sartan
0900 and applied at 2100.
17. Beta blockers are useful to improve myocardial
11. When collecting data from a patient who is not oxygen supply in angina because they slow the
obtaining relief from nitroglycerin tablets, the heart rate and force of contraction and
nurse should ask whether the patient allows the a. dilate coronary arteries.
tablet to completely dissolve in her mouth. This is b. dilate veins.
based on the nurse’s understanding that swallow- 
 


 


 
-
ing sublingual nitroglycerin tablets nary arteries.
a. allows the liver to inactivate the drug before it d. reduce blood return to the heart.
works.
b. increases tolerance to the drug. 18. The nurse would withhold a beta1 blocker and
c. prevents the small intestine from absorbing the immediately contact the prescriber if it was
drug. discovered that the patient has a history of which
d. speeds excretion. condition?
a. Asthma

]^
#

 
$

 

 

 

b. First-degree heart block


  
 
=+˜?

c. Hyperglycemia
an as-needed basis to do what? 
{


a. Discard unused tablets after 12 months.

{

 




 

19. The nurse would assess for tachycardia when
the bathroom. 
  
 



 = ?
$

c. Take a few tablets from the bottle and keep   ƒ


={

 
?
them in her purse or pocket for emergency use. a. Diltiazem
d. Write the date that the tablets are opened on the 
#   
outside of the bottle. c. Metoprolol
d. Nifedipine
š
]†

 

 




e. Nitroglycerin
receive the second dose of newly prescribed
sustained-release nitroglycerin, the patient com-
^`
 

  
 
$

  

plains of a headache. What should the nurse do? when the nurse is caring for a patient who is re-
a. Administer a prescribed PRN analgesic such as ceiving ranolazine?
acetaminophen. 
\…
‡
‰'
b. Crush the medication to speed absorption b. Penicillin prescribed for upper respiratory
through the oral mucosa. infection

+$

  
$

  
{!! 
{ 
–|
 
<

*

 

d. Withhold the drug and consult the prescriber. changes



ˆ 
$ 
14. The nurse is aware that which nitroglycerin prepa-
ration has a rapid onset?

{   
 

{  % 
  
c. Topical ointment
d. Transdermal patch

Copyright © 2016, Elsevier Inc. All Rights Reserved.


CHAPTER 51 Q Drugs for Angina Pectoris 117

DOSE CALCULATION QUESTIONS 3. What teaching should the nurse provide about
administration of the nitroglycerin patch?

^]
+ 

=^œ?

  
$

tube, and the length of the ribbon squeezed from



 

 
=}

 




$
 ?
*
 


would equal 30 mg? 4. The patient has the nitroglycerin drip turned
off, the nitroglycerin patch is applied, and the
diltiazem-CD is administered. He asks the nurse

^^
# 
=#Š?
 

  


what may have been the reason that his nitro-
rate of 5 mcg/min. The drug is available in a pre- glycerin sublingual tablets did not work. Based




]``
 '‰
$
œ
 
on knowledge of nitroglycerin sublingual tablets,

 
=&5?
!



   

‰' what factors may have contributed to why the ni-
hr. At what rate should the nurse program the troglycerin tablets did not relieve his anginal pain?
pump?

CASE STUDIES 5. The nurse asks the patient’s spouse to bring the
patient’s supply of sublingual nitroglycerin tablets
Case Study 1 in to the hospital. The patient’s spouse shows the
nurse a small plastic pill container. “Heart pills” is
A 74-year-old man with a history of stable angina,
handwritten on a masking-tape label. What teach-
asthma, and hypertension is admitted to the hospital with
ing does the nurse need to provide?
substernal chest pain that was not relieved by three sublin-
 
 
  
#

 
 


 

  
 @



 

drip.
6. The patient states that he guesses he will not need
1. What actions must the nurse take when adminis-
to get any more nitroglycerin sublingual tablets


 ƒ
now that he is using the nitroglycerin patches.
How should the nurse respond?

2. The patient’s pain is relieved after receiving a dos-


age of 330 mcg/min. He is admitted to the coro-
7. The nurse is reconciling medications during dis-
nary care unit. His vital signs are stable, and he is
charge instructions. The patient admits to adher-
currently without pain. Cardiac enzyme levels do
ence issues with taking his antihypertensive drug

  
  

 
!
Z>\


therapy because of experiencing erectile dysfunc-


normal sinus rhythm. The physicians believe that
tion. He asks what the nurse thinks about silde-
the patient is having classic angina and not a myo-
 
=Š  ?
 
$ 


 

cardial infarction. The attending physician orders


provide?
that the patient be weaned from the nitroglycerin
drip and that nitroglycerin transdermal patches
10 mg/24 hr be started. He is also prescribed

<]
 
 @%>&
=†`
 ?

 

= ?
^
 







 

#$

  
 


  

Case Study 2
of nitroglycerin when changing the patient from A farmer who lives in a remote area was admitted with
the nitroglycerin drip to the patch, when, in rela- chest pain. He has been diagnosed with angina and is be-


 


 

ing discharged with a prescription for sublingual nitro-
the nurse apply the nitroglycerin patch? glycerin tablets.

8. What teaching should the nurse provide about


administration and storage of this drug?

Copyright © 2016, Elsevier Inc. All Rights Reserved.


118 CHAPTER 52 Q Anticoagulant, Antiplatelet, and Thrombolytic Drugs

9. The prescriber has written instructions that this 11. The patient has been instructed to take one baby
patient should take the nitroglycerin before partici- aspirin tablet per day. The patient verbalizes that
pating in stressful activity. What teaching should this seems silly because one baby aspirin tablet
the nurse provide, considering that farm work will not do much to relieve his chest pain. How
includes operation of potentially dangerous equip- should the nurse respond?
ment?

10. The prescriber has instructed this patient to


continue to take nitroglycerin up to 3 tablets 5
minutes apart, take an aspirin, and to seek emer-
gency medical care if relief is not obtained after
  
$

 
   
 

tablet. The patient’s spouse states that an older


family member took nitroglycerin and had been
instructed to seek medical care if relief was not
experienced after taking 3 tablets. Why are the
prescriber’s instructions logical for this patient?

52
Anticoagulant, Antiplatelet, and Thrombolytic Drugs

STUDY QUESTIONS CRITICAL THINKING, PRIORITIZATION,


AND DELEGATION QUESTIONS
True or False
   
  T   F 7. A patient asks why heparin cannot be administered
   orally. What is the basis of the nurse’s response?
a. Heparin has a prolonged half-life when admin-
1. ___ Patients who take anticoagulant, antiplatelet, istered orally.
and thrombolytic drugs are at risk for bleed- b. Heparin can only be prepared as an oral solu-
ing. tion and is bitter tasting.
2. ___ Pregnant or nursing patients should never be c. Heparin is destroyed by proteases in the gastro-
prescribed heparin. intestinal tract.
3. ___ Anticoagulant and antiplatelet drugs only d. Heparin is large and negatively charged, limit-
affect the formation of clots, not removal. ing absorption.
4. ___ When given at prescribed doses, antico-
agulant and antiplatelet drugs can cause
unintended bleeding.
5. ___ *
  is a synonym for clot.
6. ___ "   means clustering.

Copyright © 2016, Elsevier Inc. All Rights Reserved.


CHAPTER 52 Q Anticoagulant, Antiplatelet, and Thrombolytic Drugs 119

6
<

$
 
  
 
 
=
13. Which aPTT result suggests that heparin therapy is

 


  


 ?
in the therapeutic range?
would be a priority to report to the prescriber if a a. 30-40 seconds
patient is prescribed heparin? b. 40-50 seconds
a. aPTT 75 seconds c. 60-80 seconds

–ˆ+
^^
 '‰ d. 90-120 seconds
c. Platelet count 40,000/mm3
d. WBC 11,000/mm3
]‹
# 
 
```


—



prescribed today. Doses are scheduled at 1200,


9. What should the nurse do when a patient who ]<``
^‹``

`—``
#$
$


has been receiving heparin tells the nurse that she recommendations, the nurse will enter a laboratory
thinks she could be pregnant? request for the aPTT specimen to be obtained at
a. Administer the heparin as ordered and notify what times?
the prescriber of the possible pregnancy status. a. 0800
b. Withhold the heparin. b. 0900 and 2100
c. Withhold the heparin until pregnancy status can c. 1000, 1600, and 2200

 d. 1100, 1700, 2300, and 0500
d. Withhold the heparin and consult the prescriber
regarding administration of the antidote prot- š
]




  
 
$

amine sulfate. 




 
 
=}#…?
$


$  

= ?



š
]`

 



 %
 % 
of greatest concern to the nurse after beginning
heparin therapy. What nursing teaching would be subcutaneous heparin therapy to prevent DVT?
best to prevent a common adverse effect of this a. Bruising at the injection site
type of therapy? b. Headache and faintness
a. Avoid crowds. 
| 

 
]``„
…
=†~<„
>?
b. Develop a plan for weight-bearing exercise as d. Pink saliva after brushing teeth
approved by the health care provider.

# 


 

 16. The nurse would consult the prescriber if which
d. Regular use of an over-the-counter antacid. drug was prescribed with no order for monitoring
of aPTT?
š
]]

 


$

%
 - a. Fragmin
tion at 1000 AM. Heparin 5000 units is scheduled b. Heparin


 
   
=  ˜?

^]``
c. Warfarin
the evening prior to surgery and 0900 the morning 
‰
of surgery. Which action by the nurse is appropri-
ate?
]~

 


  
=‰-
a. Administer both doses of the medication as ?ƒ
={

 
?
ordered. a. Cost of treatment is more than cost of treatment
b. Administer the 0900 dose and contact the pre- with unfractionated heparin.
scriber regarding the 2100 dose. b. Dosing may be based on patient’s weight.
c. Withhold the medication and contact the pre- c. May be used in combination with low-dose
scriber. aspirin in patients with unstable angina.
d. Administer the 2100 dose and contact the pre- d. Protamine sulfate is an effective antidote.
scriber regarding the 0900 dose. e. When ordered twice a day, should be adminis-
tered every 12 hours.

]^

]]`%
=`% ?
 


” 


subcutaneous dose of 4000 units of heparin has




 


{!!
|!!

is 155 seconds. The prescriber orders protamine


sulfate. What is the recommended dose of prot-
amine in this case?
a. 10 mg of protamine
b. 20 mg of protamine
c. 40 mg of protamine
d. 60 mg of protamine

Copyright © 2016, Elsevier Inc. All Rights Reserved.


120 CHAPTER 52 Q Anticoagulant, Antiplatelet, and Thrombolytic Drugs

6
]<

~‡% %
]ܠ%
=—% ?
$ 
 

24. Which assessment is of greatest priority when a
admitted with a hip fracture and is scheduled for  

  
 
=  ?ƒ

}#…
$



 



a. BP lying and sitting
priority to report to the prescriber if the patient is 
#²}
  
  
=‰?ƒ c. Pain with movement
a. A medical resident has ordered warfarin to be 
{
added in 2 days

\…

‰' 25. The nurse is teaching a male patient about ri-

#+
]^   
= ?
  
$
 



{ 
  



$
replacement surgery. Which statement made by
the surgery scheduled for tomorrow the patient suggests understanding of the teaching?

’#

 

$
  



19. Observations and research suggest that which razor.”


}!>
= ?
 
 

$$
$
 - 
’#






”

$ 
=> ?

 

 
$
- this drug.”
 ƒ
={

 
? 
’#



 


 





 
=!? when taking this drug.”

 

= ? 
’#$
#
 


$
 
#



>
=! ? just sit down and rest.”

\ '

€ @
=€  
   
 ? 26. While receiving an infusion of recombinant hu-
 
!
=!?

 
 
$
 

20. The nurse should teach patients who are pre- around the mouth and tightness in the chest.
 
 $ 
=> ?

 
  
Which action by the nurse should be performed

$

$= ?ƒ
={

 
?  ƒ
a. Citrus fruits a. Assess lung sounds.

‰ b. Assess vital signs.

\
 $
   
… 




 


d. Mayonnaise 
{

$ 
$

 
e. Red meat
27. Which statement, if made by a patient who has
š
^]
 


 
 

 

- been prescribed low-dose aspirin therapy, would
cautions before dental surgery when the patient is indicate a need for further teaching?
prescribed long-term warfarin therapy? 
’#


$

   
 
#

 -
a. No action is needed. ing aspirin therapy.”

#$

 
$

 

 - 
’#

"
 “
 
 @
 
=#+?
   
’#


 


 

-

{
 $ 
†
 
$
 
  ach acid while on aspirin therapy.”
d. Take half of the normal dose of warfarin for 3 
’#$
#
 

% 

#
‚


days before dental surgery. to worry about my stomach.”

^^
…


$
 $ 
=> ?
 


^<


 

=>{?
 



has been discontinued should the nurse maintain


  
 
=| ?

 %
precautions to prevent bleeding,?
brown liquid with dark brown particles. Which
a. 6 hours
action should the nurse take?
b. 8 to 12 hours
a. Complete a head-to-toe assessment.
c. 2.5 days
b. Consult the prescriber to request emesis Hema-
d. 5 days
test.

^†
#


$
  


 

c. Consult the prescriber regarding prescribing an
prescriber if the nurse notes which laboratory test antiemetic.
result for a patient who is prescribed dabigatran 
{
  
$

 
=| ?
=|   ?ƒ

‰!
—
#ˆ'‰

\…
=>>‰?

‰'

>\
^
#ˆ'‰

+ «
]‹`
Z"'‰

Copyright © 2016, Elsevier Inc. All Rights Reserved.


CHAPTER 52 Q Anticoagulant, Antiplatelet, and Thrombolytic Drugs 121

29. The initial digoxin level of a patient who is pre- 35. Which instruction is of greatest priority when
 
 
=‰ ?
`]^



 
teaching the family of a patient who is at risk for

 
=– ?
‡`


 

`—
myocardial infarction?
 '‰
–





 
a. Drugs have been developed to open clogged
What action should the nurse take? circulation.
a. Administer the digoxin as prescribed only. 
#

 
 

 

b. Administer the ticagrelor as prescribed only. c. They should seek immediate medical care when
c. Administer both drugs as prescribed. symptoms start.
d. Withhold both drugs and contact the prescriber. 
! 

 

  
 
$

bleeding.
š
†`

 
$$
$



 



recently been prescribed ticlopidine would be a


reason for the nurse to contact the prescriber im- DOSE CALCULATION QUESTIONS
mediately?
a. Change in level of consciousness
†—

 


```

$
 
 ˜

b. Diarrhea at 0900 develops hematemesis at 1030. The pre-


c. Dyspepsia scriber orders 50 mg of protamine sulfate via slow

#
   
=#Š?
 
”
| 
$ 


  

]`
 '‰
*
 




š
†]
!
 



  
 
$

nurse administer?
patient who has been prescribed ticlopidine for the
past week. The WBC count is 3000/mm3, and the
neutrophil count is 30%. What is a nursing priority
†~
!
 
  

$
 
$
 

for this patient? =   ?


$

^^`%
 
 
-
a. Assessing adequate cardiac output loon angioplasty. The drug comes in a solution

Z  
$$
 
   $
]^
 '^`
‰
!



   


c. Maintaining skin integrity milliliters per hour. What rate will the nurse enter
d. Preventing infection 




ƒ

†^

 



=|?

-
”

 
 

=|>#?
CASE STUDY
develops a hard lump at the access site and pain in
the leg. Which nursing action would be appropri- A 36-year-old, 132-lb woman whose job requires frequent



ƒ
={

 
? long airplane trips is admitted for treatment of a deep
a. Administer oxygen. 
  
=&Š!?


 
 
!
 ‚

b. Apply pressure over the sheath insertion site. spouse asks why the patient’s primary care provider did
c. Maintain bedrest. not prescribe low-dose aspirin to prevent DVTs.
d. Outline any bleeding on the dressing.
e. Prepare for a CAT scan. 1. How should the nurse respond?

††
 


 
  
   
$
-
 ' 
= ?ƒ
a. Cost
b. Dosing schedule 2. The patient is initially ordered a heparin drip by
c. Does not contain recommended dose of aspirin the emergency department physician. What is the
d. Does not contain recommended dose of dipyri- most common adverse effect of heparin, and what
damole symptoms of this adverse effect should be moni-
tored by the nurse?
34. The nurse teaches a patient who has been pre-
 
  @
=| ?
 

 
$


bleeding is increased if the patient includes which


food in the diet?
a. Cabbage
b. Dairy products

\ $
”

\
  

Copyright © 2016, Elsevier Inc. All Rights Reserved.


122 CHAPTER 53 Q
€  
$
{!%Z 
€  
#$ 

3. Heparin is pregnancy category C, which means 4. The patient’s primary care provider discontinues
that research studies suggest that risk to a develop- 

 


  
—`

 ˜


$
 



#$
 


every 12 hours. How much time should elapse be-
cross the placenta, why is it not categorized as tween the nurse discontinuing the heparin drip and
pregnancy class B?  

 ˜
  
=‰?ƒ
ƒ

53
Management of ST-Elevation Myocardial Infarction

STUDY QUESTIONS
<
#  

 

 -
diac muscle cells are damaged are
Completion called ____________________ and
____________________.
1. The medical term for a heart
attack is ____________________
‡
>
 
$
€#

-
____________________. ministering oxygen if O2 saturation is less than
____________________.

^
!
 
$

€#

-
plete blockage of the coronary artery is
____________________. CRITICAL THINKING, PRIORITIZATION,
AND DELEGATION QUESTIONS

†
!
’{!“

{!Z€#
 
$
 
$


____________________ ____________________
]`
–  
$

 





$

|˜{!
$

Z>\  
 

{!Z€#

 


expect the patient to exhibit


4. Myocardial infarctions where there is partial a. bradycardia.
blockage of the coronary artery are called non- b. edema.

€# 
  
d. rapid, deep respirations.


 
$ 
$
{!Z€#


___________________, ___________________,
]]
  
  

 = ?
= ?

___________________, ___________________, 



{!Z€#ƒ
={

 
?
___________________, ___________________, a. Administration of aspirin
and ___________________. b. Administration of a beta-blocker

  
$
 $
=+{#&?
6. Almost all coronary occlusions occur at the d. Administration of morphine
site of a ruptured ____________________ 
  
$

 
____________________. f. Administration of oxygen

7. Remodeling of the myocardium that occurs 12. What is an expected response within minutes of

€#



 

 

$
 




{!Z€#ƒ

___________________. ={

 
?
a. Decrease in pain

#
 

# 


$
  

# 
$
!
 

Z>\

Copyright © 2016, Elsevier Inc. All Rights Reserved.


CHAPTER 53 Q
€  
$
{!%Z 
€  
#$  123

]†




 
 
=Z&?

17. The nurse knows that nitroglycerin decreases the
to administer four chewable 81-mg aspirin tablets workload of the heart by
= 
†^‹
 ?

 

 
€#

a. dissolving existing clots.
student nurse asks the nurse why four chewable b. preventing clot formation.
aspirin tablets are administered instead of two c. slowing the heart rate.
  

 
=—`
 ?
 


 
d. reducing venous return to the heart.
 
  
$


$

 

dose of aspirin?
]<

 
$

 


{!Z€#



a. Aspirin is an acid, and acids are more readily prescribed alteplase, the nurse would question the
absorbed in the acid environment of the stom- prescriber if which drug was also prescribed?
ach. 
 
=|?
b. Chewable forms of aspirin are rapidly absorbed 
> 
=| ?
through the buccal mucosa. 
‰ 
=¢ ?
c. Chewing breaks the tablet into smaller par- 
‰
 
 
=‰€?
 
=‰-
ticles, which are more readily absorbed in the ?
intestines.

Z
†^%
 
 
$$ 

 - 19. Which laboratory result in the history of a patient
lation and antiplatelet effects of lower doses. 


{!Z€#


 

important to report to the prescriber who has just


6
]‹

 


 


{!Z€#


 
=> ?ƒ
prescribed metoprolol 50 mg by mouth every 6 
{!
]^`
  
 '‰
 
#





 
 

–ˆ+

 '‰

  
$

 


 - 
\…
†<
‰'
ent? 
!
!
`<
 '‰
a. Altered taste

# 
c. Rhinorrhea DOSE CALCULATION QUESTIONS
d. Wheezing

^`
# 
=#Š?

‹


  

š
]
!
Z&
 
 
 

 
$

 
$

{!Z€#
 
  


 '‰
*

who has a history of angina pectoris and chronic  



$



 ƒ
 
 
  
=>}|&?
!
 -
tient reports chest pain that has not been relieved
by three doses of nitroglycerin. The patient had 21. The drug book suggests diluting morphine to 4-5
been told by his physician that he should take ‰

$ 

‹%
 
!
 



chewable aspirin should this occur, but he does not $






 

 




have any aspirin. What should the nurse tell the 




`<

$



patient to do? push over what time?


a. Call 911.
b. Call his physician for directions.
c. Take ibuprofen and seek medical care. CASE STUDY
d. Turn on his home oxygen and rest.
A 57-year-old man who is vacationing locally is admit-

]—


  
$

{!Z€#





Z&


 
 
 
!
 

withheld and the prescriber consulted immediately radiates into his left arm, and he is experiencing diapho-
if the patient’s pulse is 118 beats/min?   
  

   
!
Z>\

 

a. Aspirin {!
 


$
  
!
!
=!!?


b. Atenolol elevated three times above lower limits, and the total cre-
c. Morphine 
 
=>?


>%€–
 @

 

d. Nitroglycerin  


!
  
  
{!Z€#
!
  



 
 
 

 
=|?

1. What assessments should the nurse monitor re-


lated to the risk of intracranial bleeding?

Copyright © 2016, Elsevier Inc. All Rights Reserved.


124 CHAPTER 54 Q Drugs for Hemophilia

2. The nurse is directed to interview the patient’s 


‰ 


 

 
$

]`

spouse. What information does the nurse need mg daily


to obtain to assist the physician with deciding if
thrombolytic therapy is appropriate?

4. Why is it important for the nurse to assess if this


 

 
 


 

 ƒ

3. The following medications are ordered. What is


the purpose of these drugs, and what assessment
should the nurse perform relating to administration
of these drugs? 5. The patient is being discharged with prescriptions
$

=‰ ?
]``



 


 
={?
<]%
 
  
‹
 -
 
=> ?



<
 



{!!
<]



 

 $ 
=> ?



once a day. The patient’s spouse asks why the


 
 
 

=#+?




# 
=#Š?

‹

$

week. How should the nurse respond?
2 mg every 10 minutes as needed to control
pain


€




^

$
†
6. When explaining warfarin therapy, the nurse
 

`

 

—

¬
‹<
 

 ‚
#+
  

 

hours recently were 1.8. The laboratory result form lists



 
#+

`‡

]]
*
 

 

 
 

  
#+

†

 



+ 


]`


 be abnormal?

54
Drugs for Hemophilia

STUDY QUESTIONS 4. ___ Hemophilia is inherited on the Y chromo-


some.
True or False 5. ___ Hemophilia has a risk factor of 50% for
every boy born to a mother who is a carrier
   
  T   F of the defective gene, no matter how many
   brothers have the disease.
6. ___ Hemophilia never occurs in females.
1. ___ Hemophilia always produces a severe bleed- 7. ___ 100% of daughters of hemophiliacs are car-
ing disorder. riers of the disease.
2. ___ Hemophilia can result from a spontaneous 8. ___ Treatment with clotting factor replacement
gene mutation. is costly.
3. ___ Hemophilia interferes with the formation of
a platelet plug.

Copyright © 2016, Elsevier Inc. All Rights Reserved.


CHAPTER 54 Q Drugs for Hemophilia 125

CRITICAL THINKING, PRIORITIZATION, 14. Calculations are available to predict patient re-
sponse to infusion of factor concentrates. For each
AND DELEGATION QUESTIONS
'
$
  
$ 
Š###
 


 ‚
 
$ 
Š###

 


6 9. The nurse is caring for a 3-year-old patient who


what percentage?


$

 

‰  
 

a. 0.1%
indicate that the patient has 0.4% of the normal
b. 0.2%

$

$ 
Š###



c. 1%
would be of most concern to the nurse?
d. 2%
a. Blood pressure 72/58 mm Hg

&$

 
15. When the prescriber is determining the proper
c. Pulse 100 beats/min
dose of factor concentrate, what is the most impor-

{

tant patient factor to consider?
a. Age
10. The nurse needs to give an intramuscular immuni-
b. Clinical response
zation to a patient with a history of mild hemo-
c. Target percentage of normal factor levels
philia B. What should the nurse do?
d. Weight
a. Administer the immunization as usual.
b. Administer the immunization subcutaneously.
š
]—
&  
 
 
]`


  

c. Apply pressure to the site for 5 minutes after


preoperatively for a 4-year-old patient with mild
the injection.
hemophilia A. The pharmacy only carries the
d. Administer only via an oral or nasal route.
spray that delivers 10 mcg/spray and is unable to
obtain any other concentration. What should the
11. The parent of a child with hemophilia calls the pe-
nurse do?
  ‚
$
!
 

$ 


a. Administer 15 sprays of the medication.


medication can be given to the child for mild pain
b. Administer 150 mcg intravenously.
and fever. The telephone triage nurse follows
c. Consult with the prescriber regarding the fact
protocol and explains that the safest choice for
that the pharmacy does not carry a needed drug.
hemophiliacs is which drug?
d. Consult with the prescriber regarding an oral

 
=!?
dose order.

 

= ?

>
=> ?
6 17. The nurse is administering factor concentrate to a

# $
=€?
child who begins to experience swelling around
the face. What is of greatest priority and should be
š
]^
 
 
  




 

$
 ƒ
with hemophilia?

 
 
=– ?
a. Anxiety

 
  

=Z|?

#$$


{

$ 
c. Risk for altered growth

{

$ 
d. Risk for injury
18. Aminocaproic acid solution may be prescribed to
13. Calculations are available to predict patient re-
prevent bleeding from dental care for patients with
sponse to infusion of factor concentrates. For each
 
= ?ƒ
'
$
  
$ 

 


a. Hemophilia A and B
 ‚
 
$ 


 

 

b. Hemophilia A only
percentage?
c. Hemophilia B only
a. 0.1%

{



–

b. 0.2%
c. 1%
19. The nurse knows that inhibitor antibodies to factor
d. 2%
concentrate are most likely to occur in hemophili-

$

 ƒ
={

 
?
a. African American
b. Asian
c. Hispanic

€
Z 
e. Native American

Copyright © 2016, Elsevier Inc. All Rights Reserved.


126 CHAPTER 54 Q Drugs for Hemophilia

^`


 


$ 
Š##
2. The patient’s parents ask why aspirin should not
=+{?

 


 $
- be given to their child. What should the nurse
sessed for what? explain?
a. Allergy to pork
b. Anorexia and nausea
c. Chest pain and shortness of breath
d. Muscle weakness and fatigue 3. The nurse provides a comprehensive list of over-
the-counter drugs that contain salicylate. Which
of these drugs would the nurse include on the
DOSE CALCULATION QUESTIONS  ƒ

{@
 
  
 

 
& 
& ‚
  
& 
Z-
21. A nurse in the perioperative area is caring for a pirin, Keygesic, Kaopectate, Momentum, Pepto-
12-year-old child with hemophilia A who weighs – 
{
  ‚


! 
99 lb. What is the recommended number of units
$
  
$ 
Š###
 

  


child’s level to 50%?

4. The home health nurse goes to the patient’s home

^^
&  
£&&Š|¤
`†
   '

to teach the parents how to administer recombi-
prescribed for a 66-lb child. The drug is available  
$ 
Š###
 


 





$
‹
 '‰


$


catheter. What are the nursing diagnoses that the
0.9% normal saline and infused over 30 minutes. nurse should consider when developing the plan of
*
 



 


$
ƒ
care for this family?
 
 


  




$

the solution of desmopressin was further diluted in


`

$
`‡œ
 
ƒ

5. Administering factor concentrate carries a risk of


complications. What should the nurse teach the
CASE STUDY caregiver to prepare for possible complications?

A 17-month-old child has been newly diagnosed with he-


mophilia A. Review of the chart indicates that the patient
has not been immunized for hepatitis A.

1. What teaching needs to be provided relating to


hepatitis immunization, and what information
relating to immunizations does the nurse need to
obtain regarding caregivers?

Copyright © 2016, Elsevier Inc. All Rights Reserved.


CHAPTER 55 Q Drugs for Deficiency Anemias 127

55
     

STUDY QUESTIONS 


     

Matching
]‹

Z 


]

‰  
  

     
]—

‰  

17. ___ Pale

]

&



=–>?

 

]<

{ % @

after incorporating hemoglobin
2. ___ Developing RBC in bone marrow before a. Hyperchromic
incorporating hemoglobin b. Hypochromic
c. Macrocyte

†

# 
 d. Megaloblast
4. ___ Oxygen-storing molecule of muscle e. Microcytic
5. ___ Measures color of RBCs
6. ___ Measures size of RBCs CRITICAL THINKING, PRIORITIZATION,
7. ___ Oxygen-carrying protein of RBCs
AND DELEGATION QUESTIONS
8. ___ Percentage of RBCs in a volume of blood š
]‡
!
 

 
$

 
 


-

‡

{  
  
$
  $

 mic who is receiving chemotherapy that prevents
the reproduction of rapidly dividing cells, includ-

]`

{ 
$
$


 
 ing the cancer cells, hair, RBCs, and epithelium of

]]

{ 
$
$



  
  
=\#?
 

  

result suggests that the anemia is getting worse?


12. ___ Transports absorbed iron to bone marrow

*  
]`
 '‰
13. ___ Variation in size of RBC b. Hematocrit 26%

–>
†<
¬
]`12'‰

Z   d. Reticulocyte count 0.002%
b. Ferritin
c. Hematocrit 6
^`
#



$

 


$


d. Hemoglobin of anemia in a patient who has a history of what


e. Hemosiderin condition?
$
#% 
  
=#–>?
£ 
#–>
a. Chronic obstructive pulmonary disease
=!#–>?¤ b. Chronic kidney disease
g. Mean corpuscular hemoglobin concentra- c. Diabetes insipidus

=€>*>? d. Hypertension

€ 
  

=€>Š?
i. Myoglobin 21. Normally the body prevents excessive buildup of
j. Proerythroblast iron in the body by




  

=&? a. decreasing intestinal absorption of iron.
l. Reticulocyte b. increasing metabolism of RBCs.
m. Transferrin c. increasing excretion of RBCs in bile.
d. increasing excretion of RBCs in urine.

Copyright © 2016, Elsevier Inc. All Rights Reserved.


128 CHAPTER 55 Q Drugs for Deficiency Anemias

^^

$= ?


 


 
š
^<
!
 


 


 



teaching for pregnant women regarding prevention receiving an oral iron preparation. The patient’s
$
%
 ƒ
={

 
? stool is greenish-black. What should the nurse do?
a. Carrots, red peppers, squash a. Administer a dose of the iron preparation.
b. Cheese, milk, yogurt b. Consult the prescriber.

>
 

 
c. Determine if this stool is usual or a change for

Z

  this patient.
e. Oranges, apples, pears d. Hold the iron preparation.

^†
#
%
 

 


6 29. A patient with peptic ulcer disease has developed
which test to be elevated from normal levels? 
 
#
 
=&…?

” 


* 
=*? been prescribed. The nurse has administered a test

*  
=*

* ?  
#



$

 

 



=! ?
% 
  
=#–>

!#–>? patient for what adverse effect?

€ 
  

=€>Š? 
Z
$

$ 

 
b. Hives
24. What is a common nursing diagnosis for patients c. Phlebitis at the site of injection

%
 ƒ d. Tissue damage from extravasation
a. Body image disturbance
b. Breathing pattern, ineffective 30. When iron must be administered by injection,
c. Cardiac output, decreased which method of administration should be used ?
d. Fatigue 
#  

#  
š
^
!

 


 


$% 
{  
grade teacher regarding a student who is pale and 
¢% 
   
tires excessively as the day progresses, and whose
academic performance has been declining. What 6 31. The dialysis nurse has administered iron dextran
action would be appropriate at this time? and erythropoietin. What is a priority nursing
a. Contact the child’s parents and inform them concern relating to adverse effects?
that their child is anemic. 
\   
 
b. Contact the child’s parents and tell them their 
#$ 

 $
child needs to be seen by their pediatrician. c. Tissue perfusion
c. Make an appointment, along with the teacher, d. Taste alterations
to speak with the parents about noted concerns.
d. Tell the child that you think he/she is anemic 6 32. A patient with chronic kidney disease is prescribed
and should tell his/her parents. $
=… ?
#






inform the consulting physician of this drug if he


26. What is the most common adverse effect of ad- or she orders which diagnostic test?
ministration of ferrous sulfate? 
>
  
=>!?
a. Discoloration of teeth 
€ 
  
 
=€#?
b. Heartburn 
| 
 
  
=|Z!?
c. Hypotension 
ˆ 
d. Weak, rapid pulse
33. Which foods are good sources of vitamin B12?
27. A patient complains of intolerable nausea and a. Organ meats
heartburn from taking an oral iron preparation. b. Fruits
What should the nurse instruct the patient to do? 
\ 
a. Consult with the prescriber. d. Vegetables
b. Take a calcium carbonate antacid with the iron.
c. Take the iron with milk. 6
†‹
#




 

 



d. Take the iron with orange juice. anemia to report which issue?
a. Fatigue
b. Joint pain
c. Paresthesias

! 
³]``‹„
…
=†<„
>?

Copyright © 2016, Elsevier Inc. All Rights Reserved.


CHAPTER 55 Q Drugs for Deficiency Anemias 129

š
†

 



 
$
2. What should the nurse teach the parents about fer-
numbness and tingling of the hands and feet. What rous sulfate therapy?
data relating to possible neurologic damage should
the nurse collect before notifying the prescriber of
the patient’s complaint?
a. Blood pressure
Case Study 2

&


c. Capillary glucose A 28-year-old woman has had gastric bypass surgery.

{
 {

 




 
^
 
{




 
 

=|>|?

 
$
$  

36. To prevent damage to the developing neural tube 


 
= 
$

 ?
of a fetus, it is important for women of child-
bearing age to have adequate daily intake of folate, 3. Because of a history of this surgery, the patient is

 

$


$ ƒ
={

at risk for what type of anemia? Why?
 
?
a. Asparagus and spinach

Z



 

Z
 

% 
  4. The patient is prescribed parenteral injections of

‰

 vitamin B12
=†`
 ?

$

^``

 


‰

 $
]

{



 
  -
amin 10,000 mcg/day and folic acid 400 mcg/day.
37. When a patient is prescribed drugs that increase The patient tells the nurse that she had learned that
the production of RBCs, it is a priority to monitor oral administration of cyanocobalamin is ineffec-
the effect of changes in serum potassium on which tive. How should the nurse respond?
system?
a. Cardiovascular system

Z
 

\   
 
d. Pulmonary system 5. The patient discusses concerns about taking “all of
these pills” every day. The prescriber discontinues
the oral cyanocobalamin and prescribes intrana-
DOSE CALCULATION QUESTIONS 
   
=+  ?

 



nostril once a week. What teaching will the nurse


38. The pediatrician prescribes ferrous sulfate drops provide about intranasal administration of the
25 mg twice a day. The child weighs 22 lb, and drug?


 


 ' ' 
#

 

safe and effective?

6. The nurse teaches the patient to report which


39. Ferrous sulfate drops are available as 75 mg in symptoms that suggest the adverse effect of hypo-
`—
‰
*



 
 
kalemia that is possible when taking cyanocobala-
with each dose? 
= 
–12?ƒ

CASE STUDIES
Case Study 1
An 11-month-old child, who drinks 8-ounce bottles
of whole milk 5-6 times a day, is diagnosed with iron-

 

1. What can the nurse teach the parents about factors


that could be contributing to the anemia and mea-
sures to change the factors that can be altered?

Copyright © 2016, Elsevier Inc. All Rights Reserved.


130 CHAPTER 56 Q Hematopoietic Agents

56
Hematopoietic Agents

STUDY QUESTIONS CRITICAL THINKING, PRIORITIZATION,


AND DELEGATION QUESTIONS
Completion

]
#



 



‡


 



$

secreted by cells of many organs including greatest priority to report to the prescriber when a
_________________, _________________,  




=|?ƒ
_________________, _________________, a. Chest pain
_________________, _________________, and b. Fatigue
_________________. c. Pallor
d. Weak dorsalis pedis pulse

^
Z  %  

=Z{ ?
 

shorten life in certain _________________ pa- 10. A patient with a history of type 2 diabetes mel-
tients. 
=!^&€?



  
=>&?





=|?
  

$ -

†
Z


 


 

ing kidneys do not produce adequate amounts of
_________________ and _________________ erythropoietin, and she has become anemic. What

  

 
 
 
]]
'‰ would be a realistic goal for therapy?

*
<
'‰Œ
  $
^œŒ
$
‡`
 '‰
4. Dosage of epoetin alfa should be reduced when 
*
]`
'‰Œ
  $
^`œŒ
$
]^
 '
  
 


 

'‰


two weeks. 
*
]^
'‰Œ
  $
]`œŒ
$
<`
 '‰

* ]‹
'‰Œ
  $
]œŒ
$
]``
 '



 
  
=+ ?


nurse should review the WBC count ______ times
per week. 11. When darbepoetin alfa is administered preopera-
tively to reduce the need for blood transfusion
6. Allergy to _________________ would be a con- postoperatively, the nurse would question the
traindication to administration of sargramostim prescriber if which drug was not also prescribed?
=‰? a. Ferrous sulfate
b. Folate

~
 
=+ ?‚



c. Furosemide
_________________ platelet production. d. Heparin

8. The nurse needs to assess for shortness of breath, 6


]^
#








  

chest pain, and change in level of consciousness $


  

=  ?

 

when a patient is prescribed the thrombopoietin  ƒ



 
=! ?
  
 

a. Blood pressure 170/98 mm Hg
increase the risk of _________________. b. Pulse 100 beats/min
c. Respirations 24

! 
]`]‹„
…
=†<„
>?

Copyright © 2016, Elsevier Inc. All Rights Reserved.


CHAPTER 56 Q Hematopoietic Agents 131

š
]†

 
 




-
]<
  
$
 
   
=+  ?

”


 

$


=|?
  
$
  
=+ ?

 ƒ
is due for the next dose. The patient’s hemoglobin 
#
 

 
 
$


has not increased. What should the nurse do? after chemotherapy.
a. Hold the next dose. b. The course of therapy is one injection lasting 2
b. Administer the medication as ordered. weeks.
c. Consult the prescriber regarding assessing for 
#


  

 
renal failure. 
!


 
$
‰&*
 
 -
d. Consult the prescriber regarding assessing for phatase, and uric acid.
neutralizing antibodies.
6 19. What would be a symptom of an extremely rare
š
]‹
Z

=|?
‹```

  

  
 
 

 
 

is ordered at 0900 pending endogenous erythro- $


   
=‰?ƒ


$


*#Š%$
 

> 
=  ?

@
=¢!?
‰
 



\ 
=?
0830, including endogenous erythropoietin level c. Pleural friction rub
$
~<
 '‰
 


 
ƒ d. Wheezes

 



=|?
{!!

 



=|?



^`
#$

 
 
$
   
=‰-
0900 medications. ?




 
]`
 '‰



c. Hold the medication and consult the prescriber. should be diluted in


d. Hold the medication. a. 0.9% sodium chloride.

`‡œ



^`
Z"
>
6
]
Z 
$




 
$$
$
c. 0.9% sodium chloride with 0.1% albumin.
  
=+ ?
 
 



d. 0.9% sodium chloride with 5% dextrose in
assessment for the nurse to evaluate relating to water.
possible elevations of uric acid?

Z
^]
 




=+ ?ƒ
={


Z 
  
 
?
c. Joint redness and swelling 
#
 

$
 
  
d. Tachycardia 
#
 


 
$
 %
karyocytes.
š
]—
!
 

 
  
$
  

#

 

 


 

=+ ?


 
$



 - adults.
tal neutropenia who weighs 88 lb. The prescribed 
#




$



 

^‹`


 ' ?
!
 


#
 




 
  


 
$
†``
 '‰


]—%‰
  
!
 

 

 

6 22. Which of these adverse effects of oprelvekin
administer which dosage? =+ ?


$
  





`—
‰
  

 
$

 
  to the prescriber?

`—
‰
  
^
 
$
 
  a. Dyspnea

`<
‰
  

 
$

 
  b. Peripheral edema

`<
‰
  
^
 
$
 
  c. Rash
d. Red eyes

]~
  

$
  
=+-
?
$

 

 

6 23. What is a priority nursing concern for a patient
which directive? 
 
 

=#!|?

a. Administering after each dose of chemotherapy 



  
 
=|  ?ƒ
b. Consulting the prescriber once absolute neutro- a. Fluid volume
phil count has reached 10,000/mm3 
# 
 
c. Keeping the drug refrigerated at all times c. Nutritional needs
d. Wiping the rubber stopper with alcohol when 
{ $
reentering the vial

Copyright © 2016, Elsevier Inc. All Rights Reserved.


132 CHAPTER 56 Q Hematopoietic Agents

^‹
#

 



 


” 

e. RBC indices
  
 
=|  ?

 
$


%%
 ƒ
={

 

? f. Ferritin
a. Analgesics
b. Antacids
c. Cough medicines
d. Decongestants g. Transferrin saturation

‰  


ˆ

DOSE CALCULATION QUESTIONS

^

 

  
   
=‰?

250 mcg/m2 after a bone marrow transplant. The


patient weighs 50 kg and is 163 cm tall. The phar- 2. What adverse effects should the nurse assess in
 

   
=‰?
``
 ' this patient?
‰


 



 
$

]`
 '‰
*

   
=‰?

should be administered?
3. The nurse is reviewing laboratory results for this
 
*  

]†
'‰
 



^—
!
 


   

‰'
nurse do?
!
   
=‰?




$ 

over 4 hours. What rate should the nurse program


into the pump?
Case Study 2

 

  
   
=‰?
^`
 '
CASE STUDIES m2 after a bone marrow transplant.
Case Study 1
4. What is the rationale for this therapy?
A 65-year-old male patient has had T2DM for 40 years
and CKD managed with hemodialysis for 5 years. He is




  

$
=|?

three times a week to help maintain his erythrocyte count.


5. What should the nurse do or not do if the patient
The prescriber has ordered his periodic laboratory tests,
experiences these adverse effects?

–ˆ+
 
  
 
–>

indices, hemoglobin, transferrin saturation, ferritin, and


uric acid. The patient is tired of “being jabbed” and asks
the nurse why these tests have to be done. a. Diarrhea

1. Describe the rationale for each test in a way the


patient can understand.
b. Weakness

–ˆ+' 

c. Rash
b. Phosphorus

d. Bone pain
c. Potassium

d. Hemoglobin

Copyright © 2016, Elsevier Inc. All Rights Reserved.


CHAPTER 57 Q Drugs for Diabetes Mellitus 133

57
Drugs for Diabetes Mellitus

STUDY QUESTIONS 15. Which antihypertensive drug might be prescribed


for a patient with DM who does not have hyper-
True or False tension?

– @
=‰ ?
   
  T   F 
> 
=> ?
   
& @
=> @?

]

\


  
$

… 
=‰ ?
carbohydrates, proteins, and fats.

^

& 
$$




   š
]—
 


 

$

 

3. ___ Foods with a low glycemic index have fewer 


 
=  ?
$

 


carbohydrates than those with a high glyce- DM?


mic index. 



 
4. ___ Blood sugar control will best be achieved b. No adventitious lung sounds
if the patient follows a plan created by the c. Palpable peripheral pulse
health care providers. 
{$
 




Z 

 
  

 -
lin. 17. What would be the most appropriate nursing
6. ___ Clear insulin is always short-acting. intervention for a 23-year-old patient who, for the

~

# 
 


 % 

 



$ 
 
 

$

that mimics the body’s basal insulin secre- ]‹


 '‰ƒ
tion and another short-acting agent to cover a. Advise the patient to include 30 minutes of
eating most closely mimics normal function- vigorous exercise in his daily activities.
ing. b. Discuss possible diet changes with the patient.
8. ___ Once the patient’s blood sugar is stabilized, 
Z 


$
 
 
 -
the patient will be able to maintain control tion to the patient.
with oral drugs. d. Teach the patient how to do a urine dip for

‡

{  
$ 





 
glucose and ketones.
10% of the total calories.
10. ___ Administering insulin or sulfonylurea drugs 18. Which type of diabetes often exists for years
and not eating can cause serious effects from before diagnosis, but fasting blood glucose is not
the blood sugar going too low. elevated because of hyperinsulinemia?
11. ___ Tight glucose control decreases the inci- 
\   
 
dence of kidney failure. b. Juvenile diabetes
12. ___ Treatment can be monitored by blood or urine. c. Type 1 diabetes
13. ___ Weight loss is always needed to decrease the d. Type 2 diabetes
patient’s insulin requirements.
6
]‡
#




$

 

 


which symptoms if exhibited by a patient who is


CRITICAL THINKING, PRIORITIZATION, receiving insulin therapy for diabetes?
AND DELEGATION QUESTIONS a. Fatigue and blurred vision
b. Perineal itching and copious urine
14. Which complication of diabetes causes the most

|$ 
 

$

 
deaths?
d. Thirst and constant hunger
a. Cardiovascular effects
b. Hypoglycemia
c. Ketoacidosis
d. Kidney damage

Copyright © 2016, Elsevier Inc. All Rights Reserved.


134 CHAPTER 57 Q Drugs for Diabetes Mellitus

20. A patient who has type 2 DM has been unable to š


^‹

  


 
$
+|*
 
$

follow the recommended diet and exercise regi- the refrigerator, the nurse notes that the suspension
men. He tries to alter his laboratory test results by is partially frozen. What should the nurse do?
eating less than usual before having blood testing a. Discard the vial and obtain a new one from the
performed. Which test would be most accurate for pharmacy.
this patient because it evaluates his glucose control 
{ 

 
  


 
over the past 3 months? c. Warm the suspension in warm water.
a. Fasting glucose d. Withdraw the unfrozen portion then discard the
b. A1c rest.
c. Postprandial glucose
d. Two-hour glucose tolerance š
^

 




 
$
 
 -
dosis. The resident orders insulin detemir 0.1 mg/
21. A patient with diabetes who has been using  '


 

 
=#Š?


traditional insulin therapy has been prescribed What should the nurse do?
intensive insulin therapy to achieve tighter glucose a. Calculate the insulin dose and mix it with
control. Which information should be included in ]``

$
 

the teaching? b. Calculate the insulin dose and mix it with
a. An insulin pump is used to provide the best ]``

$
&5W.
glucose control and requires about the same c. Calculate the insulin dose and infuse the solu-
amount of attention as intensive insulin therapy. tion prepared by the pharmacy.

# 
 
 
  
"
$

> 

  
{!!
injections of a rapid-acting insulin each day in
addition to an injection of a basal insulin. š
^—
!
 



 
 

-

# 
 
 

 

$
signed patients and is preparing to administer pre-
newly diagnosed type 1 diabetics who have scribed insulin. All patients are alert and oriented.
never experienced ketoacidosis. #

<•`~
AM, and breakfast trays are scheduled to

{
 

 
 
 
arrive at 8:45 AM. The nurse should administer the
therapy is most effective in preventing the mac-  
 


 


rovascular complications characteristic of type a. 8:00 AM
{€–\
]``
 '‰Œ
  
 

2 DM.  
=+‰ ?


b. 8:00 AM
{€–\
]]`
 '‰Œ
  
 

š
^^
!
 
 
$


$

 
= ?
~

electrolyte imbalance that is most likely when a c. 8:00 AM
{€–\
<`
 '‰Œ
  
+|*

patient is receiving large doses of insulin?  


=+
+?
†‹

a. Muscle weakness and constipation d. 8:00 AM
{€–\
‡`
 '‰Œ
  
  

b. Restlessness and irritability  


=*
?



{ 
$

 

 

 -
tion of the upper arm is constricted for several 27. A patient has been prescribed 5 units of insulin
minutes  
=+‰ ?



$
 


d. Twitching of the facial nerve when the face is =‰?




 

`<``
 

tapped over the nerve should the nurse do?


a. Draw up the two insulins in different syringes.

^†

 
 
†`

$
  
=‰  ?

& 



$
 
 
 


insulin in the same syringe as 8 units of aspart units of detemir insulin in the same syringe.
=*  ?
 
 

 

 
c. Draw up the clear insulin, then the cloudy insu-
would need only one injection. The nurse contacts lin, in the same syringe.
the prescriber and monitors the patient for which 
& 



$

 
 
 


symptom caused by the effects of this combination units aspart insulin in the same syringe.
on the absorption of the insulin?
a. Profuse sweating 28. What special administration techniques must the

# nurse use when administering NPH insulin?
c. Thirst a. Never mix with another insulin.
d. Vomiting b. Administer this insulin only at bedtime.
c. Roll the vial gently to mix particles in solution.
d. When mixing with another insulin, draw the
+|*


 
 

Copyright © 2016, Elsevier Inc. All Rights Reserved.


CHAPTER 57 Q Drugs for Diabetes Mellitus 135

29. The nurse is caring for a patient who is NPO for 33. A patient with type 2 DM that is controlled with
a diagnostic test. Which dose of insulin should be 

$
=\ ?
 



administered, if ordered, even if the patient is not  



=?



  

eating?  


=& ?


  

a. 8:00 AM
{€–\
]``
 '‰Œ
  
 
What is most likely to occur?
 
=+‰ ?



+|*
=+‰ ?
a. Development of acute hypoglycemia because
34 units of the RA exacerbation
b. 8:00 AM
{€–\
]‡`
 '‰Œ
  
  
b. Development of a rash caused by metformin-
 
=*
?
‹



 
prednisone interactions
coverage c. Needing a diet higher in calories while receiv-
c. 8:00 AM
{€–\
]]`
 '‰Œ
  
 
ing prednisone
detemir 26 units d. Prescription of insulin on a sliding scale while
d. 8:00 AM
{€–\
<`
 '‰Œ
  
+|*
on prednisone
 
=+
+?
†‹


š
†‹
!
 
 
 

 
$

 

30. A patient is prescribed 4 units of insulin aspart student who has type 2 DM who was brought to
=+‰ ?




 
 

the health clinic after getting intoxicated at a party.
{€–\
  
 

 
`

!


  
$
=\ ?


`


]``


]
‰



850 mg once a day. The nurse assesses the student.
 
 

`



`%‰
 

What assessment would be of most concern to the
administer this dose? nurse?

#



 
 
 
 
 - 
& 

 
$

  
sure this dose. b. Respirations 32 breaths per minute and deep

#



 
 




c. Pulse 100 beats/min
for administering this dose. 
{
$ 
  

#

 

 

 
 


syringe. 35. After receiving teaching about the drug glipizide



#

 




 

‹

=\?

 
$
 


 -
on this syringe. tient, would suggest that the patient understood the
teaching?
31. Research suggests that intensive insulin therapy 
’#


$


$
#
 
 

= 
?


]

^


 
$$- of sweating and shakiness.”
tive in preventing 
’#


 


 




   

=>Š ? drug because it does not stimulate the release of
b. neuropathic pain. insulin.”
c. peripheral vascular disease requiring amputa- 
’#


 

 
 
#
 



#

tion. take enough of the drug.”


d. ophthalmic complications. d. “This drug is less potent than the newer drugs.”

6 32. When performing the initial morning assessment 36. A patient with type 2 DM who is prescribed glipi-
on a patient with diabetes, the nurse notes that @

” 

  
 
=> ?

the patient is diaphoretic and confused. The nurse Which statement suggests that the patient needs


{€–\


†~
 '‰
 


additional teaching?
priority assessment before administering orange 
’#



{€–\
$
 


juice? 



#
‚
$
 “
a. Blood pressure b. “This drug can make my low blood sugar

&

 worse.”

{ 
 
’#



 


 


d. Temperature }
$
#
‚
$

  
 
#
  




{€–\

“

’#

 

 
$
 


carvedilol and consult my doctor if it is less


than 60 beats/min.”

Copyright © 2016, Elsevier Inc. All Rights Reserved.


136 CHAPTER 57 Q Drugs for Diabetes Mellitus

37. Which laboratory test result increases the risk that 43. Which nursing action is appropriate when a patient
a patient who is prescribed glyburide might expe- 
  

=> ?
$

^

rience hypoglycemia? DM?




|
]‹^
  
 '‰ a. Change positions slowly.

–ˆ+

 '‰ 
Z 

 
$


$
†`

$


\…
‹^
‰' eating.

\—|&
^‹
ˆ'
$
   
€

 


d. Weigh daily.
38. Which assessment change from yesterday morning


 


  
  @
= ?
44. What are the only drugs currently used to treat
would be of most concern to the nurse? 
^


  

  ƒ
={-
a. Abdominal pain and two loose stools this 

 
?
morning 
   
=| ?

> 
 



^1§2-lb 
\@
=\?
weight gain 
# 
c. Decrease in BP from 130/82 to 118/78 mm Hg 
€$
=\ ?
and pulse drop from 82 to 74 beats/min 
|  @
= ?
d. Temperature 102.4° F and cough
45. After administration of glucagon for severe hypo-
š
†‡
 

  

 

 
glycemia, what is the priority goal of therapy?
 
=| ?

 

- a. Correct acidosis.
scriber? b. Correct cachexia.

… 

 
‡
 '‰ c. Restore stores of glycogen in the liver.

\  
  
=]?
œ d. Treat the adverse effect of diarrhea.
c. Patient is NPO for a colonoscopy
d. Patient is scheduled for hemodialysis at 1000  46. The nurse is preparing to perform the last assess-

$

]‡``

`~``
$

`—``
{ 

40. Which nursing outcome would be most appro- assigned patients have diabetes. The patient
priate for a patient who is prescribed acarbose receiving which drug should be assessed for hypo-
=| ?

  
={ ?ƒ 
 ƒ

–
{€–\
]—`
 '‰ 
   
=| ?
`

†


 



… 

 

 
‡`
 '‰ meals

\  
  
=]?
~œ 
\  
=‰ ?
 




d. Two-hour postprandial blood sugar less than 


 
 
=+‰ ?



]`
 '‰  
 

{€–\

€$
=\ ?
``

$
 -
 41. A hospitalized patient who has diabetes and who fast and supper




   
=| ?
`

†

+ 
 
* 
=+|*?
=*

times a day with meals. Because of possible ad- +?


†



verse effects of the drug, the patient is most likely
at risk for which nursing problem?
a. Activity intolerance DOSE CALCULATION QUESTIONS

#$$


#” 47. A 132-pound patient is prescribed an intravenous
d. Powerlessness infusion of regular insulin at 0.1 units/kg/hr, which
calculates to 6 units per hour. The insulin comes
 42. A patient with diabetes who receives glipizide and 



$



]``

$
-
 
=\ ?

 
  

 

!


   


-
 
!
{€–\

ܠ
 '‰
 

ters per hour. What rate should the nurse program
would be most effective to quickly raise the pa- $



 
ƒ
tient’s blood sugar?

\ 
 
b. Honey
c. Orange juice

{ 
 

Copyright © 2016, Elsevier Inc. All Rights Reserved.


CHAPTER 58 Q Drugs for Thyroid Disorders 137

CASE STUDY 4. What is the disadvantage of using basal glargine


=‰  ?
 ƒ
The nurse educator is presenting a seminar on insulin
to nursing students. What would be the best response to
these questions?

1. Why are regular and rapid-acting insulins the only


insulins administered intravenously?

{
  


  
=‰  ?

insulin in the morning rather than in the evening


as the literature recommends. Why would they do
this?

^
#$
  
 

 

 
 


do doctors prescribe rapid-acting insulins such



 
 
=+‰ ?

 
 

= ?ƒ 6. Patients sometimes need both basal and bolus


doses of insulin at the same time. What could
 
$



=‰?

  

=‰  ?
 





$
 

 
=+‰ ?


 %

$ % 

insulins?

†
*

 

=‰?

  
=‰ -
 ?
 

 
 
+|*
 ƒ

7. What should the patient or nurse do if a vial of


insulin aspart is cloudy?

58
Drugs for Thyroid Disorders

STUDY QUESTIONS 4. ___ Maternal hypothyroidism has the most


  
$$


$




True or False trimester when most fetal weight is gained.


5. ___ Assessment for congenital hypothyroidism
   
  T   F = ?





$ ‚

   2-month checkup.



—

Z 





 
$

1. ___ The 3 and 4 in T3 and T4 indicate the number hyperthyroidism.


of atoms of iodine in the molecule. 7. ___ Natural thyroid hormone products from

^

‰
 



$$
$
animals are more likely to contain the exact
thyroid hormones. amount of thyroid hormone than synthetic
3. ___ Free T4 is the count of T4 hormone not hormone replacement products.
bound by protein.

Copyright © 2016, Elsevier Inc. All Rights Reserved.


138 CHAPTER 58 Q Drugs for Thyroid Disorders

<

#


 

 


6
]‹

 



 
 

replacement at the same time each day than for the nurse to report to the prescriber immedi-
with or without food. ately if noted in a patient admitted with urosepsis

‡

–  
!{*

 
$




 
$
\  ‚
  
 



hormone replacement is taken by a euthy- antithyroid drug?


roid person who is trying to lose weight, a a. Blood in urine on urinalysis
dose that produces the adverse effects of b. Burning when urinating
hyperthyroidism would be needed to achieve c. Flank pain
weight loss. 
! 
]`‹„
…
=‹`„
>?

15. The nurse is caring for a patient who receives


CRITICAL THINKING, PRIORITIZATION, levothyroxine. Which OTC drugs should not be
AND DELEGATION QUESTIONS administered within hours of the administration of
ƒ
={

 
?
š
]`
!
 

 

 

- 


=€ ?

={?

  
 

> 
   
=} > ?
$
  




 
$


>
=! 
*–?
nurse to withhold the medication and consult the 
…
$ 
=
?
prescriber? 
€
$
  
=€}€?
a. Free T3
b. Free T4 š
]—
–  
$

  

 



!{* monitor a patient who is taking warfarin and levo-
d. T3 thyroxine for which condition?
a. Bleeding
6
]]
#



$

 

 


b. Dysrhythmias
pregnant woman that adequate thyroid hormone 
# 
replacement therapy is critical during which stage d. Tachycardia
of the pregnancy?
a. First trimester 17. Which statement, if made by a patient who has

{
  type 1 diabetes mellitus and was recently diag-
c. Third trimester nosed with hypothyroidism and prescribed levo-
d. Postpartum period 
={?

 


$

further teaching?
6 12. Which would be of greatest priority when a patient 
’#




 
$
#

 

is admitted in thyrotoxicosis?  





$
 “

‰

 
  
’#





 

#

 -
b. Providing adequate nutrition 
{“

  


$
!{* 
’#


 




 


{ 


 before meals and at bedtime.”

’#



$

{


13. Which statement, if made by a patient receiv- off in my blood and my basal insulin dose can




 
=|!ˆ?
be adjusted accordingly.”
would indicate a need for further teaching?
a. “After this drug gets to a therapeutic level, it 18. A patient who has hypothyroidism tells the nurse
should help my heart to stop feeling like it is that her insurance company now requires that she
racing.”  



$


  
ˆ




’#


 

 


#



 
 % 
{
!

am on this drug.” nurse should advise the patient to


c. “This drug should help decrease the bulging a. ask the pharmacist what is best.
appearance of my eyes.” b. discuss this change with her prescriber.

’!




$
 
#

c. pay for the brand name.
always too hot.” 
 


 Œ



$$

Copyright © 2016, Elsevier Inc. All Rights Reserved.


CHAPTER 58 Q Drugs for Thyroid Disorders 139

19. A patient has received instructions regarding ad- š


^‹
| 
=# ?


  
$

  
$

={?

patient in thyrotoxicosis. Which outcome is ap-
of these statements made by the patient would propriate for this therapy?
indicate that the patient understood the directions? a. BP 110/80-90/60 mm Hg

’#

 






b. Free T4 `<%^†
 '‰
breakfast.” c. Pulse 60-80 beats/min

’{ 
 

$



 


!{*
`‹%‹
ˆ'‰
an antacid.”
c. “Taking the drug on an empty stomach 30-60
minutes before breakfast increases drug absorp- DOSE CALCULATION QUESTIONS
tion.”
d. “Taking the drug with orange juice increases
^
‰
={?
`


  

drug absorption.” for a 4-month-old child with congenital hypothy-


 

 
‡
 
#


$
 ƒ
6
^`
#







  

 
 


  
$
 @
=!  -
@?ƒ
^—
‰
={?
`^

 



‰!
`
#ˆ'‰ available. The patient has been prescribed 100

–ˆ+

 '‰ mcg. How many tablets should the nurse adminis-

…–\
]†<
 '‰ ter?

>\
‹^
#ˆ'‰

š
^]
–  
$

 
$
   

 
CASE STUDIES
should teach the patient who has been prescribed
 @
=!  @?



- Case Study 1
tom?
a. Anorexia A 30-year-old woman visits her family physician com-
b. Bleeding gums plaining that she has been experiencing unusual fatigue,
c. Pale conjunctiva lethargy, intolerance to cold, and weight gain. At the visit,

{
  her vital signs are BP 100/58 mm Hg, P 62, R 16, and T
97.8° F.
22. A 34-year-old female patient is prescribed io-
dine131 for toxic nodular goiter. Which statement, 1. What thyroid disorder do these symptoms sug-
if made by this patient, would indicate a need for gest?
further teaching?

’#
 

 " 



secretion from this therapy.”



’#
 
 



 


2. What laboratory test results would the nurse ex-
exposing others to radiation.” pect to see for this patient if she is hypothyroid?

’#


 

 

$


control.”
d. “This drug takes months before it is fully effec-
tive.”
3. After appropriate history, physical examination,

^†

$
 
$  
$



 
and laboratory studies, the patient is diagnosed

‰ ‚






with primary hypothyroidism and started on levo-
the nurse? 
={?
]]^



 
 

a. Decrease in free T3 teaching should the nurse provide about expected


b. Weight gain of 2 lb in 2 weeks timing for relief of symptoms?
c. Beverages having a brassy taste
d. Drug taken with grapefruit juice

4. What teaching should the nurse provide to this


patient about administration of levothyroxine
={?ƒ

Copyright © 2016, Elsevier Inc. All Rights Reserved.


140 CHAPTER 59 Q Drugs Related to Hypothalamic and Pituitary Function

5. The nurse should teach about possible adverse 9. The patient asks the nurse why two drugs have
effects of levothyroxine. What adverse effects been prescribed. What information should the
warrant notifying the prescriber? nurse provide regarding the need for both drugs?

6. The patient tells the nurse that her sister has asked
]`
!

 @
=!  @?




for some of her thyroid pills to help her lose  


|!ˆ
 



  
 


 
{
 


$




 - 
  
 
|!ˆ
$

 ƒ
lem. How should the nurse respond?

11. What assessments should the nurse teach the

~
!
 

$

]% 

{
 



 




wonders if she still needs the medication because reported to the prescriber?
she sometimes forgets to take the drug and does
not feel any different on those days. How should
the nurse respond?


]^
{

 

 ‚
 
 


 
!{*

$
!3 levels are within
normal limits. The prescriber provides a plan for
Case Study 2
gradual decrease, then discontinuation of pro-
| 
=# ?
]`


 
=|!ˆ?
 
=# ?




 
 

100 mg 3 times a day are prescribed for a 25-year-old rather than discontinued if vital signs are normal?
 



  

\  ‚
  

8. What laboratory results would the nurse expect to


see when reviewing this patient’s chart?

59
Drugs Related to Hypothalamic and Pituitary Function

STUDY QUESTIONS 3. ___ Acts on the ovaries to promote ovulation and


development of the corpus luteum, and acts
Matching on the testes to promote androgen produc-
tion.

 #    $     4. ___ Acts on the ovaries to promote follicular
growth and development, and acts on the

]

*
  
\*
  testes to promote spermatogenesis.
2. ___ Acts on the adrenal cortex to promote


{ 


$
 
synthesis and release of adrenocortical hor-
—

{ 


  

 

mones. and organs.


7. ___ Acts on the thyroid gland to promote synthe-
sis and release of thyroid hormones.

Copyright © 2016, Elsevier Inc. All Rights Reserved.


CHAPTER 59 Q Drugs Related to Hypothalamic and Pituitary Function 141

8. ___ Promotes uterine contractions during labor š


]‹

 




 


and stimulates milk ejection during breast- consult the prescriber regarding administration of
feeding.   ƒ
={

 
?
9. ___ Acts on the kidney to cause reabsorption of a. BP 82/60 mm Hg
water. b. Chest pain

\…
‹
‰'
a. ACTH 
…
 
^```
‰' 
b. ADH

…{* 15. The nurse is teaching a patient how to administer

\* intranasal desmopressin. What is the proper way

‰* to administer a dose to ensure accuracy?
f. Oxytocin a. While lying down with the head turned toward
g. Prolactin the side of administration

{    
ˆ
 




!{* c. Tilt bottle so tube draws from deepest portion
of medication
d. When inhaling
CRITICAL THINKING, PRIORITIZATION,
AND DELEGATION QUESTIONS 16. A patient who was recently diagnosed with hypo-
thalamic diabetes insipidus and prescribed vaso-

]`




 




pressin reports weight gain of 1 to 2 lb per day for






$

- the past week. The nurse should recognize that this

=\*?ƒ may indicate

> 
 a. failure to comply with directions for adminis-

‰



 tration of the drug.
c. Profuse sweating b. failure to limit water intake.

{
 

 
 c. improper diagnosis.
d. inadequate drug dosage.
š
]]

 
$

~% %

 

increased risk for severe adverse effects of soma- š


]~

 
  


 


tropin?  
  



 

a. Anterior cervical lymphadenopathy results would indicate that therapy has achieved

–€#
† the desired effect?

&
 


  a. 1.0002
d. Pulse 100 beats/min b. 1.002
c. 1.02
6
]^
#





 




d. 1.2
been prescribed mecasermin, and the child’s par-
ents, to seek medical care if the child experiences
]<
#









a. diaphoresis and tremor. patient is prescribed tolvaptan?


b. dry skin and itching. a. Calcium
c. nausea and vomiting. b. Phosphate
d. sore throat and fever. c. Magnesium

{
š
]†

$
 
 
 
$




 



   
={- 19. Which laboratory result would be a priority to re-
 ?
 

  
  

”
port to the prescriber when a patient is prescribed
and should be reported to the prescriber? pegvisomant for acromegaly?
a. Headache 
 
]`
#ˆ'‰
b. Nasal congestion 
‰!
^~
#ˆ'‰
c. Malaise c. A1c 6.8%
d. Tea-colored urine 
*  

'‰

Copyright © 2016, Elsevier Inc. All Rights Reserved.


142 CHAPTER 59 Q Drugs Related to Hypothalamic and Pituitary Function

DOSE CALCULATION QUESTIONS Case Study 2


#   
  








^`



  
 
=* ?
4 hours as needed, are prescribed for a patient with post-
`^
 ' '
{ ˜





 
operative abdominal distention.
to be administered every other day. The nurse
teaches the parents of the child, who weighs 30 kg, 4. What should the nurse assess before administering

 


 
=* ?
this drug?
at each dose?

21. Mecasermin 800 micrograms is prescribed for a


‹‹%

!


  

]`
 '‰


 
 




 


How much drug will the nurse administer? withhold the drug and contact the prescriber?

CASE STUDIES
Case Study 1 Case Study 3
The nurse works in an endocrine clinic at a major pedi- 




  

 
=*-
atric center that sees children with short stature. When  ?


  
 
= %

\*
 

 
=* ?

  

$ %]?
!
 

 

 


the nurse must teach the patient and parents self- parents about possible adverse effects and management.
administration of the drug.
6. What would the nurse include in teaching about
1. What should the nurse include in the teaching of the following?
the technique of subcutaneous injection of this a. Hypoglycemia
drug?

b. Tonsillar hypertrophy

2. What will the nurse include when explaining the


importance of monitoring weight while on this
drug? 
‰ 

 
 
 

†
{
 
 
 

\*
$


child, but the prescriber decides that the risks are


 
 
 
 
$ 
 


nurse provide to reinforce prescriber teaching




 

\*
 


 

for children with the following conditions?


a. Prader-Willi syndrome

b. Diabetes mellitus


ˆ


Copyright © 2016, Elsevier Inc. All Rights Reserved.


CHAPTER 60 Q Drugs for Disorders of the Adrenal Cortex 143

60
Drugs for Disorders of the Adrenal Cortex

STUDY QUESTIONS
]†

 ‚
  

 $

secretion of adrenocorticoids and Cushing’s


Matching syndrome is excessive production of adreno-
corticoids.

       14. ___ Cushing’s syndrome is usually treated by
administration of drugs that suppress adrenal
1. ___ Androgens
secretion of hormones.

^

\
3. ___ Mineralocorticoids
CRITICAL THINKING, PRIORITIZATION,

#
   
  


other processes
AND DELEGATION QUESTIONS
b. Modulate salt and water balance
15. The adrenals of the full-term fetus release a burst
c. Contribute to expression of sexual char-
of glucocorticoids during labor and delivery.
acteristics
These steroids act to:
a. accelerate maturation of the fetal lungs.
True or False
b. lower maternal blood pressure.
   
  T   F c. prevent uterine contractions.
   d. stimulate fetal weight gain.

4. ___ The terms          


]—
# 

$

 
  
 
$

and    only refer to glucocorticoids. glucocorticoid release, glucocorticoid replacement


5. ___ Normal secretion of glucocorticoids should therapy would be administered at what time?
not cause hyperglycemia unless the patient a. Midmorning
is a diabetic. b. Just after the patient wakes
6. ___ Thin extremities from muscle wasting is a c. Midafternoon
symptom of glucocorticoid excess such as is d. Just before the patient falls asleep
seen in Cushing’s syndrome.

~

\



 - 6
]~
#










down of fat for energy. caring for a patient with Cushing’s syndrome?

<

\
  
–|




… 

 
^†
 '‰
vessels to constrict and limiting capillary b. pH 7.44, HCO3
^<
Z"'‰
CO2 45 mm Hg
 


$


   
| 
!
 

   
=Z>\?

‡

Z



 

{


Z>\
with depression.
10. ___ Adverse effects of water retention and š
]<
ˆ 
>  ‚



 

muscle wasting are expected when a patient dangerous if a patient had a history of what condi-
takes glucocorticoid therapy because of tion?
 $



   a. Aspiration pneumonia
11. ___ Patients who take glucocorticoid therapy b. Bowel obstruction
  
$
 $



- c. Cerebral palsy
nals are at risk for hypotension and hypo- d. Heart failure
glycemia any time they experience elevated
levels of physical or emotional stress.
12. ___ A patient who takes multiple drugs that
block/decrease the release of aldosterone is
at risk for metabolic acidosis.

Copyright © 2016, Elsevier Inc. All Rights Reserved.


144 CHAPTER 60 Q Drugs for Disorders of the Adrenal Cortex

]‡
Z

 

  
$
DOSE CALCULATION QUESTIONS
a patients with adrenal hyperplasia because these
drugs 24. Fludrocortisone acetate is available in 0.1-mg tab-
a. cause greater excretion of water than other lets. The patient is prescribed 0.05 mg once a day.
diuretics. How many tablets should the nurse administer for
b. increase renal reabsorption of hydrogen. one dose?



 @ 
$
 
 
d. protect the heart.
25. The recommended dose of hydrocortisone for a
20. What is the priority teaching for a patient who has  

 
 $

]^%]
 '2.
been prescribed hydrocortisone for adrenal insuf- A patient who is 5 feet 11 inches tall and weighs
ƒ ]~—



  

 
#

 


# 
$
" 



 safe?

# 
$
 

 
$


when ill

# 
$

$ 
  


fruits in the diet CASE STUDY



# 
$
  
–|
 
A 32-year-old man has been admitted with severe episodic
6
^]

$
 






 

"  
=ˆ˜?
 
 

   

report to the prescriber if a patient was prescribed Oral cholecystography reveals acute cholecystitis, and an
 ƒ open cholecystectomy is planned. When gathering data
a. Blood pressure 135/88 mm Hg regarding medical history, the patient reveals he has been
b. Flushed, dry skin experiencing extreme fatigue, dizziness when changing
c. Nausea and vomiting position, and muscle weakness. He has felt this way for a
d. Weight gain of 2 lb in 24 hours long time, but thought the symptoms were caused by his
stressful job. He also states that he loves every kind of
22. The nurse is caring for a patient who is scheduled salty food and eats at least one bag of potato chips daily.
for an adrenalectomy for adrenal adenoma. During He states that he has experienced impotence on occa-
the night, the nurse should assess the patient for sion. The nurse notes that he has increased pigmentation
which symptom of the hyperaldosteronism? around the face and hands and a decrease in body hair.

–
 
 

$

 –

$ 
 
—^
 '‰


b. Postural hypotension when getting up to the ]†^


Z"'‰
 
†
Z"'‰



 

bathroom =–ˆ+?
^<
 '‰
{



c. Tremor and cool, clammy skin from hypogly- =>!*?



 
#Š

 
 


cemia 

‰
$ 

 
$

>!*
-
d. Vomiting, diarrhea, and subsequent electrolyte lation. He is diagnosed with Addison’s disease. He asks
imbalances the nurse how this causes his symptoms.

š
^†

$
 
  
  
$



1. Describe how the nurse relates current and pos-
patient with Cushing’s syndrome who is receiving sible symptoms to decreased levels of adrenal
800 mg of ketoconazole to inhibit corticosteroid cortex hormones.
synthesis, should the nurse report to the prescriber
immediately?

‰!
]`
  
 '‰

–ˆ+

 '‰

> 
]
 '‰
^
!
 

  
 
`



\ 
]`
 '‰ every 8 hours. Text states that daily doses of hy-
drocortisone should mimic normal daily secretion
=^`%†`


 ?



 


such a high dose?

Copyright © 2016, Elsevier Inc. All Rights Reserved.


CHAPTER 61 Q
Z  

|   •
– 
|   

+ 
  145

3. Because the patient is ill with cholecystitis and 5. What teaching should the nurse provide to a
 
 $
 


 
 
patient who has been prescribed oral glucocorti-
concern of the nurse? What should be included in coids?
the nursing plan of care to address this concern?

4. The patient is prescribed hydrocortisone 20 mg/


day. What administration regimen would be used
for the hydrocortisone if the prescriber wanted to
mimic the body’s natural basal secretion of gluco-
corticoids?

61
Estrogens and Progestins: Basic Pharmacology and Noncontraceptive Applications

STUDY QUESTIONS CRITICAL THINKING, PRIORITIZATION,


AND DELEGATION QUESTIONS
Matching

      
   6. Which site is not acceptable for application of
 transdermal formulations of estrogen?
a. Breast
1. ___ Causes the dominant follicle to swell rap- b. Calf
idly, burst, and release its ovum c. Forearm
2. ___ Following menstruation, promotes endome- d. Thigh
trial restoration
3. ___ Occurs when progesterone levels are insuf- 7. What is not a function of progesterone during


 

 

pregnancy?
of estrogen on the endometrium a. Promotes growth and proliferation of breast
4. ___ Converts the endometrium from a prolifera- structures that produce milk
tive state into a secretory state b. Prevention of immune rejection of the fetus
5. ___ Acts on the developing ovarian follicles, c. Promotes follicular maturation and ovulation
causing them to mature and secrete estro- 
{ 
$

 
gens
 8. A 45-year-old patient has not had a menstrual
a. Dysfunctional uterine bleeding period for 2 months. The nurse is reviewing labo-

Z   ratory tests. Which result suggests that the patient

…{*  


  
=  ?ƒ

‰* 
Z 
  
e. Progesterone 
Z 
 

 

Z 
@


Z 
  

Copyright © 2016, Elsevier Inc. All Rights Reserved.


146 CHAPTER 61 Q
Z  

|   •
– 
|   

+ 
 

 9. The nurse is caring for a 56-year-old patient who


]

 

  

=|€{?

is post–total knee replacement. The patient has 



  

=|@ ?
$
|€{

been taking estrogen to relieve menopausal symp- Which statement, if made by the patient, would
toms and increase bone mineral density. Which suggest that the patient understands the potential

 



$
 


$


 
$
|€{ƒ
to the nurse? 
’#
 
 



 

 

a. Dizziness with position changes my symptoms are present only part of the
b. Dyspnea while resting month.”
c. Nausea and vomiting 
’#
 
^

‹

$
 
 
$
#

d. Vaginal spotting of sanguineous discharge 






#
$“
c. “This drug may help reduce my breast tender-
10. Research suggests that hormone replacement does ness, bloating, and headache.”



 ƒ
={

 
? d. “This drug should improve my depression and
a. Cardiovascular protection irritability.”

# 


 
c. Prevention of vaginal atrophy
d. Protection from dementia CASE STUDY

$
$
 


 
=
  ?
A 46-year-old patient has just received the recommenda-
11. Which symptom, if exhibited by a patient who is tion that she undergo a total hysterectomy and bilateral

  
 
=Z!?

 


 %
$

  
be an effect of this therapy?
a. Constipation 1. The patient should be informed that which symp-
b. Nausea toms should be expected postoperatively relating
c. Profuse vaginal drainage 

 
$

 
=   
-
d. Orthostatic hypotension   ?ƒ

6 12. The nurse is assessing a 62-year-old female patient


who is post–knee arthroplasty. The patient is
  
 $
=Z  ?

 
  

What would be of greatest priority to report to the


surgeon? 2. The prescriber orders laboratory tests, including

>  

†


 
   

&Z
 



&$
 
 
 tells the patient that results are needed before they
c. Dry mucous membranes can discuss options to prevent surgically induced
d. Pain and increased circumference of the right menopausal symptoms. The patient asks the nurse
calf why these tests have been ordered. What informa-
tion can the nurse provide?
6 13. The nurse is reviewing laboratory test results for a
42-year-old patient who has been taking medroxy-
  
 
=| ?
$
 $ 

uterine bleeding. Which test result would be a


priority to report to the prescriber?

>\
^^^
  
 '‰
†
‰
 


 
 
]


‰&‰
]`^
 '‰   
 

 
‰&‰
]`
 '

#+
]] ‰
*&‰

 '‰



$
 


d. WBC 8,765/mm3 mammogram. The patient is interested in post-


operative hormone replacement. What additional
14. Research suggests that which natural strategy is information must be obtained before the prescriber
most effective in relieving symptoms of premen- considers ordering hormone replacement?
 

=|€{?ƒ
a. Taking black cohosh every other day
b. Taking evening primrose oil every day
c. Taking red clover twice a week

Z 
%  
$

 

Copyright © 2016, Elsevier Inc. All Rights Reserved.


CHAPTER 62 Q Birth Control 147

4. There are no contraindications for estrogen 6. What teaching should the nurse provide that can
replacement therapy. The woman states that her help reduce the risk of cardiovascular events?
mother received relief of menopausal symptoms
by taking estrogen-progesterone combination
=|?



  




prescribed for this patient?

7. The patient asks if there are any alternative treat-


ments for menopausal symptoms. What informa-
tion can the nurse provide?


$
  
 

 

  

orders Climara, a transdermal estrogen patch, and


asks the nurse to teach administration directives.
What should the nurse include?

62
Birth Control

STUDY QUESTIONS 10. ___ Combination OCs do not increase the risk of
breast cancer for most women.
True or False 11. ___ OCs can speed the growth of existing breast
cancer.
   
  T   F 12. ___ OCs are contraindicated during pregnancy.
   13. ___ Progestin-only OCs are safer and more ef-
fective than combination OCs.
1. ___ Abortion is a common approach to deal with 14. ___ Newer progestins in combination OCs are
unplanned pregnancies. not more effective but they have different

^

ˆ 
  
 

$

adverse effects.
20 years. 15. ___ OC use can prevent anemia in some patients.
3. ___ Tubal ligation and vasectomy are the most 16. ___ When using a continuous-cycle OC, once
commonly used form of birth control in the the pills have been taken daily for at least 3
ˆ{ weeks, up to 7 days can be missed with little
4. ___ The most effective pharmacologic forms of or no increased risk of pregnancy.
birth control have the most adverse effects. 17. ___ Cervical caps are more effective in women
5. ___ Oral and transdermal contraceptives are the who have already had babies.
most effective forms of birth control.
]<

#ˆ&



$
 


—

€ 
 
 
=}> ?
  
- 
 
$
{!#
taneous abortion of a fertilized ovum. 19. ___ A diaphragm with spermicide is more ef-

~

{

  
  
 
- fective than male condoms with actual and
creases the risk of serious blood clot events theoretical use.
when a woman uses OCs.
^`

+%‡
 
 

 
$
*#Š

<

Z  

}> 


  
transmission through promotion of vaginal,
increases the risk of developing a thrombus. cervical, anal, and rectal lesions that facili-
9. ___ Combination OCs are not associated with  
*#Š
 

 
an increased risk of ovarian and endometrial
cancer.

Copyright © 2016, Elsevier Inc. All Rights Reserved.


148 CHAPTER 62 Q Birth Control

CRITICAL THINKING, PRIORITIZATION, š


^—

 


 
 

  


phenytoin comes to the college health center


AND DELEGATION QUESTIONS
requesting OCs for birth control and menorrhagia.
{


  
$
}>
 
|-
6 21. What would be the priority nursing teaching when
nytoin decreases the effectiveness of OCs. What is
a woman is prescribed an OC containing the pro-
the best strategy in this situation?
gestin drospirenone?

! 

 

 


=?


a. “Do not eat too many bananas while you’re


addition to prescribed OCs.
taking this drug.”
b. Tell the patient to start OCs and stop taking

’{
 
 
$

 
 
 


phenytoin and see if seizures resume.


unexplained shortness of breath.”
c. Teach the patient that abstinence is the only
c. “You should have less bloating with this drug
alternative in this situation.
than with other OCs.”
d. Teach the patient how to predict the times
d. “You should have your blood pressure checked
when she is most fertile.
at every health visit.”
š
^~

 



  
 
$
 -
22. What is a potential cause of OC failure?
 

 

 
 
=}


…–\
³
]^
 '‰
Z ?


 ƒ
b. Hypertension
a. 19-year-old with multiple sexual partners
c. Multiple sexual partners
b. 25-year-old who weighs 210 pounds

{$

 
 


  
c. 37-year-old who is anemic
d. 40-year-old who smokes 2 packs of cigarettes
6
^†
#


$
  

$

 


per day
report which symptom if she is prescribed an OC
containing drospirenone and naproxen for osteoar-
6 28. A 19-year-old healthy patient who has been tak-
thritis?
ing a combination OC for 2 years is involved in
a. Bleeding between menstrual periods
an automobile accident and undergoes surgical
b. Fatigue


 
$

$ 
$


c. Chest pain

 
 
 


$


{
  
greatest priority to report to the prescriber?

>  

$
^

24. Which supplement can decrease the effectiveness
b. Diaphoresis
of OCs?

{
 

Z 

! 
]``„
…
=†~<„
>?

\ 

\ 

6 29. The nurse is working in a women’s health clinic.

{
 ‚

{ 
 %
 

 
}>



the same time. Which patient should be given the


6 25. A patient who has been using OCs for birth control
highest priority for seeing the APRN?


  
 $ 
=> ?

-
a. 17-year-old who is experiencing a migraine
vent DVTs during a period of prolonged impaired
headache with aura
mobility. Which action by the patient is of greatest
b. 26-year-old who is experiencing breakthrough
priority in this situation?
bleeding
a. Consuming adequate calcium in the diet
c. 35-year-old who smokes and needs her pre-
b. Complying with scheduled blood tests



# 

 

^```
‰' 
d. 42-year-old who has had episodes of severe

ˆ 

$
  
right upper quadrant pain

š
†`

 
$
 
  
 

 


is prescribed ethinyl estradiol and drospirenone


=™ @?
 
 
 



hyperkalemia?
a. Confusion
b. Tachycardia
c. Positive Trousseau’s sign
d. Restlessness

Copyright © 2016, Elsevier Inc. All Rights Reserved.


CHAPTER 62 Q Birth Control 149

š
†]
*


 
 


 ‚
35. What is the best way to evaluate teaching regard-
concern that she has not had a period since starting ing administration directives, including what to do
low-dose oral contraceptives 3 months ago? if a dose of OC is missed?

’{



 


 
a. Demonstrate taking the pill from the blister
oral contraceptives.” pack.
b. “You should see a gynecologist to decide if b. Have the patient read the directions to you.
another oral contraceptive should be used.” c. Have the patient repeat the directions.
c. “You should have a biopsy to test for uterine d. Have the patient write the directions in her own
malignancy.” words.
d. “You should have a pregnancy test immedi-
ately.” 36. The nurse instructs the patient who is prescribed

   
 
 
}
Z 

š
†^
–  
$

$$
$
 
 




 


={

 
?
glucose levels of diabetics, it is most important for a. breasts.
the nurse to assess for b. buttocks.
a. anorexia. c. lower abdomen.
b. diaphoresis. d. upper torso.

 
 e. upper inner arm.
d. tachycardia.
37. What nutritional strategy could prevent adverse
33. A nullipara patient, who plans to get pregnant after $$
$

  
=&%
1 year of using contraceptives, is discussing differ- | ?ƒ
ent pharmacologic agents for birth control with the a. Decreasing intake of bananas and other foods
nurse. What should be included in the teaching? high in potassium
a. Alternative forms of birth control should be b. Decreasing intake of red meat and other foods
used for 3 months after stopping oral contra- high in iron
ceptives if the patient wants to minimize the 
# 
 
$
 




risk of multiple births. foods high in calcium


b. When medroxyprogesterone injections are 
# 
 
$

 $
  


discontinued, return of fertility is delayed by an other foods high in vitamin K


average of 9 months.
c. Contraception usually continues for 2 to 3
†<


 


#ˆ&

 

 

months after subdermal implanted contracep- should assess for which symptom of a common
tive capsules are removed. adverse effect?

!
€
#ˆ&

 

 

$
a. Breast tenderness
this patient. b. Dull pelvic pain
c. Dysmenorrhea
š
†‹

 
 

 
 
 - 
Z
ing mestranol complains of right upper quadrant
discomfort. Because of the possibility of a highly 39. Which method of birth control is contraindicated
vascular benign hepatic adenoma, which assess- when a patient has a pattern of multiple partners?
ment technique should the nurse use for assess- 
Z  
  
 
=# ?
ment of the liver after inspection? 
#   
  
 

a. Deep palpation =&%| ?



‰ 
   
|  
!
=|   ?
#ˆ&
c. Percussion 
Š  
 

=+  ?
d. Auscultation
40. What are advantages of polyurethane condoms

 
 ƒ
={

 
?
a. Cause fewer allergic reactions
b. Do not have increased risk of failure with lubri-
cant use
c. Are more effective in preventing pregnancies
d. Prevent transmission of genital herpes
e. Are thinner and more pleasurable to use

Copyright © 2016, Elsevier Inc. All Rights Reserved.


150 CHAPTER 63 Q
&
! 
$
#$

CASE STUDY 3. When educating the patient in the use of OCs, it


is important to stress the need to take them every
A 17-year-old sexually active female patient has used con- day at the same time as prescribed. The patient



$
$




$
{!# 
is started on a triphasic OC and is concerned that
{

 




 


}>

 
$ 

 


  
#$


The patient’s history includes asthma that is controlled does forget, what procedures should she take to
with Theo-Dur, dysmenorrhea, and irregular menses. minimize the risk of pregnancy?
Her last normal menstrual period was 2 months ago, but
she says she is not concerned because this is a common
occurrence for her. The patient is anxious and asks the
nurse about the medical exam and what procedures will 4. The patient asks about using an alternative form
be performed. of contraception. What teaching should the nurse
provide about the use of condoms and spermi-

]
–
  
 





- cides?
administration assessment and how frequently the
patient will need to have it repeated.

5. Knowing that the patient has asthma and is pres-



 
!%&
=
$
$
?

what assessments relating to interaction of OCs


2. The patient asks the nurse, “How do oral contra- and theophylline need to be taken and why?
ceptives prevent pregnancy?” How does the nurse
respond?

6. What possible adverse effects warrant discontinu-


ation of the OC, and what symptoms of these
conditions should the nurse teach the patient to
report to her prescriber?

63
Drug Therapy of Infertility

STUDY QUESTIONS 
…%  

=…{*?

\ % 

=\*?
Matching 
‰@

=‰*?


     True or False
1. ___ Hormone that stimulates testicular cells to    
  T   F
produce testosterone and the corpus luteum   
to secrete progesterone
2. ___ Hormone that stimulates maturation of ovar- 4. ___ (   




 


ian follicles reproduce.

†

{ 




- 5. ___ Treatment for low sperm count is more
thalamus that affects the anterior pituitary successful than treating female causes of
infertility.

Copyright © 2016, Elsevier Inc. All Rights Reserved.


CHAPTER 63 Q
&
! 
$
#$ 151

6. ___ &   is the medical term for implanta- š


]‡
…
 

$


 
$$
$

tion of the products of conception. ovarian hyperstimulation syndrome from therapy


7. ___ The drug clomiphene has similar effects as 

=

€?




\* nurse would be most concerned if, after starting
8. ___ Menotropins and follitropins act on mature therapy, the patient reported
follicles to cause ovulation. a. a fever higher than 100.4° F.

‡

* 

 
=>\?
b. a gain of 2 lb in 24 hours.
stimulates ovarian follicles to mature. c. a pulse rate higher than 80 beats/min.

]`

{ 






 
d. redness across cheeks and nose.
at the neck of the uterus can impair motility
of sperm into the uterus. š
^`
!
 

 
$

 



]]

{
$
 
 

ovarian hyperstimulation syndrome after infertil-
secretion of milk when not breast-feeding. 
 

>\
#





$

]^

‰

 $ 


 
$
which possible effect of this syndrome?
infertility associated with endometriosis. a. Abdominal distention
13. ___ The priority reason for using drugs to treat b. Dyspnea
 
 

=|>}{?



Z


decrease the number of ovarian cysts. d. Weight gain

]‹

ˆ$
=– ?


 


  
$
…{*
 
$


21. The nurse is explaining drug therapy to a patient
of pregnant women. who is prescribed cetrorelix. Which statement, if

]

>\

 



 

made by the patient, indicates a need for further
action to luteinizing hormone. teaching?

’#

 

 
$

”-
tion because it must go into a muscle.”
CRITICAL THINKING, PRIORITIZATION, 
’#

”




$ 



AND DELEGATION QUESTIONS skin about 2 inches away from my belly but-
ton.”
616. The nurse is explaining possible adverse effects 
’#
 

 



 


$

=>?

 
$$
$
#


 “
experienced by the patient, would be a priority for d. “This drug prevents immature eggs from being
the patient to report to the prescriber? released.”
a. Breast engorgement

*
 
=
  ?
^^
–
=| ?
 
 
-

‰
 
  tion by activating receptors for which neurotrans-
d. Nausea mitter?
a. Acetylcholine
17. Which laboratory test result would be an absolute b. Dopamine
contraindication to the administration of clomi- c. Nicotine

=>?ƒ d. Norepinephrine

{!
‹]
  
 '‰

> 
]‹
 '‰

>\
<``
%  
 '‰ DOSE CALCULATION QUESTIONS

{

~`œ
23. Cabergoline is supplied in 0.5-mg tablets. The ini-
 ]<
| 


  

=>?
tial dosage is 0.25 mg. How many tablets should
may experience increased viscosity of cervical be administered?
 



Z  


possible treatment to thin the cervical mucus.


Z  
 


$
  

$

^‹
* 

 
=>\?
=| ?

the patient also has which chronic condition? <```


ˆ{|
 

  
   

a. Asthma causing bronchoconstriction when   



^````


]`
‰
*
 

exposed to mold should be administered at each site?


b. Hypertension requiring diuretic therapy
c. Type 2 diabetes mellitus with peripheral neu-
ropathy
d. Venous stasis resulting in thrombophlebitis

Copyright © 2016, Elsevier Inc. All Rights Reserved.


152 CHAPTER 64 Q
&
 
$$
ˆ
…

CASE STUDY 2. One year later, the couple has not conceived.
€ ž  
€\
=?
]

A heterosexual married couple has no children. Both the ampule intramuscularly for days 9 through 12
husband and the wife are in their late thirties. They have $

  

$

>\
```

been trying to conceive for the past 2 years but have not units—are prescribed for the wife. The nurse pro-
been successful. During a lengthy diagnostic work-up for vides counseling relevant to the need for follow-up
infertility, the wife was found to have no increase in her and early detection of ovarian hyperstimulation
basal body temperature throughout her menstrual cycle. syndrome. What symptoms should the nurse teach
The husband has a normal sperm count and no health the patient to report immediately?
issues. The physician diagnoses primary infertility and



 
>
={?


>\
=| ?

  


  
$-
lar maturation and ovulation. 3. What will the nurse teach the patient about the

$
  
$
>\ƒ
1. What organ functioning must be present for this
drug to work?

4. The couple asks whether a multiple pregnancy


=

 

$ ?






drug regimen. How should the nurse respond?

64
Drugs that Affect Uterine Function

STUDY QUESTIONS CRITICAL THINKING, PRIORITIZATION,


AND DELEGATION QUESTIONS
True or False
   
  T   F 6 6. What should women be taught to decrease the
   incidence of the most common risk factors for

 ƒ
={

 
?
1. ___ Tocolytic drugs are used to induce and a. Teaching the patient to avoid the use of alcohol
strengthen uterine contractions and to con- during pregnancy.
trol postpartum bleeding. b. Teaching the patient to eat nutritious foods and
2. ___ Magnesium sulfate is the most effective exercise during pregnancy.
drug available to suppress preterm labor. c. Promoting patient behaviors that decrease the

†

Z 
 


 
 

- incidence of sexually transmitted infection.
duce contractions during labor because they d. Counseling the patient on smoking cessation.
can cause prolonged uterine contractions.

‹

#


$
 

 
 

temperature after receiving a dose of carbo-


 
 
=* ?



postpartum hemorrhage.
5. ___ The best way to administer carboprost tro-
 
=* ?

   

Copyright © 2016, Elsevier Inc. All Rights Reserved.


CHAPTER 64 Q
&
 
$$
ˆ
… 153

6 ~


 




- 613. The labor and delivery nurse is preparing to
ity to report to the obstetrician when the nurse is administer methylergonovine to a woman who has
caring for a patient who is 35 weeks pregnant and just delivered the placenta. Before administering
receiving terbutaline to suppress preterm labor? the medication, it is a priority for the nurse to as-
a. Patient is experiencing diaphoresis. sess the patient for what?
b. Patient’s heart rate is 90 beats/min between 
Z 
–|
contractions. 
‰

c. Patient is very thirsty. c. Respiratory depression
d. Patient is short of breath, and her sputum is 
ˆ

foamy.
14. Dinoprostone for cervical softening is con-
6 8. The priority nursing focus when caring for the traindicated in patients with a history of what
neonate immediately after birth, relating to recent = ?ƒ
={

 
?
administration of nifedipine to the neonate’s 


  
  
mother, is to provide adequate 
–
 
 
 
^``
 '‰
a. hydration. c. Asthma
b. nourishment. d. Hypertension
c. ventilation. e. Hypotension
d. warmth. f. Previous cesarean section
g. Wheezing
 9. Based on adverse effects of nitroglycerin on the
pregnant patient, when this drug is being admin- 15. What are the responsibilities relating to dinopros-
istered, the nurse needs to closely monitor the 

=|?
$
 
 



patient’s  
   ƒ
={

 
?
a. blood pressure. a. Calculating the proper dose of the drug
b. blood glucose. b. Determining the number of doses needed
c. respiratory status. c. Ordering the correct number of doses for the
d. urine output. patient
d. Positioning the patient during and after admin-
š
]`
#$

 


  
$ 

- istration
press preterm labor, which of these laboratory test 
{





 
results will be of most concern to the nurse?

‰!
`
  
 '‰ 6]—
!
 
=>?
  
 



> 
]]
 '‰ has been inserted in a patient’s posterior fornix.

> 
‹
 '‰ Monitoring indicates that moderate contractions

€  
]^
Z"'‰ lasting 15 seconds have begun. What is the nurse’s
priority nursing action?
 11. What nursing outcome best indicates that intrave- 
# 
 
$ 
$

=|-

=#Š?

=|?
 


?

 
$$


 

$
- 
# 

 
 

 





ery of the placenta? for at least 2 hours.


a. Blood pressure is 120/80 mm Hg. c. Place the patient on a fetal monitor.
b. Pulse is 80 beats/min. d. Remove the dinoprostone pouch.

{  
$

 
 
 
 
 

20 minutes.

{

 

 CASE STUDY

]^

 = ?


 




^% %
 
=  
^
 
]?




$ 
$

=|?ƒ
={

 
- 
 



$

$
 
{


? at 41 weeks gestation. Vital signs and fetal heart rate are
a. Contractions lasting longer than 1 minute  
& 

=|?


$
 

b. Contractions occurring more frequently than ripening.


every 4-5 minutes

‰ 
 
$
$ 
 
  1. What will the nurse teach the patient about this

# 
 

^`

*

therapy?
contractions

ˆ  

  



Copyright © 2016, Elsevier Inc. All Rights Reserved.


154 CHAPTER 65 Q Androgens

2. The cervix has softened and is 50% effaced. The 5. The patient has suddenly started having contrac-
fetal head was at 0 station in a left occiput anterior tions lasting 90 seconds every 2 minutes. What
position. The medication order reads: “oxytocin should the nurse do?
=|?
]`
ˆ
€ 
 
]

^
ˆ'


every 30 minutes until normal pattern of uterine


contractions are established.” What is a normal
pattern of uterine contractions?
6. When reporting the patient’s response to oxytocin
therapy to the obstetrician, what should the nurse
include?

3. When is oxytocin augmentation of labor contrain-


dicated?


~
!
 



—%
‹%@

{

received one dose of carboprost tromethamine


=* ?
^`




  

‹


$ 
$
]```

$


hemorrhage. The next day she spiked a tempera-
]``

$
œ
 

`‹œ
 


$
]`]—„
…
 
=!?
—`


is infusing via a secondary line into a primary every 4 hours as needed is included in the postpar-
infusion line. The oxytocin infusion was regulated tum orders. Fever is a common adverse effect of


$ 



 

]
ˆ' carboprost. Why is it important to notify the obste-
min. Why is the oxytocin infusion “piggybacked” trician rather than just administer the antipyretic?
into the primary infusion line rather than added to
the primary infusion solution?

65
Androgens

STUDY QUESTIONS 4. A weight gain of 2 lb in 24 hours suggests


that a patient who is receiving androgen
Completion therapy may be experiencing the adverse ef-
fect of retention of ___________________ and
1. The body responds to increased tes- ___________________.
tosterone levels by suppressing the re-
lease of ___________________ and 5. When taken by women in high doses, androgens
___________________ from the anterior pituitary. can cause clitoral growth, hair loss, and lowering
of the voice that may be ___________________.
2. Androgens increase absorption of iron from the
  
=\#?
 

 

 

 

refractory___________________.

3. Because methyltestosterone can cause cho-


lestatic hepatitis, the nurse should monitor
___________________, ___________________,
and ___________________ lab tests when a pa-
tient is receiving this drug.

Copyright © 2016, Elsevier Inc. All Rights Reserved.


CHAPTER 65 Q Androgens 155

CRITICAL THINKING, PRIORITIZATION, 10. The pediatrician’s triage nurse receives a call at
2:00 PM
=]‹``?
$

]‹% %
 


AND DELEGATION QUESTIONS


was recently prescribed testosterone buccal tab-
lets. The patient applied the buccal tablet at 7:00
 —
‰  
 
 
$

 


-
AM
=`~``?


$



*


 
 
 

‰!
‹
 '‰Œ

what he should do. The nurse should direct the


 
‡
 '‰Œ
…–\
<^
 '‰Œ
*
]‹^Œ
*

patient to do what?
‹œŒ

*&‰
^^
 '‰
#






a. Do not replace the tablet until the next dose is


teach this patient the importance of
due at 7:00 PM.
a. eating a cardio-healthy diet.
b. Replace the tablet with a new one and remove
b. increasing protein in the diet.
it as scheduled for the original tablet at 7:00 PM



   


=]‡``?
d. performing weight-bearing exercise.
c. Replace the tablet with a new one and remove

]^

 
= 
`^``?
6 7. When a patient is prescribed an androgen, which
d. Replace the tablet with a new one and remove
adverse effect would be a priority for the nurse to
it the following morning at 7:00 AM
=`~``?
report to the prescriber?
a. Breast enlargement
11. Which behaviors, if occurring when topical andro-
b. Change in menstrual pattern
gens are applied, would be of most concern?

Z



a. Chewing tobacco

# 
 
b. Drinking hot tea

{
$
 
6 8. Which adverse effect of testosterone is a medical

{
 
emergency?
a. Closure of epiphyses before age 12 years
 12. The high school nurse is planning a teaching pre-

Z 
‰&‰
 
 
^``
 '‰
sentation on the use of anabolic steroids. Develop-
c. 1+ pitting edema of ankles
mentally, discussion of which effects of anabolic
d. Priapism lasting longer than 4 hours
steroid use would most likely discourage their use
by male high school athletes?
9. A patient has received instructions regarding
a. Atherosclerosis
  
$
  
 
 
={-
b. Hypertension
?

$
 
 
 


 -

‰
  
tient would indicate the need for further teaching?
d. Testicular shrinkage

’#



 






incisor.”
6]†

$
 
 
 
$




’#

 

$



 

a patient who is receiving oxandrolone for muscle


dose.”
 
 

 
*#Š
  


’#



 

 

 

would be a priority to report to the prescriber?


over the tablet from the outside of my mouth
a. Acne
for 30 seconds to ensure adhesion.”
b. Fatigue, jaundice, and nausea

’#



 





‰
$
 
 “
d. Weight gain of 1§2 lb per week

614. During admission history, a patient who has a his-



$
 

  


of dark-colored urine and clay-colored stool. What


is the greatest priority for the nurse to do at this
time?
a. Assess the patient’s abdomen.
b. Contact the prescriber and ask for an order for a
complete metabolic panel.
c. Continue obtaining all admission data and
review liver function tests.
d. Notify the pharmacy and tell them not to de-
liver any androgenic drugs.

Copyright © 2016, Elsevier Inc. All Rights Reserved.


156 CHAPTER 66 Q
&
$
Z
& $

– 
|  
* 

DOSE CALCULATION QUESTIONS testosterone. The patient is prescribed 1% testosterone


   

= ?
*
 
’
 ‚
#
 

15. Testosterone cypionate =&%!  ?


`

pill?”


  
^``
 '‰

  
*

much should be administered? 1. How should the nurse respond?

]—
!  

=! ?



 -
able. Prescribed is 300 mg. How many pellets 2. The patient’s mother is 36 years old and may wish
will the nurse have available for insertion into the to have another child. What precautions does the
patient’s abdomen? nurse need to teach the patient’s mother and the
patient?

CASE STUDY
A 16-year-old male patient has come to the health care 3. What teaching should the nurse provide to the
provider because of his small testes and penis. The nurse patient about administration of the gel?

 

 


 

$
]`
 ?



=]—`
 ?
*


  

$



He tells the nurse that he has frequently been in trouble in






 


$  

 


4. The patient asks if there are any alternative routes


this year. He is considering dropping out of school.
of administration of testosterone. How should the
‰  
 
 
 


$




nurse respond?



 
$



 

than would be expected for a male. The diagnosis of


Klinefelter’s syndrome is made. The nurse knows that
this is not extremely rare, occurring in 1 of every 500
live male births, but is often not discovered until the male
5. What information does the patient need regarding
patient comes in for an infertility workup. Therapy will
the adverse effects of testosterone?
be directed toward administration of the male hormone

66
Drugs for Erectile Dysfunction and Benign Prostatic Hyperplasia

STUDY QUESTIONS
†

{ 

 


  
 



$
Z&
$
 
 
  
True or False
‹

|   


=|&Z%?
 -
tors activate the parasympathetic nervous
   
  .    system.
  


#  

$
  -



=|&Z%?
 

 


]

|   


=|&Z%?
 
impaired for 3 days if the patient regularly
are contraindicated if the patient has a his- ingests grapefruit juice.
tory of hypertension or diabetes mellitus.
—

| 


  
 
=Š-

^

|   


=|&Z%?
 
 ?


 
$
  

$

are most effective if the cause of the sexual vision and/or hearing.
dysfunction is depression.

Copyright © 2016, Elsevier Inc. All Rights Reserved.


CHAPTER 66 Q
&
$
Z
& $

– 
|  
*  157

~

Š  
=‰ ?

$ 
 

6]~
#



$

 

 



 
  

  

$
 
 
$
-

<

!   
=>  ?


$
 
 




 



  
 

lower than an as-needed dose if the patient =Š  ?ƒ


anticipates sexual activity at least once a a. Chest pain when engaging in any moderate
week. physical activity
9. ___ Finasteride is protective against high-grade 
…  
 

]

$
 


prostate cancer. c. Rhinorrhea in the spring season
10. ___ Finasteride can interfere with the laboratory 
ˆ 
$"

$
 


test for prostate cancer. stream of urine


11. ___ Patients should not donate blood while tak-
ing dutasteride.
]<


  


 
 
 

12. ___ There are no concerns for patients taking =Š  ?








 -
   


=|&Z%?
 - rant consulting the prescriber before continuing to
tors if they are also prescribed the alpha1a  


$
Z&ƒ

 
=  ?

   
a. Blurring of vision
=… ? b. Chest pain during sexual activity
13. ___ Tolterodine, a muscarinic antagonist 
&$
 

$

 
developed for urge incontinence, is contra- d. Dizziness with position changes that resolves in
indicated in patients with benign prostatic less than 1 minute
 
=–|*?
6]‡

 


  
=‰ ?

-
ning comes to the emergency department com-
CRITICAL THINKING, PRIORITIZATION, plaining of palpitations and dizziness. Assessment
AND DELEGATION QUESTIONS 

 
 
$
]^^
  '


BP of 80/46 mm Hg. The triage nurse recognizes


14. The nurse who works in a reproduction clinic that the priority is to report this information and
counsels patients who are experiencing sexual consult the emergency department physician
 $
€ 
 

$
 
=Š-  


$


{!!ƒ
 ?


!
 

 



Z   
will be effective if the basis of the male’s problem 
Z   

={

 
? 
{
 
a. desire for a more intense experience in a nor- 
{


mally functioning male.
b. improvement of erection quality in a patient
^`


   
=>  ?

  
$


 

  


 % 
   
=… ?ƒ
c. lack of desire for sexual activity. 
!   


  

–|
d. improved duration of erection in a patient with b. Tamsulosin is a short-acting drug.
erectile dysfunction. c. Tamsulosin is taken in the morning.
e. premature ejaculation. 
!   





   
š
]

$
 
 

‹

$
21. What is an effect of injection of papaverine plus
sexual activity, would be an emergency situation phentolamine that differs from the effects of oral
$

 

 
 
=Š  ?ƒ 
$
Z&ƒ
a. Blood pressure 160/80 mm Hg a. An erection occurs without sexual stimulation.
b. Diarrhea 
 


 
c. Persistence of erection c. Hypotension will not occur.
d. Headache 
}
$



 
6]—
#





$

 
 

 

 
=Š  ?


 


$


nurse is preparing to administer which medica-


tion?

>
=! ?

& @ 
=>  ?

… 
=|  ?

#   
=#?

Copyright © 2016, Elsevier Inc. All Rights Reserved.


158 CHAPTER 66 Q
&
$
Z
& $

– 
|  
* 

^^
 



–|*ƒ
={

 
? 27. What is a disadvantage of the herbal preparation

#
 
  

   saw palmetto?

#
 

 
$


$

#

$


 

  
 



#

  


 @
 
size of the gland. c. Research suggests that it is not effective.

#


 

 
$
  
  
#

  
 
$$ 

#



 


@
$

the gland.

#
 




 
$
 ”- CASE STUDY

 

 ”
 
The nurse is providing medication teaching for a 45-year-
š
^†

 
 
=|  ?

old male patient who was admitted with cellulitis of the
nursing outcome would be appropriate? left lower leg. The patient has a history of type 2 diabetes
a. The erection lasts long enough to achieve mellitus and hypertension. During the discussion, the
sexual satisfaction. patient’s wife mentions that he has been experiencing

!
  % 
 
=|{?



’$

“


 
 
 

 
‹
 '‰ for his high blood pressure.

| 
  



 
`



bladder scan. 1. Why is it particularly important to address sensi-


d. The size of the prostate gland is reduced by 
 
$$ 


Z&
  


50%. they occur as a result of antihypertensive medica-


tions?
624. Which action is a priority when a female nurse is
 
 
= ?ƒ
a. Administering on an empty stomach.
b. Administering with food.
c. Delaying blood donation for at least 6 months.
^
!
 


  
$

 ‚

d. Avoiding touching the capsule. and wife’s concerns. After discussing the problem


 

  

 

6^
#




$

  

- =Š  ?
`






 


sessment before the nurse administers terazosin explain administration of the drug. What should be
=*?


 



  

included in this explanation?
BPH?
a. Bladder distention
b. Blood pressure
c. Respirations
d. Temperature 3. Why is it important for the patient to have infor-
mation on his person stating that he takes silde-
626. What is the priority teaching regarding effects of  
=Š  ?


 

$ 



 @ 
=>  ?
  
$
–|*ƒ prescribers?
a. Nasal congestion is possible.
b. Orthostatic blood pressure precautions must be
taken.

˜ 
$
”  
 
 

{
 



$
 

the drug.

Copyright © 2016, Elsevier Inc. All Rights Reserved.


CHAPTER 67 Q

$

#
{  159

67
Review of the Immune System

STUDY QUESTIONS 14. ___ The major antibody in blood


15. ___ Found only on the surface of mature B cells
Matching

# 

   
   
# &

# Z
1. ___ Antigen-presenting cells found in lymph 
# \
nodes and other lymphoid tissues 
# €
2. ___ Attack and kill target cells directly
3. ___ Attack and destroy foreign particles that Completion
 

 

 
$

# Z

class 16. Antibodies are also known as


4. ___ Devour cells that have been tagged with ____________________ and
 
$

# \
 ____________________.
5. ___ Mediate immediate hypersensitivity reac-
tions
]~
{

  

  

6. ___ The principal scavengers of the body because ___________________ and


7. ___ Produce all antibodies ___________________ possess receptors that can
8. ___ Promote delayed-type hypersensitivity recognize individual antigens.
9. ___ Molecule that binds to a bacterium, thereby
promoting phagocytosis CRITICAL THINKING, PRIORITIZATION,
10. ___ Makes possible the discrimination between
self and nonself
AND DELEGATION QUESTIONS
18. What is true about natural immunity?

–


 ?

#
 

 
 


 
b. Basophils and mast cells

#



 
 @


>
!

=>&<?
individual antigens.
d. Dendritic cells

#




  
 

Z 

#




$
*
!

=>&‹
 ?
g. Macrophages
19. The nurse assesses for declining immune status of
h. Major histocompatibility complex
patients who are infected with human immunode-
=€*>?


=*#Š?


$


i. Neutrophils
levels of
j. Opsonin
a. basophils.

>&‹

=
!
 ?


       



>&<

=
!
 ?
d. macrophages.

]]

{ 
 
$
  
 


other mediators from the mast cells, thereby


20. What is true about breast-feeding?
causing symptoms of allergy

#


$$

$ ‚

12. ___ First antibody produced in response to an

#

$

$

  

antigen
for which the mother has immunity.
13. ___ First line of defense against microbes enter-

#
  $
  
# 


$ ‚
\#
 




  
=\#?
 


#
 

 
$
 
$
 

and lungs
cells.

Copyright © 2016, Elsevier Inc. All Rights Reserved.


160 CHAPTER 67 Q

$

#
{ 

21. When do autoimmune diseases occur? 26. Activation of the complement pathway occurs
a. A new antigen is introduced to the body, and 

 

$


 -
the immune system does not recognize the 
=>]?

antigen. a. a free antibody.


 


  
 


b. a free antigen.
antigen. c. an antibody-antigen complex.
c. An antigen is introduced, and the immune d. phagocytic cell
response fades too quickly.
d. There is a failure in MHC molecules’ ability to
identify self. CASE STUDY

22. A person who is allergic to penicillin could expect The nurse is preparing to perform allergy testing by intra-
a more severe reaction with exposure to penicillin dermal injection of house dust, molds, foods, and other
in the future. Why? common allergens.
a. Helper T cells attack the penicillin molecule.
b. Higher doses of penicillin would have to be 1. Why are injected extracts called   ?
used.

€

 




an antigen allow for a more rapid, intense, and


prolonged response.
d. The immune system cannot eliminate the peni- 2. The allergic reaction causes degranulation of mast
cillin. cells. What cells respond in the skin, what do
they release, and what dermal symptoms will the
23. What is an immunologic reason for the fact that patient have if allergic to the antigen?



=–> ?
 

  $ 
$


person to another?

* 
–>


 

#
€*>
-
ecules.
b. Human blood is exactly the same in all indi-
†
!
 

"

 


$
$

viduals. 20 minutes after injection of any antigen because



* 



 


##
€*>
that is the usual time frame for an immediate
molecules. hypersensitivity reaction. Describe what the nurse
d. There is no reason because the above statement should do if the following reactions occur.
is not true. 
#





$
”

24. The nurse is reviewing the laboratory tests for



 


” 
‰  
 



 

 
$ 
=!+…?
b. Tingling around the mouth
This suggests the patient has what condition?
a. Cirrhosis or hepatitis
b. Diabetes mellitus or insipidus

#  



d. Kidney cancer or renal failure 4. Why is it important to teach this patient about pos-
sible anaphylaxis?
25. Cells that provide signals to regulate cell prolifera-
tion and function during immune responses are
called
a. antigen-presenting cells.
b. cytokines.
c. MHC molecules.
d. opsonins.

Copyright © 2016, Elsevier Inc. All Rights Reserved.


CHAPTER 68 Q
>
#@  161

68
Childhood Immunization

STUDY QUESTIONS CRITICAL THINKING, PRIORITIZATION,


AND DELEGATION QUESTIONS
Matching

      
]†
!
 
$
 

   
$
 -
taining vaccine information sheets to be provided
1. ___ Refers to production of both active immu- to parents before they consent to vaccinations
nity and passive immunity for their children. What is the best source of this

^

\

 
$
  information?

†

{  
 
 


 - a. www.cdc.gov
tem to produce antibodies b. www.immunize.org
4. ___ Not pathogenic c. pubmedcentral.nih.gov



&
 
  



d. www.webmd.com
 
$

 
6. ___ A mercury-based preservative found in some 14. Because cases of anaphylaxis associated with
vaccines MMR can be severe, the nurse should ensure
7. ___ Preparations made from donated blood  



$




which contain a high concentration of anti- before administering any immunization?




 


  
 
=|?
8. ___ Preparation containing whole or fraction- 
>@
=¢?
ated microorganisms, causing the recipient’s 
& 
=– ?
immune system to manufacture antibodies 
Z
= ?
directed against the microbe

‡

\

 


š
]
 


 
@ 
  
$

10. ___ Protein produced by the body’s immune a child who is behind in getting immunizations
 

  



  





 
$
$

-
11. ___ Develops in response to infection or to  
$
]``—„
…
=†<]„
>?

 

administration of a vaccine or toxoid acute otitis media that is treated with amoxicillin?
12. ___ Bacterial toxin that has been changed to a a. Administer needed immunizations at the next
nontoxic form, causing the recipient’s im- scheduled well-child visit.
mune system to manufacture antitoxins b. Administer needed immunizations during this
visit.
a. Active immunity c. Do not administer immunizations until the child
b. Antibody is afebrile.
c. Antigen d. Do not administer immunizations until the child
d. Avirulent 
 

 

#
 
$
#@  6]—
#


$
  

$

 


g. Passive immunity contact the prescriber if a child experienced which


h. Thimerosal symptom after administration of a DTaP immuni-
i. Toxoid zation?
j. Vaccine a. Drowsiness
k. Vaccination b. Refusal to eat

Š 
$ 
 
=Š#{? 
{



! 
$
]``‹„
…
=†<„
>?

Copyright © 2016, Elsevier Inc. All Rights Reserved.


162 CHAPTER 68 Q
>
#@ 

17. When should the nurse provide parents or legal 6^]


#


$
  


 

-
guardians with vaccine information sheets? scriber if MMR was ordered in which situation?

–$

 
 
$

  
 


a. The child’s mother is receiving chemotherapy
be administered for breast cancer.
b. Before each dose of each vaccine b. The child has had a positive Monospot test
c. Once, before any vaccines are administered =Z–Š
$?
d. Only when requested c. The child is being treated with chemotherapy
for leukemia.
š18. The nurse would consult with the prescriber if 
!


 

*#Š
 
6  which immunization was prescribed for a child
who is receiving long-term high doses of predniso- š
^^
!
 

 

 
\  


ƒ
={

 
? an 18-year-old college-bound female patient. The
a. DTaP nurse tells the patient, “You cannot get pregnant
b. Hib $
^

$
#



“





#|Š not a good statement for the nurse to make to this
d. MMR adolescent?
e. Varicella vaccine a. Pregnancy is not contraindicated after MMR
vaccination.
19. Foster parents bring a 14-month-old child in for b. The statement assumes that the patient is sexu-

 
 


  ‚
$
!
>
ally active.
Protective Agency is unable to obtain any immuni- c. The statement is a communication blocker.
zation records. The child has no contraindications d. The patient may interpret the statement as
to vaccination. What approach to immunizations is meaning that the injection prevents pregnancy.
appropriate?
a. Administer immunizations as if the child has 23. Which nursing actions would be critical when the
not received any vaccines per the catch-up $
 

  


€€
 -
schedule. cination with the parents of a 12-month-old boy?
b. Administer the immunizations regularly sched- ={

 
?
uled for 12-14 months of age. a. Determining whether the child has allergies to
c. Delay immunizations until the records can be gelatin
found. b. Teaching that any adverse effects from receiv-

#@




@ 
ing the vaccine should occur within 48 hours
scheduled for 2 months of age. c. Asking whether the child has received blood or
blood products in the past 6 months
20. What are appropriate measures to decrease dis- 
# 

 

 

$
$

$
 

@  ƒ
={
they notice unusual bleeding or bruising

 
? 
# 

 




$

a. Administer the appropriate dose of acetamino- having close contact with anyone who is preg-
phen or ibuprofen 30-60 minutes before the nant for 3 weeks after receiving the vaccine
immunization.
b. Apply topical anesthetic to the injection site š
^‹
!
 
 


 
$
 


prior to administering the injection. are concerned about their 2-month-old son, who
c. Aspirate before injecting an intramuscular vac- received a DTaP vaccination 5 days ago. Which
cine. symptoms would be of most concern to the nurse?
d. Tell the child when the nurse is done with a. Crying inconsolably for the past 24 hours
administration of the immunization. b. Breast-feeding for 20 minutes every 3-4 hours

ˆ 
^%

^—%  


 - c. Redness and swelling at the injection site
ing the injection. d. Temperature of 103° F for 12 hours
25. Why does the CDC currently recommend ad-
ministration of the inactivated poliovirus vaccine
=#|Š£#|}‰¤

{ ?
  
$

 

 -

=}|Š'{ ?ƒ
a. OPV can cause vaccine-induced polio.

#|Š


 
 
}|Š
c. OPV has not been studied long enough.

}|Š


$$
 
#|Š

Copyright © 2016, Elsevier Inc. All Rights Reserved.


CHAPTER 68 Q
>
#@  163

26. Which information would be a reason to consult


†`
 

 
$
 

 ƒ
={-
the prescriber before administering the varicella 

 
?
vaccine to a 15-month-old child? a. All children age 1 to 12 years
a. The child had a severe reaction to the MMR b. Children with hemophilia that requires clotting
vaccine. factor infusions
b. The child is allergic to eggs. c. Children with renal failure
c. The child’s aunt and her newborn are temporar- d. Children who will be traveling to the Caribbean
ily staying with the family. e. Children who will be traveling to Japan

!
$

 

$

 


1-inch needles. š
†]
| 
$





 


 

†% %



 

š
^~
!
 
$
 


 
$


because they were afraid that she would get the


father of a 16-year-old who received a varicella
disease from the injection. The pediatrician con-
vaccine 10 days prior. The teenager experienced
vinced them to allow immunization in January of
a fever after the injection, which was treated with
this year. When should the child receive the next
aspirin. Yesterday he stayed home from school
 

 ƒ

 

*



 


a. As soon as the new vaccine is available



$ 

 
$
 



{ 
$

 
What directions should the nurse provide to the
c. November of this year
father?
d. January of next year



 

 

 


intake.
632. The nurse administers Rotarix rotavirus vaccine
b. Make an appointment for his son to be seen in
to an infant at a medical clinic in the Dominican

$



  
 
#

$
  

$

 

c. Reassess the teenager’s temperature after ad-


to teach the child’s caregivers to seek immediate
ministering acetaminophen and call back in the
emergency care if within the next month the infant
afternoon.
exhibits which symptom?

{
 
 
 
$


-

‰ 

¬
^



mediately.
b. Runny nose for 3 days
28. Which child should return for a booster dose of 
{
 

 
 
 
]

$

 
 ƒ 
{
 


”
a. A 12-year-old who did not experience a vesicu-
 
 
$

 
  33. A patient who has had a splenectomy asks why
b. A child who was12 years old when receiving he needs to get a meningitis vaccine. What is the

 
 



 
 basis of the nurse’s response?
c. A child younger than 13 years old at the time of a. The spleen destroys old red blood cells.

 
 


 
 b. The spleen forms blood cells.
d. A child who is younger than 18 months c. The spleen is a critical defense whenever a
patient has a fever.
29. The nursery nurse would expect to administer 
!


 

  
-

  = ?


 

]^
monia and meningitis.
hours of birth if the neonate’s mother is HBsAg-
 ƒ
={

 
?

* 
–
  
=*–#\? CASE STUDIES
b. Havrix

#@
  Case Study 1
d. Pneumovax
Parents bring their 7-month-old child to the clinic. They
e. Recombivax HB
state that they were told by an emergency department
f. Twinrix
=Z&?
  
$

 
  






for “baby shots.” The parents are concerned, stating that


they have heard that children can die from these shots.

1. What teaching can the nurse provide about the


$


$

@  ƒ

Copyright © 2016, Elsevier Inc. All Rights Reserved.


164 CHAPTER 68 Q
>
#@ 

^
!
 
 
$
  

Case Study 3
concepts. How can the nurse explain reactions to
immunizations? During pharmacology class, a nursing student remarks
that he would not have his 11-year-old daughter vacci-
 

\  
  
’ 



“

8. What should be included in the discussion about


3. How would the nurse explain the laws requiring this parent’s reaction?
children to have immunizations before they can
attend school and, in some cases, day care?

9. The concerned parent/nursing student agrees with


the explanation but states that his daughter has
4. What subjective data and medical history does the heard that the injection is painful. What might
nurse need to review before administering an im- the parent discuss with the pediatrician about this
munization? concern?

Case Study 2 10. The literature reports a number of adolescent


patients fainting when given the human papillo-
| 
$

 



‰  
|-  
=*|Š?
 
 
 
 


sylvania have verbalized to the school nurse that their 


 

 
 

$
 



children have never been “contaminated” with immuniza- prepare for this possible reaction?
tions. The parents state that their unimmunized child has
not contracted the measles, mumps, diphtheria, or other
illness for which immunizations are routinely adminis-
tered.
11. Another nursing student states that she will have
5. How should the school nurse respond? her daughter vaccinated because the adolescent
will not have to have Pap smears and pelvic ex-
aminations until she is ready to get pregnant. What
should be included in response to this statement?

6. The parents state that they are particularly con-


cerned about MMR because of its association with
autism. What has research suggested about the
association of MMR and autism?

7. The school nurse is aware that exposure to certain


populations or situations puts unvaccinated chil-
dren at risk for contracting vaccine-preventable
disease. What are possible populations and situa-
tions that would put children at risk in this school
and geographic area?

Copyright © 2016, Elsevier Inc. All Rights Reserved.


CHAPTER 69 Q
#   165

69
Immunosuppressants

STUDY QUESTIONS 8. Which symptom should the nurse teach a patient


who has been prescribed immunosuppressant
Completion drugs to report because it is an early sign of infec-
tion?
1. Because of the possible adverse effect of a. Chills and rigor
prednisone on blood glucose, the nurse b. Cough
should assess the patient for symptoms of c. Rash
_______________________. 
{
 

^
ˆ 



  
 
$$

9. A 16-month-old child receives a heart-lung trans-


of _______________________.
plant. What teaching should the nurse provide
3. A liver transplant patient’s orders include discon- about immunizations relating to immunosuppres-
 
$

 
=|  $?

 
sant therapy?
 
  
!
 

 

 
a. The child may need more than the usual recom-
at 10:00 AM
!
 
 
 


 - mended doses of inactivated vaccines.
tered no sooner than _______________________. b. The child may need more than the usual recom-
mended doses of live vaccines.
4. Because sirolimus has a half-life of 2.5 days and it c. The child should not receive any inactivated
is categorized as pregnancy category C, the nurse vaccines.
teaches women of child-bearing years to continue d. The child should not receive any live vaccines.
using birth control for ______________________ e. The child should not receive any vaccines.
after discontinuing use of sirolimus.
10. Body image is developmentally important to many
5. The nurse should teach patients who take siro-   
#

 
$

 

 

limus to take the medication consistently either body image when cyclosporine has been pre-
without food or with consistent amounts of scribed to a young woman, because the drug can
_______________________ in foods. cause
a. acne.
6. The nurse should teach a patient who is
b. facial hair.
  
 
$
=>-
c. rash.
>?


 
%%
=}!>?

d. weight gain.
_______________________.
611. Because of the risk of nephrotoxicity with admin-
CRITICAL THINKING, PRIORITIZATION,   
$
 
={ ?



AND DELEGATION QUESTIONS nursing assessment is


a. intake and output.
6 7. The nurse is providing postoperative teaching for b. pulse.
a patient who has had carpal tunnel release surgery c. temperature.
and who is receiving immunosuppressant therapy d. weight.
for rheumatoid arthritis. Based on the surgery and
medication, the priority nursing concern in this
case would be preventing
a. airway constriction.
b. impaired tissue perfusion.
c. infection.
d. pain.

Copyright © 2016, Elsevier Inc. All Rights Reserved.


166 CHAPTER 69 Q
#  

6]^
!
 

 
 
=#Š?

]~
! 

$
 

]`
 '‰
 

 
={ ?



should the nurse do?
after beginning the infusion, the patient complains a. Repeat the trough level.
of feeling hot and says her chest feels tight. Which b. Consult the prescriber before doing anything
action would be of greatest priority? else.

 
  

= ? c. Administer the drug.
b. Assess vital signs. d. Withhold the drug.
c. Assess lung sounds.

{

$ 
$
  618. Which symptom, if reported by a patient receiv-


=  ?






 13. When reviewing the laboratory results of patients report to the prescriber or attending physician?

 
={ ?

 - a. Productive cough

=|  $?

 = ?



 
b. Fatigue

 

  ƒ
={

 
? c. Joint pain

‰!
^<
  
 d. Tinnitus

–ˆ+
]~
 '‰

> 
†^
 '‰
]‡

 
$
 


=¢ ?



| 
‹
Z"'‰ most likely to cause toxicity?
e. WBC 3800/mm3 
Z
b. Phenobarbital
14. The nurse is providing discharge teaching regard- c. Phenytoin
ing prescribed oral cyclosporine to prevent rejec- d. Rifamycin
tion of a transplanted kidney. Which statement, if
made by the patient, suggests understanding of the 20. Due to the adverse effect on bone density, it would
  ƒ
={

 
? be a priority to protect the patient from falls when

’#

 





- taking which drug?

$
#
 
 
$
  

 
a. Azathioprine
pain or if my BM changes color.” b. Prednisone

’#

 


  
$

$


{
prescriptions.” d. Tacrolimus

’#



 

 
#
$



taking this drug.” 21. Which laboratory results should be immediately



’#

 



 
  “ reported to the prescriber when a patient is admin-
 
@ 
=# ?ƒ
 15. A liver transplant patient who is prescribed tacro- 
‰!
—
#ˆ'‰

=|  $?
 
 





>\
†^<
#ˆ'‰
grapefruit juice, but he has made sure it has been c. Platelets 145,000/mm3
at least 4 hours between the juice and the medica- d. WBC 4900/mm3
tion. Which lab result, if a change from recent
 
  

 %$
 ƒ
^^

"= ?





a. Blood pressure 88/47 mm Hg  



 
= ?ƒ


\…
`
‰' ={

 
?

… 

 
^†
 '‰ a. Administer via a central line using an inline
d. Temperature 102° F 

 
$
  
  

 @

6]—
#



$

 

 

 

anxiety.

  
 
=|  $?

 
 
c. Dilute in 5% dextrose in water.
of over-the-counter drugs and not take drugs that 
#$ 
ž
‹



contain
a. acetaminophen.
b. acetylsalicylic acid.
DOSE CALCULATION QUESTIONS
c. diphenhydramine.

^†

 

  
 
 
=\  $?

d. ibuprofen.
9 mg/kg/day divided into 2 doses 12 hours apart.
The patient weighs 132 lb, and the medication is
  



$
]``
 '‰
*


medication should the nurse administer?

Copyright © 2016, Elsevier Inc. All Rights Reserved.


CHAPTER 69 Q
#   167

^‹
€ 
$
]



^


- Case Study 2
scribed before renal transplant surgery. The drug
is diluted in 5% dextrose in water at a concentra- A 38-year-old kidney transplant recipient is prescribed

$
]```
 '^<
‰
!
 


  
 


 
={ -
  

‰'
 
 


 
?
 

 @
program into the pump?
6. The patient asks why she is prescribed ketocon-
azole in addition to cyclosporine. What should the
nurse explain is the main reason these two drugs
CASE STUDIES are administered concurrently?

Case Study 1
A 4-year-old girl received an allogenic heart transplant at
age 16 months for transposition of the great vessels and

~
#

 
$

 

 

 

only one ventricle, which resulted in cardiac failure.


about the cyclosporine’s possible effects on a
developing fetus and that current contraceptive
1. What is an allogenic transplant?
recommendations include use of which type of
birth control?
The patient was discharged on cyclosporine, azathioprine
=# ?

 
{

 




transplant hospital monthly for bloodwork to monitor


rejection status and performance of her new heart. During
8. What teaching should the nurse provide relating to
the past year, she has shown no sign of rejection or limita-
the adverse effect of increased risk of neoplasms
tion of her activities. Her growth has more closely fol-
from taking cyclosporine?
lowed normal growth charts, and outwardly she appears
to be a healthy 4-year-old. Recently, following a trip to a
national park, she developed fatigue and appeared tired.
Bloodwork indicated evidence of macrophages and mono-

 

”
 
{

  -

‡
!
  
$
 
={ ?


ized and diagnosed with evidence of transplant rejection.


ordered a trough level after the fourth dose. When
should the trough level be drawn?
2. What are nursing priorities for this patient?

†
# 
 %>&†





10. The patient’s cyclosporine orders are changed


patient’s medication regimen to prevent rejection

 
 
=\  $?
!
  

of the transplanted heart. What actions should the


  


{ 
$ 

nurse take in the hospital to prevent the patient


=]``
 '‰?
 


 
ƒ
from contracting an infection?

4. What special steps must be taken when adminis-


11. The patient has a history of osteoarthritis and a



#Šƒ
seizure disorder. What should be included in the
plan of nursing care relating to immunosuppres-
sant therapy that she is receiving and her medical
history?


…

$

 


 
 

of muromonab-CD3, the patient’s parents tells the


nurse that the patient is shivering. Her temperature
is 102.6° F. These symptoms could be indicative
of further organ rejection or adverse effects of the
drug. What should the nurse do?

Copyright © 2016, Elsevier Inc. All Rights Reserved.


168 CHAPTER 70 Q Antihistamines

70
Antihistamines

STUDY QUESTIONS 12. The parent of a 3-year-old child who was recently
diagnosed with asthma calls during phone hour
True or False 

  ‚
$
!





which often triggers an asthma attack. The parent


   
  T   F has forgotten which drug the child should be given
   when he starts wheezing. Which drug would be
safe and effective?
H1-blocking drugs for allergic reactions

& 
=– ?ž %  

H1 antagonist
1. ___ are the active ingredient in most over-the-

‰  
=> ?ž %  
*1

=}!>?




antagonist
2. ___ can cause excitement, nervousness, and

…$ 
=  ?ž %  
*1
tremors.
antagonist
3. ___ can cause urinary retention.
d. No antihistamines are safe and effective for
4. ___ cause the skin to become red and warm.
wheezing of asthma
5. ___ decrease release of histamine present in high
levels in the skin.
13. Why is it important to teach patients to be pre-
6. ___ decrease pruritus.
pared for possible allergic reactions every time
7. ___ elevate the pH of stomach secretions.
they take a certain drug?
8. ___ have sedation as the most common adverse
a. Because allergic reactions are common.
effect.
b. Because allergic reactions are most likely to oc-
9. ___ prevent local edema.
cur on the third or fourth day of taking a drug.
10. ___ prevent the release of histamine from mast
c. Because allergic reactions frequently occur
cells and basophils.
even if a patient has never been exposed to the
drug or its components.
CRITICAL THINKING, PRIORITIZATION, d. Because allergic reactions usually occur after
AND DELEGATION QUESTIONS previous exposure to a drug or its components.

14. On admission, the nurse is reviewing all of the


11. A patient is scheduled to receive a radiocontrast
drugs that a 78-year-old patient takes at home. The
media for a CAT scan. The patient has never
patient has a history of hypertension and diabetes.
received any radiocontrast media. Why must the
!
 
 
}!>
 
=– -
radiology nurse carefully assess for an allergic
?





! 
 
 


reaction?
$$
$
 %  
  




a. Allergic reactions are common because patients


body and on a cold. Which statement, if made by
often are unaware if they have had radiocon-
the patient after the teaching, suggests a need for
trast media in the past.
additional teaching?
b. Allergic reactions can occur even without prior
a. “Benadryl does not prevent colds but it can
exposure to radiocontrast media.
make my cold go away faster.”
c. Allergic reactions occur in more than 30% of
b. “Benadryl can make mucus from a cold hard to
patients.
expel because it thickens the mucus.”
d. Allergic reactions to contrast media are usually
c. “Benadryl can make my constipation worse.”
asymptomatic until anaphylaxis occurs.
d. “My 3-year-old granddaughter could have con-
vulsions if she took some of my Benadryl.”

Copyright © 2016, Elsevier Inc. All Rights Reserved.


CHAPTER 70 Q Antihistamines 169

15. The term   


 is used to refer to drugs DOSE CALCULATION QUESTIONS
that block
a. H1 receptors. 20. Fexofenadine, 30 mg twice a day, is prescribed
b. H2 receptors. $

‹% %

 %
 
#

c. H1 and H2 receptors. 
  


  
 
—
 '‰

*
 



 

 

16. Why do second-generation H1 blockers cause less


dose?
 
 
 %  
*1 blockers?
a. They are less potent.
b. They bind reversibly to histamine receptors.

^]
| @
=| ?



  
$

c. They do not cross the blood-brain barrier.


a patient with nausea and vomiting. The drug is
d. They are rapidly metabolized.
  


 '‰



$

-
 
`]





 
$




6]~

$
 
 
 
$


to infused no faster than 25 mg/minute?


by the delivery room nurse in a full-term neo-
nate whose mother has taken diphenhydramine
=– ?
” 
$


 



priority to immediately report to the pediatrician? CASE STUDY



{ 
–|
—`

*

! 
†~^„
>
=‡‡„
…? A 25-year-old taxi driver has come to his health care pro-
c. Pulse 180 beats/min vider to obtain information and help for his “hay fever.”
d. Respirations 25 breaths per minute He says he doesn’t know what to take because OTC drugs
seem so confusing. His condition is diagnosed as allergic
18. The nurse is teaching a patient who has a history rhinitis and is seasonal, with reactions lasting from May
$
 
  
 
=–|*?

 
until the end of June each year. He also says that he is oc-


 
   
=> ?
casionally bothered by dust during the rest of the year. He
 

 
ƒ
={

 
-  
$
  Œ
  Œ
@ Œ


? =
 ?Œ


 
 
 
*







>+{
   how allergies cause these symptoms.
b. Avoid any activity that requires coordination or
alertness. 1. What information could be included in the expla-
c. High doses are more likely to cause sedation. nation?
d. Taking doses higher than recommended does
not help the drug work better.
e. The drug may aggravate urinary retention from
the enlarged prostate.
2. What are the disadvantages to antihistamines for
š
]‡
!
 

 
—
AM medications, in- those who drive or use heavy equipment at work?

 
=>%!?
‹


every 6 hours for urticaria. The patient complains


of nausea associated with the medication. What
should the nurse do?
a. Change the timing of the medication to 8 AM 3. The patient is concerned about cost. He asks if
=`<``?
]^

=]^``?

PM
=]~``?

<
prescription antihistamines are more effective than
PM
=^```?

 

 


}!>
 
=– ?
*


snack. the nurse respond?


b. Decrease the dose of the medication.
c. Hold the medication and consult the prescriber.
d. Provide a snack with each dose of medication.

4. How do second-generation antihistamines work


but not cause sedation?

Copyright © 2016, Elsevier Inc. All Rights Reserved.


170 CHAPTER 71 Q
>  
#  •
+  
%$  
&

 


!
  

@ 
= ?
^
 
6. What suggestions can the nurse make if the patient
of 250 micrograms in each nostril twice a day. chooses to use one of the antihistamines that cause
Why is this medication appropriate for this pa- sedation?
tient?

71
  !" #
  $      !

STUDY QUESTIONS CRITICAL THINKING, PRIORITIZATION,


AND DELEGATION QUESTIONS
Completion
1. The half-life of aspirin is 15-20 minutes, but the 6 9. The following information was included in the
antiplatelet effects last _________ ____________. change of shift report on a patient who is pre-
scribed aspirin for its anticoagulant effects. Which
2. Preoperative teaching for knee replacement patient symptom, if present, would be a priority
surgery should include that the patient should for the nurse to report to the attending physician?
discontinue high-dose aspirin therapy for a. Abdominal bloating
___________________ before surgery. 
Z 
$
 % 
 
c. Heartburn when recumbent
3. When taking aspirin for antiplatelet action, other d. Two liquid stools in the past 24 hours
+{#&


  
$


 

by at least _______ hours. š


]`





 
 
$


nurse to administer an as-needed dose of 650 mg


4. Aspirin should not be taken by children, especially of aspirin?
for symptoms of viral illness, because of the risk a. Hand pain and stiffness when arising
of ________________ ________________. b. Knee pain with ambulation
c. Right temporal headache associated with a
5. The nurse should instruct patients to dispose of tense neck
aspirin tablets if they develop an odor that smells d. Abdominal pain with distension
like ___________________.
11. What should the nurse do to prevent the most
6. Chronic alcohol use interferes with the metabo- common adverse effect of long-term aspirin
lism of large doses of __________________. therapy?

 





$
7. _________________________ is the antidote for b. Assess lung sounds for wheezing before admin-
acetaminophen overdose. istering the drug.
c. Monitor urine output.
8. The recommended maximum dose of acetamino- d. Teach the patient to report tarry-colored stool.
phen for individuals who regularly consume
alcohol is ____________.

Copyright © 2016, Elsevier Inc. All Rights Reserved.


CHAPTER 71 Q
>  
#  •
+  
%$  
&

  171

12. Which statements, if made by a patient who is 18. A patient has been prescribed aspirin 81 mg once
receiving aspirin therapy for rheumatoid arthritis,
 
$
 
  
#

 

suggests that the patient needs further teaching? for the nurse to teach this patient to avoid using
a. “Drinking a full glass of water with my aspirin 
%%
=}!>?
 ƒ
can help prevent pill particles from getting 
 
=!?
trapped in the inside folds of my stomach.” 
> 
   
=!ˆ€{?

’#


$


  

  

\ $ 
=  ?
it will cause abdominal pain.” 
# $
=€
?

’#$
#



 
 

 

and make it easier for aspirin to cause it to 19. Which aspirin preparation is the least irritating to
bleed.” the stomach?

’#


 


 

a. Buffered aspirin solution

  
$
#



 

Z% 

runny nose.” c. Four chewable children’s aspirin
d. Timed-release aspirin
13. Which diagnostic test is performed before starting
long-term aspirin therapy to identify an increased
^`
#





$


$

risk for gastric ulceration? patient has taken aspirin before delivery of a neo-
a. Colonoscopy nate?
b. Complete blood count and differential a. Boggy uterus

Z     b. Fatigue
d. Test for * c. Perineal discomfort

{ 
6]‹
#

$
  

$

 

-
cate self-prescribed aspirin use to the primary care š
^]




$
  


provider if the patient has which history? when a patient reports regular use of sodium sa-

\
  
  licylate for joint pain?

{ 

 

 

$
$
a. Chronic obstructive pulmonary disease
birth control b. Heart failure

{

 
$
 

  c. Diabetes mellitus
d. Type 2 diabetes mellitus 
Z 


 


615. A patient brought into the emergency department 22. A patient has been prescribed naproxen/esome-
=Z&?
$
 

$

  
 
 @
=Š?

 
  
 

the following symptoms. Which symptom would patient teaching regarding administration of this
be of greatest priority for the nurse to address? drug has been successful?
a. Bronchospasm a. “Taking this drug combination is better than
b. Profuse, watery nasal discharge taking the two drugs separately.”
c. Tachycardia b. “This drug combination will protect my kid-

ˆ  neys.”
c. “The additional drug decreases acid production
16. Which symptom suggest that blood levels of a in my stomach.”
salicylate such as aspirin are too high? d. “The addition of the second drug will make the
a. Fatigue naproxen more effective.”
b. Heartburn after meals
c. Ringing in the ears
^†
# 
=#?

  
$
-
d. Vomiting mature neonates to promote closure of the duct
located between the
š
]~
!
 


  
 
$

 

a. inner ear and cochlea.
minute when assessing an 18-month-old child with b. liver and the duodenum.
suspected salicylate poisoning. Which laboratory c. pulmonary artery and the aorta.
results support this diagnosis? d. umbilical vein and the neonate’s abdomen.

*
~†^Œ
| CO2
‹`

* Œ
*>}3´
%^`
Z"'‰

*
~†Œ
| CO2
†~

* Œ
*>}3´
^^
Z"'‰

*
~‹]Œ
| CO2
†

* Œ
*>}3´
^`
Z"'‰

*
~‹—Œ
| CO2
†]

* Œ
*>}3´
]<
Z"'‰

Copyright © 2016, Elsevier Inc. All Rights Reserved.


172 CHAPTER 71 Q
>  
#  •
+  
%$  
&

 

š
^‹

 
 

  
 


š
^‡
 
 


 


 -
   
=#€?
$

 


" 

 

  ‚
$
 
 


pain relief during labor. Which nursing action is 15-month-old child just ingested an unknown
most appropriate at this time? quantity of children’s chewable Tylenol tablets?
a. Administer the drug and monitor the neonate’s a. Assess for the presence of nausea, vomiting,
respirations for depression after delivery. abdominal pain, or diaphoresis.
b. Assess the patient’s stage of labor and question b. Direct the parent to seek immediate medical
the order if the patient is in transition. care for the child.

˜ 
 


”
$

c. Make an appointment for the child to be seen as
woman in labor. soon as possible.
d. Withhold the drug and question the prescriber. d. Take the opportunity to teach the parent about



š
^
!
  


=> ?

100 mg to be administered twice a day. The nurse 6†`



$ 
 
= ?
$

would consult the prescriber before administering acetaminophen overdose, the nurse notes that the
the drug if the patient is allergic to patient is scratching her arms. Which action is of
a. amoxicillin. greatest priority at this time?
b. Bactrim. a. Assess respirations, breath sounds, and vital
c. Ceftin. signs.

¢  b. Consult the prescriber regarding changing to an
oral formula.
6^—

 





$
c. Report the itching to the prescriber.
the nurse to report if a patient was receiving cele- 
{

$ 

 

  


=> ?ƒ {!!
a. Bruising of arms and shins
b. A weight gain of 1 lb each day for 3 days š
†]

†% %


 


Z&
!

c. Heartburn at night child’s mother states that he had a rash and fever

# " 
”
 
$
$
 
$
]
over a week ago. He vomited several times last
week night. This morning he was so drowsy that she
could not rouse him. The child is lethargic and
6^~
#

$
  




  


  
  
#

$
  
-
when a 65-year-old female patient is prescribed ity to ask the mother if she administered which of

=> ?ƒ the OTC drugs to her child during his illness?
a. Belching a. Bayer low-dose aspirin
b. Bruise on left arm b. Robitussin cough syrup

&$
  
{ $
   
d. Headache d. Tylenol elixir
e. Nausea

š
^<

 




% 
$ 
DOSE CALCULATION QUESTIONS
for rehabilitation after an open reduction and inter-
 
 
=}#…?
$


$ 
{
" 

†^
 
= ?
^^`

 

|
=  
†^
 %
is to be infused over 30 minutes. The drug is

 ?
]
 


‹





^``

$
œ
 
!
$ 

before going to physical therapy. The patient states 



   

‰'
 
 



that her pain is 2/10 currently but that she experi- nurse enter as the infusion rate?
ences unbearable pain if she does not take the Per-
cocet before therapy. The patient has not received
a dose of this drug in the last 4 hours. What should 33. The nurse is teaching parents about use of infant
the nurse do?  

=<`
 '`<
‰?

%
a. Administer the drug.  
 
*
 



-

Z 

 

 

$
 
istered per dose to a 13-month-old child?
to take the drug so that it is longer between
doses.
c. Withhold the drug and question the prescriber.
d. Withhold the drug because the patient’s pain
does not warrant administration of a narcotic.

Copyright © 2016, Elsevier Inc. All Rights Reserved.


CHAPTER 72 Q
\

+
&  173

CASE STUDIES Case Study 2


A 21-year-old patient comes to the family planning clinic
Case Study 1 to determine what she can do for relief of moderate
The nurse is assisting a 58-year-old science teacher who is   
{
 
 

 


   -
postoperative total abdominal hysterectomy and bilateral  

 
 $


 
 
$


 %
=!*%–{}?

  
  
{




 

 
 

in the hallway when the patient complains of knee pain her sleepy and that she has tried acetaminophen without
and stiffness. The patient has a history of hypertension relief. After further assessment, the nurse practitioner

  
=  
”
  
& &?
!
suggests that she try ibuprofen as a beginning drug to see
 
 


  

>}%^
 
$



  
{



 
^
 $
=^``%



$

 
 

{


 
  ?

‹

$

 
^%†
 
$

-
that the drug really helped her knee pain and did not cause  

 


 

$
 


 
 


+{#&
 
{


$
cramping.
she could have the medication again.
4. Why is this schedule appropriate in this situation?
1. What could the nurse tell the patient?

5. What information should be provided to this

^
!
 
 
’#

 





patient about possible adverse effects of this drug
 
#
 
 
$


 


  
therapy?
side effects.” How should the nurse respond?

6. Based on the developmental stage of this patient,


3. What nonpharmacologic teaching can the nurse what teaching should the nurse provide about use
provide this patient? $
 $


%%
+{#&

drugs?

72
Glucocorticoids in Nonendocrine Disorders

STUDY QUESTIONS a. Bisphosphonates



# 
Matching 
‰


+{#&
          e. Vaccines

1. ___ May require increased dose

^

# 
 
$
 
3. ___ Can decrease antibody response

‹

# 
 
$
 
5. ___ Can help prevent osteoporosis

Copyright © 2016, Elsevier Inc. All Rights Reserved.


174 CHAPTER 72 Q
\

+
& 

CRITICAL THINKING, PRIORITIZATION, 12. Which are common symptoms of hypokalemia


caused by glucocorticoids with high mineralocor-
AND DELEGATION QUESTIONS
ticoid activity?



 

6. Which possible effect of glucocorticoid therapy in
b. Hypotension and cool, clammy skin
the doses used for nonendocrine disorders would
c. Muscle weakness and fatigue
be a priority to report to the prescriber?

! 




 

> 
‡
 '‰

>
‡
 '‰
13. Alternate-day glucocorticoid therapy is most ap-

| 
†]
 '‰
propriate for a patient who is

{
]‹
 '‰
a. 9 years old.
b. 29 years old.
6 ~
#







  

c. 45 years old.
test result to the prescriber if a patient is receiving
d. 65 years old.
therapeutic doses of glucocorticoids?

‰!
†`
  
 '‰
š
]‹

 







–ˆ+
^`
 '‰
therapy for 2 months is being tapered off the
c. A1c 5.9%
glucocorticoids. Which symptom suggests a with-
d. WBC 2000/mm3
drawal syndrome?
a. BP 84/47 mm Hg
 8. The school nurse would notify the parents of a

… 
 
^
 '‰
fourth-grade child who is receiving oral glucocor-

| 
†
Z"'‰
ticoids to control asthma symptoms if height and
d. Pulse 55 beats/min
 




a. change from the 35th to 75th percentile in height.
š
]

 

 
  
$

 -
b. change from the 75th to 35th percentile in height.

 
  
=>}|&?

 

c. gain of 2 lb in the last year.


$
 
$ 
=*…?
–  
$

 

d. gain of 5 lb in the last year.


risk for digoxin toxicity, the nurse should assess
$
 
$$= ?ƒ
={

 
?
9. The nurse should be especially vigilant in as-

‰
$

sessing for adverse systemic effects of a topical
b. Positive Chvostek’s sign
glucocorticoid therapy when administering the
c. Flushed skin
  
={

 
?
d. Confusion
a. to intertriginous areas.
e. Visual changes
b. to mucous membranes.
c. to well-hydrated skin.
d. to infants. DOSE CALCULATION QUESTIONS
e. under an occlusive barrier.
16. A patient is prescribed dexamethasone 8 mg
š
]`





 
$
 

  
 
=#Š?
 
!


  



if which immunization was administered when the ” 



$
]`
 '‰
# 






 ƒ
={
to administer the dose over 1 minute. How many

 
? ‰


 
 


 ƒ

# 
@

#”
@

€  
 

 
=€€? 17. Dexamethasone 5 mg orally every 12 hours is
d. Pneumococcal prescribed for a 10-year-old child who weighs
e. Varicella 82.5 lb. The recommended maximum safe dose for


`† ' ' 
#

 
$ƒ
š
]]
| 

 %

 
 

minimize a common complication of therapy by


having adequate servings of which foods in their
diet?

–

 
b. Citrus fruits

‰ 
d. Whole grains

Copyright © 2016, Elsevier Inc. All Rights Reserved.


CHAPTER 73 Q Drug Therapy of Rheumatoid Arthritis 175

CASE STUDY 
#$•
 

  


phagocytic activity of neutrophils and macro-


A 43-year-old man with severe persistent asthma is admit- phages
ted with an acute exacerbation. After the crisis is averted,
the patient is prescribed dexamethasone sodium succi-
 
‹


 
!
  

  

\ 
 
 

 '‰

1. How much should the nurse administer?


e. Myopathy

$
Z 
  

 
2. The patient is switched to oral glucocorticoid
therapy. Because of the severity of the patient’s
asthma, the prescriber explains that long-term oral
glucocorticoids will probably be necessary in ad- g. Mood
dition to an inhaled glucocorticoid and bronchodi-
lators. What instructions should the nurse provide
this patient regarding minimizing the following h. Cataracts and glaucoma
adverse effects of glucocorticoid therapy?

 
 $
i. Peptic ulcer disease

b. Osteoporosis

73
Drug Therapy of Rheumatoid Arthritis

STUDY QUESTIONS a. Disease-modifying antirheumatic drugs


=&€& ?
Matching 
\

+  
%  


     =+{#& ?

1. ___ Provide rapid relief of symptoms and can



  
  Œ

 %

use they can cause serious toxicity.


2. ___ Provide rapid relief of symptoms, but do
not prevent joint damage and do not slow
disease progression.
3. ___ Drugs that reduce joint destruction and slow
  
  Œ




Copyright © 2016, Elsevier Inc. All Rights Reserved.


176 CHAPTER 73 Q Drug Therapy of Rheumatoid Arthritis

CRITICAL THINKING, PRIORITIZATION, 9. A patient with RA is being treated with oral pred-
 
$

 
$


 



AND DELEGATION QUESTIONS


patient would suggest to the nurse that this patient
4. Which medication use for rheumatoid arthritis needs more teaching?
=?
$

 ƒ
={

 

’#$
#

 






? may not be able to deal with infection, healing,


a. Do not discontinue glucocorticoid therapy once and stress.”
 
  


  
’  “ 
’#

 



$
#


 


#$

 




this drug because it can make my bones weak.”
+{#&


&€& 
’#



 





{ 


&€&

†

$
 - guidance from my doctor.”
nosis. d. “Taking this drug with food will prevent me

{ 


+{#&

 

from having any bad effects.”
$
’  “

! 

+{#&



&€&

 10. A 16-year-old female patient takes methotrexate

 
$

$

 
$
 once a week for juvenile RA. Which laboratory
test result for this patient would be of greatest


 


  
$
>}%^
 

concern for the nurse?


=> ?

 %  

–

 
=–ˆ+?
^`
 '‰
>}%]

%^
 


 ƒ 
Z
 
 
=Z{?
†`
'
a. They decrease production of stomach acid. hr
b. They do not decrease gastric mucus production. 
ˆ
>\
 
c. They do not irritate the stomach as much. d. White blood cell count 11,000/mm3
d. They are more effective in reducing pain and
   11. Which laboratory test result should be monitored


 


$  @
=@-
6. Which teaching is of greatest priority if a patient ?ƒ
has taken prednisone 10 mg twice a day for 6 a. CBC

$


 


 

 ƒ 
Z
a. Avoid people who have infections. c. Fasting blood glucose
b. Be sure to get enough rest. 
ˆ

 
c. Follow instructions for tapering off of the drug.
d. Take steps to manage stress.  12. When performing a nursing assessment on a
 


=  ?

 -
7. A patient has been prescribed methotrexate servations would be a reason for the nurse to hold
=&€&?
 
$

  
the medication and contact the prescriber?
with RA. The patient has heard that these drugs a. Abdominal pain and dark urine
can be dangerous and asks the nurse why the b. Alopecia and skin rash
prescriber has not ordered a prescription-strength c. Nausea and diarrhea
+{#&
 


 
  


 ƒ d. Rhinorrhea and sneezing
a. “Methotrexate delays joint degeneration.”
b. “Methotrexate has fewer adverse effects than 613. When a female patient is prescribed hydroxy-
 %  
+{#& “ "
=| "?
$



$
  


’€ 

$ 
 
+{#& “ priority for the nurse to assess if the patient has

’| %  
+{#&
 
  
- adhered to recommended follow-up care with

 Œ
 

“ which specialist?

Z  
 8. A patient who is unwilling to take other drugs 
\   
$




  

‡~



\  
  ?

—
 


$
- d. Ophthalmologist
enced by this patient, suggests aspirin toxicity?
a. Bruising

Z 
 
c. Jaundice
d. Ringing in the ears

Copyright © 2016, Elsevier Inc. All Rights Reserved.


CHAPTER 73 Q Drug Therapy of Rheumatoid Arthritis 177

 ]‹

 
 
$



 
620. What is the priority nursing concern when admin-
who is receiving a biologic DMARD, should the  
 

= ?ƒ
nurse report to the prescriber immediately? a. Airway
a. Dizziness b. Hydration
b. Fever 
#$
c. Headache 
# 

#”% 


^]
 



{!!


  

15. Which symptom suggests that a patient who is when a patient has been receiving tocilizumab
  
 
=Z ?
 

- = ?



$

 
^
  ƒ



 
$$
$
{ %  
- ={

 
?
drome? 
‰!
†^
#ˆ'‰
a. Diminished breath sounds and wheezing b. ANC 1750/mm3
b. Nausea, vomiting, and diarrhea 
–|
]^`'~^

*
 Œ
]]`'—<
 
c. Painful, red rash and blisters 
{
‰‰˜
 
d. Perioral edema and tingling 
! 
 
]<
 '‰

 ]—
!
 
$
 

 

$ 

to a rheumatologist for a child who has juvenile DOSE CALCULATION QUESTIONS



 


  

 
#

would be most important for the nurse to identify


^^
Z 
=Z ?



  
$

%
whether the child has received or is scheduled to 
]`% %

 
#

 
$ƒ
receive which immunization before the child sees
the specialist?
a. DTaP
^†
Z 
=Z ?



  
  -

#|Š 
$


  


 '‰
 


#@
#€ amount should the nurse administer?
d. Varicella

]~
#


$
  




 
CASE STUDY
 


  
$

= ?ƒ
a. ANC 3500/mm3 A 34-year-old woman who works as a grocery store clerk

{!
‹
#ˆ'‰ has been diagnosed with RA after seeking care for carpal

–+|
^<~
   '‰ tunnel syndrome.

!
!
`]
 '‰
1. The patient asks how the disease developed from

]<

 




$
$
 
$
carpal tunnel syndrome. How should the nurse
the PPD tuberculin test administered 50 hours ago respond?
before initiation of treatment for RA with adali-

=* ?
!
 




test results as positive and consult the prescriber if


the skin reaction was greater than
a. 5 mm of erythema. 2. The patient’s physician has recommended that the
b. 5 mm of induration. patient see a vocational rehabilitation counselor.
c. 5 cm of erythema. The patient asks the nurse, “What type of job
d. 5 cm of induration. 
#


ƒ“
 
 
 

the nurse provide?


19. A male patient who has been taking a DMARD is
planning to try to conceive a child with his wife.
The nurse must counsel him about the need for a



 


$


$

the patient has been taking which DMARD?



 
=* ?

 
=?

#
= ?

‰
=  ?

Copyright © 2016, Elsevier Inc. All Rights Reserved.


178 CHAPTER 74 Q
&
! 
$
\

3. When developing a long-range plan of care for 5. What symptoms of methotrexate toxicity to these
this patient, what interventions might the nurse organs should the nurse teach the patient to report
include to address the four goals of RA therapy? to her prescriber?

‰
a. Relieving symptoms

b. Kidney
b. Maintaining joint function

c. Bone marrow
c. Minimizing systemic joint involvement


\#
d. Delaying disease progression

4. The rheumatologist has recommended that the


 
 


 
 

+{#&


 
= ?
– 



developmental stage of this patient, what teach-


ing would be especially important for the nurse to
provide to this patient?

74
Drug Therapy of Gout

STUDY QUESTIONS CRITICAL THINKING, PRIORITIZATION,


AND DELEGATION QUESTIONS
True or False
   
  T   F 7. A patient who experiences an attack of gout once
   every 1 or 2 years has been self-treating with
%%
=}!>?
 




1. ___ Colchicine is the most common drug used to the nurse expect to provide the least relief of gout
prevent gout attacks. pain?
2. ___ Chemotherapy for cancer can cause gout. 
 
=!?

†

\
 



 
 

=– 
?

‹

\

  

 
   
# $
=
€?



#
 


@
 

+ 
=?
when white blood cells break down damage
the joints.

—

ˆ 
  
 
  
 

Copyright © 2016, Elsevier Inc. All Rights Reserved.


CHAPTER 74 Q
&
! 
$
\ 179

 8. A male patient develops an acute gouty attack


]†
#

$
  



$

 -
$

$
 

*

  

sible adverse reaction to pegloticase?
=> ?








‹<
 
a. Dyspnea
The patient calls the prescriber stating that he 
Z

”

is experiencing nausea and abdominal pain. He 
#
wants to know what he should do. The telephone d. Flare of pain
triage nurse should advise the patient to
a. stop taking the medication.
b. take the medication before going to bed. DOSE CALCULATION QUESTIONS
c. take the medication with food.
d. take the medication with milk. 14. A patient is prescribed allopurinol 150 mg once
a day. Available are 100-mg tablets. How many
6 ‡

 


  

=¢- tablets should be administered per dose?
?
$

 



laboratory test result would be a priority to report


to the prescriber?
]
|  
<


  



^`%‰


–ˆ+
^^
 '‰ bag of normal saline solution. The drug is to be

> 
†<
 '‰ administered over 120 minutes. What is the rate of

Z{
^<
' $ 

‰'ƒ

ˆ

‡
 '‰

10. The nurse teaches the patient to stop taking allo- CASE STUDY

=¢?

 

  
$


experiences which effect? A 5-year-old boy who is scheduled to start chemotherapy


a. Diarrhea in 2 weeks for acute leukemia is prescribed allopurinol
b. Drowsiness 150 mg once a day. His father researches the drug on the
c. Fever #



 




  

d. Headache when his son does not have gout.

]]
!
 

 

=¢?
1. How should the nurse respond?


 

 

 $ 
=> -
?

  
 
 



-
ity to report to the prescriber of the Coumadin?

–ˆ+

 '‰

\…
‡`
‰' 2. The patient’s father does not like to give drugs to

#+
‹— his children. What protective effects of allopurinol
d. WBC 9200/mm3 would be most important for the nurse to include
in an explanation of why this drug is prescribed?
12. What is an important nursing action to prevent
complications when initiating therapy with pro-
benecid for gouty arthritis?
a. Avoid taking BP or drawing blood from the af-
fected extremity.

# 

 

^``%†```
‰' 

Z 

$$

d. Measure intake and output.

Copyright © 2016, Elsevier Inc. All Rights Reserved.


180 CHAPTER 75 Q
&
$$
> 
‰

–
€ @ 

75
Drugs Affecting Calcium Levels and Bone Mineralization

STUDY QUESTIONS CRITICAL THINKING, PRIORITIZATION,


AND DELEGATION QUESTIONS
Completion
1. The three factors that regulate serum cal- 610. The priority goal of the body’s regulation of cal-
cium levels are _____________________, cium levels is maintaining normal calcium levels
_____________________, and in which body system?
_____________________. a. Cardiovascular
b. Muscular
2. Because free calcium is the active form of cal- c. Nervous
cium, a patient with low protein levels can exhibit 
{ 
symptoms of ____________________________.
 11. A 55-year-old female consumes 6 oz of plain, low-
3. An inadequate level of free, ionized cal- fat yogurt and 16 oz of skim milk each day. How
cium may result in blood taking longer to 
 
 
 
=
 

____________________. &?


$

 ƒ
a. 300 mg once a day
4. Central nervous system symptoms of hypercal- b. 300 mg twice a day
cemia include ______________________ and c. 600 mg once a day
______________________. d. 600 mg twice a day

5. Hypocalcemia increases neuromuscu- 12. What is a known adverse effect of excessive cal-
lar excitability, which can cause symp- cium supplementation?
toms such as ____________________, a. Bleeding
____________________, b. Diarrhea
____________________, and c. Dysrhythmias
____________________. 
Z

6. Parathyroid hormone secretion 613. The nurse is caring for a patient whose lab results
____________________ calcium absorption in the 
 
]^
 '‰
 


 

small intestine. priority?


a. Administering medications on time
7. Patients should not take calcium supple- 
Z 
" 
 
ments with ____________________ or c. Preventing falls
____________________ because they interfere 
{
  
with calcium absorption.
614. The nurse is caring for a patient whose lab results
8. Patients should take calcium supplements that also 
 
—<
 '‰
 


 

contain _____________________ because this priority?


would increase absorption of the calcium. a. Administering medications on time

Z 
" 
 
9. Calcium supplements decrease the ab- c. Preventing falls
sorption of ____________________, 
{
  
____________________,
____________________,
____________________, and
____________________.

Copyright © 2016, Elsevier Inc. All Rights Reserved.


CHAPTER 75 Q
&
$$
> 
‰

–
€ @  181

 15. A 62-year-old woman is receiving chemotherapy 19. A 58-year-old woman is at risk for osteoporosis
for metastatic breast cancer. Which laboratory caused by a history of hypothyroidism treated with
result suggests the patient is at increased the risk 
={?
`]


 

for hypercalcemia? {




 

 
 
 -

> 
†^
 '‰ 
{

 
 
 
  -

*'*
]]
'‰'††œ 
=!ˆ€{?

‹``

$
 
 


*&‰
†
 '‰ Which statement, if made by the patient, would

…–{
]‹
 '‰ indicate a need for further teaching?

’#
 
 
!ˆ€{

$






16. The plan of nursing care for a patient with bone



 
$
#
 


 
 

cancer who is at risk for hypercalcemia should cereal.”


include teaching to avoid which food in the diet? 
’#

 

!ˆ€{


  

$

a. Bran water.”
b. Canned sardines 
’!
 



 
$
#
 


{  each tablet at separate times throughout the
d. Whole-grain cereal day.”

’!ˆ€{






$
 

 17. Which question would provide the most useful oyster-shell calcium, but they are less expen-
information when taking the history of a child sive.”
diagnosed with rickets?

’&


 

$


^`
#

$
  

$

 



school?” for which symptom when administering parental


b. “Has your child ever had a kidney infection?” calcium to a patient who is prescribed digoxin
c. “Have you noticed a change in your child’s =‰ ?ƒ
energy level?” a. Hypertension
d. “How many hours a day does your child watch b. Decreased urine output
television?” c. Dehydration
d. Bradycardia
 18. The nurse is assessing a postoperative thyroidec-
tomy patient. Which assessments suggest that the 21. The pediatric nurse is explaining why vitamins
parathyroid glands may have been damaged or re- should be kept out of the reach of children. Which



 ƒ
={

 
? 
  
$$
$
 
&
ƒ
={


& 
$

 


$ 
$


 
?
other toes when the sole of the foot is stroked a. Diarrhea
with a blunt object from lateral heel to medial b. Constipation
toes c. Bone pain

# 
$

–|
$$
$
†


d. Tetany
involuntary spasms of the wrist e. Vomiting

‰
$
 

 




closed, arms at sides, and legs together 22. Patients with which chronic disorders are less able

{ 
 $
 

 
$

$ 
 
 
&
$
 ƒ
={


e. Twitching of facial muscles when the facial  


?
nerve is tapped a. Chronic renal failure

>
 
 
  
=>}|&?
c. Cirrhosis
d. Osteoarthritis
e. Type 2 DM

23. A 3-year-old child is admitted after ingesting an


unknown number of chewable multiple vitamins

 
&

 
 
$

present, would be of most concern to the nurse?


a. BP 80/50 mm Hg
b. One watery stool in past 4 hours
c. Pulse 102 beats/min
d. Confusion and ataxia

Copyright © 2016, Elsevier Inc. All Rights Reserved.


182 CHAPTER 75 Q
&
$$
> 
‰

–
€ @ 

24. Which statements are true regarding calcitonin  29. A patient with hypercalcemia of malignancy
=€  ?ƒ
={

 
? =*>€?

@ 
=¢ ?



#

  
 


 symptom, if present, suggests that the patient may

#


$ 
$$

$
  

be experiencing drug-induced hypomagnesemia?
men. ={

 
?
c. Nasal pumps must be primed before each use. a. Anorexia
d. The drug promotes bone formation. b. Dry, sticky mucous membranes
e. The drug should be administered using hand c. Muscle weakness
opposite of nostril. d. Muscle spasms

6^

 
 
$



 
30. When a patient is prescribed a biphosphonate,
who is receiving a biphosphonate such as alendro- which nursing measure would be most effective to
 
=…  ?


$
  

$
prevent the possible adverse effect of osteonecro-
the nurse to report to the prescriber? sis?
a. Dysphagia 
Z @

 
$

 

b. Dysphasia hygiene
c. Headache b. Taking calcium supplementation with the drug
d. Muscular pain c. Teaching the importance of weight-bearing
exercise

^—
€
$
  
 
 
d. Teaching the importance of avoiding alcohol
serum creatinine, urine output, weight, and intake and tobacco
and output is especially important for the nurse to
monitor when a patient is receiving which biphos- 31. What causes osteonecrosis when a patient is pre-
phonate? scribed biphosphonates?

 
=…  ? a. Decreased bone breakdown

Z 
=&? 
# 

 

! 
={? 
# 
$ 



¢

=¢ ? d. Reduction in blood calcium

27. Taking calcium supplements at the same time as


†^



$
 $
=Z  ?



a dose of a biphosphonate for osteoporosis does estrogen receptor modulator?


what? 
#
 
  




 



# 
%  teach to report blurring of vision.

#$


$

   
#
 
 


$

 

c. Potentiates the action of the biphosphonate =


  ?
d. Prevents adverse effects 
#


 
$

$ 

  

fractures.
28. A patient who has just started taking tiludronate 
#







 
$

={?
$
| ‚
  
 

  ‚
breast cancer.
$


 



  


 

 
 


 


633. Which information, if elicited from a patient who
nurse should do? 

 $
=Z  ?


$
 -
a. Advise the patient to stop taking the drug. est priority for the nurse to communicate to the
b. Determine if the patient is taking the drug with prescriber?
a full glass of water. a. Has an aunt who has been diagnosed with

# 

 

 




estrogen receptor–positive breast cancer
antacid. b. Has had a hysterectomy

# 

 

 



$ 
#


 
d. Does work that requires frequent, long airplane
trips

Copyright © 2016, Elsevier Inc. All Rights Reserved.


CHAPTER 75 Q
&
$$
> 
‰

–
€ @  183

634. Which laboratory test result, if found in a woman


‹`
# 
$ 


  
$

who has just been prescribed raloxifene, would be patient in a hypercalcemic emergency. The nurse
the priority concern to the nurse? knows that which imbalance would be least likely

> 
]]]
 '‰ to occur as a result of this therapy?

*  
=* ?
]]—
'‰ a. Hypocalcemia
c. Hemoglobin A1c 6.8% b. Hypoglycemia

* 

 
=>\?
††`
c. Hypokalemia
#ˆ'‰ d. Hyponatremia

35. To increase bone mineral density, teriparatide


=…?


 
ƒ DOSE CALCULATION QUESTIONS
a. Continually

#” 41. The nurse’s drug book recommends that calcium

#    

$ 


 
$
`

^
‰'

d. Orally #$

 
$ 


$
]`œ
 
-
conate, how long will it take to infuse the entire
36. Which action should the nurse perform when amount?


 
  
=…?


patient with osteoporosis?


a. Administer in the deltoid or vastus lateralis
‹^
& 
=  ?
]^`

  
”-
muscle with a 1-inch needle. tion is prescribed every 4 weeks. Available is 120
b. Administer subcutaneously in the upper arm,  ']~
‰
 



 ƒ
alternating arms daily.
c. Date the injection pen for disposal 28 days after
 
  CASE STUDY
d. Remove the pen from refrigeration 20 minutes
before administration to allow the solution to A 50-year-old patient who states that she is going through
warm. menopause is undergoing a thyroidectomy today. The
 
 
 

 




 

37. Which symptoms would be a priority to report is available postoperatively. The patient has just been
to the prescriber if a patient who is receiving admitted to the nursing unit from the postanesthesia care
chemotherapy is scheduled to receive denosumab 
=|>ˆ?
=  ?ƒ
a. Fatigue and anorexia 1. Knowing the anatomy of the thyroid gland, why
b. Flank pain and fever would the nurse anticipate a potential need for
c. Nausea and vomiting #Š
 ƒ
d. Redness at injection site after previous injection

38. Which action should the nurse perform when




 
 
=| ?


 

   ƒ
={

 
? 2. What is the average normal value for total serum
a. Administer into the posterior gluteal muscle. calcium?
b. Assess current dental needs.

# 

$
 


d. Discard if particles present in solution.
e. Warm to room temperature for 15-30 minutes.

†
!
 ‚
 



—<
 '‰

 †‡
!
 


  
={  ?
# 
]`œ
 
 





 


  
$


- = ?
$ 

  
!
 



ing was discovered during patient assessment? in a chair when the nurse prepares to administer
a. Anorexia the calcium gluconate. Why is it important for the
b. Diarrhea nurse to assist the patient back to bed before the
c. Headache drug is administered?
d. Numbness

Copyright © 2016, Elsevier Inc. All Rights Reserved.


184 CHAPTER 76 Q Drugs for Asthma and Chronic Obstructive Pulmonary Disease

4. What should be monitored while the nurse is 5. The patient’s spouse has assumed that calcium is
administering the calcium gluconate? in the bone. The spouse asks how surgery on the
neck can cause an imbalance in the calcium in the
body. What could the nurse include in the discus-
sion of the functions of calcium and the mecha-
nism for calcium regulation?

76
Drugs for Asthma and Chronic Obstructive Pulmonary Disease

STUDY QUESTIONS 
&%
 
=&|#?
e. Dysphonia
Matching $
\

#  

      
€% 
 
=€&#?
i. Nebulizer
1. ___ Adventitious breath sound that occurs with j. Prophylaxis
bronchoconstriction of asthma 
{ 
2. ___ Drug that may increase the number of bron- l. Wheezing
chial beta2 receptors as well as their respon-
siveness to beta2 agonists

†

&

 



€&#


CRITICAL THINKING, PRIORITIZATION,


increase delivery of drug to the lungs and AND DELEGATION QUESTIONS
decrease deposition of drug on the oropha-
ryngeal mucosa 613. Which outcome would be of greatest priority for

‹

*   
 
$ most asthma patients?
5. ___ Measures taken to maintain health and pre- a. Avoiding pollution
vent illness 
# 
 
 
6. ___ Most effective drugs available for reliev- 
|
 
  
ing acute bronchospasm and preventing 
{ 
 
$
 
exercise-induced bronchospasm
7. ___ Persisting for a long period of time 14. A patient has received instructions from the nurse
8. ___ A small machine used to convert a drug to administer two puffs of a beta2-agonist drug via
solution into a mist
% 
 
=€&#?

 

‡

ˆ 





$
$


by the patient would indicate a need for further
micronized powder directly to the lungs teaching?
10. ___ Handheld, pressurized devices that deliver a 
’#


 







measured dose of drug with each actuation goes deep into my lungs.”

]]

{ 
$
 

 
 
 - 
’#




€&#
 
 
#
 


rows airways  
#




“

]^

ˆ
  
$
 
’#

 

 
$
 


inhaler.”
a. Beta2 agonist 
’#

 

 
]
$

$
#
 


b. Bronchoconstriction second puff.”


c. Chronic

Copyright © 2016, Elsevier Inc. All Rights Reserved.


CHAPTER 76 Q Drugs for Asthma and Chronic Obstructive Pulmonary Disease 185

15. What are current recommendations for use of š


]‡

  @
 


$
 

inhaled glucocorticoids for persistent asthma? =|


… ?


={

a. Administer daily even on days where there are &  ?



  

]^


 -
no symptoms. 
=|?
€&#

$$

‹



 
 




- needed. The patient has not required albuterol for
lator. the last 24 hours. When preparing to administer

}
 


 

$

 
$
the budesonide and salmeterol, the nurse notes
wheezing. audible wheezing and dyspnea. Which action by
d. Only use on a regular basis if acute attacks oc- the nurse is correct?
cur every day because of adverse effects. a. Administer albuterol only.
b. Administer albuterol, wait 5 minutes, and then
16. Which symptom suggests that a child with asthma administer the salmeterol and then budesonide.
is not rinsing his or her mouth after using a gluco- c. Administer salmeterol and then budesonide,
corticoid inhaler? and reassess in 15 minutes for the need for

Z  
  albuterol.

\ 
  


  d. Administer budesonide, wait 5 minutes, admin-
c. Persistent cough ister the salmeterol, and reassess in 15 minutes
d. White patches in mouth for the need for albuterol.

17. The nurse would question the prescriber if which 620. The nurse is caring for several patients who
drug regimen is ordered for an asthmatic patient? receive beta-agonist inhalation treatments for
={

 
? asthma. After treatments are administered, it is of
a. Albuterol inhaler, two puffs before exercise, not greatest priority to reassess the patient who also
exceeding eight puffs in 24 hours for a 14-year- has a history of which condition?
old patient 
* 
$ 
=*…?

 
=+ ?
$

 

&

  
=&Š!?
  
=|
  ?

 

\    

=\Z&?
nebulizer for a 6-year-old patient 
 

=?

– 
  
=|
  ?

$

 
=+ ?

”
 -  21. A 42-year-old patient with asthma is admitted for
lizer with a mask for a 4-year-old patient a surgery. Which information would be of greatest

{ 
={
&  ?

  
priority for the nurse to share with the anesthesi-
every 12 hours for a 20-year-old patient ologist?

{ 
={
&  ?

  
a. This patient experiences an asthma attack

 
=|
… ?

]^
whenever exposed to tobacco smoke.
hours for a 40-year-old patient 
!
 

  
”
  
= -
  ?
$

 
 ]<
#

$
  


 
  

 - 
!
 
 
 
=|?
€&#


tain adequate bone mass if an asthmatic patient is puffs every 4 hours as needed.
prescribed which drug? d. The patient was recently changed from long-

… 
=–?
€&# term oral methylprednisolone to inhaled beclo-

… % 
= ?
&|# methasone.

+ @
  
=?
d. Prednisolone oral tablets  22. The nurse administers albuterol via nebulizer
when a patient with asthma experiences severe
@



 

{ 

minutes after the treatment, the patient is rest-


ing quietly with his eyes closed. What should the
nurse do?
a. Allow the patient to rest.
b. Assess the patient’s breathing and lung sounds.

#$

 
 
 


lunch will be needed later.


d. Wake the patient and encourage him to try to
 



Copyright © 2016, Elsevier Inc. All Rights Reserved.


186 CHAPTER 76 Q Drugs for Asthma and Chronic Obstructive Pulmonary Disease

 ^†

 



 


 ^<
¢  
= ?

 


€


prescribed a beta2-adrenergic agonist would be a be administered at 7:00 AM


=`~``?
– $ 


reason for the nurse to consult the prescriber? served at 8:30 AM


=`<†`?
 


 

a. BP 140/90 mm Hg do?

{€–\
]—
 '‰ a. Administer the drug as ordered.

ˆ 
$  b. Change the timing of the drug to 8:30 AM

ˆ 
 =`<†`?
c. Change the timing of the drug to 10:00 AM
 ^‹
#$
 
=–?

 
=€  ?
=]```?
are both ordered to be administered at 0900, how d. Consult the prescriber.
should they be administered?

… 
 

 
]

 
29. The nurse is explaining therapy with cromolyn to
using a spacer for both drugs a patient who has asthma. Which statement should

… 
 
 

 

 
not be included in the teaching?
3 minutes later a. Cromolyn must be taken on a regular basis to

| 
 
 

 

 

  
5 minutes later b. Rinse the mouth immediately after administra-

| 
 

 
]`

tion to remove the unpleasant taste.
later, using a spacer for both drugs c. Timing doses 15 minutes before activities
involving exertion or exposure to known al-
25. Why does the FDA recommend that patients with lergens may prevent bronchospasm.
asthma who use an inhaled long-acting beta- d. The drug is not effective in stopping an episode
 
 
=‰–?
 


once it has begun.
prescribed a combination product with a glucocor-
ticoid rather than the two drugs as separate inhal- 630. The nurse is caring for a patient who has asthma
ers? 
 
 @
= ?


 


#
 

 
 

 

 
department. Which symptom would warrant im-

‰–



 mediate, priority care?
b. The two drugs given together as one inhalation a. Fever
produces faster bronchodilation. b. Headache

!
  



&|#

$

c. Perioral edema
less harmful to the environment. 
{
  
d. The combination drug is less expensive than
the two drugs given separately. 631. An adolescent patient with asthma has controlled
her asthma using a drug regimen that includes
26. The nurse is caring for a patient who is prescribed theophylline. Which new behavior would be of
@  
= ?
$


 $ 
$
greatest priority to report to the prescriber?
 
  



  
 
$

a. Joining the soccer team
interaction of these two drugs? b. Occasionally skipping school when not ill
a. Bradycardia c. Becoming sexually active
b. Hemorrhage 
{

 
$
 

 
c. Pulmonary emboli
d. Myocardial infarction  †^

 
$



 




  
 %  
=> -
27. The nurse would be most concerned about drug in- ?


$
  






teractions if a patient who has asthma is prescribed prescriber?


both zileuton and a. Allergy to peanuts
a. albuterol. b. Complaint of sore throat
b. prednisolone. c. Drinking 4-6 cups of coffee per day
c. terbutaline. 
{

 
$
 

 
d. theophylline.

Copyright © 2016, Elsevier Inc. All Rights Reserved.


CHAPTER 76 Q Drugs for Asthma and Chronic Obstructive Pulmonary Disease 187

33. Which adverse effect would the patient be most CASE STUDY
likely to experience when prescribed the anticho-
 
 


={ ?ƒ 
~% %



 
  



a. Blurred vision at age 4 years, has been admitted to a medical unit from
b. Constipation intensive care after experiencing a severe acute exacerba-
c. Dry mouth tion. Based on the frequency, characteristics, and sever-

ˆ 
 
$

 



 

 

severe persistent asthma. The pediatrician has ordered oral


34. A patient who has asthma has had peak expiratory  
]`


<
 
 
=|


 

 
~`œ
$

  
  ?
`^
 

 
]^


 @

best despite regular and as-needed use of drugs. every 6 hours as needed.
What are current treatment recommendations for
this patient? 1. What is the difference between administering
a. Continue the regularly prescribed treatment. budesonide and albuterol via nebulizer?

ˆ 

 
 


  

prescribed treatment.

ˆ 

 
 

 

  

for possible treatment changes.



{
 
 
 
 2. The boy’s parents tells the nurse that they have
heard negative things about steroids and are con-
š
†



 


  
- cerned about their son receiving them. What does
cause she has just learned that she is allergic to the nurse need to teach regarding the importance
the family dog. Which intervention would most of steroid therapy for this child?
effectively address the physical and psychosocial
needs of the child?
a. Advise the family that there are no interven-
tions that will help.
b. Advise the family to increase the dose of the 3. The parents have heard that steroids prevent
child’s asthma drugs when the dog is around. growth. What information can the nurse provide
c. Advise the family to take the dog to the local about the effects of steroids on growth?
shelter.
d. Advise the family to train the dog to stay out of
bedrooms and off of furniture.

DOSE CALCULATION QUESTIONS 4. What teaching does the nurse need to provide
about the inhaled steroid therapy to prevent ad-

†—
– 
=|
  ?
`^


- verse effects?
scribed via nebulizer. Budesonide is available as
^`
 '^
‰
*

 



administered via nebulizer at each dose?


5. The nebulizer is more time-consuming to use than

†~
} @
= ?

 

%
 
=&|#?

% 
-
  



$
]`
 ']^
‰
†``

 
=€&#?
 

  
 



subcutaneous is prescribed once every 4 weeks. route was prescribed for this child?
How much drug should be administered?


—
 


 



-
gic to house dust mites. What teaching should be
provided regarding exposure to allergens?

Copyright © 2016, Elsevier Inc. All Rights Reserved.


188 CHAPTER 77 Q Drugs for Allergic Rhinitis, Cough, and Colds

~
#


  
 



8. After 2 years of standard allergy treatment, the
be avoided because they are common triggers of child is still experiencing asthma attacks two to
acute asthma episodes? three times a week, and he is missing many days
of school. The allergist has prescribed omalizumab
= ?
^``



 
  -
ously every 4 weeks. What is the procedure for
administering this drug?

77
Drugs for Allergic Rhinitis, Cough, and Colds

STUDY QUESTIONS 13. ___ The most effective OTC nonopioid cough
medicine, and the most widely used of all
Matching cough medicines

]‹

{




       15. ___ Cough suppression is achieved only at doses
that produce prominent sedation
1. ___ Conjunctivitis
]—

{ 


 



^

Z carries a greater liability for abuse

†

#  
4. ___ Perennial a. Acetylcysteine
5. ___ Pruritus b. Benzonatate
6. ___ Rhinitis c. Dextromethorphan
7. ___ Rhinorrhea d. Diphenhydramine

<

{   
\ $ 
f. Hydrocodone
a. Antibodies 
# 

#  
$


  h. Omalizumab

#  
$

  
-
ing the eyelids and eye surface

# CRITICAL THINKING, PRIORITIZATION,
e. Occurs during spring and fall in reaction AND DELEGATION QUESTIONS
to outdoor allergens
f. Nonseasonal, triggered by indoor aller- 17. A patient exhibits watery nasal discharge and
gens sneezing every winter when the house is closed
g. Redness as a result of injury or irritation and the forced-air furnace is running. How is this
h. Runny nose  ƒ
a. Perennial rhinitis
        
{  


9. ___ Renders cough more productive by stimulat-


]<
 



 
$
={





$
  
 
  
?
10. ___ Decreases sensitivity of respiratory tract a. glucocorticoids.
stretch receptors b. histamine.

]]

 

 
# Z c. leukocytes.
12. ___ Blocks cholinergic receptors, thereby de- d. leukotrienes.
creasing rhinorrhea e. prostaglandins.

Copyright © 2016, Elsevier Inc. All Rights Reserved.


CHAPTER 77 Q Drugs for Allergic Rhinitis, Cough, and Colds 189

19. What are the most effective drugs for prevention š


^

$
 
 
 
$


and treatment of symptoms of seasonal and peren- in a patient who is taking an oral decongestant,
nial rhinitis? would be a priority to report to the prescriber?
a. First-generation oral antihistamines a. Agitation

#  
 b. Chest pain
c. Oral glucocorticoids 
Z  

{%  
 
   
{
 

š
^`
!

 

 




š
^—
#

 



 
 
$

long-term therapy with intranasal glucocorticoids the nurse to administer a prescribed as-needed
for seasonal rhinitis. The nurse must monitor the dose of an antitussive medication?
child’s a. Barking cough of croup
a. blood sugar. b. Cough that interferes with work because the
b. hearing. patient cannot carry tissues at work in which to
c. height. expectorate the mucus
d. weight. c. Cough associated with upper respiratory infec-
tion that keeps the patient awake at night

^]

 

  
 
=… ?
^

{
 
$
 


-
 

 
 
^


{
 

duction that only occur on arising
nurse to report that the drug has not helped. Which
question should the nurse ask to identify a com- 27. A patient who recently saw a commercial for
mon cause of early treatment failures?  
={  ?


$
 


a. “Do you have nasal burning after administra- the prescriber will not order this for her allergic
tion?” symptoms. The nurse knows that prescribers may
b. “Do you have nasal congestion?” not readily prescribe this drug for allergic rhinitis
c. “How often are you administering the sprays?” for what reason?
d. “What is the expiration date on the bottle?” 
#
$"
  
 
 

 

#

 
 
$$ 
22. The nurse teaches a patient with allergic rhinitis 
#



$$


   
that antihistamines are not effective in reducing 
#
 

@

 
which symptom?
a. Nasal itching
^<

 

  
 @
= ?
$

b. “Runny nose” allergic rhinitis. Which statement, if made by the



’{$$
 “ patient, suggests a need for further teaching about

{@ this drug therapy?

’#
 
 



‰ 
& 




23. A patient has received instructions regarding ad- sneeze all day.”
ministration of a second-generation oral antihista- 
’#



 





mine for seasonal allergic rhinitis. Which of these winter to control my allergy to house dust.”
statements made by the patient would indicate that c. “The doctor has prescribed this drug to control
the patient needs further teaching? my allergy symptoms because allergies trigger

’#


 

 

#

my asthma.”
experiencing symptoms.” d. “This drug will be injected into the fat under

’#

 

 

  
my skin.”
 

 

#
 
 

symptoms.” 29. Research suggests that codeine, dextrometho-



’!

 

  
 

#


rphan, and diphenhydramine are not effective in
be careful when driving.” suppressing coughs induced by
d. “This drug is not any more effective than over- a. chemical irritation.
the-counter antihistamines.” b. common cold.
c. mechanical irritation.

^‹
 


  
$
  

=+ - d. smoking.
?

 
 ƒ

#
 

 
  

#

% % 
  

#


$$
 

 

#

$

Copyright © 2016, Elsevier Inc. All Rights Reserved.


190 CHAPTER 77 Q Drugs for Allergic Rhinitis, Cough, and Colds

30. A patient has received instructions regarding CASE STUDIES


  
$
@  
=! 
| ?

Which statement made by the patient would indi- Case Study 1


cate that the patient needs further teaching?
a. “The drug should not be given to infants.” A 19-year-old college student presents to the student

’#
 
 





 “ health clinic during spring semester with complaints of

’#



 $
  


 
runny and itchy nose, sneezing, and nasal congestion. He
make me drowsy.” is diagnosed with allergic rhinitis. The student states that

’#

 

 





he has been using an over-the-counter nasal spray for 2
soft food such as applesauce.”  
#
 



  




seems to be getting worse.


631. Despite instruction by the nurse to swallow the
@  
=! 
| ?


 

]
Z 
 

 



 
 -
proceeds to chew the capsule. What is a priority tinue use of this drug.
nursing concern until the effect of this drug has
diminished?
a. Aspiration
b. Bronchospasms 2. The nurse practitioner prescribes loratadine
c. Respiratory depression => ?
]`

 

 
=+ -

{
   ?
^
 


 
 
 
 


nurse provide?
32. Which statements are true about the common
ƒ
={

 
?
a. Antibiotics are not effective.
b. Antihistamines do not help.

#

 
 



-
†
#$









cations. change his medications?


d. Fever usually means there is a bacterial infec-
tion.
e. Vitamin C prevents colds.
Case Study 2
A patient asks the nurse why she doesn’t need a prescrip-
DOSE CALCULATION QUESTIONS 

 ‚

{ $
$$

$
{

 


had to go to the pharmacy counter and sign a paper to get


Diphenhydramine 50 mg orally every 6 hours is pre- 
{ $


‚



 

 

scribed. The patient has purchased 25-mg capsules. on her Peace Corps assignment.

33. How many capsules should the patient take for 4. What should the nurse include in the response?
each dose?

34. The nurse would teach the patient not to take more
than how many capsules in 24 hours?

!
 



 

{ $
|Z
$$


$
$
{ $
 

 


 

substance. What should the nurse include in the


explanation?

Copyright © 2016, Elsevier Inc. All Rights Reserved.


CHAPTER 78 Q
&
$
|
ˆ
&   191

Case Study 3 7. The parents ask what they can do to help their
child feel better. What current nonpharmacologic
Parents of a 17-month-old girl ask the pediatric on-call recommendations can the nurse provide?
nurse why their doctor did not give them samples of
PediaCare cold medication like she did with their older

!
 
 


$
^
 



their daughter is still coughing.


8. The parents ask the nurse what they should do if
6. How should the nurse respond? their 4-year-old catches the cold from his younger
sister. What instructions should the nurse provide
if the parents choose to use cold remedies for this
child?

78
Drugs for Peptic Ulcer Disease

STUDY QUESTIONS 
€  
=>?
$
} @
=| ?
Matching 
{ $ 
=>  $ ?

! @
=!  ?
    

 
True or False

]

> 
  

  %
 


hence must not be combined with alcohol    


  T   F
2. ___ Works quickly to neutralize acid in the stom-   
ach

†

{


 
$
 
9. ___ Aluminum hydroxide antacids bind phos-
prostaglandins phate, warfarin, and digoxin, decreasing
4. ___ Creates protective barrier in stomach against absorption.
acid and pepsin 10. ___ Aluminum hydroxide antacids frequently
5. ___ Disrupts the cell wall of * , thereby cause diarrhea.
causing lysis and death 11. ___ Calcium carbonate antacids can constipate.
6. ___ Causes irreversible inhibition of H+, K+- 12. ___ Calcium carbonate antacids do not cause
ATPase, the enzyme that generates gastric 

  
acid
]†

ˆ 
$
 
   


-

~

{ 

$
* by inhibiting cause of acid rebound.
protein synthesis 14. ___ Magnesium hydroxide antacids are used to
8. ___ Blocks H2 receptors thereby reducing both help diagnose abdominal pain.
the volume of gastric juice and its hydrogen 15. ___ Magnesium hydroxide antacids should not
ion concentration be used in severe renal impairment.

]—

{
   
 
 

-
a. Aluminum hydroxide sorbed into systemic circulation.
b. Bismuth
]~

{
   
 

*

> 
=– ?

>
=! ?

Copyright © 2016, Elsevier Inc. All Rights Reserved.


192 CHAPTER 78 Q
&
$
|
ˆ
&  

CRITICAL THINKING, PRIORITIZATION,


^‹

 

   

  

=\Z&?


$
$

 -
AND DELEGATION QUESTIONS
mens with multiple doses has been prescribed

=! ?
<``



 

-

]<
#




 
 

 
|ˆ&


time. The patient complains that the medication


importance of
effects wear off before taking the next dose. To
a. avoiding use of alcohol.


 
$$
$

 

 

b. consuming more dairy products.


instructs the patient to take the medication
c. not using tobacco products.
a. twice a day.
d. using ibuprofen, not acetaminophen, for head-
b. with food.
aches.
c. on an empty stomach.
d. with an antacid.
19. Hypersecretion of gastric acid is the etiology of
what?
š
^
>
=! ?
 

>™|^&—


a. Duodenal ulcers
CYP3A4 hepatic enzymes. The nurse must be par-

\ 

ticularly cautious for toxic effects of which drugs
c. Peptic ulcers
that are metabolized by these enzymes when they

¢ %Z 


are administered with cimetidine, because of their
 
  
$
$ƒ
={

 
?
20. Drug therapy that prevents recurrence of peptic

+  
={ ?ž 
ulcers associated with * must include

|
=& ?ž @
a. antacids.

| 
=& ?ž %  
b. antibiotics.
d. Theophylline—asthma
c. antisecretory agents.

 $ 
=> ?ž   
d. mucosal protectants.
26. The generic names of histamine2 receptor antago-
21. Which dietary alteration may facilitate recovery
 
=*2 ?
 


$ƒ
from ulcers?
a. -azole
a. Avoiding caffeine intake
b. -lol

Z 

 
$
c. -dine

{
 
 

 
d. -sartan
d. Frequent intake of milk

^~

 

¢ %Z 



22. A 36-year-old woman who is taking bismuth





  
 
=¢  ?


subsalicylate, tetracycline, and metronidazole ex-



 
 

=! ?
$
\Z&

periences black-colored stool. What is the priority


The nurse will base the response which fact?
nursing action at this time?
a. Ranitidine only needs to be taken once a day.
a. Completing an abdominal assessment
b. Ranitidine has fewer adverse effects than ci-
b. Consulting the prescriber
metidine.

>
 
 Œ



 
$-
c. Ranitidine is less expensive than cimetidine.
fect of bismuth
d. Ranitidine is less potent than cimetidine.
d. Teaching that tetracycline can discolor fetal
teeth if the patient gets pregnant
28. The generic names of proton pump inhibitors
=||# ?
 


$ƒ
623. A patient is prescribed drug therapy including
a. -azole
bismuth subsalicylate, metronidazole, and tetracy-
b. -lol
cline. The patient reveals to the nurse that she does
c. -dine
not like taking medications all at once. The nurse
d. -sartan
should explain that it is important for the patient to
take the therapy as prescribed, because taking the
drugs alone may cause what to occur?

# 

 
$

   

# 


$
 
$$ 
c. Need for increased dose of the individual drugs.

Z
$
  

  

Copyright © 2016, Elsevier Inc. All Rights Reserved.


CHAPTER 78 Q
&
$
|
ˆ
&   193

29. Which action by the nurse would be of greatest š


†‹

‹~% %
 



 





 


 
+{#&
$
 



 
 -





  

||#ƒ 

  
  
=>?

 

a. Consult the prescriber for an order for stool for \#


 
 

+{#&
 
{
$


ova and parasite. nurse that she has stopped taking her oral con-

Z 
 
 
$
^```

$


traceptives because she has not had a period for
24 hours. 2 months and thinks she could be in menopause.
c. Place the patient in isolation. What should the nurse do?
d. Wash hands with soap and water after caring a. Administer all the medications as ordered and
for the patient. consult the prescriber regarding tests for meno-
pause.
š
†`

 
 

  
 @
=|- 
 

+{#&

  
=>-
 ?
$
\Z&
!
 

 

?


 
 

$


 
 
 
=|Z\?
 
prescriber.
inserted, and the pharmacy substituted omepra- c. Administer all the medications as ordered and
@
=¢ ?
 % 
 
  
inform the prescriber that the patient has not
because Prilosec capsules cannot be crushed. Be- been taking the oral contraceptives.
  
$
$$

| 

¢ 
d. Hold the medications and consult the prescriber
which concurrent diagnoses would be a reason for regarding a pregnancy test.

 

 

  ƒ
={

 

?
†
 
$
|ˆ&


 

>
 
 
  
=>}|&? a. half an hour before meals and bedtime.

* 
$ 
=*…? b. 1 and 3 hours after meals and at bedtime.

& 

=&€? c. as soon as symptoms occur.
d. Hyperthyroidism d. as infrequently as possible.

ˆ
 

31. The nurse is preparing to administer lansoprazole DOSE CALCULATION QUESTIONS


=| ?
#Š
  




$

$ ƒ
={

 
?
†—
{ $ 

  


  
=]
']`
‰?


#
 



 
 for oral dosing. The patient is prescribed 2 g a day

#
 



  
 ‚
 


  
*
 





#
 



œ
 

  administered at each dose?

#
 

$ 


 

#
 

 





†~
>

$ 



]``

$

š
†^

  
 



 
$

normal saline solution and is to be administered
nurse to consult the prescriber regarding adminis- 

 
!

 



$ 
$

 
$
  @
=*?ƒ ]`
 '‰
 



 


 ƒ

& 
^
 '‰

#+
]

‰&‰
]†<
 '‰

d. WBC 10,000/mm3

33. A patient who has recently been prescribed


 @
=| ?









 
 $ 
=>  $ ?
!
 ‚

response is based on what knowledge?


a. The two drugs are not necessary.
b. The sucralfate prevents the absorption of the
omeprazole.
c. The adherent coating of sucralfate requires a
gastric pH of less than 4.
d. Both drugs are metabolized by the CYP450
cytochrome system.

Copyright © 2016, Elsevier Inc. All Rights Reserved.


194 CHAPTER 78 Q
&
$
|
ˆ
&  

CASE STUDIES 6. At discharge, the patient is prescribed drug


therapy including bismuth subsalicylate, metroni-
Case Study 1  @

 
$
|ˆ&
 


* . What teaching should the nurse provide


A 47-year-old woman with a history of type 2 diabetes relating to therapy with metronidazole?

=!^&€?

 




 

 
 
=|>|?


 
$
 -
tric pain, especially at night. The pain radiates to the back
and is relieved by antacids, but returns within an hour.
The pain is worse when bending forward and better after 7. The patient asks why she is not on a special diet.
  
!
|>|
 



|ˆ&


 
 
{

 
 

 

 
 

and gastroesophageal-duodenoscopy are ordered. The foods and drink a lot of milk. How should the
patient asks the nurse how breathing into a bag can test nurse respond?
for the presence of bacteria in the stomach.

1. Describe how the nurse explains this test.

Case Study 2
A 47-year-old man with a history of T2DM and os-
2. The urea breath test results indicate the pres-  




\#
 

ence of * Œ


   %  

 
$
+{#&
$
”
 
Z 
 

duodenoscopy results reveal a duodenal ulcer. $"


 






\#
 -
 


 
$

 
$
|ˆ&ƒ tress. The patient asks how his arthritis medication can
cause stomach distress.

8. Describe the nurse’s response.

†
# 
 


 

=#Š?
$ 
!
 

 

 




$
†``

$

 
-
sessments, actions, and teaching should the nurse
‡
 
=¢  ?

*2RA, is prescribed.
perform when administering this medication? Based on the patient’s need to take medications
for his chronic conditions and his developmental
level, describe why ranitidine is a better choice
 

=! ?
$

 

4. The drug handbook states that cimetidine is the


drug of choice to prevent aspiration pneumonia.
Why is it important to do a careful respiratory as-
sessment of this patient? 10. Why is this patient at risk for accumulation of
ranitidine, and what laboratory tests should be
monitored?


  

$

 




day include severe pain and a rigid abdomen.


What should the nurse do and why?

Copyright © 2016, Elsevier Inc. All Rights Reserved.


CHAPTER 79 Q
‰   195

79
Laxatives

STUDY QUESTIONS 7. The nurse is teaching a patient about measures to


prevent and treat constipation, which is a common
Matching adverse effect of a newly prescribed drug. Which
statement made by the patient indicates a need for
   

  additional instruction?
a. “Drinking 6-8 glasses of water each day will

]

# 

$
 




keep my bowels regular.”
intestine, and reduce water and electrolyte 
’Z 
$

  


 

absorption. every day will help prevent constipation.”

^

‰
$ 
 

$  

’#

 


 




penetration of water into the feces.  


$



 

$
#




†

{

 

$

 


of times a day.”
gel, thereby softening the fecal mass. 
’‰  


 

 “
4. ___ Draw water into the intestinal lumen causing
the fecal mass to soften and swell, thereby 8. The nurse teaches a patient that which is the best
stretching the intestinal wall, which stimu- 
$
 




$ƒ
lates peristalsis. a. Dietary bran
b. Methylcellulose
a. Bulk-forming c. Psyllium
b. Osmotic 
Š  
 

{ 

{$   9. A nursing student is sharing research on laxatives.
Which statement by the student would indicate a
need for further study?
CRITICAL THINKING, PRIORITIZATION,
a. “Bulk-forming agents can form a mass in the
AND DELEGATION QUESTIONS esophagus if the patient does not drink enough
water when taking the drug.”
5. What is a reason to use castor oil? b. “Cathartics are useful as bowel preparations for
a. Avoid straining with defecation colon procedures.”

> 
$
 
$
 

 c. “Patients should be warned that stimulant laxa-
c. Prevent constipation due to chronic opioid use tives can cause electrolyte imbalances.”
d. Prepare for a colonoscopy 
’{ 
  





-
vent constipation associated with pregnancy.”
6 6. When clarifying a patient’s complaint of constipa-
tion, what priority information does the nurse need
to obtain?
a. Amount of stool
b. Color of stool
c. Consistency of stool
d. Frequency of stool

Copyright © 2016, Elsevier Inc. All Rights Reserved.


196 CHAPTER 79 Q
‰  

610. The nurse is caring for a 45-year-old woman who š


]

 



 


 

is receiving antibiotic therapy. The patient takes a Milk of Magnesia, prescribed as needed for consti-
 
  


 
#





pation, and to consult with the prescriber?


 



  ƒ 
–ˆ+
]`
 '‰
a. Bubbling sounds throughout the abdomen and 
\…
†`
‰'
dull sound with percussion of the left lower 
…–{
=…–\?
]†
 '‰
"  
=‰‰˜? d. Na+]‹—
Z"'‰

\ 


 


and one soft, liquid stool this morning 16. A patient who has a history of type 2 diabetes mel-
c. High-pitched tinkling bowel sounds in the right 
=!^&€?

 

 
 -

"  
=‰˜?




lin, hydrochlorothiazide, and valsartan is having
in other quadrants bowel preparation for a colonoscopy. The patient

{$


 


asks why the prescriber has ordered polyethylene
occurring every 1-2 minutes in all quadrants on %

=\‰™!Z‰™?
  

awakening $

…
| %{
= 
  ?

that she took in past. Which statement would not


š
]]
 


 
 
 

 
be included in the nurse’s response?
should provide regarding bulk-forming laxatives? 
’\‰™!Z‰™


  

 -
a. They should never be taken more than once a ances.”
day. 
’{
  
 
 

 “
b. They should be taken with 8 oz of water or c. “You are at greater risk for kidney damage due
juice. to your medical history and drug therapy.”

!
 
 




$$


’!
  
=\‰™!Z‰™?



peristalsis. because you do not have to drink as much


d. They are contraindicated if the patient has ir- liquid.”
ritable bowel syndrome.
š
]~

 
  
 
 
$


12. Which directive should be included in instructions constipation. Which would be the best initial inter-
for administration of bisacodyl tablets? vention?
a. “You will have the best response if you take the 
–%$
  
 

 
 

laxative with a meal.” 





b. “You should chew the bisacodyl tablets to b. Moderate exercise after meals and increase
achieve the maximum effect.”  




c. “You should not take antacids within 1 hour of 
{
$

 
 




taking the laxative.” diet


d. “You should take the laxative with a full glass 
{
$

 
 
$

of milk.” meals

š
]†
#

 
$

 



 
18. Which would be a reason to withhold administra-
who regularly uses Milk of Magnesia as a laxative 
$
  
=@ ?

 


for signs of hypermagnesemia. What is a sign of prescriber?


hypermagnesemia? a. The patient takes multiple drugs that are me-
a. Hypertension and rapid, bounding pulse tabolized by the P450 metabolizing enzymes.

|   


  

 b. The patient experiences mild nausea after tak-
c. Tremors, twitching, and hyperactive deep ten- ing the drug.


=&!? c. The patient is older than 60 years.
d. Weakness, diminished bowel sounds, and bra- d. The patient is taking the drug for constipation
dycardia associated with chronic opiate use.

6]‹
!
 

 
  
†`



619. A patient has been using mineral oil daily as a
patient who has hepatic encephalopathy. Which of laxative. Because excessive use can interfere with
these outcomes is a priority as the nurse plans care vitamin K absorption, it would be a priority to
with this patient? report which symptom?


]]`
 '‰ a. Bruising

‰!
†
  
 '‰ b. Fatigue
c. One soft, formed stool within 24 hours c. Pallor
d. Relief of constipation 
{
 

Copyright © 2016, Elsevier Inc. All Rights Reserved.


CHAPTER 80 Q
}
\   
& 197

20. Which laxative is best for reestablishing normal 1. What additional information does the nurse need
bowel functioning when discontinuing chronic to know about this patient before she can address
stimulant laxative use? the problem of laxative overuse?
a. Castor oil

\
 

‰  
d. Mineral oil
Further data collection reveals that this patient describes
her daily bowel movement as light brown, mushy, and
DOSE CALCULATION QUESTIONS with some watery discharge.

21. Milk of Magnesia, 1 oz, is prescribed. How many 2. What lifestyle changes are appropriate to help



 
 ƒ establish an acceptable bowel pattern for this
patient?

^^
‰  
```<



 

  

for irritable bowel syndrome with constipation.


Available are 8-mcg soft gelatin capsules. How
many capsules should the nurse administer at each
dose? 3. What problems does this patient’s medical history
present when trying to address normalizing bowel
patterns and laxative use for this patient?
CASE STUDY
During a routine physical examination, the nurse discov-
ers that a 78-year-old woman who lives alone uses several
%%
=}!>?
  

 

 

 


{


 
$
 
4. What laxatives are contraindicated for this patient?
and heart failure.

80
Other Gastrointestinal Drugs

STUDY QUESTIONS
‹
{  
 
 
  


 

via the ________________ nerve.


Completion
5. Drugs for chemotherapy-induced nausea and vom-
1. The vomiting center is a group of neurons located 
=>#+Š?

 
$$
$
 

in the ________________ _________________. ________________ ________________.

^
Š

=?


—
}  
=¢$ ?





by blocking ________________ receptors.


3. Chemotherapy drugs cause vomiting by di-
rectly stimulating the ________________
________________ ________________.

Copyright © 2016, Elsevier Inc. All Rights Reserved.


198 CHAPTER 80 Q
}
\   
&

~
!
$$
$
  
=¢$ ?
614. What action would be of greatest priority if a pa-
is increased by also administering tient with postoperative vomiting reports burning
__________________. 


 @
=| ?
$ 
ƒ
a. Apply ice.
8. First-line therapy for nausea and vomit- b. Contact the prescriber.
ing of pregnancy is ________________ plus 
| 


 
________________. 
{


$ 

‡
{

  




15. What is an appropriate nursing focus when a pa-
uncomplicated diarrhea can ________________ 

  
 
=* ?
$
  

traveler’s diarrhea. and vomiting after knee replacement surgery?


a. Anxiety
10. ________________ ________________ b. Body image
________________ is the most common disorder 
{ $
$

\#
  
{ 

6]—
#


$
  






CRITICAL THINKING, PRIORITIZATION,   







 


AND DELEGATION QUESTIONS   


 
=€ ?
$
>#+Šƒ
a. Need to take two naps per day
6]]
#


$
  




b. Blood pressure of 110/72 mm Hg
laboratory test results when a patient is experienc- c. Pulse increase from 82 to 122 beats/min
ing the adverse effect of diarrhea when prescribed d. Two-pound weight gain in past month
  
=¢$ ?
$
 
 ƒ

‰!

  17. Why is it recommended that a patient with nausea

–ˆ+

 and vomiting of pregnancy take doxylamine at
c. CBC and differential bedtime?

#+

|! 
#


$$


  

{ 



$$
12. What is the primary reason why aprepitant 
{
$
  

  

 

 
=Z?

$
 



$
d. There is a longer time between doses.
>#+Šƒ

#


 
$$ 18. Which planned activity would be of most con-

#

$
 
$$



cern if a patient has been prescribed scopolamine
other drugs. =!  %{µ?ƒ

#


$$
 

 

- 
! 

 
 

tonin receptors. b. Taking a cross-country rail trip

#

$$

$

  

- c. Driving a delivery truck
ing. 
! 

”



Z

]†

  
 

  

- 19. Which symptoms would be of greatest concern
perazine as needed for nausea and vomiting. After when a patient is experiencing prolonged diar-
being transferred from the bed to a chair with the rhea?
assistance of three people, the patient vomits and 
–ˆ+
^^
 '‰
requests the medication. What should the nurse b. Hct 35%
ƒ
={

 
? 

††
Z"'‰
a. Administer the medication while the patient is 
+
]†
Z"'‰
sitting in the chair.
b. Assess vital signs before and after administra-
^`
 


 
 

‰


tion. antidiarrheal drug, contains atropine in addition to


c. Hold the medication if the patient is hyperten- diphenoxylate?
sive. 
#
 
$$

 
$

 

# 
$
 
$
 

#
 @

 
$$ 
administration. 
#
 



e. Transfer the patient back to the bed before 
#

  
administering the medication.

Copyright © 2016, Elsevier Inc. All Rights Reserved.


CHAPTER 80 Q
}
\   
& 199

š
^]



 

$

 

š
^~

  
 
$

†~% %
$ 

who has had two loose stools in the past 12 hours? patient suggests possible adverse effects of sul-
a. Difenoxin $  @
=@?ƒ

‰  
*  
]`
'‰
c. Paregoric b. MCV 90/mm3
d. Watchful waiting c. Neutrophils 75%
d. WBC 10,500/mm3
š
^^



  


 -
ent to immediate medical care when his or her 6 28. A 43-year-old female patient who has been taking
]`% %

 


 

š

 @
=*&#ˆ#‰?
$
-
treated with acetaminophen and bismuth sub-  

  @
=&?
$
 

 
=|%– ?ƒ colitis is admitted to the hospital. What would be
a. Two loose stools in the last 24 hours the greatest nursing priority if nursing assessment
b. Black tongue and black stool 


 
 


c. Fatigue and poor appetite paresthesias?


d. Vomiting and confusion a. Anxiety
b. Breathing pattern

^†

 

  
 
=‰?
]


…


once a day. How long after administration of the 
 

drug should the patient expect relief from symp-
toms? 29. A patient has received acute relief from dexameth-
a. 30-40 minutes asone for an exacerbation of Crohn’s disease at
b. 1-2 hours the ascending colon. He is concerned because the
c. 3-4 days prescriber is discontinuing this drug and prescrib-
d. 1-4 weeks 
 
=Z
Z>?
*



 
 ƒ
={

 
?
 ^‹
#

 
$

 

 

$ 

’{ 
$$


 

 -
 


 
 
=‰?


methasone.”
taking the medication and immediately report b. “Budesonide is released in the area of the colon
symptoms if she experiences where it needs to work.”
a. abdominal pain and dyspepsia. c. “The drugs are the same, but budesonide is less
b. abdominal pain relieved by defecation. expensive.”
c. constipation or bloody diarrhea. d. “Tolerance develops to long-term use of dexa-
d. fever or headache. methasone.”

6^
#

$
  


 

š
†`

  
 
 


$
 
-
the presence or absence of chest pain and dyspnea cern to the nurse when caring for a patient who is
before administering which drug? receiving cyclosporine for severe Crohn’s disease?

 
=‰? 
–ˆ+
^^
 '‰

‰  
=@ ? 
> 
†—
 '‰

{$  @
=@? 
Z{
^^
'

! 
=¢? 
…–\
]^^
 '‰

^—

 

  
$  @
=@?
6†]
#


$
  

$

 



for moderate ulcerative colitis. The nurse would   



= ?
$
>‚
 -
consult the prescriber if which allergy was listed ease to report which symptom for over 6 weeks?


 ‚
 ƒ
={

 
? a. Abdominal pain

>$ @
=$? b. Blood in the stool

\@
=\? c. Fatigue

€
=€? d. Productive cough

+ $
=ˆ?

˜ 
=?
$
!% $  @
=– ?

Copyright © 2016, Elsevier Inc. All Rights Reserved.


200 CHAPTER 80 Q
}
\   
&

š
†^




 
$

 

- CASE STUDIES

@
=>@ ?
  
$


ulcerative colitis and consult the prescriber? Case Study 1



–
 


$ 

”
 
b. Headache and photophobia A patient who has a history of motion sickness receives a
c. Pruritic eruption on the ankles  
$

 
 
$
 




{



  cross-country and embarking on a 7-day cruise. The pa-


 
"  


  


$

33. The nurse would consult the prescriber if a patient nurse to explain the use of this medication.
with Crohn’s disease reported new paresthesias
after prolonged use of which drug? 1. How should the nurse describe the medication’s

– 
=Z
Z>? action and administration?

> 
=>?

#
= ?

€ @
=… ?

634. A patient with diabetes has been experiencing 2. Research the drug in a drug handbook and list
episodes of abdominal pain, nausea, and vomiting precautions that the nurse should teach the patient
of undigested food, especially at night. Metoclo- to take.
 
=  ?
]`

 
$


 

is prescribed. Because the drug promotes gastric


motility, the nurse would expect to administer the
doses within which time frame?
a. After meals and at bedtime 3. What interventions can the nurse suggest to pre-
b. Before meals and at bedtime vent adverse effects?

Z
—

d. With meals and at bedtime.

š
†




 
$

 



 
 $
= ?ƒ Case Study 2
a. Absence of nausea and vomiting after chemo- 
 % 
 


  

  %
therapy 
 


=#–{?


 


‰!'{!

 
 health center.
c. Comfortable consumption average of 70% of
meals and snacks each day 4. What nonpharmacologic measures can the nurse
d. Weight gain of 1 lb each week teach to assist this patient with controlling his
#–{ƒ
š
†—
!

 
 
$



 

   






 -
lipase?
a. With lunch
b. One half-hour before lunch

 
=‰?

  
!
 

c. One hour before lunch should stop taking the medication and return to the
d. Two hours after breakfast and lunch health center if he experiences what symptoms?

DOSE CALCULATION QUESTIONS

†~
& 
=€ ?



  
^


before the start of chemotherapy for a patient who



‹‚

 

 
~`
 
– 

–{


the dose safe and effective?

38. What is a safe and effective maintenance dose


$

= ?
$

]——%
 


ulcerative colitis?

Copyright © 2016, Elsevier Inc. All Rights Reserved.


CHAPTER 81 Q Vitamins 201

81
Vitamins

STUDY QUESTIONS 8. Routine vitamin supplementation is recommended




 = ?ƒ
={

 
?
Matching 
 %
= 
Z?


 

heart disease and cancer



      
  

= 
>?




–
 
= 
?
$


-

]

 
 
 
 
$

tect from lung cancer
meet the nutrient requirements of nearly all 
>   
= 
–12?


-
healthy individuals mia in people older than 50 years
2. ___ An estimate of the average daily intake e. Folic acid for women before and during preg-
required to meet nutritional needs nancy
3. ___ Five reference values on dietary vitamin
intake 6 ‡
#




$

 




4. ___ The highest average daily intake that can laboratory test result when a patient is taking high
be consumed by nearly everyone without a doses of vitamin A for acne?
  
 
$
 
$$ 
‰!
^`
#ˆ'‰
5. ___ The level of intake that will meet nutrition 
–ˆ+
^`
 '‰
requirements for 50% of the healthy indi- 
>\
`—
#ˆ'‰
viduals in any life-stage or gender group 
#+
]


" 
 
=#? 10. A patient with rough, scaling skin and a sore

& 
$
 
=&# ?  
 


 

!


Z  
 
"
=Z? nurse teaches that which foods are high in niacin?


 
 
=&? 
Z
 
 

 $
  -

! 

 
=ˆ#? bles, and legumes
b. Chicken, peanuts, and cereal bran

& 


$
 

 
CRITICAL THINKING, PRIORITIZATION,
d. Pork and enriched breads and cereals
AND DELEGATION QUESTIONS
 11. The preoperative patient’s medication history
6. The nurse knows that which statements are true includes hydrochlorothiazide 25 mg once a day,
about vitamins? calcium 400 mg 4 times a day, a senior multivita-
a. They are inorganic compounds. 


 
  
=˜  ?
‹

‹

b. They are needed for energy transformation and 



 

 
Z
]```



 

regulation of metabolic processes. !


 





$

c. They are required in megadoses for growth and the surgeon of the medication history because the
maintenance of health. patient is at risk for what issue?
d. They are sources of energy. 
Z 


 
b. Hypotension during surgery
7. The nurse knows that which statement is true c. Poor wound healing
about published RDAs? d. Vomiting during surgery
a. They do not consider increased needs during
illness.
b. They include values for older adults.
c. They may be excessive for a chronically ill
person.
d. They need to be ingested every day.

Copyright © 2016, Elsevier Inc. All Rights Reserved.


202 CHAPTER 81 Q Vitamins

 12. The nurse would teach a patient who follows a 618. The nurse is providing nutritional teaching to a pa-
vegan diet the importance of supplementation with 

 
   
 
#

 

which vitamin? important for the nurse to assess if the patient is


a. Alpha-tocopherol self-medicating with which vitamin that has been
b. Ascorbic acid falsely promoted as a treatment for cancer and can
c. Cyanocobalamin   

\#
 ƒ
d. Folic acid a. Vitamin A
b. Vitamin B
6]†
#





 

 

c. Vitamin C
the possible adverse effects of self-prescribing 
Š 
Z
megadoses of vitamin A for healthy skin?
a. Adolescent female 19. Which characteristic of the older adult could cause
b. Adolescent male
 
&
ƒ
c. Older adult female a. Older adults buy less fresh fruit because it is
d. Middle-aged male expensive.
b. Older adults consume fewer dairy products.
 14. The nurse should be aware that which disorder c. Older adults do not eat fresh vegetables be-
puts the patient at greatest risk for bleeding and cause they cause indigestion.
 
  
$
 

ƒ 
}

 
$

 
a. Addison’s disease
b. Celiac disease  20. A teacher sends a Hispanic migrant worker’s

\    

  
=\Z&? 



 ‚
$
  




|

  
=|ˆ&? concerned that the child has a smooth, swollen
tongue and cracks in the corners of her mouth and
615. A patient tells the nurse that he takes a multivita- lips. The nurse knows that this may be a vitamin
min containing vitamin K. The nurse reports this 
  
  



 
 

information to the patient’s prescriber because the component is


patient has been prescribed which drug? a. home-ground corn.
a. Digoxin b. commercial cereals.
b. Furosemide c. dairy products.
c. Heparin d. freshly picked vegetables.
d. Warfarin

16. The nurse includes in her health promotion teach- DOSE CALCULATION QUESTIONS
ing that vitamin C supplementation has been ap-
proved for which use? 21. Niacinamide is available in 100-mg tablets. The
a. Decreased bronchoconstriction of asthma medication administration record indicates the
b. Prevention of colds dose is 150 mg. How many tablets should the
c. Promotion of wound healing nurse administer?
d. Treatment of scurvy

 17. An oncology nurse is providing health teaching 22. Vitamin K1


`




$

 

to a patient who is starting chemotherapy. The Available is vitamin K1


^
 '‰
>  


patient has heard that chemotherapy can cause dose.


painful mouth ulcers. The patient would like to
know if there is anything that she can do to help
prevent these ulcers. The nurse could teach about
adequate consumption of which foods that are rich
in vitamin A ?
a. Citrus fruits, strawberries, and red peppers
b. Deep yellow, orange, and green vegetables and
fruits
c. Nuts, nut oils, and vegetable oils
d. Pork and enriched breads and cereals

Copyright © 2016, Elsevier Inc. All Rights Reserved.


CHAPTER 81 Q Vitamins 203

CASE STUDIES 2. Why is the patient less likely to be showing symp-



$
$ %  
 
 ƒ
Case Study 1
A 46-year-old man has been admitted to a medical unit
$
‹
 



 

*


admitted because of complaints of extreme weakness 3. Blood studies include hemoglobin 8.4 g and he-
and an unsteady gait. His wife provided a history that his matocrit 25%. Which vitamins are essential in red
alcohol intake has steadily increased since he lost his job blood cell production?
3 years ago. During the past 6 months, he has been living
on the street and drinking 1-2 quarts of wine daily. He
is a poor historian and does not recall being admitted to
the hospital. His response when asked about his diet is,
’ 
#
 
“
|  
 
 
  
4. The patient’s wife has agreed that he may come
edema of the lower extremities, nystagmus, dry skin with home with her after his discharge from the hospi-
cracks in the corners of his mouth, and multiple bruises. tal as long as he continues to stay in an outpatient
He states that the bruises are caused by any slight pres-   
  




{


*


 

$


 

asks the nurse to tell her some of the foods she
a few bites of his lunch. He complains that his mouth is should prepare to be certain that he gets the neces-
’
“

 
 
$ 
Z 
 

sary vitamins. What foods or food groups would
gastritis with no obvious bleeding. The patient is diag- you suggest to her to ensure intake of vitamin A,
 

 


 




 
>
 

  
 

with one ampule of vitamins C and B complex per liter. and folate?

1. Describe the symptoms exhibited that suggest



$

$
  
a. Thiamine
5. The patient’s wife asks the nurse whether it would
be a good idea to go to the health-food store and
buy him some high-dose vitamin pills that include
b. Niacin and pyridoxine all vitamins. What is the best response?


  
Case Study 2
A high-school nurse has developed a rapport with a fe-
male student who has acne. The student states that she has
d. Ascorbic acid been seeing a dermatologist without success. At the last
appointment, the doctor stated that she would be consid-
ering isotretinoin, a megadose form of vitamin A, as the
next step in therapy.
e. Cyanocobalamin and folic acid
6. What precautions and instruction should the nurse
provide?

Copyright © 2016, Elsevier Inc. All Rights Reserved.


204 CHAPTER 82 Q
&
$
 
‰

82
Drugs for Weight Loss

STUDY QUESTIONS  6. What is the most important factor when the nurse
is devising a weight management plan for a pa-
Matching tient?
a. Developing strategies to help the patient mini-
         mize stress

#
 
 

  



]

{ 


 

 - patient’s plan
ability of norepinephrine at receptors in the c. Advising the patient to limit overly processed
brain foods in the diet

^

{ 


 

 
$

{

 ‚


 


satiety revising the plan

†




\#
 



$

fat 7. Which statement, if made by a patient who has


been prescribed a drug for weight loss, suggests
a. Orlistat that the patient needs additional teaching?

‰  
’#$
#
$


 


c. Phentermine  


 


#





 


 



 
“

’#$
 

$

#
 


 


CRITICAL THINKING, PRIORITIZATION, long-term to maintain my weight loss.”


AND DELEGATION QUESTIONS 
’€


 

 
  
#

have followed the recommended diet and exer-


 4. The nurse is teaching health promotion to ado- cise prescribed for the past 7 months and lost
lescent girls. Developmentally, which health risk only 4% of my starting body weight.”
would most likely be a motivator for change to d. “My doctor is starting drug therapy for me
healthier eating and exercise habits?   
#


 

 “
a. Cardiovascular disease
b. Decreased fertility 6 8. Which laboratory test result would be a priority to
c. Diabetes mellitus report to the prescriber of warfarin if a patient has
d. Hypertension $%  
  
=?ƒ

‰!
†`
#ˆ'‰
5. Research suggests that which change has not been 
–ˆ+
^^
 '‰
shown to occur when patients lose weight? 
#+
^
a. Decrease in high-density lipoproteins 
| 
^
Z"'‰
b. Decrease in low-density lipoproteins
c. Decrease in level of hypertension  9. Research on drugs for weight loss suggests that
d. Decrease in hemoglobin A1c in patients with some patients will lose weight even when receiv-
type 2 diabetes mellitus ing a placebo.
a. True
b. False

10. Which result would be of most concern if a patient


is prescribed phentermine?
a. BP 180/98 mm Hg
b. P 52 beats/min
c. R 24/minute

!
]`^^„
…
=†‡„
>?

Copyright © 2016, Elsevier Inc. All Rights Reserved.


CHAPTER 82 Q
&
$
 
‰ 205

]]

 


  

={- 2. The patient is prescribed orlistat 120 mg three
?
$
 

$%  
times a day with meals. What strategies can the
  
=?
$
 
 

 
nurse suggest to minimize these adverse effects,


$
  

$

 ƒ
={- which can be serious?


 
?

\#
$$

#

 

 
 
$ 
 -
tion.

#

 

 

$
 

B and C.

‰
  

! 

  
=?
$
 $ 


and dinner.

! 

  
{
†`

-
fore breakfast.
c. Reduced absorption of vitamin D

! 
 



 
 


\#
$$ 

DOSE CALCULATION QUESTIONS d. Reduced absorption of vitamin K

]^
| 
  
= ?
 

" 



 
  
$
}!>
  
=?ƒ

3. The patient is concerned about the adverse effects


13. Phentermine 18.75 mg is prescribed twice a day. of orlistat and asks if there are ways to improve
Available are 37.5-mg tablets. How many tablets his health without taking diet drugs. How should
should be administered at each dose? the nurse respond?

CASE STUDY
A 48-year-old man with a history of sleep apnea, hyper-
tension, and type 2 diabetes weighs 285 lb and is 5’ 8”
tall. His waist circumference is 44”. His total cholesterol
is 330 mg.

1. The patient asks the nurse what is considered a


realistic goal for weight loss, and how much he
needs to cut back in his eating. How should the
nurse respond?

Copyright © 2016, Elsevier Inc. All Rights Reserved.


206 CHAPTER 83 Q Basic Principles of Antimicrobial Therapy

83
Basic Principles of Antimicrobial Therapy

STUDY QUESTIONS
]—

#$ 


 

%
%
 



 


>+{

True or False more readily than in a child or adult.



]~

 
 
$
 


   


  T   F abscess.
   18. ___ The American Heart Association has
recently stressed that prophylactic use of
1. ___ Mammalian cells do not have a rigid cell antibiotics is more important than previously
wall. thought.
2. ___ Patients should not take folic acid supple- 19. ___ Antibiotics that are added to animal feed
ments when prescribed sulfonamide drugs. can cause humans to develop a resistance to
3. ___ Cephalosporins kill bacteria by weakening those antibiotics.
the cell wall and promoting bacterial lysis.
4. ___ Aminoglycosides inhibit folic acid produc-
tion by the microbe. CRITICAL THINKING, PRIORITIZATION,
5. ___ Metronidazole inhibits DNA synthesis in AND DELEGATION QUESTIONS
certain microbes.

—

€ 

 

  
$
*#Š
š
^`

 




 


-
infection inhibit enzymes needed for viral  
 
=#Š?
'   

reproduction. =ˆ ?


$




$



~

€{
ŠZ

/   are a problem  
=>²{?
$

 
 -
because infection with these microbes is 
=Z&?!
 



 
$

usually fatal. notifying the prescriber of culture results as soon


8. ___ When patients take antibiotics as prescribed, as they are available is that ampicillin/sulbactam
resistance to antibiotics does not occur. =ˆ ?
9. ___ Bacteria can become resistant to antibiotics a. has many more adverse effects than most other
by producing enzymes that inactivate the antibiotics.
antibiotic. b. is a broad-spectrum antibiotic, and there may

]`

#$


  

 

- be an effective narrow-spectrum antibiotic.
biotic may not be able to bind to the microbe c. is a very expensive antibiotic.
and exert effects. d. is a narrow-spectrum antibiotic and may not be
11. ___ The human body can make compounds effective for the cultured organism.
that prevent an antibiotic from exerting the
desired effect.  21. Which patient would most likely have an infection

]^

{
 
 
  

 


that is resistant to antibiotic therapy?
taking antibiotics into the cell. a. A child with asthma who develops pneumonia

]†

!
+
&
€ %– %‰   
b. An adult construction worker who drinks from
]
=+&€%]?





$
a worksite water supply and develops giardiasis
# 

|   
 c. An adult who developed a wound infection
14. ___ Broad-spectrum antibiotics kill more com- while in the hospital after surgery
peting organisms than do narrow-spectrum d. An older adult who got an infected paper cut
drugs and do the most to facilitate emer-
gence of resistance.
15. ___ Narrow-spectrum antibiotics tend to pro-

 
$
 

 
 -
sess mechanisms for resistance.

Copyright © 2016, Elsevier Inc. All Rights Reserved.


CHAPTER 83 Q Basic Principles of Antimicrobial Therapy 207

^^

$


 


 - 28. Which are valid reasons for prescribing two differ-
fection? 
  ƒ
={

 
?
a. Monilial vaginal infection that develops during a. Cases of infection with  
 -
antibiotic therapy 
b. Peritonitis that develops after surgery for a 
 
$

>²{
 




ruptured appendix effective


c. Pneumonia in a patient with chronic bronchitis 
 
$
>²{
 

  

d. Varicella outbreak after injection with varicella sensitive to different drugs are present
vaccine 
{
$


 

patient
23. Microscopic examination of gram-stained prepa- e. Foreign material is present on the sample speci-
 


  = ?
$
={

 
men
?
a. detecting microbes when only a minute amount 29. Which is an acceptable reason for giving antibiotic
are present. prophylaxis?

–

 


 
- a. Before cardiac surgery
 
 
 
=|>? b. Before examination of the eyes where dilating
c. Providing rapid results. drops are instilled
d. Being a more simple test. c. When patient experiences yellowish or yellow-
green nasal discharge
 24. The nurse has consulted the prescriber because a d. Whenever the patient has a fever
patient reports an allergy to the prescribed penicil-
lin antibiotic. The prescriber is aware of the allergy,
but the patient is experiencing a life-threatening CASE STUDIES
infection and no other suitable antibiotic is avail-
able. What is the priority nursing action? Case Study 1
a. Administer the antibiotic. A 78-year-old nursing home patient with a history of
b. Ask the patient if he or she is willing to take the hypertension, type 2 diabetes mellitus, and chronic ob-
antibiotic. 
 
  
=>}|&?

 



c. Obtain orders for treatment of a possible al- Z&




 
$
$
$
~^


 
 

lergic reaction. The extended care nursing report states that he was very
d. Refuse to administer the antibiotic.  
 
 



$

 


 
  


 
]`†„

6^
#







 
 
$
…
=†‡‹„
>?
 
]]^
  '
–|
]``'—

* 

a patient is prescribed tetracycline?   




 

 
  
 


\—|&
<^
ˆ'
$
   =  ?
 


 
\ 

 
]^


>\
†^
#ˆ'‰ > 
% 
  

  
# 


#+
]] 

$  
}







{
]†^
Z"'‰ =>²{?

 26. What test results should the nurse monitor when 1. What are nursing responsibilities regarding these
caring for a patient who is at risk for a glucose- orders?
—%  
  




prescribed a sulfonamide antibiotic?



{!

‰!

–ˆ+

 

\ 

] 2. What type of antibiotic would the nurse expect to
d. Hemoglobin and hematocrit be ordered and why?

^~
#
 
$ 


$
 




of infection needs to be
a. at the minimum inhibitory concentration
=€#>?

^%†


€#>

‹%<


€#>

]`%^`


€#>

Copyright © 2016, Elsevier Inc. All Rights Reserved.


208 CHAPTER 83 Q Basic Principles of Antimicrobial Therapy

†
!

 
 



$ 
Case Study 2
=>$ ?
]


]^
 
!

 



$
=>²{?
  
 

A neighbor asks a student nurse why her doctor will not
should the nurse take? phone in prescriptions when she has a “sinus infection”
anymore.

Organism: Moraxella catarrhalis 6. How can the nurse explain these changes in antibi-
Antibiotic Sensitivity otic prescription practices?

Amikacin R
Amoxicillin R
Azithromycin { 7. The neighbor states that she has antibiotics left

$

 
$
’#
 



Cefepime R 
 
#

“

 
{


 

Cefotetan R if it is okay to take the leftover drug since it was


prescribed for her. How should the student nurse
Clarithromycin R respond?
\   {
‰  {
Piperacillin R
8. The neighbor asks what she can do to prevent
Tobramycin R resistance to antibiotics. What suggestions should
the student nurse make?
{

 Œ


   

4. The cultured organism is sensitive to more than



 
#

 
 

$ 


considered when choosing among the effective 9. Where can the student nurse direct people for
antibiotics? more information on preventing antibiotic resis-
tance?

5. What assessment should be monitored by the


nurse to determine clinical response to the antibi-
otic?

Copyright © 2016, Elsevier Inc. All Rights Reserved.


CHAPTER 84 Q
&
! 
 

–  
>
 

| 209

84
Drugs That Weaken the Bacterial Cell Wall I: Penicillins

STUDY QUESTIONS 15. Parents ask why their son has been prescribed


 
   
= ?

True or False $

=?


$$



past. The nurse’s response is based on the fact that


   
  T   F the addition of calcium clavulanate
   a. aids the penicillin in attaching to microbial
1. ___ All penicillins are able to penetrate the penicillin-binding proteins.
gram-negative cell membrane. b. affects a wider spectrum of bacteria.
2. ___ Penicillins are able to destroy many bacteria c. prevents penicillinase from inactivating the
when taken as prescribed. amoxicillin.
3. ___ Penicillins are in the same antibiotic family d. provides additional activity to disrupt the bacte-
as macrolide antibiotics. rial cell wall.
4. ___ Penicillins are only active against bacteria
that are undergoing growth and division. 16. Which organism was previously susceptible to
5. ___ Penicillins are more effective against gram- penicillin but has developed resistance to penicil-
negative than gram-positive bacteria. 

6. ___ Penicillins are toxic to human tissue when 
\
 
  

/   
  -
prescribed in high doses. 
7. ___ Penicillins can cause diarrhea by altering the 
\
  

&   
 
\#
  c. Meningitis caused by )  -
8. ___ Penicillins must bind to special proteins on
 
the outer surface of cytoplasmic membrane 
{
  

. 
 

to be effective.
9. ___ People can drastically reduce their chance 17. A child is prescribed amoxicillin. When asked
of catching methicillin-resistant )- if their son is allergic to penicillin, the child’s
  
=€{?


 
parents state that the child has never received
measures. any medication except immunizations. Why is it

]`

€{
 

 

 % important to assess for an allergic response despite
resistant penicillin such as nafcillin. this history?

]]

€ 

 
€{


 

a. Most people who experience a penicillin al-
do not know it.  


 




]^

€ 
$

€{

$
b. Most people who are allergic to penicillin do
sports equipment. not know of the allergy.

]†

!

 
  
€{


c. Parents often are poor historians.
found on the skin. d. People can have an initial exposure to penicillin
present in foods.

CRITICAL THINKING, PRIORITIZATION, 6]<



 







AND DELEGATION QUESTIONS report to the prescriber if it occurred after adminis-


 
$

  

 
$
ƒ
614. Which assessment is of greatest priority for the 
& $



nurse to complete before administering a penicillin 
…
 
 
]``‹„
…
=†<„
>?
antibiotic? c. Wheezing
a. Allergy history 
{

–ˆ+
c. Temperature
d. Wound drainage

Copyright © 2016, Elsevier Inc. All Rights Reserved.


210 CHAPTER 84 Q
&
! 
 

–  
>
 

|

š
]‡
!
 

 

 
<
AM
=`<``?
624. The nurse reviews current laboratory test re-
medications to a patient who is to receive nafcil- sults before administering ticarcillin/clavulanate

^


 
 
=#Š?
$ 
=!?
#








!


 

]``

$
 

laboratory result to the prescriber?
solution. The drug handbook states that the drug 
–+|
—
   '‰
should be infused over 30-90 minutes. Just before 
–ˆ+
^^
 '‰
the nurse hangs the nafcillin, the nurse is informed 
*
<
'‰
the patient is to be placed on a cart to go off the d. WBC 15,000/mm3
unit for a diagnostic test in 30-45 minutes. The pa-





$$


$
†`
 
 25. The nurse would be concerned about toxicity if a
What should the nurse do? patient receiving penicillin had which laboratory
a. Hold the drug infusion until the patient returns result?
from the test. 
‰!
^
  
 '‰

#$ 



†`

$
 

–ˆ+
^`
 '‰
the patient on the cart. 
> 
^—
 '‰

#$ 



‹


 


| 
††
Z"'‰
patient on the cart.

{

$ 



‡`
 
DOSE CALCULATION QUESTIONS
20. What does the nurse do when assisting with skin
testing for penicillin allergy with the minor deter- 26. Amoxicillin 125 mg every 6 hours for 10 days is
 

=€&€?ƒ prescribed for a child with acute otitis media who

# 

 



$

 
weighs 16.5 lb. The recommended dose is 20-40
reaction during the following week.  ' ' 
#

  
 
$ƒ
b. Be aware that the test carries little risk of a
systemic reaction.
c. Make respiratory support and epinephrine
^~

]
'   
`

=ˆ ?

-
available. 

`

$



$ 


d. Be aware that the test involves injection of a  


!
 

  




small amount of penicillin under the skin. 




 


ƒ

21. The nurse would be concerned about the increased


possibility of an allergic reaction when adminis-


 = ?
$

 ‚
 
CASE STUDIES



 ƒ
={

 

? Case Study 1



'   
=ˆ ? An 8-year-old is diagnosed with streptococcal pharyngitis.

@
=¢ ? {

  

^`

†


 
$
]`


> 
=>? days.

\  
=!"?

| ' @  
=¢ ? 1. What important information should the nurse
$
Š 
=Š ? obtain before the penicillin is administered?

^^
 



$

$
€{ƒ
a. Dicloxacillin
b. Nafcillin
c. Oxacillin 2. What information about the possible side effects
d. Vancomycin and adverse reactions of penicillin does the nurse
need to provide to the patient’s parents?
 ^†

$
 
 
$



 




 '   
=!?

should the nurse report to the prescriber immedi-


ately?

>  

^

b. Respirations 16 per minute
c. Temperature 101° F
d. Weight gain of 2 lb in 24 hours

Copyright © 2016, Elsevier Inc. All Rights Reserved.


CHAPTER 85 Q
&
 
 

–  
>
 
## 211

3. Describe the types of allergic reactions that might Case Study 2


develop with the administration of penicillin and
the interventions that should be included in nurs- The nurse is working in a public health clinic. A 15-year-
ing care to prevent complications from an allergic old with no known allergies is diagnosed with syphilis
reaction. and prescribed 1 dose of 2 million units of procaine peni-

\^
   
=#€?

7. Developmentally, why is this drug a good choice


for this patient?
4. The patient’s parents ask how and when amoxi-
cillin should be given. How should the nurse
respond?
8. The patient asks why he cannot get this medica-
tion in a pill. How should the nurse respond?

5. Why is it critical that the nurse teach the patient’s


parents not to stop the medication even if the
child’s throat stops hurting in 4-5 days? 9. Describe the technique the nurse should use to
 


”

 

to prevent administration complications.

6. What outcome would indicate that the antimicro-


bial effects were successful? 10. The patient has denied an allergy to penicillin.
Why does the nurse need to be cautious about pos-
sible allergy?

11. What actions/policies should the clinic have in


place to prevent death from an anaphylactic reac-
tion?

85
Drugs that Weaken the Bacterial Cell Wall II: Cephalosporins, Carbapenems,
Vancomycin, Telavancin, Aztreonam, Teicoplanin, and Fosfomycin

STUDY QUESTIONS 3. First-generation cephalosporins are not effective


against __________________.
Completion

‹
{%  
  
 

1. Cephalosporins are often resistant to __________________ resistance to beta-


_____________________________. lactamases produced by gram-negative organisms.

2. First-generation cephalosporins are highly active 5. Third-generation cephalosporins are considerably


against gram-__________________ bacteria. more active against gram-__________________
aerobes.

> 
Ÿ
^`]—
Z 
#

 
 
212 CHAPTER 85 Q
&
 
 

–  
>
 
##

6. __________________ is the only cephalosporin  13. A patient with hospital-acquired pneumonia has
with activity against methicillin-resistant )- just been prescribed cefotaxime and probenecid.
  
=€{? The patient has no history or evidence of gout.
What should the nurse do?

~
| 
$
   

=>{…?
a. Administer both medications as ordered.
by fourth-generation cephalosporins is b. Administer the cefotaxime as ordered and ask
__________________. the patient if he or she was taking probenecid at
home.
c. Administer the cefotaxime, but contact the
CRITICAL THINKING, PRIORITIZATION, prescriber before administering the probenecid.
AND DELEGATION QUESTIONS d. Contact the prescriber before administering
either of the medications.
8. Which result would be of greatest priority to
report to the prescriber of cefotaxime? 614. An alert and oriented patient with a history of

 
†‹
'‰ penicillin allergy is prescribed cephalexin. What is

‰!
]<
  
 '‰ the priority action by the nurse?

\…
‹<
‰' a. Administer the cephalexin.

#+
] b. Administer the cephalexin and carefully assess
for allergic reaction.
9. Which result would be of greatest priority to c. Assess the type of reaction that the patient had
report to the prescriber of ceftriaxone? to the penicillin.

 
†‹
'‰ d. Notify the prescriber of the allergy and ask for

\…
<<
‰' a different antibiotic order.

*'*
‡^
'‰'^~œ

#+
]  15. The nurse is reviewing new laboratory results,

 
  
=\…?
<^
‰'
10. When performing shift assessment, the nurse notes min for a patient receiving cefotetan 2 g every 12
a maculopapular rash over the trunk of a patient hours. What should the nurse do?
who has been taking a ceftriaxone for 4 days. a. Administer the medication.
What is the priority nursing action? b. Withhold the medication and notify the pre-
a. Administer PRN epinephrine. scriber of the laboratory results.
b. Complete the assessment.
c. Consult the prescriber. 16. Cefazolin has been prescribed at discharge for a
d. Withhold the ceftriaxone.  


  
  
&



  
$

  %
 


11. Nursing interventions when administering cepha- discharge teaching it is important for the nurse to
 

={

 
? teach the patient to avoid consuming what?
a. always administer on an empty stomach. a. Alcohol
b. assess for severe allergy to carbapenems due to b. Antacids
cross-allergy. c. Aspirin
c. instruct the patient to immediately report pain 
# $

 



d. store oral suspensions in the refrigerator.  ]~
!
 

 

 


-

$
$ 

`

$
œ
 



 12. A patient who is receiving ceftazidime has three $ 


!
$ 



$
  
 ‚

loose, brown bowel movements in 24 hours. What 


$ 

†
‰'
 



should the nurse do? nurse do?


a. Administer the drug and continue nursing care. 
{ 



$

  

b. Continue to administer the drug and notify the $






 
prescriber of the change in bowel movements. b. Run the minibag as a piggyback at a Y site.
c. Discontinue administering the drug and notify c. Consult with the prescriber.
the prescriber. 
{


#Š
$ 

 



| 

  
{!!   


#Š

> 
Ÿ
^`]—
Z 
#

 
 
CHAPTER 85 Q
&
 
 

–  
>
 
## 213

š
]<

  
 
$
 



23. The nurse is preparing to administer an intermit-
reason to withhold administering cefotetan to an 
$ 
$
 
=# @?
!

 

adult male and to notify the prescriber? be administered concurrently with which intrave-

‰!
^‹
  
 '‰ 
=#Š?
ƒ

–ˆ+
†‹
 '‰ a. 5% dextrose in water

> 
^]
 '‰ b. 0.9% sodium chloride

…–{
^†`
 '‰ c. 0.45% sodium chloride
d. 0.45% sodium chloride/5% dextrose solution
19. Cefditoren is prescribed on discharge for a patient
diagnosed with bronchitis caused by   š
^‹
Š 


 


$

  

^
&€
\Z&

%  
pseudomembranous colitis caused by /   

renal failure. Why is this antibiotic a good choice   . The nurse receives laboratory results on
for this patient? 
 
 

 
^^
 '‰
 


#



$$

   should the nurse do?
b. Cefditoren is effective for beta-lactamase pro- a. Administer the medication.
ducing   . b. Hold the medication and notify the prescriber.
c. Cefditoren is inexpensive.
d. Cefditoren has once-a-day dosing.
^

%
  
 
-
ing would be of most concern when a patient is
š
^`
>$ 
]
 
   


$
prescribed telavancin?
a 130-lb woman. The drug is reconstituted to a 
…  
 

$
^`
 '‰
*


 
b. Foamy urine
administer it? c. Maculopapular rash on cheeks and nose

]

 


 

$
 
  
d. Bibasilar crackles
and deltoid muscles

^

 


 

$
  

muscles DOSE CALCULATION QUESTIONS



^

 


 

$

 

‹



 

$

 
26. Telavancin 0.75 grams is prescribed for a patient
muscle 
 
~^


 
\…
=>>?

†‹

‰'
#

 
$ƒ

^]


=| ?

  

-
scribed for a patient, what is the priority nursing
concern?
^~
€
=€?
^`


  


a. Hydration 8 hours for a 14-lb child. The recommended safe


b. Nutrition  
 
$

=€?
$
-

{ $ 

‹`
 '

<
 
#


$
 ƒ

{
 

^^
!
  
 

=| ?
CASE STUDIES
is a combination of the antibiotic and cilastatin.
What is the purpose of the additive cilastatin? Case Study 1
a. To decrease adverse effects of nausea and vom-
iting A 45-year-old, gravida 4, para 4 woman is admitted to the

!




\#
   



  
$

 
{

c. To prevent destruction of the antibiotic by beta-  


$
 
 
   

 
{

lactamase has a fever of 101°F and a white count of 18,000/mm3.


d. To prevent inactivation by an enzyme present 
  
 

 



 

]

$

in the kidney ceftriaxone is ordered every 12 hours. Pain management








1. The nurse should assess the patient for which pos-


sible adverse reactions to the antibiotic?

> 
Ÿ
^`]—
Z 
#

 
 
214 CHAPTER 85 Q
&
 
 

–  
>
 
##

2. How can the possibility of thrombophlebitis be Case Study 3


minimized?

 

  
 
=Š ?
]



]^



`‡``

AM?

^]``

PM?
!

  



$
]


^``
‰

3. Why would a broader spectrum antibiotic not be


]`



 

 




-
prescribed for this patient? istering this drug rather than hanging this drug by
gravity?

4. What should the nurse do if the nurse discovers


the patient has had an anaphylactic reaction to
]]
#$



 

`‡``

 


penicillin? should blood be drawn to assess trough levels of


the drug?

Case Study 2
12. The drug will reach its therapeutic level after how
!
 

 
$

 


  

many doses of the drug?
$ 
^


]^


 
=?

300 mg 3 times a day after surgery for a ruptured appen-


dix.

5. What assessments and laboratory results should 13. Describe red man syndrome and the nursing mea-
be monitored while this patient is on this drug sures that can be taken to prevent this reaction.
therapy?

14. Does experiencing red man syndrome create


6. What precautions does the nurse need to take a contraindication for further administration of
when administering these two antibiotics through vancomycin?



ƒ

15. Describe how the nurse will assess for the possible

~
!
 

 

 ‚
#+

†
 
adverse effects of
should the nurse do? a. ototoxicity.

b. immune-mediated thrombocytopenia.
8. Three days postoperatively, the patient has pro-
gressed to a soft diet and develops severe watery
diarrhea. What are possible causes of the diarrhea?

9. What are possible electrolyte imbalances that


could occur and what are their symptoms?

> 
Ÿ
^`]—
Z 
#

 
 
CHAPTER 86 Q
–   
# 
$
|
{  •
!  
€  

} 215

86
Bacteriostatic Inhibitors of Protein Synthesis: Tetracyclines, Macrolides, and Others

STUDY QUESTIONS  12. A patient who was admitted with severe ab-
dominal pain has been diagnosed with * –
True or False associated peptic ulcer. Tetracycline 500 mg,
metronidazole 250 mg, and bismuth subsalicylate
   
  T   F 525 mg have been prescribed 4 times a day. The
    

 

 ‚
^‹%

 



 
^``





1. ___ Tetracyclines should be taken on an empty 



—``%<``

$
 
$

 
^
  

stomach. What should the nurse do?

^

‰
 
$

 

 

a. Administer the medications and continue nurs-
prevent destruction of gingival connective ing care.
tissue. b. Administer the medications and report the
3. ___ Resistance to tetracycline is increasing. changes.

‹

! 


 

$

$
c. Withhold the medication and continue nursing
most nonseptic infections. care.
5. ___ Tetracycline easily crosses mammalian cell d. Withhold the medication and notify the pre-
membranes. scriber of the changes.
6. ___ Tetracycline is active against the bacilli that
cause anthrax. 13. Tetracycline can cause esophageal ulceration.
7. ___ Tetracycline is a narrow-spectrum antibiotic. What can the nurse teach to minimize the risk of
8. ___ Tetracyclines are bactericidal. this adverse effect?
9. ___ Minocycline reduces symptoms of rheuma- 
{ 
 
$

 
†`

$
 

toid arthritis. the medication.


10. ___ Tetracycline should be used for mild acne. b. Take the medication at bedtime.
c. Take the medication with an antacid.
CRITICAL THINKING, PRIORITIZATION, d. Take the medication with milk.
AND DELEGATION QUESTIONS  14. A patient is prescribed a tetracycline antibiotic.
Which patient information is a reason for the
11. The nurse teaches a patient who has been pre-
medication to be withheld by the nurse and the
scribed oral tetracycline that the medication
prescriber consulted?
should not be taken with which over-the-counter
a. Patient has an allergy to penicillin.
 = ?ƒ
={

 
?
b. Patient is a 12-year-old child.
a. Ascorbic acid
c. Pregnancy status of patient is unknown.
b. Centrum silver
d. Theophylline for asthma is also prescribed.
c. Ferrous sulfate
d. Folic acid
15. The nurse should assess for adverse effects of

!ˆ€{
lightheadedness and dizziness when a patient is
receiving which medication?
a. Demeclocycline
b. Doxycycline
c. Minocycline
d. Oxytetracycline

Copyright © 2016, Elsevier Inc. All Rights Reserved.


216 CHAPTER 86 Q
–   
# 
$
|
{  •
!  
€  

}

616. A patient is prescribed tetracycline for /


  21. The nurse teaches that to be well-absorbed, which
 
 

  

 

$
$
 
=– ?
 

 -
would be a priority to report to the prescriber? tered with food?
a. Burning on urination 
{  
 
b. Perineal itching 
Z% 
 
c. Vaginal discharge 
\ 
d. Watery stool 
{  

17. A patient with a penicillin allergy is prescribed 22. What teaching can the nurse provide regarding
erythromycin ethylsuccinate 250 mg every 6 hours administration of antibiotics that helps decrease
for pneumonia caused by *
   %'. the development of resistance?
!
 
  

=€?

a. Always take antibiotics with food.
the medication scheduled at 0600, 1200, 1800, and b. Only take antibiotics when symptoms are pres-
2400. On the second day of therapy, the patient ent.
complains that he does not like taking the drug 
\





$
 



because it causes heartburn. What would be an as you think you have an infection.
appropriate intervention by the nurse? d. When prescribed, take the full course as di-
a. Administer the drug with food. rected even if symptoms are gone.
b. Change the timing of the drug so that most
doses are administered with meals.  ^†
#

 
$

 



 -

Z 
 
 



$

 - ratory test when a patient is prescribed azithromy-
ids will prevent absorption of the drug. cin or erythromycin and warfarin?
d. Withhold the drug and notify the prescriber. 
–ˆ+
b. CK-MM
18. How should an enteric-coated erythromycin base 
#+

 ƒ
={

 
? d. RBC
a. On an empty stomach
b. Once a day  ^‹
!
€

@
=¢ ?
 

c. Whole, not chewed to be administered at 0800. Breakfast arrives on


d. With a full glass of water the nursing unit around 0800, lunch at 1230, and
e. With food dinner at 1730. The patient does not receive any
antacids. What should the nurse do?

]‡
–  
$

 
$
 
˜!
 

a. Administer the drug at 0730, within the accept-
torsades de pointes, the nurse would consult the able time frame.
prescriber before administering erythromycin to a b. Administer the drug at 0800 and hold the pa-
patient who has experienced tient’s breakfast until 0900.
a. frequent headaches. c. Consult the prescriber.
b. nausea. d. Reschedule the drug for 1030.
c. unexplained fainting.
d. wheezing.
^

 
 


 
=>-
?
$

 

 
$
 
 

20. A patient is prescribed erythromycin. What is one by this patient, could decrease bowel motility
reason why careful review of all drugs this patient and prevent the body’s attempt to rid the colon of
is prescribed is important? overgrowth of /  ?

Z‚





– 
=ˆ?
clindamycin. 
& 
=‰ ?

Z

$$
  

!
=&?
metabolism is increased by nondihydropyridine 
 $ 
=> ?
CCB.
26. The nurse would consult the prescriber regard-

Z
 
 

$
 $ 


 
  
$
@
=¢?
$


and cause bleeding.


nurse discovered the patient has a history of which

Z



$
% @

disorder?
antifungals.
a. Asthma
b. Celiac disease

\
d. Phenylketonuria

Copyright © 2016, Elsevier Inc. All Rights Reserved.


CHAPTER 86 Q
–   
# 
$
|
{  •
!  
€  

} 217

 ^~
#







  - 632. The nurse is caring for a patient whose peak chlor-
tory test when a patient is prescribed linezolid 


^<
 '‰

 -
=¢?ƒ 











{!

‰!
prescriber relating to administration of this drug?

–ˆ+ a. BP 135/80 mm Hg
c. CBC and differential 
…  
 


d. Fasting blood glucose c. Pallor
d. Vomiting
28. A patient with a history of hypertension controlled
by an angiotensin-converting enzyme inhibitor is
††
 


 ”
  
$
 
=! ?

  
@
=¢?
$

 % 
 
ŠZƒ
   
 
=ŠZ?
$
!
 
a. No change to the metabolism of any drugs
should be instructed to avoid consuming what including warfarin
substance while on this antibiotic? b. Decreased development of antibiotic resistance
a. Aged cheese c. Oral administration
b. Milk 
{ $
$
 

  
 

 

c. Red meat patients



{ $

†‹

 

 
@
€{


29. The nurse should include in teaching to a patient =–  ?



 


 ƒ




@
=¢?

- 
#  
mediately report which symptom of a rare adverse 
#  
effect that has been associated with prolonged c. Orally
therapy? d. Topically
a. Dizziness when changing positions
b. Headache
c. Numbness or tingling in any extremity DOSE CALCULATION QUESTIONS
d. Vomiting

†
Z
 
  
^``

‹


6†`

 



=?
$
day, has been prescribed for a child. The pharmacy
% "

=>|?
  



^`
 '
‰
*


/
 
 . The patient developed erythromycin should the nurse teach the parent to
nausea, vomiting, dark urine, and fatigue. The administer per dose?
prescriber discontinued the drug after evaluating
‰!


^<~
  
 '‰
 


priority teaching for this patient? 36. Chloramphenicol sodium succinate 1 g every 6

’#$


 







  

$

—~%
 
!

take it with food to minimize adverse effects on recommended dose is 12.5 mg/kg every 6 hours

\#
 “ 

 
 

@ 
#

-

’#$


 






scribed dose safe?

<`%]``
@
$

 
 “

’#$


 






look at your stool and contact the prescriber if


it turns black or bloody.”
d. “You should never take this drug again.”

6†]

  
 
$



 




 $ '" 
={-
?


 



  

immediately?

‰!
^`
  
 '‰

–ˆ+
^`
 '‰

>%€€
`
 '‰

…–\
^`
 '‰

Copyright © 2016, Elsevier Inc. All Rights Reserved.


218 CHAPTER 87 Q
   •
–  
# 
$
|
{ 

CASE STUDIES Case Study 2


A 15-year-old patient is readmitted after being discharged
Case Study 1 following an appendectomy with shaking chills, fever of
A 30-year-old attorney reports to the health care facility 103° F, and purulent drainage from the incision site. Cul-
with a fever of 101° F and a nonproductive cough that ture and sensitivity results of the wound drainage reveal
has lasted 10 days. Her medical history reveals that she 0   
that is resistant to vancomycin.

 
 

=!%&?


 
& $ '" 
={?
``

 

to penicillin. On physical examination, her respiratory every 12 hours, is prescribed.


rate is 24/min and nonlabored, and bibasilar rhonchi are
heard. The patient is given a prescription for erythromy- 3. What laboratory tests and symptoms should the
cin, 250 mg 4 times a day for 10 days, to treat what is nurse monitor to assess for hepatotoxicity?
 


 
 
{

 


take Tylenol 325 mg every 4 hours for fever, increase her




—%<
 
$
 

 

 

$

work for 2-3 days.

1. What serious adverse effect is this patient at risk 4. What nursing assessments and interventions
for based on drug interactions, and what symp- should the nurse include in the plan of care
toms would suggest this syndrome?  


  
 
$
$ % 

thrombophlebitis?

2. What should the nurse do regarding the possibility


of this syndrome?

& $ '" 
={?
 

CYP3A4 hepatic drug-metabolizing enzyme.


 


  
$
 
 ƒ

87
Aminoglycosides: Bactericidal Inhibitors of Protein Synthesis

STUDY QUESTIONS 3. Aminoglycosides are rapidly excreted by the


______________________.
Completion 4. Aminoglycosides are __________________
1. Aminoglycosides kill bacteria so their action is -spectrum antibiotics.
______________________. 5. Aminoglycosides are primarily used to
2. Aminoglycosides are highly polar polycations and treat serious infections with aerobic gram-
therefore cannot enter the __________________ ______________________ bacilli.
__________________.

Copyright © 2016, Elsevier Inc. All Rights Reserved.


CHAPTER 87 Q
   •
–  
# 
$
|
{  219

6. Aminoglycosides can be toxic to  14. The nurse is assessing a patient who is sched-
the ______________________ 



 
$
 
=\  ?

and ______________________ #

 
]^
 

 


‡``
‰

______________________. 



†``
‰


 ‚

bladder is not distended. What should the nurse


7. Aminoglycosides can kill bacteria for several
do?
______________________ after serum levels
a. Administer the drug.
drop below the minimal bactericidal concentra-
b. Administer the drug and notify the prescriber of
tion.
the output.
8. Cell kill by aminoglycosides is 
# 

 



$

$
 

__________________________ each time the medication is administered.


__________________________. d. Withhold the drug and notify the prescriber of
the output.
9. The principal cause of bacterial resistance is
production of ______________________ that can  15. The nurse is taking a history from the spouse of
inactivate ______________________. a patient who was admitted in a septic state and
10. Aminoglycosides cannot kill prescribed an aminoglycoside antibiotic. Which
______________________. question is most important to ask the patient’s
spouse about the patient’s history?
a. “Has your spouse ever had any surgery per-
CRITICAL THINKING, PRIORITIZATION, formed?”
AND DELEGATION QUESTIONS b. “Has your spouse been told to follow any spe-

ƒ“
11. Major adverse effects of aminoglycoside antibiot- 
’*

 



 




  

={

 
? year?”
a. cochlea. d. “What medications is your spouse currently
b. heart. taking?”
c. kidneys.

]—
!
 

 
$
 
$$
$

d. lungs.
tobramycin. Which change would be a priority to
e. stomach.
report to the prescriber?
f. vestibular apparatus.
a. Dilute urine
b. Headache
12. Facultative bacteria survive in what condition?

‰
 
 
a. anaerobic
d. Ringing in the ears
b. both anaerobic and aerobic
c. aerobic 17. The nurse is aware that the risk of ototoxicity is
  
 
$

 
 


6]†

 


  
  
=+ ?
also receiving which medication?
complains of a headache. What is the priority a. Bumetanide
nursing action? 
Z 

a. Assess the onset, characteristics, and associated c. Furosemide
symptoms of the headache. d. Hydrochlorothiazide
b. Medicate with acetaminophen and reassess in 1
hour. 618. What is the priority nursing action before admin-
c. Withhold the tobramycin and notify the pre- istering an aminoglycoside to a patient with an
scriber when making rounds. \…
$
`
‰'ƒ
d. Withhold the tobramycin and notify the pre- a. Ask the patient if he or she has had a headache.
 
{!! b. Assess peak levels of the drug.
c. Assess liver function tests.
d. Compare prescribed dose to recommended
dose.

Copyright © 2016, Elsevier Inc. All Rights Reserved.


220 CHAPTER 87 Q
   •
–  
# 
$
|
{ 

š
]‡

  

  



CASE STUDIES
once-daily dose, it is important to monitor trough
levels Case Study 1
a. 30 minutes before the next dose.
b. 1 hour after completing the infusion. A 72-year-old male patient with a history of type 2
c. 1 hour before the next dose.  

=!^&€?
 

$


d. 30 minutes after completing the infusion. rheumatoid arthritis treated for many years by nonsteroi-
 
%  
 
 

  

 

š
^`
!
 

 
$

 



earlier. He is readmitted to a medical-surgical unit for a
gentamicin twice a day. Peak and trough levels   

$
  


–|

were drawn after the fourth dose. Results were 100/70 mm Hg, pulse 98 beats/min, respirations 24/min,
 
†
 '‰

 
`—
 '‰
 

 
]`‹„
…
=‹`„
>?
*

$

 

should the nurse do? and his skin is hot and dry. 0    cultured from
a. Administer an additional dose. 

  

 

 
=?
*

b. Continue nursing care. dose of amikacin is less than the normal recommended
c. Consult with the prescriber. dose.
d. Withhold future doses until the prescriber can
be consulted. 1. What are possible reasons for the prescribed dose
not being equivalent to the recommended dose?
621. Which of these laboratory tests would be a priority
for the nurse to assess when a patient is receiving
an aminoglycoside?
a. Creatinine
b. Fasting blood glucose

^
!




 

`—``

AM?

c. Hemoglobin and hematocrit


]‹``
=^
PM?

^^``
=]`
PM?
| 

 


#+
levels of the amikacin are ordered. When will the
nurse schedule the collection of the blood sample
š
^^

 


  



for this testing?



 
#

    
 



  

 
‹`


{!!

What is the most appropriate nursing action?


a. Administer the drug as quickly as possible.
b. Assess the patient’s vital signs.
c. Clarify the order.
†



 


  
 

d. Refuse to administer the drug. serum peak levels?

DOSE CALCULATION QUESTIONS


23. Based on the recommended dose of gentamicin,
‹
!
 

$

 
=?


what is the recommended dose range for a patient ]


 '‰
 


 
ƒ
with a gram-negative infection, who weighs 220
lb, if the gentamicin is to be administered every 8
hours?

Case Study 2
24. A patient weighs 65 kg and has been prescribed
 
]~


<
 
#

  
A 40-year-old male patient is recovering in the trauma
dose appropriate for this patient? unit from a major accident. He is on a ventilator and has a
 

$


  

{ %\ @
 -
eter to measure cardiac status, an arterial line to measure
continuous blood pressure, a small-bore feeding tube,
two chest tubes, and a Foley catheter. After 3 days, he
develops gram-negative septicemia and pneumonia and is
 

 
$ 
=\  ?



=ˆ ?
#Š

Copyright © 2016, Elsevier Inc. All Rights Reserved.


CHAPTER 88 Q
{$ 

! 221

5. Describe the assessments the nurse should perform 8. How can the nurse prevent drug interactions?
to detect nephrotoxicity and neurotoxicity.

9. What type of urine collection bag should the nurse


6. How does nephrotoxicity increase the risk of use with this patient?
developing ototoxicity?

10. What changes in the dosages of medications


7. Which toxicity is most likely to be permanent? would be necessary if the patient were 80 years
old instead of 40?

88
Sulfonamides and Trimethoprim

STUDY QUESTIONS
]`

{$ 


  



applied topically.
True or False
]]

{$ 
 
  
 
$

folic acid.
   
  T   F
]^

{$ 


 
 
 -
   otics developed.

]

{$ 
 

 
-
lated to glipizide, glyburide, furosemide, and CRITICAL THINKING, PRIORITIZATION,
hydrochlorothiazide. AND DELEGATION QUESTIONS

^

{$ 

 % 
 -
ics. 613. Which type of rash would most likely indicate the

†

{$ 


 

 
 
$
{ %  



 

many microbes for which they initially were is receiving sulfonamides?


effective. a. Amber-colored, crusty rash on the cheeks

‹

{$ 


 

 
b. Papular rash on the shoulders
 
 
$
=ˆ!# ? c. Pruritic rash on lower arms



{$ 

  
 
 - d. Vesicular rash in the mouth
muscularly or intravenously.

—

{$ 


 
  
$
614. What is the priority reason why the nurse teaches a
low toxicity. patient who is prescribed sulfamethoxazole to take

~

{$ 

$$

 

this medication with a full glass of water?
organism common in pneumonia occurring a. Decrease the risk of esophageal irritation

 

#&{ b. Minimize crystal formation in the urine

<

{$ 
  
$



c. Prevent nausea
the patient. 
{ 
$"


‡

{$ 


  

Copyright © 2016, Elsevier Inc. All Rights Reserved.


222 CHAPTER 88 Q
{$ 

!

615. Which laboratory test result would be a priority to


^]
!
=| ?

 

  

report to the prescriber of a sulfonamide?   


>
 


 ƒ
={

 

a. Neutrophils 65% ?


b. Platelets 200,000/mm3 a. Animal studies have shown a risk for fetal
c. RDW 18.4% problems, but human studies have not shown
d. WBC 11,800/mm3 any problems.
b. Controlled studies have shown no risk to a

]—
{$ 
 
  

 
$$
$
- developing fetus.
nicterus. How does this present? c. This drug must not be used while the patient is
a. Damage to the brain in neonates caused by pregnant.
deposition of bilirubin in the brain d. Risks are possible during pregnancy but the
b. Damage to the kidney tubules by crystals in the  

 
” $

 
urine 
!



$
 


-

# 
$


 



ing fetus.
cells

ˆ  
 

  622. Because of the risk of hyperkalemia when pre-
scribed trimethoprim, it is a priority for the nurse
 17. A patient who takes glyburide for type 2 diabetes to teach the patient to report what symptom?

=!^&€?

  
$  @% a. Bruising

=!€|'{€¢?
$

ˆ!#
–  
b. Weakness
of the possibility of intensifying the effect of c. Pallor
glyburide, the nurse should assess for 
{
 
a. bleeding and bruising.
b. fever and photophobia. 23. The nurse is preparing to administer trimethoprim
c. hot, dry skin and thirst. `

  
 

]^
 
‰  -
d. paresthesias and abdominal cramps. 
 
 

\…
†`
‰'
 

should the nurse do?


 18. The patient has been receiving mafenide applica- a. Administer the drug.
tion to a second-degree burn for over 2 weeks. b. Check the patient’s intake and output.
Which laboratory results suggest that the drug c. Consult the prescriber.
might be causing acidosis? d. Withhold the drug.
a. pH 7.32, HCO3
]<
Z"'‰
CO2 31 mm Hg
b. pH 7.35, HCO3
^^
Z"'‰
CO2 37 mm Hg
^‹
 



$
%
c. pH 7.41, HCO3

Z"'‰
CO2 40 mm Hg $  @
=!€|'{€¢?


-
d. pH 7.45, HCO3
^]
Z"'‰
CO2 47 mm Hg nents used alone?

‰ 
$
 


 
 

 -
619. The nurse is preparing to apply topical mafenide row therapeutic range
to second-degree burns on the anterior of a 7-year- b. Fewer adverse effects
old patient’s arms, upper legs, and trunk. What is 
‰
   


the priority nursing action, aimed at preventing the 
‰

$

common adverse effect relating to application of
this drug? 25. A physician who is on call for another physi-

Z

" cian gives a verbal order for sulfamethoxazole-

Z 

 
$
  
=!€|'{€¢?
$

 
!
 

c. Prevent pain. would consult the prescriber about this order if the

{


 
 


  patient had a history of which condition?
a. Megaloblastic anemia
20. The nurse is administering trimethoprim to a pa-
b. Chronic obstructive lung disease
tient who has a history of alcohol use disorder. Be-
c. Diabetes mellitus
  
$ 


 



d. Hypertension
use disorder, the nurse should assess for symptoms
of possible adverse effects of trimethoprim, which
include
a. nausea and vomiting.
b. rash and malaise.
c. pallor and sore throat.
d. photosensitivity and drug fever.

Copyright © 2016, Elsevier Inc. All Rights Reserved.


CHAPTER 88 Q
{$ 

! 223

 ^—

#+
 



 

 

5. What can the nurse teach to decrease the incidence
  


 

 
  
 
and severity of these possible adverse effects to
warfarin who has also been prescribed a sulfon- !€|'{€¢ƒ
amide? a. Photosensitivity

#+
^

#+
^

#+
†

{ %  


#+
†

DOSE CALCULATION QUESTION c. Renal damage

^~
!


  
~^

!€|'†—`

{€¢

Available is an oral suspension of 8 mg TMP and


‹`

{€¢

‰
*
 




nurse administer? 6. What forms of sulfonamide drugs have less risk of


{ %  
ƒ

CASE STUDIES
Case Study 1 Case Study 2

†`% %





*#Š% 

The nurse has volunteered to make a trip to provide health

 

¢!
$

 
]
 
{
 
 

  
$ 

ˆ  





to the clinic with a low-grade fever, nonproductive cough, set up, and doctors and nurse practitioners are diagnosing
and shortness of breath. A chest x-ray shows diffuse cases and providing donated samples of medications. The
  

  
$
 

=|>|?
+


 



{$ 

caused by !
  1  is made. are being provided to patients with urinary tract infec-
tions.
1. The patient states that she knows about getting
pneumonia from !
    . Now she 7. What adverse effect would these patients be at
has something new and she is sure that she is go- greater risk for than the general population?
ing to die. How should the nurse respond?

8. What type of sulfonamides would be more likely


2. The patient is started on trimethoprim- to cause hemolytic anemia?
$  @
=!€|'{€¢?


!€|'{€¢

a good choice for this patient?

9. The nurse should assess for what symptoms that

†



 

 
$

 
$
>\
suggest possible development of hemolytic ane-

$

 
 
 
!€|'{€¢ƒ mia?

4. What adverse effects should the nurse teach the


]`

*#Š% 
 





patient to report, should they occur? with a 2-week-old infant who appears toxic. The
only antibiotic the team has in a liquid form is a
sulfonamide. Why is this dangerous to give to this
infant?

Copyright © 2016, Elsevier Inc. All Rights Reserved.


224 CHAPTER 89 Q
&
! 
$
ˆ 
! 
#$

89
Drug Therapy of Urinary Tract Infections

STUDY QUESTIONS 9. A female patient has been prescribed an antibiotic


for 3 days. What are some advantages of shorter-
Matching  
 ƒ
={

 
?
a. Fewer adverse effects

      
‰
 
$
 
   

‰
 

]

#  
$




 
\ 

$
 


^

#  
$

 
 
 

therapy as prescribed
from the bladder to the urinary meatus e. Treatment of upper urinary tract involvement

†

#  
$

 



bladder neck in men 610. A prescriber’s orders for a new admission include

‹

#  
$

 
  


 

 

5. ___ Recolonization with the same organism ‹``



  
=#Š?

]^
 
 

6. ___ Colonization with a new organism is a priority nursing responsibility?



>  


 
$


$ 
a. Cystitis 
… 


 
b. Prostatitis 
€

 





c. Pyelonephritis d. Obtain the urine culture specimen before ad-
d. Reinfection ministering the antibiotic.
e. Relapse
$
ˆ 11. What is true about nitrofurantoin and methena-
mine?

!

 %

$
   
CRITICAL THINKING, PRIORITIZATION, 
!


 
$

\#
 
AND DELEGATION QUESTIONS c. Therapeutic levels are achieved only in urine.
d. They have few adverse effects.
7. What is an example of a complicated urinary tract
$
=ˆ!#?ƒ 6]^

$
 
 
 
$




ˆ!#
"


  
 



$ 
=€ -

ˆ!#
  


  
    ?


 



  


ˆ!#
  


   immediately?

ˆ!#
$


 a. Brown-colored urine
b. Dyspnea
6 8. The nurse is caring for an incontinent patient who c. Nausea

 
 
$
ˆ!#
=0 ?
 
d. Headache

!€|'{€¢
 






prevent recurrence of infection? 13. A patient is receiving 50 mg of nitrofurantoin


a. Assessing for sore throat and fever =€  ?


$
 
$
-
b. Carefully wiping from front to back during rent cystitis. This means that the patient
perineal care a. is taking low doses of the drug to prevent rein-

Z 
  
$

 
^```

$
fection.

 
  b. is taking low doses of the drug to prevent
d. Offering cranberry juice each day relapse.
c. is taking very high doses of the drug because he
or she cannot get rid of the infection.



ˆ!#
  


   
  

Copyright © 2016, Elsevier Inc. All Rights Reserved.


CHAPTER 89 Q
&
! 
$
ˆ 
! 
#$ 225

]‹

 


  
$ 
=… - CASE STUDIES
 ?
$

ˆ!# 
!
 

 


 


 ƒ
={

 
? Case Study 1
a. Not to take the drug if there is any chance that
she might be pregnant. A 75-year-old male patient with known benign prostatic
b. Report any shortness of breath and coughing.  
=–|*?

 

 
$
  
c. Report the onset of numbness or tingling.
d. Take the medication on an empty stomach.
]
#$

 

 
 
 
$



ˆ 
 


  


 
other signs of infection higher in the urinary tract
cause drowsiness. suggesting pyelonephritis, what actions should be
taken?
615. Which laboratory test result is of greatest priority
to review before the nurse administers nitrofuran-

=…  ?ƒ

‰!
b. Bilirubin 2. The patient has been successfully treated with

\… !€|'{€¢



  
$ 


{ =€  ?



$
ˆ!#


he can be evaluated for surgery for his enlarged


 16. The nurse is reviewing the other over-the-counter prostate. The nurse should teach the patient to be
products taken by a patient who has been pre- alert for what possible adverse effects from Mac-
 
 
=€  ?

$
rodantin?
these home treatments might reduce the urinary
antiseptic action of methenamine?
a. Baking soda in water for heartburn
b. Drinking 10 glasses of water each day for gen-
eral health 3. What nursing assessments and teaching can the
c. Taking ibuprofen twice a day for joint pain nurse employ to prevent complications from nitro-
d. Taking megadoses of vitamin C to prevent $ 
=€  ?
 ƒ
colds

DOSE CALCULATION QUESTIONS



‹
!
 
 



$
 
 


]~
+$ 
   
=€  ?
`
urine has changed to a brown color. What should
mg twice a day is prescribed. Available are 25-mg the nurse tell him?
capsules. How many should be administered per
dose?

]<
€ 
 
=*?
``


5. Why is it important to monitor creatinine levels in
a day is prescribed for a 10-year-old child. The this patient?
pharmacy dispenses methenamine hippurate
=*?
]%
  
*
 
 



administered at each dose?

Case Study 2

$ 
 




 
 

{

constantly feels that she needs to urinate, but she does not
want to because it hurts, and the urge to urinate persists
$

 
{

$

 
 



not looked at her urine.

6. What additional questions should the nurse ask the


patient to assist the practitioner with diagnosing?

Copyright © 2016, Elsevier Inc. All Rights Reserved.


226 CHAPTER 90 Q Antimycobacterial Agents

7. Why is it important to determine how frequently 9. What instruction should the nurse provide the

 

 
ˆ!#


 ƒ patient to prevent future infections?

<
!
 
 
 

 
 
ˆ!#

treated with sulfonamides are due to Escherichia


 infection. Why are female patients more prone
to get this infection?

90
Antimycobacterial Agents: Drugs for Tuberculosis, Leprosy, and Mycobacterium avium
Complex Infection

STUDY QUESTIONS CRITICAL THINKING, PRIORITIZATION,


AND DELEGATION QUESTIONS
True or False
   
  T   F 12. A male patient who has been diagnosed with ac-
   tive TB has been prescribed isoniazid, rifampin,
ethambutol, and pyrazinamide. The patient asks
1. ___ The principal cause underlying the emer- the nurse why he has to take so many drugs. The
gence of resistance is inadequate drug basis of the nurse’s response should include that
therapy. 

 


$$= ?ƒ
={

2. ___ Microscopic examination of sputum is the 


 
?
best way to diagnose TB. a. Decreased adverse effects
3. ___ Culture and sensitivity for  
 b. Decreased risk of relapse
  takes 24-48 hours. c. No increased risk of suprainfection

‹

# 
$

!–

% 
Z 
$



 

free, cannot infect other people. tuberculosis mycobacteria



# 
$


 

 
$
e. Prevention of mycobacteria from developing
developing active TB without additional resistance to a drug
exposure to the bacteria even if the infection
has been dormant for many years. 13. A male patient asks the nurse why he has to

—

#$



 
 


 - continue to take medication after completion of
pital. the initial induction phase if his sputum is “clean.”
7. ___ TB is an infection limited to the lungs. What does the nurse say?
8. ___ TB is more prevalent in jails and homeless a. Dormant bacteria are still present inside cells
populations than the general population. and can become active at a later time.
9. ___ Primary infection with TB usually is evident 
{



 
 


on chest x-ray. ensure that the active bacteria are eliminated.


10. ___ Resistance to drugs for TB is increasing. 
{




$
 -
11. ___ Treatment of active infection with TB rium.
should always include at least two drugs. d. This therapy is in case the patient comes into
contact with the person who originally infected
him.

> 
Ÿ
^`]—
Z 
#

 
 
CHAPTER 90 Q Antimycobacterial Agents 227

]‹

*#Š%$
 




19. A patient with a history of peripheral vascular
 

 
= ?

- disease, type 2 diabetes mellitus, and latent TB is
quinavir, is diagnosed with an active TB infection. prescribed isoniazid and pyridoxine. What is the
Because of the drug interaction that decreases the purpose of the pyridoxine?
$$
$
 

$
*#Š
$

 

#
 


should not be prescribed which drug? b. Prevent hypoglycemia

Z   c. Prevent peripheral neuropathy

#  @ d. Treat resting mycobacteria within cells
c. Pyrazinamide
d. Rifampin 20. Why is the interaction of isoniazid and phenytoin
especially important?
15. A 19-year-old nursing student with no history of 
#  @

 

  
medical problems, no symptoms of disease, and no b. Phenytoin has a narrow therapeutic range.
risk factors for contracting TB is being screened c. Phenytoin levels can become subtherapeutic.
for TB before starting a clinical nursing course. d. The risk of hepatotoxicity increases.
He has a 20-mm area of erythema surrounding an
11-mm area of induration 48 hours after receiv-  21. A patient who is paranoid schizophrenic who has
ing a PPD tuberculin test. Chest x-ray and sputum been involuntarily committed to a psychiatric in-
culture are negative. The nurse would expect stitution has been diagnosed with latent TB and is
prophylactic treatment to involve refusing drug therapy with isoniazid. What would
a. multidrug therapy. be an appropriate nursing response?
b. watchful waiting. a. Consult the prescriber and psychiatrist regard-
c. isoniazid. ing possible intramuscular administration of
d. rifampin. isoniazid.

Z 

 
$

 
616. A nurse is receiving isoniazid therapy for latent c. Withhold the drug and isolate the patient until
!–
#


$
  


 

he agrees to take the drug.
prescriber if the nurse experiences 


 Œ

 
 
$

a. frontal headache. others.


b. dry mouth.
c. heartburn at night. 22. The nurse knows that oral rifampin may not reach
d. persistent nausea. therapeutic levels if it is administered
a. on an empty stomach.
 17. The public health nurse would withhold adminis- b. with food.
tration of TB drug therapy with rifampin and pyra- c. with other drugs that are metabolized by the
zinamide and contact the prescriber if the patient’s hepatic cytochrome P450 enzymes.
laboratory test results include d. with protease inhibitors and nonnucleoside

{!

  
 '‰  
   
 
=++!# ?
$


 
 
†
 '‰ *#Š
$

–ˆ+
^^
 '‰

 
]
 '‰  23. A patient takes prophylactic warfarin because
$

 
$
 
  
!
 


18. The nurse should be particularly vigilant when recently been diagnosed with TB and is prescribed
monitoring intake and output and creatinine levels antimicrobial drugs, including rifampin. Because
when a patient is receiving which antitubercular of the interaction between warfarin and rifampin,
 ƒ
={

 
? it is important for the nurse to assess the patient
a. Amikacin for which symptom?

\   a. Abdominal pain
c. Kanamycin b. Bleeding

#  @ c. Change in mental status

{ d. Oliguria

> 
Ÿ
^`]—
Z 
#

 
 
228 CHAPTER 90 Q Antimycobacterial Agents

24. A patient has received instructions regarding TB 6†`



$
 
 
 
$



therapy with rifampin. Which of these statements a patient who is receiving clofazimine, is of great-
made by the patient would indicate the patient est priority to report to the prescriber?
needs further teaching? a. Darkening of the skin

’#

 

   
 
b. Diarrhea, nausea, and vomiting
use of my contact lenses.” c. Hyperactive bowel sounds in right upper

’#

 

  
$
#

quadrant and absent bowel sounds in left lower
reddish-colored urine.” quadrant
c. “My oral contraceptive birth control may not d. Red-tinged sputum and urine
work.”
d. “This drug can stain my contact lenses.”
DOSE CALCULATION QUESTIONS

^
{ 
%

$

 

-
tial to damage the eighth cranial nerve. The nurse 31. The nurse is preparing to administer rifampin
would assess for this damage by noting changes in 420 mg intravenously. The solution was prepared
 ƒ
={

 
? by dissolving 600 mg of powdered rifampin in
a. Balance ]`

$

 
{

$

-

Z  
  





``%‰
 

c. Facial movements =#Š?



$
œ
 
!
$ 




d. Hearing over 3 hours. What is the hourly rate of infusion



{ 
$
  

 

  



$ 

pump?
6^—
#

$
  


 

 



prescribed ethambutol to
a. call the prescriber if experiencing nausea.
†^
!
$
 
$
 
={
| ?

b. contact the prescriber if experiencing any visual is 10-20 mg/kg per day. An 88-lb child is pre-
changes.  

^`%
  


 
#

 

c. take the drug on an empty stomach with or safe?


without food.
d. take the drug with food if it causes upset stom-
ach. CASE STUDIES

^~
€
 
$
*  ‚
  
= ? Case Study 1
a. is curative.
b. is prophylactic. A 36-year-old woman who works as a social worker
c. is futile. and has sole custody of her two children is prescribed
d. is unnecessary. rifampin, isoniazid, ethambutol, and pyrazinamide after
being diagnosed with active TB.
28. The current recommendation for use of rifampin
$
*  ‚
  
= ?


 

1. Why is therapy for active infection always initi-
drug how often? ated with at least two drugs?
a. Once a day
b. Once a week
c. Once a month
d. Once a year
2. What information should the nurse provide to

^‡

 

\%—%|&



- ensure that these medications are taken exactly as
ing dapsone for leprosy, is at increased risk for prescribed?
a. destruction of red blood cells.



c. infection.
d. liver damage.
3. What obstacles might this patient face regarding
compliance with drug therapy?

> 
Ÿ
^`]—
Z 
#

 
 
CHAPTER 90 Q Antimycobacterial Agents 229

4. What will be necessary to determine the effective- 9. What organ function is the nurse most concerned
ness of drug therapy? about when a patient is taking this combination of
drugs? What symptoms would suggest this adverse
effect?

5. The public health nurse is consulted regarding


the need to identify all of the people who share
facilities with the patient to screen them for TB
]`
 



$

 
 

and prophylactically treat the individuals without =&}!?


$

 ƒ
active infection with isoniazid. Why is it essential
to treat TB contacts prophylactically for TB?

11. What interventions could the public nurse imple-


ment to improve compliance with drug therapy
6. When evaluating the contacts associated with an with this patient?
active TB patient, what considerations are made to
determine whether they are candidates for isonia-
zid prophylactic therapy?


]^
#$

 

$


*#Š% 
 

drug-drug interactions may occur between the TB



 


*#Š

 ƒ
7. What is the drug of choice as the primary agent for
treatment and prophylaxis of TB, and why is this
true?

13. The nurse is evaluating the patient when he comes


in for DOT. The patient reports vision changes in
which he feels like he is looking through a tun-
8. Two months into therapy, the nurse is review- nel. What could possibly be causing this visual
ing laboratory results for the original patient, change? What should the nurse do?


‰!
—
  
 '‰
{!

<`
 '‰
 
 
`†
 '‰
 

`<
 '‰



]^
 '‰
 
" -
tions should the nurse ask this patient?
14. The nurse is writing a letter to a legislator because
of proposed cuts to funding for health care that
would affect patients with TB. What could the
nurse include in the letter to justify the expense of
Case Study 2 the government providing assistance for people
who cannot afford to pay for therapy for TB?
A 45-year-old man who is homeless comes to the emer-
gency department with weight loss, lethargy, a low-grade
fever, and a productive cough streaked with blood. His
chest x-ray indicates a suspicious area in the middle right
lobe. He is hospitalized, and sputum cultures are ordered.
The sputum cultures reveal   . His active
TB is to be treated with a combination of drugs based on
the sputum culture drug sensitivity. The patient is started
on isoniazid, ethambutol, and pyrazinamide in the initial
phase of therapy.

> 
Ÿ
^`]—
Z 
#

 
 
230 CHAPTER 91 Q Miscellaneous Antibacterial Drugs

91
Miscellaneous Antibacterial Drugs: Fluoroquinolones, Metronidazole, Daptomycin,
Rifampin, Rifaximin, Bacitracin, and Polymyxins

STUDY QUESTIONS 8. All patients are at risk for developing tendon




  

"


Completion  
 

 
 ƒ
={

 

?

]

  
 
$$
$
"

a. Older adult patients
__________________ rupture. b. Patients who frequently take antacids for heart-
burn
2. Resistance has become common in c. Patients who are prescribed calcium supple-
Œ
$
 


ments for osteopenia
no longer recommended for this infection. d. Patients who are prescribed glucocorticoids
e. Patients who are post-solid organ transplanta-

†
> 
=>?


 $
 
$- tion
tions caused by __________________.
 9. Which situation would warrant assessing for
4. Fluoroquinolones pose a risk of candida infection when the nurse is caring for a
__________________. 6-month-old, breast-fed infant who is prescribed
 
$
 ƒ


!
 


$

$

- a. Parents supplement breast milk with rice cereal.
quinolone is the __________________. 
#$ 


$

$ 

#$ ‚


 



#$ ‚




 
CRITICAL THINKING, PRIORITIZATION,
AND DELEGATION QUESTIONS š
]`

 
  

 

250 mg twice a day for a 72-year-old woman with


6 —
!
 

 

 

-
ˆ!#
!
 

 

$
$ 

 


 



$
- 300 mg for anemia and calcium carbonate 400 mg
 
 
#






4 times a day for osteopenia. What should the
which ordered diagnostic test result as soon as it is nurse do?
available? 
 

 
]

$



>

 
=>²{? other medications.

€# 
 

 
^

$



ˆ   other medications.

%  c. Consult with the prescriber for directions.
d. Hold the ferrous sulfate and calcium during
6 7. The nurse assesses a 6-year-old child who is  
 

 
=>?
$

 

ˆ!#
 



ƒ š
]]
!
 

 
 
=>?

a. Abdominal pain to a patient who receives theophylline for asthma.


b. Fluid and electrolyte balance Because of potential drug interactions, the nurse
c. Nutrition should monitor theophylline levels and assess for

{
  a. constipation.
b. drowsiness.
c. tachycardia.
d. weakness.

> 
Ÿ
^`]—
Z 
#

 
 
CHAPTER 91 Q Miscellaneous Antibacterial Drugs 231

12. A common symptom of suprainfection that can  18. A nursing measure to prevent the most common



 
 
  
$$
$
 
=> ?


a. circumoral cyanosis. 

 
 




 -
b. high fever. tient’s diet.
c. pinpoint maculopapular rash. b. dim the lights in the patient’s room.
d. white patches in the mouth. 
 

 ‚


d. limit noise in the halls during the night.
š
]†

 

  
 

and should be reported to the prescriber of moxi-  19. Which isoenzyme of CPK would be most helpful
 
=?ƒ when monitoring for the most common serious
a. Confusion and irritability adverse effect if a patient is prescribed daptomycin
b. Constipation and weakness => ?

  
=¢?ƒ
c. Diarrhea and vomiting a. CPK
d. Tremor and numbness b. CPK-BB
c. CPK-MB
 ]‹


 

  

"
d. CPK-MM
 



  
 
$

˜!

 

Z>\

 



 20. The prescriber has asked the nurse to provide
electrolytes for which result that is most likely to teaching for a 34-year-old female patient who has
 

 
$

 
$$ƒ
={

" 

 
$
$ 
=$  ?

 
? before a trip to Central America. Teaching should

>

 
‡<
Z"'‰ include not administering the drug if the patient

€  

 
]†
Z"'‰ 
 
 ƒ
={

 
?

| 

 
†
Z"'‰ a. Amenorrhea

{

 
]†
Z"'‰ b. Bloody stool
c. Fever
15. Which goal is most appropriate when a patient is
d. Flatulence
  
 @
=… ?
$
/  
e. Nausea
=>&&?ƒ
a. Clear lung sounds
b. No burning on urination DOSE CALCULATION QUESTIONS

{$
$

d. Temperature within normal limits 21. The drug handbook states that the therapeutic
dose of metronidazole is 7.5 mg/kg every 6 hours.

]—
!
 


  % 

$




  

 @
=… ?

metronidazole in the patient’s medication drawer


``


]``

&5W every 6 hours. The child
and notes that it has been mixed to a concentration

‚—“

 
]‹
 
#

 
$

$-
$

 '‰
 


 
ƒ
fective?
a. Administer the drug.
b. Discard the solution and get a new bag from the
pharmacy.

^^

 
 
$
]

$
 @
=… -

&




^
 '‰

?



$

 

 
]~—
 

d. Return the bag to the pharmacy and request a


!

 


 ' 
#

 

refrigerated preparation.
safe?
 17. The nurse is preparing to administer daptomycin.
When should the nurse withhold the medication
and contact the prescriber?

#$

 

  

€{
$

CASE STUDIES
skin.

#$

 ‚
#+

]< Case Study 1

#$

 



 
A 68-year-old female patient returns to her prescriber
pain. after taking 10 days of ampicillin for her upper respira-

#$
 

 
 

 



$
{



%  
$


ordered. 



 

{




cough and thick secretions. Her x-ray does not indicate


 
!
 


 
$
 
  

> 
Ÿ
^`]—
Z 
#

 
 
232 CHAPTER 92 Q Antifungal Agents

$


 

  
{


- Case Study 2
 
^`
 
 
^


 
$
~

  

The patient’s nurse wants to be sure she understands this A 15-year-old male patient is admitted to the nursing unit
medication. after emergency surgery for a ruptured appendix. He is
  

 @
=… ?
``


]``

1. What in the patient’s history may contribute to D5



—


'   
=ˆ ?

potential drug-drug interactions? ]



=
`

+{{?

<
 
–





 
$
]``




5. Metronidazole is scheduled at 0000-0600-1200-


]<``


 
$
]``
‰'
'   

2. What foods should the patient avoid when taking is scheduled at 0600-1400-2200. Which would the
 
 ƒ  
 
 

`—``





nurse start each drug?

†
!
 
 

  ‚
$

^
 



 




{


6. What procedures need to be followed when ad-
she can stop taking the medication since it is so ministering these drugs?
expensive and she is improving so much. How
should the nurse respond?

7. Because the family delayed seeking treatment for



‚
 
 

\#



4. What new problem might the patient develop expelled into the peritoneum, the nurse would
while taking this antibiotic? monitor for what symptoms that suggest the bacte-
ria have invaded the retroperitoneal region?

92
Antifungal Agents

STUDY QUESTIONS True or False


   
  T   F
Completion   
1. The term for fungal disease is
_______________________. 4. ___ Amphotericin B can bind to cholesterol in
human cell membranes causing toxicity.
2. )   means _______________________. 5. ___ Amphotericin B damages the kidneys.
6. ___ Amphotericin B doses must be decreased
3. ) 
 means _______________ if initial creatinine clearance is less than
_______________ _______________. ‹`
‰'
7. ___ Amphotericin B is rarely associated with
fungal resistance.

Copyright © 2016, Elsevier Inc. All Rights Reserved.


CHAPTER 92 Q Antifungal Agents 233

8. ___ Amphotericin B has broad-spectrum bacteri- 20. How does dantrolene relieve rigors and fever as-
cidal activity. sociated with amphotericin B infusion?
9. ___ Amphotericin B is only used for systemic a. Blocking the action of amphotericin B
fungal infections. b. Counteracting an allergic reaction
10. ___ Amphotericin B is the best drug for systemic c. Decreasing the set point of temperature in the
fungal infections despite the potential for hypothalamus
toxicity. d. Relaxing muscles and preventing shivering
11. ___ Amphotericin B readily penetrates the cen-
 

 
=>+{? 621. The nurse is reviewing laboratory tests before
12. ___ Amphotericin B remains in human tissue preparing to administer a dose of amphotericin B.
more than a year after treatment is discontin- Before administering the drug, it is a priority to
ued. review which lab result?
a. Creatinine levels
Matching 
Z>\

‰
$
 
      
d. WBC count
13. ___ Body
 22. Which laboratory results of an adult male patient
14. ___ Foot
suggest bone marrow suppression caused by am-

]

\
photericin B?

]—

{ 

*  
]`
'‰Œ
 
†`œŒ
€>Š

110 mcg3Œ
€>*
^<
 '
a. Tinea capitis

*  
]`
'‰Œ
 
†`œŒ
€>Š

b. Tinea corporis
67 mcg3Œ
€>*
†`
 '
c. Tinea cruris

*  
]`
'‰Œ
 
†`œŒ
€>Š

d. Tinea pedis
89 mcg3Œ
€>*
^<
 '

*  
]`
'‰Œ
 
†`œŒ
€>Š

CRITICAL THINKING, PRIORITIZATION, 105 mcg3Œ


€>*
^`
 '
AND DELEGATION QUESTIONS
23. A patient with a systemic fungal infection is pre-
 17. When administering drugs that are potentially  

–

 
!
 

nephrotoxic, the nurse should consult the pre- expects what drug interaction?
scriber before administering which over-the- a. Decreased risk of amphotericin toxicity

=}!>?
 = ?ƒ 
# 
 
$
 
  
a. Antacids 
# 
 
$
 
b. Acetaminophen d. Needing a higher dose of amphotericin B

+{#&

‰  
^‹
 


$
  @
={ ?ƒ

#

 
 
  
=#Š?
18. At what point is the patient most likely to experi- 
#

  
$
 

  
$

ence fever, chills, rigors, nausea, and headache other drugs.


when receiving amphotericin B? 
#

$$
 

$ 
 
-

# 
$

$ 
  tericin.
b. 20-30 minutes after the infusion begins 
#


c. 1-3 hours after the infusion begins
d. 3-6 hours after the infusion begins 25. To achieve maximum absorption of itraconazole
={ ?
   

 
 


 19. A patient experiences sudden episodes of shaking drug with which liquid?
chills after receiving a dose of amphotericin B. a. Apple juice
Which drug, ordered as-needed for this patient, b. Milk
should the nurse administer at this time? c. Cola

 
=!? d. Water

& 
=& ?

& 
=– ?

‰ @ 
= ?

Copyright © 2016, Elsevier Inc. All Rights Reserved.


234 CHAPTER 92 Q Antifungal Agents

626. The nurse is reviewing test results for a patient 6†^



 

  
  @
=+ ?
$



  
  @
={ ?

 
= 
   ?
 

$
 -
result would warrant immediate consultation with 
$



  
#


the prescriber? be of greatest priority to promptly inform the



{!
†
  
 '‰ prescriber if drug reconciliation reveals that the

–+|
~‹
 '‰  
 

 = ?
$
 
   ƒ

Z   ž”
$ 
—`œ 
 
 
 $$
=Z


| 
†<
Z"'‰ €  ?

&  
=€   ?
 ^~
!
 

 
  @
={- 
# $
=€
#–?
 ?


 

 

  
 - 
{  
=#?
 
=¢?
–  
$

$$
$
  @

on hepatic isoenzyme CYP3A4, the combination


††
 


 


 




increases the risk of rhabdomyolysis from the  



 
 $ 
=>  ?ƒ
-statin drug. The nurse should monitor the patient 
 



for what symptoms? b. Check the dose provided with the dose ordered.
a. Bleeding and bruising c. Reconstitute the drug in sterile saline.
b. Hypoglycemia and tremors 
|


  
c. Hypotension and dizziness
d. Muscle pain and dark urine  34. The nurse would be concerned that a patient who


 $ 
=€ ?

-
28. Which drug may prevent absorption of itracon- ing a histamine reaction and might have an ana-
@
={ ?

 



 - phylactic reaction if the patient reports a sudden
tered? feeling of

> 
={ ? a. bloating.

& 
=‰ ? b. headache.

Z  @
=+? c. itching.

 
=¢  ? d. nausea.
6^‡
#









in a patient who is prescribed the -azole antifungal 35. The nurse reviews the CBC of a patient who is

 @
=& ?ƒ   
 
= ?


-
a. Abdominal pain and diarrhea trophils 27%. This patient is especially at risk for
b. Fever and blisters in the mouth which issue?
c. Headache and photophobia a. Fatigue
d. Nausea and vomiting 
…



# 

 
 30. The nurse reviews the laboratory results of a 
#$
female patient who is prescribed voriconazole
=Š$?
^``




  
36. The medication administration record lists a
result warrants withholding the drug and immedi- clotrimazole troche to be administered at 0900.
ately notifying the prescriber? The nurse administers this medication by

{!
†~
  
 '‰ a. applying it to the skin.

…–\
^^^
 '‰ b. dissolving it in 8 ounces of water.

*
]]<
'‰ c. instructing the patient to chew it.

>\
]~^
  
 '‰ d. instructing the patient to let it dissolve in the
 31. A patient who has been receiving ketoconazole mouth.
for 5 days experiences nausea and vomiting. What
should be the initial response of the nurse?
a. Administer the medication with food.
b. Assess the skin, abdomen, urine, and stool.
c. Consult the prescriber.
d. Withhold the medication.

Copyright © 2016, Elsevier Inc. All Rights Reserved.


CHAPTER 92 Q Antifungal Agents 235

 37. A postoperative patient who had knee replacement 2. The treatment of choice for this patient’s fun-
 

  
‰



$- 
$

 @
#

  
 

mation related to immobility. The patient develops extreme care be taken in providing nursing care
candidiasis under the breasts and in the groin. The to this patient because she is at high risk for ad-
prescriber orders miconazole cream. What should ditional infections that could now be lethal. The
the nurse do? nurse must consider many areas when treatment

 


‰

 @
includes combining the treatment for fungus with


#+ the numerous other physical problems a patient

 

‰

 

- 
€‰
$  
 
  
 
$$

scriber regarding the miconazole. of intravaginal clotrimazole vaginal suppositories


c. Administer the miconazole but consult the should the nurse instruct this patient to report?
  
 

‰




‰

 @

and consult the prescriber.

38. What is a potential problem of patients’ self- 3. The patient asks why she must use the vaginal
prescribing OTC miconazole for vaginal dis-  


 



$
-
charge? motherapy administration. What should the nurse

‰ 
  tell her?

{ 

c. The cause may not be a yeast infection
d. The treatment is usually not effective

4. The patient’s fungal infection has advanced be-


DOSE CALCULATION QUESTIONS cause of her severely immunocompromised state.
{


 
 
"
-
39. The nurse is preparing to administer anidulafungin tericin B for treatment. All of her medications
=Z  ?
]``




 

   
must be carefully evaluated to determine whether
candidiasis. The drug books states a 100-mg vial they are toxic. What system is almost always

 

†`


 


damaged in some way by amphotericin B and can
 
$
†††
 '‰

$

increase the risk of toxicity of other drugs?

]``

$
`‡œ
 

!
$ 





‡`
 
!










 
‰'ƒ

5. What intervention can the nurse employ to reduce


40. The drug book recommends after a successful test the risk of kidney damage by amphotericin B?
 
 

 
 
$

–


be 0.25 mg/kg of drug. The patient weighs 110 lb.


!
  


]^
 
#


$

dose?

—



 
$

  

 



]%
 
 
$

–
=… @?

CASE STUDIES before starting full therapy?

Case Study 1
A 25-year-old female patient was recently diagnosed with

 

=€‰?
{


$
7. Amphotericin B vial is supplied as a powder for
the induction phase of chemotherapy. Her platelet count reconstitution using sterile water that does not
is low, and she has vaginal candidiasis. OTC topical treat- contain a bacteriostatic agent. Why must the drug
ments for vaginal candidiasis are available. be mixed in sterile water without a bacteriostatic
agent?
1. What problems might this produce?

Copyright © 2016, Elsevier Inc. All Rights Reserved.


236 CHAPTER 93 Q
 
 

&
$
+%*#Š
Š 
#$

8. What nursing measures should the nurse take 9. What teaching does the nurse need to provide this
when infusing amphotericin to prevent infusion- patient regarding drug therapy?
related adverse effects?

10. Why is this patient more at risk for hepatotoxicity


Case Study 2 and drug interactions than the average person?
A 42-year-old male truck driver has been prescribed vori-
 @
=Š$?
$
   
  
 




  

*#Š

93
Antiviral Agents I: Drugs for Non-HIV Viral Infections

STUDY QUESTIONS 9. ___ Famciclovir decreases the number of epi-


sodes of postherpetic neuralgia associated
True or False 

@ 
=   ?

]`

|
=& ?

 

   


  T   F =  ?
 
  
 


   duration of pain of cold sores by 50%.

]

#

 


 

 


not harm human tissue. CRITICAL THINKING, PRIORITIZATION,


2. ___ Oral acyclovir will cure active herpes labia- AND DELEGATION QUESTIONS

=
?
3. ___ Oral therapy with acyclovir is only effective 11. A college student who has been diagnosed with
$
 
=?
$
 

 

 
 

$

  


within 24 hours of rash onset. disorder and prescribed topical acyclovir treatment

‹




 


with the college health center nurse. Which state-


  

$



 
ment, if made by the patient, would suggest that
should be well-hydrated. the patient needs teaching about this disorder and



#

  

 

$

drug therapy?


=¢ ?
 
’#
 


$$
$



$

6. ___ Research suggests valacyclovir eliminates #




 $

 
$
 

the risk of transmitting genital herpes be- bowel movement.”


tween monogamous heterosexual partners. 
’#

 




-

~

#

 

 

 


ment, immediately dispose of them, then wash
prescribed valacyclovir to report unexpected 
 
  
#
 
 

$


bruising. other parts of my body.”


8. ___ Valacyclovir should not be administered to 
’#


 




 “
a person who is also prescribed a drug that 
’ˆ 





#





causes bone marrow suppression. stop the outbreak.”

Copyright © 2016, Elsevier Inc. All Rights Reserved.


CHAPTER 93 Q
 
 

&
$
+%*#Š
Š 
#$ 237

 12. What should the nurse teach regarding condom  17. The nurse would withhold ganciclovir and notify
use to a patient who is currently in a long-term the prescriber if patient laboratory test results
committed monogamous relationship and who has included
been prescribed continuous oral acyclovir therapy 
–ˆ+
^`
 '‰
for recurrent genital herpes? 
>\
^~~
#ˆ'‰
a. Condoms prevent the spread of infection, even c. neutrophils 2000/mm3.
during active outbreaks. d. platelets 75,000/mm3.

ˆ 


$


$
  


present.  18. The nurse is discussing planned care with the



ˆ 




 
 

CNA who is assigned to a patient receiving
$


 Œ



ganciclovir. Because of possible serious adverse
planned when symptoms are absent. effects, which precaution would the nurse include
d. Your partner is already infected, so condom use in instructions regarding care?
does not matter. a. Do not provide any food 1 hour before or
2 hours after drug administration.
613. Which action by the nurse would be a priority for 
Z 

$

  
preventing the most common complications of c. Turn and reposition the patient every 2 hours.
 
=#Š?

=¢ ?
 ƒ 
ˆ 


 @

 

 

  



$
$ 



Z 


 
$
^``

†```
 19. Which step is most important for a female nurse to
‰'^‹
 follow when administering valganciclovir?
c. Reporting vomiting to the prescriber a. Administer with the patient in an upright posi-
d. Teaching perineal hygiene tion.
b. Assess vital signs before administering the
614. The nurse is preparing to administer intravenous drug.

#




$

 

c. Avoid touching the drug.
provide nursing interventions for which assess- d. Dissolve the drug completely in 8 ounces of

 ƒ water or juice.
a. BP 150/85 mm Hg

> 
`‡
 '‰  ^`
‰  
 
$

 


  

c. Dry, sticky oral mucous membranes $  


=…  ?

…–{
‡`
 '‰
{!


Z %


~`
‰ <~
 '‰
 
†—
Z"'‰
 
<‹
 '
‰

  
^
Z"'‰
!
 


 ]
#







  
assess for what symptom of electrolyte imbalance?
result to the prescriber if a patient is prescribed a. Cool, clammy skin
  
=Š ?ƒ b. Headache
a. CD4 less than 100/mm3 c. Muscle spasms

*
=* ?
]]
'‰ d. Thirst
c. Platelets 220,000/mm3
d. WBC 12,800/mm3 21. When explaining prescribed pegylated interferon
= $?

 




16. The nurse is preparing to administer famciclovir information about the drug that makes it different
=… ?
``

  
 


 
from conventional interferon?
to a patient admitted with exacerbated chronic 
#
 

 
 
 
 
  
=>}|&?



#

  

 
developed shingles. The nurse reviews laboratory 
#

$
 
$$ 
 
 
 

\…
†
‰'
 

#


  
 


should the nurse do at this time? levels.


a. Administer the drug as ordered.

> 

  
{!!
 

 ^^

 
$
=*  ?


of doses. assessment would suggest that therapy could be



> 

  
{!!
 


toxic?
for more laboratory tests. a. Decreased sperm count
d. Withhold the drug and consult the prescriber b. Nausea
during her next hospital rounds. c. Oliguria
d. Yellow-colored sclera

Copyright © 2016, Elsevier Inc. All Rights Reserved.


238 CHAPTER 93 Q
 
 

&
$
+%*#Š
Š 
#$

^†
#

 
$

 


$
 

^<
#


  
 

 = ?

ideation if a patient is prescribed which drug? 





‰#Š
=…€ ?
  



#$
$ is a live attenuated virus and is administered by

‰   


  
 ƒ
={

 

c. Ribavirin ?
d. Valacyclovir a. Children or adolescents receiving aspirin
therapy
 ^‹

 



$
  
b. Children younger than 5 years of age who have





  
$



had recurrent episodes of wheezing with respi-
 



 
=Z
*–Š?
ratory infections
$
=*  ?

 
=–  ?
$

|


*#Š% 
hepatitis B because it suggests possible lactic d. People prescribed disease-modifying antirheu-
acidosis? matic drugs for rheumatoid arthritis
a. Deep, rapid breathing e. People with asthma
b. Fever
c. Poor appetite
^‡
!
 

 
 

 
  
$ 

d. Nausea in determining the effectiveness of oseltamivir


=! ?

 
@

 ^

 

  
$
=*  ?
]`

a. administration with food.
by mouth once a day. Results of recent laboratory 
@



  
  
 

>>
—^
‰'
 
 
- c. starting therapy as soon as symptoms occur.
tion is warranted? 
  
$
@

 


a. Administer the medication as prescribed. administered.


b. Consult the prescriber regarding the need for
dose reduction.  †`


 



 

c. Withhold the medication and contact the pre-   


$



$ 




scriber immediately.  


 
$
{Šƒ
d. Request dosing be changed to every other day. 
>  

=   ?


 
b. Pulse 120 beats/min
26. The nurse is reviewing the chart of a patient c. Respirations 30 breaths per minute
  
 
=!@ ?
!
 


@
 


consult the prescriber if what is noted?

‰!
 
$
^<~

†`
#ˆ'‰


 

year DOSE CALCULATION QUESTIONS



Z
$

*–Š
 
c. HBV, acute episode, recently diagnosed 31. The recommended dose of acyclovir for immuno-
d. Patient is 17 years old competent children under 12 years is 15-20 mg/kg/
day divided in three doses. What is the safe dose
š
^~
!
 




 - range for a child who weighs 44 pounds?



 


 ƒ
a. 7-year-old boy with a seizure disorder
b. 12-year-old girl receiving aspirin for its cardio- 32. Foscarnet 7.2 grams is prescribed. The drug must
protective effects 





$




$ 

c. 28-year-old man with type 1 diabetes 



 

ƒ
d. 38-year-old woman with multiple sclerosis

CASE STUDIES
Case Study 1
A 32-year-old female patient with chronic HCV has just
had boceprevir added to therapy with interferon alfa and
ribavirin.

1. What instructions should the nurse provide regard-


ing administration of this drug?

Copyright © 2016, Elsevier Inc. All Rights Reserved.


CHAPTER 94 Q
 
 
##•
&
$
*#Š
#$

 
} 
#$ 239

2. Why is it important to discuss sexual activity with 3. How should the nurse respond?
this patient?

4. What intervention would be most important for the


Case Study 2 school nurse to emphasize when discussing with
the teacher prevention of becoming infected with
The public health nurse receives a call from a school
@
$

 


 ƒ
nurse questioning whether a child who received intranasal
‰#Š
=…€ ?




!
 

-
cerned because the child’s teacher is pregnant.

94
Antiviral Agents II: Drugs for HIV Infection and Related Opportunistic Infections

STUDY QUESTIONS CRITICAL THINKING, PRIORITIZATION,


AND DELEGATION QUESTIONS
Matching
   

  7. The nurse is teaching a patient about the need for
%
 
$
*#Š

 

]

Z$ @ if made by the patient, would indicate a need for

^

Z$ further teaching?
3. ___ Maraviroc 
’#$
#
‚
 

$



  

4. ___ Raltegravir the virus is more likely to become resistant to


5. ___ Ritonavir drugs.”

—

¢ b. “The higher the number of viruses in my body,
the greater the chance one will become resis-
a. Binds with CCR5, thereby blocking viral tant.”
entry into CD4 cells. c. “The virus recognizes the antibiotic and is able

|

*#Š

$
$ 
to change to prevent being destroyed.”
with the cell membrane of CD4 cells, 
’

*#Š

  
+

&+

thereby blocking viral entry and replica- within the host cell, genetic changes can occur
tion. spontaneously.”

{ 
 
$
 
&+

thereby blocking growth of the viral 8. The prescriber has informed a patient who has
DNA strand. been prescribed a nucleoside/nucleotide reverse

–


*#Š
 
  -    
 
=+!#?
$

 
$
-
tase disrupting the active center of the patic steatosis. The patient asks the nurse “What is
enzyme, thereby suppressing enzyme hepatic steatosis?” The nurse should explain that
activity. this is a possible severe adverse effect involving

|
 
$
*#Š
&+

a. fatty degeneration of the liver.
material into the DNA of CD4 cells, b. increased secretions of glands.
 

*#Š
  c. infection of the liver.
$
# 
>™|†‹
  
 


 

$ 

 
raising lopinavir levels and enhancing
antiviral actions.

Copyright © 2016, Elsevier Inc. All Rights Reserved.


240 CHAPTER 94 Q
 
 
##•
&
$
*#Š
#$

 
} 
#$

6 ‡

*#Š% 
 

  
 ]


 



 
=Š-
trimethoprim-sulfamethoxazole for a urinary tract  ?

 
= ?
 
$

$
=ˆ!#?


 



conjunctivitis or muscle and joint pain, it would be


  
 = ?ƒ
={

 
a priority to assess for
? a. dizziness.

{!

‰! b. nausea.

–ˆ+

  c. paresthesias.
c. CBC and differential d. rash.

Z

‰  ]—

*#Š% 
 





$
|!

#+   
*!


++!#
$ -
@
 

  ‚
$


-
10. A patient has received instructions regarding riencing drowsiness and dizziness. An appropriate
  
$
  
=Š
Z>?

recommendation by the telephone triage nurse is
statement made by the patient would indicate the for the patient to do what?
patient understood the directions? a. Discontinue all drugs and make an appointment

’#
 




 “ to be seen as soon as possible.

’#


$

  
$
#
 
 - b. Discontinue taking efavirenz, but continue the
low this capsule whole.” other drugs in the regimen.

’#








ˆ 
$
 

 
$ @



 

#
 

 


bedtime.
morning.” 
ˆ 
$
 

 
$ @



’#

 



  “ food.

 ]]

 
 



 
17. Weight-bearing exercise and adequate calcium
  
$

 


  ƒ intake are most important if HAART includes a
a. Flatulence a. fusion inhibitor.
b. Headache b. nucleoside/nucleotide reverse transcriptase

#  
=+!#?
d. Vomiting c. nonnucleoside reverse transcriptase inhibitor
=++!#?
612. A patient who is prescribed highly active antiret- 
 
 
=|#?
 
 
=*!?



reports loss of interest in usual activities. Which 18. The nurse would be most concerned that a patient
nursing action is of greatest priority? may not adhere to HAART therapy with which

 

$


class of antiviral drugs if the patient verbalized
therapy. that body image is an important priority?
b. Assessing for thoughts of harming self. a. Fusion inhibitor

#$
 
$

$
  b. Nucleoside/nucleotide reverse transcriptase
d. Providing a safe environment.  
=+!#?
c. Nonnucleoside reverse transcriptase inhibitor
 13. A patient receiving HAART develops a sore throat =++!#?
and cough. The nurse should consult the prescriber 
| 
 
=|#?
immediately if the therapy includes which drug?

  
=¢ ?
]‡
!
 

 
 

*#Š%  
-

&  
=Š? munity often shares the belief that garlic supple-

{ 
=¢?  
 
*#Š


 


¢
=? the immune system. This belief can be particularly
dangerous if the patient is prescribed which drug?
14. When planning nursing interventions and teaching

# 
for the most common adverse effects of efavirenz

‰ 
={  ?

 
 





+ 
focus for the nurse?

{ " 
a. Circulation
b. Nutrition

{ $

{
 

Copyright © 2016, Elsevier Inc. All Rights Reserved.


CHAPTER 94 Q
 
 
##•
&
$
*#Š
#$

 
} 
#$ 241

20. Patients who are prescribed protease inhibitors  26. Which action, when administering enfuvirtide
=|#?


 


$%  

=…@?

 

 
$



 

{
 ‚

 


 - injection-site reaction?
sible effect of this combination? 
#”





 

# 

  


$

|# b. Not adequately cleaning the injection site.

# 


$
 
  c. Reconstituting the drug in sterile water.

# 


$

$

|# d. Refrigerating the solution for 2 hours after

€}
 
$$
$

{
 ‚
 reconstituting.

 21. What is not a known reaction if a patient who is


^~

*#Š% 
 



*!

taking the oral solution of lopinavir/ritonavir is 


  
={@?




 
  
 @
=… ?ƒ nurse that he has vomited and is experiencing

&$
  severe abdominal pain. Assessment reveals a gen-
b. Headache eralized pruritic rash. Which action is appropriate?
c. Hypertension 
}  


$
 
=– -
d. Nausea and vomiting ?
$

 
b. Withhold all prescribed drugs and contact the
 ^^

  @
*#Š% 
 


- prescriber.
 
 
=> ?
 
$
 



$





$


 
 
#


$


 
 


prescriber, which nursing intervention would be 


  
={@?

 

most appropriate? the prescriber.


a. Assess bowel sounds.

Z 

 
$

 28. What is the most important role of the nurse when

# 
Š{


‹
  treatment failure occurs?

{ 
 
#$
$ 
 
 
 
 


treatment failure
 23. Which drug must be taken 12 hours apart from 
#$

 ‚

 
rilpivirine? 
{

 
 

–$$

=–$$? d. Teaching the new drug regimen

> 
   
=!ˆ€{?

} @
=| ?
^‡
|
$
 
*#Š%  
  


 
=¢  ? 

={

 
?
a. prophylaxis is recommended for the infant/neo-
24. What are known adverse effects of raltegravir nate.
=#  ?ƒ
={

 
? 

$
 
 
 
  
a. Facial edema and angioedema c. HAART should not be used during pregnancy.
b. Flulike symptoms d. the drugs are not teratogenic.

‰
$

  e the drug regimen should only include one drug
d. Painful rash that blisters followed by shedding at a time.
of the epidermis
e. Teratogenesis  †`



 
$

*#Š%  

patient, the nurse splashes urine on intact skin.


6^
#


$
  


 

What should the nurse do?
  
$
$
=…@?
$

 

\


 
 
=Z&?



experienced end of the shift for bloodwork.


a. headache. 
# 



Z&
$
 
$
 -
b. pain and tenderness at injection site.  
 
=|Z|?
c. positive pregnancy test. c. Make an appointment with a primary care
d. muscle weakness in legs and arms. provider to discuss if postexposure prophylaxis
=|Z|?

 
d. Report the incident immediately to a supervisor.

Copyright © 2016, Elsevier Inc. All Rights Reserved.


242 CHAPTER 95 Q
&
! 
$
{ 
!  
&  

 †]

 


 
 
 

]
 
 
$
*#Š




-
HAART is currently effective? ring?
a. CD4 T count 400 cells/mm3
b. Neutrophils 2750/mm3

| 
*#Š
+

^`%~
 '‰
d. WBC 5000/mm3

32. Current recommendations state that antiretroviral 2. The patient is diagnosed with CMV retinitis. The
therapy for patients with chronic asymptomatic treatment of choice for CMV retinitis caused by
*#Š
  

 


>&‹

 
=>€Š?


 

drops below what level? concern does the nurse have about the addition of
a. 200 cells/mm3 ganciclovir to a drug regimen including zidovu-
b. 350 cells/mm3 dine?
c. 500 cells/mm3
d. 850 cells/mm3

DOSE CALCULATION QUESTIONS


3. The prescriber wants to begin with intravenous

††

@
]^`






‹
 '‰
therapy. What strategies can the nurse employ
How much 5% dextrose for injection should be when administering intravenous ganciclovir to
added to the drug? reduce the risk of and identify possible damage to
the patient’s renal system?
34. What is the recommended dose of enfuvirtide
=…@?
$

<% %


 

44 lb?

CASE STUDY 4. The patient tells the nurse that he has no job or
insurance at present. What would the nurse be

†% %
 
 


*#Š%  
$
concerned about?
12 years. His HAART includes zidovudine. When being
interviewed by the nurse during a routine clinic appoint-
ment, he complains of declining vision, headaches, and
 
 
  
|  
  
-
ings include multiple enlarged lymph nodes and several
white retinal patches.

95
Drug Therapy of Sexually Transmitted Diseases

STUDY QUESTIONS 2. ___ Annual screening for /


  
-
tis is recommended for all sexually active
True or False women younger than 25 years.
   
  T   F
  
1. ___ More than 1 million cases of /
 
 



ˆ{
 
 



reported/treated.

Copyright © 2016, Elsevier Inc. All Rights Reserved.


CHAPTER 95 Q
&
! 
$
{ 
!  
&   243

CRITICAL THINKING, PRIORITIZATION, 6 8. The throat culture of a 6-year-old boy is positive


AND DELEGATION QUESTIONS for &   . What is the nursing
priority in this situation?
3. Where does the nurse obtain the most accurate in- a. Assessing for eye exudate.
formation about nursing care of sexually transmit- b. Calculating the dose of ceftriaxone to be ad-

   '$
={!& '{!# ?ƒ ministered.
a. The Centers for Disease Control and Prevention 
Z 

$
$


b. Drug manufacturers d. Removing the child from his home.

!
ˆ{
…

&
  
d. Textbooks 9. A patient is admitted to the hospital with a diagno-

$

  
  
=|#&?
{


6 4. A neonate was delivered vaginally to a mother the nurse why her prescriber has recommended
with an active /
  
 infection.   @ 

 
=#Š?
 

Relating to this exposure, what is the priority instead of oral antibiotic therapy at home. What is
system when caring for the neonate? the basis of the nurse’s response?

\    
# " 
 
|#&





b. Ophthalmic   
 
$


=$  ?
  
c. Renal 
|#&


  


 



d. Respiratory be in isolation.

!

 
$
|#&

 -
5. Why would doxycycline not be prescribed for 




a female patient for whom pregnancy status is 


 
|#&
 

 


unknown? of cervical cancer.



#
 
  
$ 
 

#
  
    10. A 32-year-old woman develops septic arthritis.

#

  When obtaining a history, it is important to evalu-

#


$$
^œ
$

 ate for which previous infection?
a. /
  

 6. A friend tells the nurse that a sexual partner has b. &   
$

 


 
{


c. . 
 

 
 
$
 
=>?
$
d. . 
  

  
{


 



that she has used this drug in the past for gonor- 611. The nurse is caring for a neonate whose mother
rhea. What is the most important reason why the has an active infection with &   -
 

  
 

 ƒ  A priority nursing outcome for the neonate
a. The nurse’s friend is likely to develop an al- relating to possible infection during vaginal deliv-
 
 

 
=>? ery is that the neonate will
b. The law requires reporting of gonorrhea infec- a. blink in response to direct light shone in the
tions. eyes.
c. This drug is no longer effective against Neis- b. breast-feed for 10-15 minutes at least every 3
  . hours without becoming dyspneic.
d. The dose is too low to treat gonorrhea. c. have clear breath sounds.
d. not experience conjunctival discharge.
 ~

 



   
=#€?
$- 12. The recommended treatment for acute epididymi-
triaxone 1 g once a day for disseminated gonococ- tis is different depending on the age of the patient
cal infection complains of a stiff neck. Which of because usually
 
 



$
 ƒ a. older men have developed a resistance to the
a. Assess for history of arthritis. antibiotic used for younger men.
b. Complete the nursing assessment. b. the method and organism of infection is differ-
c. Consult the prescriber. ent.

&

 
c. younger men have better kidney functioning.
d. younger men are less likely to adhere to multi-
dose therapy.

Copyright © 2016, Elsevier Inc. All Rights Reserved.


244 CHAPTER 95 Q
&
! 
$
{ 
!  
&  

 13. The nurse teaches that the primary cause of recur- CASE STUDY

{!#
  



  

$ 

to A sexually active 19-year-old woman presents to the


a. prescribe the correct antibiotic. emergency department with a 2-week history of dull bi-
b. seek treatment early in the disease. lateral abdominal pain, low back pain, and mucopurulent
c. take the medication as prescribed.   
   

 
 
 
{


d. treat the sexual partner. fever, nausea, vomiting, diarrhea, and urinary symptoms.
14. A 35-year-old woman reports a yellow-green {

  




  
*
 

  
   
{

  

-   


^


{


 


 

  
 @
=… ?
#
 
$


{

 


 

is important for the nurse to teach the patient the department 3 months ago for an “infection” and states that
importance of not consuming she took the medication when she remembered and did
a. alcohol. 

$

$%
  
{

 


b. antacids. sexual partners in the past 6 months, one of whom was


c. grapefruit juice. also treated for an infection 3 months ago at the health
d. milk.  
{

 






his treatment plan or whether he is symptomatic at this

]


 
 
=> ?
"

{



 
 

 


only one dose vaginally to treat bacterial vagino- medication on a regular basis.
sis? Her vital signs are: BP 110/72 mm Hg, pulse 88 beats/
a. Bacterial vaginosis is not a serious infection. min, respirations 20 /min, and temperature 99.4° F. The
b. Clindamycin has a half-life of several days. physician orders a CBC, sedimentation rate, rapid plasma
c. The cream adheres to the vaginal mucosa for reagin, serum pregnancy, and a catheterized urinalysis.
several days. The physician performs abdominal and pelvic examina-
d. The medication immediately kills the infecting tions and obtains cervical cultures for gonorrhea and
microbes. chlamydia and specimens for saline and KOH wet preps.
The physical examination reveals a soft abdomen, right
16. What is a major goal of therapy with famciclovir
and left lower quadrant tenderness without rebound, and
=… ?
$
 
 ƒ
normal bowel sounds. The pelvic exam reveals mucopu-
a. Analgesia
rulent vaginal discharge and mild right and left adnexal
b. Decreasing length of active episodes
tenderness with bimanual exam. The laboratory results

Z  
$


reveal Hgb 11.1, WBC 7.0, and negative results for uri-
d. Prevention of superinfections during outbreaks
nalysis and pregnancy tests.
After review of the medical history, physical exam, and
DOSE CALCULATION QUESTIONS   
 
=

 ?

  

 

  
$
|#&
}

 

17. A 7-lb neonate develops /


  
 $ 
^`




 
$
-
pneumonia following intrapartal exposure. The cline 100 mg twice a day for 14 days, metronidazole 500
recommended safe dose of erythromycin succinate mg twice a day for 14 days, and Tylenol #3 1 tablet every
for neonates is 50 mg/kg/day divided into 4 doses. 3-4 hours as needed for pain. The patient is told that she
What is the safe dose of this drug? 


 

 
$

 

1. Why is it important to determine whether the pa-

]<
>$ 
]


]^
 

  

tient is pregnant before administering doxycycline
for a patient with disseminated gonococcal infec- or tetracycline?

=&\#?
#

  




`

of D5·
+{{


 

†`
 

 


 
$
$ 

‰'ƒ

Copyright © 2016, Elsevier Inc. All Rights Reserved.


CHAPTER 96 Q Antiseptics and Disinfectants 245

2. The patient is to be observed for 30 minutes after 4. The patient is instructed to inform her sexual
administration of the cephalosporin injection and partners of the need for examination. The pa-

   

{


 
‚

 




$

for bedrest for 2 days with a reexamination in  


  
{

 
 
 

24-48 hours if the signs and/or symptoms increase the culture results will be reported to the health
or persist. What teaching regarding the prescribed department if they are positive, and the health de-
medication and follow-up care should the nurse partment will then follow up with her. The patient
provide to this patient? states that she does not understand why she has
to inform her sexual partners of her infection and
why she has to take all of the medication if she is
feeling better in a couple of days. What informa-
tion and instructions can the nurse provide to the
3. Why is it important for the patient to avoid all patient to help her understand the importance of
forms of alcohol? adherence to her treatment plan?

5. After completing the teaching, the patient’s verbal-


@ 

 




 @


  
$

$

 
$

adhering to therapy. How might this alter the plan


of care for this patient?

96
Antiseptics and Disinfectants

STUDY QUESTIONS a. Antiseptic


b. Disinfectant
Matching 
\ 

\  

       e. Prophylactic
$
{ @ 
1. ___ Anything used to prevent an infection or 
{@ 
disease
2. ___ Harsh chemical used to decontaminate ob- True or False
jects
3. ___ Kills germs #

 

 

% 
 

  

4. ___ Prevents germs from reproducing the hands instead of soap and water in which of the fol-
5. ___ Reduction of contamination to a level com- lowing situations?
patible with public health standards
6. ___ Anti-infective cleanser that can be applied to 8. ___ After contact with an unknown powder pres-
living tissue ent on a patient
7. ___ Complete destruction of all microorganisms 9. ___ After eating
10. ___ After removing gloves

Copyright © 2016, Elsevier Inc. All Rights Reserved.


246 CHAPTER 96 Q Antiseptics and Disinfectants

11. ___ After setting up a patient for lunch who is in 21. Alcohol hand sanitizers are not effective against
  
$
ŠZ which organisms because they produce spores?
12. ___ After setting up a patient for lunch who is in ={

 
?
isolation for TB a. ,   
13. ___ After taking linens into a patient’s room b. /   
  
14. ___ After using the restroom c. Methicillin-resistant )  
15. ___ Before taking a pulse =€{?
16. ___ When the hands have been exposed to d.  
 
wound drainage 
Š %   

=ŠZ?

22. When the nurse is preparing to cleanse a wound,


CRITICAL THINKING, PRIORITIZATION, which preparation would be most appropriate and
AND DELEGATION QUESTIONS effective?

\  
=>?
17. Which is an example of sterilization? 
#

a. Autoclaving surgical instruments 
#


\
$





 - 
|%
=– ?
tient is in protective isolation
c. Personal respiratory equipment for tuberculosis 23. Why is hexachlorophene no longer recommended
=!–?

 
 ƒ
={

 
?

{  
  
#
 

 
 
 


#
 


 

 

 ]<


 
  



  
=>+{?
  


 
incision-site infection? 
#
 
 


$
 % 

a. Preoperative scrubbing of surgical site with an bacteria.


antiseptic 
#




$
  
b. Preoperative scrubbing of scrub nurses’ hands 
#
$"
  


c. Rigorous disinfection of operating room and

$
  24. What is the surgical scrub that is fast-acting and

{@ 
$
  
  antibacterial and remains active on the skin after
rinsing?

]‡
!
 
$$
$


= ?

–@ 

=–>?
as an antiseptic, the nurse should do what? 
>
=*  ?
a. Apply the alcohol directly to open wounds after 
* 
=|* *?
medicating the patient for pain. 
|%
=– ?

ˆ 


  


-
  

=? 625. The operating room nurse is preparing to apply

ˆ 




 
 
BAC as a surgical scrub. Before applying the solu-
than 75%. tion, it is a priority for the nurse to do what?

ˆ 


 

  
$

- a. Rinse the skin with water and alcohol.
hol. 
{ 

 
c. Wash the skin with soap.
620. The priority reason for not wiping a subcutaneous d. Warm the solution.
heparin injection site with an isopropyl alcohol
pad immediately after injecting the medication is 26. The nurse should scrub the hands and forearms
to avoid what? with an antimicrobial soap for surgical scrub asep-
a. Bruising sis for how long?
b. Neutralizing the heparin a. 1-2 minutes
c. Poor absorption of the heparin b. 2-6 minutes

{  
  c. 6-8 minutes
d. 8-10 minutes

Copyright © 2016, Elsevier Inc. All Rights Reserved.


CHAPTER 97 Q Anthelmintics 247

27. The nursing professor is orienting student nurses 2. How long do the instruments need to soak?
to the intensive care unit. Hygiene instructions
should include which directive?
a. Alcohol handrub should be used when the
hands are visibly dirty.
b. All forms of jewelry are not allowed.

&

 
 
   3. What precautions does the nurse need to take
d. Trim natural nails to no more than one-half 
 $

  
=>?ƒ



 

CASE STUDY
The delivery room nurse needs to prepare used instru- 4. Describe the handwashing technique the nurse

$
 $

  
=>?
$
would use after completing this disinfection pro-
being sent for sterilization. cess.

1. What steps should the nurse take before soaking


the instruments?

97
Anthelmintics

STUDY QUESTIONS 7. ___ Treatment may be deemed not needed if


there is a high probability of reinfestation or
True or False the patient cannot afford the drug.
8. ___ Roundworms can cause pancreatitis by
   
  T   F blocking the pancreatic duct.
   9. ___ Pinworms live in the human body for 7-10
days.

]

* 


 

$

of humans.
2. ___ Anthelmintics treat parasitic worm infesta- CRITICAL THINKING, PRIORITIZATION,
tions. AND DELEGATION QUESTIONS
3. ___ Parasitic worm infestation always causes
symptoms. 10. A patient has just started treatment with meben-
4. ___ When individual treatment is impractical,  @
=Š?
$

$  
!

improved hygiene may be the most valuable nurse assesses for the most common complication
intervention. of hookworm infestation, which may be what?
5. ___ The sooner drug treatment is started after in- a. Feculent vomiting
festation, the less reproduction of the worm b. Pale conjunctiva
occurs in the body. c. Perianal itching
6. ___ Helminthiasis can involve the liver, lym- d. Rectal prolapse
phatic system, and blood vessels.

Copyright © 2016, Elsevier Inc. All Rights Reserved.


248 CHAPTER 97 Q Anthelmintics

11. A patient who has been treated with mebendazole  ]—


!
 

 
  @
= @ ?

=Š?
$
 


  
- to an older adult patient who has been diagnosed
sone. What is the purpose of this medication? with the larval form of the pork tapeworm. Which
a. Kill larvae that have migrated. laboratory result would warrant consultation with

|
  
$
 
  

 
the prescriber regarding administration of the
muscle. medication?
c. Prevent migration of larvae. 
‰!
]~
  
 '‰


  

  
   
–ˆ+

 '‰

> 
]‹
 '‰
12. Nursing examination of a missionary who was 
| 
†<
 '‰
admitted after becoming ill in Haiti reveals severe
scrotal and peripheral edema. What other assess- 617. A resident has just prescribed diethylcarbamazine

 
$
 
  
  
 - =* @ ?
$

$

2  -
tode infestation?   . Which symptom, occurring after the start of

‰
  
$

$
” administration of the drug, would be the priority
b. Multiple bruises concern for the nurse?

{

 a. Dizziness
d. Weak hand grasps b. Headache
c. Nausea
13. A patient who works for the World Health Or- d. Confusion
@ 
=*}?

 
”



worldwide travel, complains of recent noticeable DOSE CALCULATION QUESTIONS


decreased visual acuity. Because of the possibil-

$
 
 
$

 

]<
€  @
=Š?
``



 


the nurse should assess for recent travel to which prescribed. How many 100-mg tablets should be
 = ?


 ‚
 ƒ
={

 
- administered at each dose?
?
a. Argentina

\  
]‡
 


$
 
$
  @
= @ ?
$

c. Kenya a 27-kg child if the recommended dose is 7.5 mg/


d. Mexico kg?
e. Peru
f. Rwanda
g. Venezuela
CASE STUDIES

]‹
#







  

test result to the prescriber treating a patient for Case Study 1


intestinal roundworms?
The father of a 2½-year-old child who attends day care

{!

#ˆ'‰
notices that his son has been restless, is not sleeping

–ˆ+

 '‰
well, has been scratching the perineal area, and has been

>\
†‹``
#ˆ'‰



!
$ 
 

 
$



#+
]
is told by the nurse to put a loop of transparent tape in
the child’s anal area in the early morning before the child
15. To prevent complications from therapy with
awakens. The nurse further instructs the father to remove
 @" 
=–?
  
$
  

the tape later in the morning, put it in a plastic bag, and


the nurse teaches the patient to




$
}
  
$

 


a. avoid hazardous activity.



 
 
 
=|+|?
  
-
b. chew the tablet completely before swallowing.
biasis pinworms.
c. monitor daily urine output.
d. take the medication on an empty stomach.
1. What is the probable mode of transmission of the
parasite in this case?

Copyright © 2016, Elsevier Inc. All Rights Reserved.


CHAPTER 97 Q Anthelmintics 249

^
!
|+|
  
  @
=Š?
$


<
{ 






entire family. What did the prescriber need to de- massive diarrhea. The nurse should be assessing
termine before prescribing the drug for the entire $
 

$
 
  



family? electrolyte imbalances?


a. Dehydration

3. How could the pinworm infestation have been b. Potassium


transmitted from the child to other family mem-
bers?
c. Magnesium

4. What teaching can the nurse provide these family 


{
members to prevent future infestation and spread
of pinworms?


‡
{
 

  

 
-
$  

 


={?

What follow up care should the patients expect to


5. The entire family is adherent to the prescribed
determine if the infestation has been adequately
therapy. Why is there little problem with adher-
treated?
ence with this therapy?

6. The parents are embarrassed and do not want to 10. What can the nurse do in this situation to prevent
notify the day care of the child’s infection. How spread of these infections?
should the nurse respond?

Case Study 2
A nurse is caring for victims exposed to contaminated
 

 

 


  $

clinic in a temporary shelter.

7. The nurse knows that which segments of the popu-


lation would be more at risk for helminthiasis?

Copyright © 2016, Elsevier Inc. All Rights Reserved.


250 CHAPTER 98 Q
@ 
&

   
 

98
Antiprotozoal Drugs I: Antimalarial Agents

STUDY QUESTIONS CRITICAL THINKING, PRIORITIZATION,


AND DELEGATION QUESTIONS
Completion
1. Malaria deaths are most likely in 613. What is the most important action by the nurse to
__________________ children. prevent transmission of malaria?
a. Administer drugs at the correct time.
2. Malaria eradication has not been successful be- b. Place the patient in strict isolation.
cause the transmitting insect and the parasite have c. Practice universal precautions.
become __________________ to the drugs. 
ˆ 

 


 
 

3. Malaria is transmitted by the 14. The repeating episodes of fever, chills, and profuse
__________________ __________________. sweating that are characteristic of malaria are due
to what?
4. Malaria kills more people than any other infection 
#$



$

 "

except __________________. 
#  
$



 "




=–>?



#$


 
 

@
d. Toxicity of the suppressant drugs
invading the __________________.
615. Prompt treatment of suspected cases of falciparum
6. The liver releases merozoites that infect malaria is most urgent if the patient experiences
__________________. what symptom?
a. Chills
7. __________________ malaria is the most com- b. Dark urine
mon form. c. High fever

{ 
8. Falciparum malaria does not
__________________ because it does not form 616. To prevent a serious complication, what is the
hypnozoites that remain in the liver.  
  



 
 
$

patient who has malaria caused by !


 

9. __________________ __________________   
?
__________________ is the term for fever and a. Assessing for crackles and respiratory distress
dark urine associated with falciparum malaria. b. Assessing vital signs every 4 hours
c. Performing glucometer checks every 2 hours
Matching d. Monitoring for elevations in liver function
studies
 
     
617. A patient with falciparum malaria is at risk for
10. ___ Prophylaxis   
#






 

11. ___ Prevention of relapse for what?


12. ___ Treatment of an acute attack a. Cold, clammy skin
b. Hot, dry skin
a. Clinical cure c. Hunger and thirst
b. Radical cure d. Nausea and vomiting

{ 
 

Copyright © 2016, Elsevier Inc. All Rights Reserved.


CHAPTER 98 Q
@ 
&

   
  251

18. Primaquine is not used for relapse prevention of


^†
˜

  @

  
  
>

infection with malaria caused by !  


 This means that the nurse should
Why? a. administer the drug to a pregnant woman if

#



 


 
  

prescribed, because there are no known risks to
prevent infection of erythrocytes. the human fetus.
b. !  
is resistant to primaquine. b. administer the drug to a pregnant woman if the
c. The infection is mild, and relapses will de- risk of not treating with the drug is greater than
crease and disappear over time without treat- the risk of the drug itself.
ment. c. administer the drug to a pregnant woman and
d. There are no hypnozoites in the liver to be carefully assess for any adverse effects.
eradicated. d. not administer the drug until the pregnancy
status of the woman has been determined.

]‡

 


 
"
= -
?

 




 




^‹
!

 

 

of the nurse’s response? quinidine gluconate is monitoring what?


a. Absorption is very slow after oral administra- 
Z>\
tion. 
Z
b. Deposits in tissue are slowly released into the c. Respirations
blood. 
ˆ


#

 




    25. Which change, if noted in a patient receiving me-

#
 
$ %  

 


"


 
$

 




by the kidneys. drug and contact the prescriber?


a. Headache
6^`
#








 - b. Heartburn
ment with primaquine if laboratory results for an 
‰ 
$
 


 
$



adult female patient include what? pleasurable



*  
<
'‰ d. Nausea
b. Hematocrit 40%

–>
‹‡
¬
]`6'‰  26. Because of adverse effects, developmentally,
d. Reticulocytes 2% which American patient would be least likely



 "'  
=€  ?

6^]
#




 
 


  
therapy?
quinine for malaria to report which effect? a. 15-year-old girl
a. Fatigue b. 35-year-old man
b. Headache c. 55-year-old man
c. Palpitations d. 75-year-old woman
d. Pallor  27. Tetracycline is prescribed for a female patient
who has chloroquinolone-resistant malaria. Which
22. Primaquine and quinine should not be adminis-
laboratory test result would warrant withholding


 

\—|&

  

the drug and contacting the prescriber?





‰!
—`
  
 '‰
a. causes cinchonism.

> 
]
 '‰
b. increases hemolysis.

>\
‡`
%  
 '‰
c. increases the risk of hypotension.

–>
‹^
¬
]`6'‰
d. prevents the drugs from working.

DOSE CALCULATION QUESTIONS



^<
˜
 
``^
 ' '


infusion is ordered for a patient who weighs 165


lb. The drug is supplied as 800-mg quinidine glu-
 


`

$
œ
 

 

 
 '



$
‰



  



ƒ

Copyright © 2016, Elsevier Inc. All Rights Reserved.


252 CHAPTER 99 Q
@ 
&
##•
€  
 

29. Clindamycin 20 mg/kg/day in three divided doses the frequency and intensity of the symptoms. The
is the recommended for a child with malaria. What patient is prescribed a 3-day treatment of chloro-
is acceptable dosing for a child who weighs 55 lb? quine for his acute attack of malaria. What teach-
ing could the nurse provide to decrease adverse
effects and to prevent serious effects?

CASE STUDY
A 45-year-old man has just returned from Africa after
4 years of service with the World Health Organization 4. The patient tells the nurse that he knows that mis-
=*}?
&
 


 

 
"
 

 
=#Š?
"


 

= ?
  


  

 

He asks why he wasn’t prescribed that drug. What
was bitten by an infected " mosquito. information can the nurse share about the decrease
in use of quinine?
1. What does this treatment actually accomplish?

5. The patient was advised that he should return


home for treatment with primaquine to prevent
2. What other measures should he have been taught relapse. Why was the patient not treated in Africa
to decrease the risk of malarial infection? where he could continue his work?

3. Before his service in Africa was completed, the 6. When the patient completes his acute treatment, he
patient quit taking the chloroquine because he was asks why he cannot continue taking the chloro-

  

 
{
$
quine to prevent recurrence of the malaria symp-
he started experiencing episodes of high fever toms. What is the basis of the nurse’s response to
followed by chills, and then diaphoresis occur- this question?
ring every 2 days. He was diagnosed with vivax
malaria and was unable to perform his work due to

99
Antiprotozoal Drugs II: Miscellaneous Agents

STUDY QUESTIONS 
#$

"
 



eating undercooked meat.


Matching b. Transmission is fecal-oral, often by
ingesting water contaminated with live-

       stock feces.
c. Principal site of infestation is the in-
1. ___ Amebiasis  Œ
 
  





2. ___ Cryptosporidiosis abscesses may form.

†

\   
ˆ  

$
$  


-

‹

‰    nary tract.
5. ___ Toxoplasmosis e. Disease is acquired through the bite of
6. ___ Trichomoniasis 
 
$
#$  
  




contaminated water.

Copyright © 2016, Elsevier Inc. All Rights Reserved.


CHAPTER 99 Q
@ 
&
##•
€  
  253

CRITICAL THINKING, PRIORITIZATION,  12. The nurse notes yellowish discoloration of the
AND DELEGATION QUESTIONS sclera of an immunocompetent 7-year-old girl who


 @ 
= ?
$
   

6 7. A farmer is hospitalized after developing cryp- What should the nurse do?
tosporidiosis during treatment with high-dose a. Administer the medication and continue nurs-
prednisone for exacerbated chronic obstructive ing care.
 
  
=>}|&?
 


 



 

–ˆ+

 
priority for this patient? c. Withhold the medication and review laboratory
a. Activity intolerance tests.
b. Fatigue d. Withhold the medication and contact the pre-

…

 scriber.

#  

613. Which result, if noted in the urine of a patient who
 8. Which symptom, if present in a patient who is 

 

=\ ?
$
 


"
=™?
$
   
Z 
$ 

 





would be of most concern to the nurse? to report to the prescriber?


a. Anal itching a. Multiple casts
b. Blurred vision b. pH 5.7
c. Facial pustules 
|
^
 '‰
d. Palpable thyroid 
{
 
]`]

 9. The nurse teaches a patient who is prescribed  ]‹


!

  
 
 
  
=-
 @
=… ?
 



 

 ?

*%
 


$"

report which possible adverse effect of the drug to adverse effect. The nurse observes for this immune
the prescriber? reaction, which presents as
a. Darkening of the urine a. a maculopapular rash.
b. Metallic taste 

 
c. Mouth ulcers c. lesions on the oral and conjunctival mucosa.
d. Paresthesias d. tingling around the mouth and throat.

]`

 
 
=Z&?
 


š
]

  
 
 

  
-
discharge teaching to a patient who is breastfeed- 

=ˆ?
  
$
 %
ing her infant who is prescribed metronidazole stage African sleeping sickness and consulting the
=… ?
$
   
#

 
$

 
prescriber?
to teach this patient to feed her baby formula and 
‰!
††<
  
 '‰
discard her pumped breast milk throughout the 
–+|
^—`
   '‰
time the medication is being taken and for how 
–ˆ+
†^
 '‰
long afterward? 
>|
<`
 '‰
a. 1 day
b. 3 days 616. A patient is admitted with American trypanoso-
c. 1 week  
=> ‚
  ?
#$
$
=‰ ?

d. 2 weeks is prescribed, it would be of greatest priority to


inform the prescriber of the patient’s history of
 11. A patient who has been prescribed metronidazole a. asthma.
=… ?







!
b. bowel obstruction.
nurse’s response is based on the fact that alcohol c. type 2 diabetes mellitus.
combined with metronidazole can cause what? 
\—|&

a. Bradycardia

&  
>+{
  DOSE CALCULATION QUESTIONS
c. Dysrhythmias
d. Psychotic reaction
]~
&

$

]‹`%
 


-

  
=| 
†``?
^^`



day for 7 days. The recommended dose is 4 mg/


 ' 
#

  
 
$

 -
tic?

Copyright © 2016, Elsevier Inc. All Rights Reserved.


254 CHAPTER 99 Q
@ 
&
##•
€  
 

18. A 65-lb child with amebiasis is prescribed iodo- 4. One duty is to review laboratory tests and notify
"
†~


<

$
^`
  
#

the PCP of results. The nurse is reviewing labora-
dose safe? tory results for the patient with amebiasis. Abnor-
mal results include WBC 12,800/mm3, sodium
]†~
Z"'‰
 
†—
Z"'‰



small abscesses in the liver on CT scan. Which test


CASE STUDIES result is most important to promptly review with
the PCP and why?
Case Study 1
The nurse works in a busy family practice. A 17-year-

$ 
 
 


$

 

$
 
 

"  
=ˆ˜?
 
 

chills, and diarrhea for 10 days. 5. The PCP asks the nurse to have the patient come
in immediately. After explaining the laboratory
1. What subjective information would be helpful to test results, the patient agrees to be faithful for a

$

 
 
 
=|>|?
course of metronidazole followed by iodoquinol.
sees the patient? What teaching should the nurse provide?

The patient was on a camping trip in the White Mountains Case Study 2
$
+
*  

$


{


nausea and fatigue toward the end of trip, and the symp- A patient who has a history of diabetes, seizure disor-

 

 



{


 
  

 
 

  

alcohol and tobacco use and sexual activity and is cur-  @
=… ?
$
   
!
 ‚





  

{

 
‹

 
 
 
$
=\ ?
``




 
^
 
  


!
]`^~„


 

=& ?
]``




…
 
‡<
  'Œ
–|
]``'—‹

* Œ


 
 $ 
=> ?
^



 

€ 

membranes, abdominal guarding, and weakness. The PCP and Thursday and 2 mg once a day on the rest of the days,
orders a CBC and differential, electrolytes, and a CT scan 

   
=Z  ?
†``




 
of the abdomen. The PCP diagnoses amebiasis. Because

 
 
 


 
$

- 6. What are the possible effects on drug levels of the
pliant with three times a day dosing with metronidazole, interaction of each of these drugs with metronida-

|>|
  
 @
=!  ?
^



 
@
=… ?ƒ
for 3 days.

2. During teaching regarding the adverse effects of


this drug, the nurse should explain the importance
of contacting the prescriber if what symptoms oc- 7. The nurse should assess for which symptoms of
cur? these drug reactions?

3. Based on the patient’s exposure to the organism, 8. What are interventions the nurse should employ if
symptoms, and developmental considerations, symptoms occur?
what other teaching should the nurse provide?

Copyright © 2016, Elsevier Inc. All Rights Reserved.


CHAPTER 100 Q
Z   255

100
Ectoparasiticides

STUDY QUESTIONS 13. Which treatment for head lice must be applied at
least twice to be effective?
Matching 
–@

=ˆ  ?

#
={?
(   
    #!$ - 
‰ 
=?
  #)$ #,$ 
|
=+?

1. ___ Do not live on pets


]‹
‰ 


 



 

2. ___ Burrows visible as dotted lines on skin    



$
 



ƒ
={

3. ___ Common site of adult infestation is in webs 


 
?

  
#
  
  



‹

# 
 
#


 
 




‰
$   c. Rinsing off the drug can contaminate ground
6. ___ Mite infestation water.

~

{ 
 
 

 
$ 
{ 

 
  
@ 
8. ___ Transmission possible via contact with e. Treatment usually involves reapplication once a
inanimate objects week for 3 weeks.
9. ___ Treatment differs by site of infestation

]`

ˆ  
 

 

DOSE CALCULATION QUESTIONS
CRITICAL THINKING, PRIORITIZATION, 15. The recommended dose range of ivermectin
AND DELEGATION QUESTIONS ={?

^`%‹`
 ' 
 


-
mended dose range in mg for a 30-lb child? A

]]
#$
 


=+?


$
44-lb child?
head lice, the experts recommend that it should be
done in how many days after the last treatment?
a. 1 day
]—
#
={?
‡```
  


b. 7 days prescribed. How many 3-mg tablets should be


c. 9 days administered?
d. 14 days

12. What could be associated with a severe reaction



  
=}?


  
$
CASE STUDY
head lice?
a. Absorption of the lotion through scratches on A second-grade teacher consults the school nurse about a
the scalp girl in her class. The child is one of three siblings attend-
b. Administration concurrently with drugs that ing the elementary school. The child has very long hair,
inhibit the P-450 hepatic cytochrome system and over the past week, the child has been constantly

… 

 

%



 

 
{

 

 



nits after application that her hair is tangled and almost impossible to brush,

{



 and her scalp is bleeding. The school nurse notes small




 
 $
 


 
Z  -
tion under the microscope reveals nits. The nurse contacts

‚
 


 

= ?



that it is probably !  


  
= 
?

Copyright © 2016, Elsevier Inc. All Rights Reserved.


256 CHAPTER 101 Q Basic Principles of Cancer Chemotherapy

1. How should the school nurse handle the possibility 4. The parents contact the pediatrician, who orders
that other children in the school may be infested? 
=+?
 
 


$

nurse include in the explanation for using this


product?

2. The girl’s parents are quite upset about the lice.


They want to know how their child could have
gotten “these bugs” and how to tell whether others

 


 
$

%



in the family have them. How should the nurse head lice treatment?
respond?

6. The child’s father tells the nurse that when he had


3. The parents were told by a friend to cut all of their head lice as a child, his prescriber had his parents
daughter’s hair off to get rid of the lice. What are 
 
=?
 
*
 



the reasons why the nurse would not want the why that is not used anymore. What would the
parents to do this? nurse tell him?

101
Basic Principles of Cancer Chemotherapy

STUDY QUESTIONS CRITICAL THINKING, PRIORITIZATION,


AND DELEGATION
Completion

]
#
 

7. Which characteristic best describes cancer cell
_______________ causes more death than cancer. reproduction?
a. Abnormal
2. Cancer is the leading cause of death in women age b. Rapid
______ to ______ years. 
{

ˆ  
3. _______________ cancer is one of the most com-
mon locations of cancer in men. 8. A patient is admitted for surgical removal of a can-
cerous tumor of the colon. He is concerned about
4. The most common treatment for solid tumors is the prospect of living with a colostomy and asks
_______________. the nurse why the prescriber cannot just give him
some drugs to kill the cancer. What is the basis of
5. _____________________ drugs are the class of 
 ‚
  ƒ
={

 
?
anticancer drugs used most often. 
{


  

{

 
$

 



6. The enzyme telomerase, which is active in most dividing.


cancer, allows cancer cells to _______________ 
{

 
$



\0 phase.
_______________. 
{

 






core.

{

 
 
&+

Copyright © 2016, Elsevier Inc. All Rights Reserved.


CHAPTER 101 Q Basic Principles of Cancer Chemotherapy 257

9. The spouse of a cancer patient asks why some š


]†
!
 

 
$

—^% %
 


tumors become resistant to chemotherapy. Which developed pancreatitis during chemotherapy after
statement would not be included in the explana- mastectomy. Most recent laboratory results include
tion? RBC 2.8 106'‰
–>
‹]``'3, neutrophils
a. A transport molecule may be produced to trans- 18,000/mm3, and platelets 147,000/mm3. What
port drugs out of the cancer cell. is an expected nursing diagnosis, based on these

> 



\0 phase have time to repair laboratory results?
drug-induced damage before it does serious a. Activity intolerance
harm. b. Decreased cardiac output
c. Chemotherapy drugs alter the cancer cell’s 
# 

  
DNA, making the cell resistant. 
#$$
  
 
$ 
d. Drug-resistant mutant cells have a competition-
free environment because of the death of drug- š
]‹
!
 

 
$

 
 

sensitive cells. who has neutropenic precautions. The nurse


washes her hands and dons a gown, mask, and
10. The father of a child with leukemia is concerned gloves before entering the patient’s room. The
about his son’s fear and discomfort during chemo- patient refuses care by the nurse until the nurse
therapy. He asks the nurse why the child cannot washes her hands at the sink in the room and puts
receive one large dose of a single powerful drug on new gloves. What should the nurse do?
instead of multiple doses of different drugs. After 
Z 
 





  


teaching, which statement, if made by the father, infection if she takes off her current gloves
suggests that that this parent needs further expla- while in the room.
nation? 
Z 
 

 

 
” 
 

a. “Different drugs kill cancer cells in different the door before entering the room.
ways.” c. Rewash her hands in the room and put on new

’#

  
 
$




gloves.
multiple doses because one of the drugs that d. Rewash her hands, put on new gown, gloves,
my son is getting only kills cancer cells at a and mask inside the room.
particular point of cell reproduction.”

’#$



 

 


15. For a cancer patient who is receiving chemother-
likely to develop resistance to all of them.” apy, which food would be least likely to cause an
d. “My son will not experience adverse effects if infection if the patient develops neutropenia?
he receives multiple drugs administered at dif- a. Commercially canned fruit
ferent times.” b. Vegetables from the patient’s garden
c. Rare meat
11. The nurse would expect what when two or more d. Yogurt
anticancer chemotherapeutic agents are used
together? 16. The nurse is administering subcutaneous insulin to
a. The drugs’ adverse effects primarily affect dif- a chemotherapy patient who is experiencing bone
ferent organs. marrow suppression. Which technique would be
b. The drugs are less effective than if used alone. best for the nurse to use to prevent an adverse ef-
c. The drugs can be mixed in the same intrave- fect from the bone marrow suppression?
nous solution. 
 

 


 


d. The drugs have the same mechanism of action. b. Apply pressure to the site following injection.
c. Delay the administration of insulin until after
12. A patient with brain cancer asks the nurse why the patient has eaten his meal.
they are putting the drug into her back instead d. Wipe the injection site with Betadine after
of into a vein. The basis of the nurse’s response injection.
should be that this route
a. allows higher doses of chemotherapy to be 17. A patient receiving chemotherapy develops hyper-
used. uricemia. A nursing priority addresses the effect of
b. allows the drug to get to the tumor cells. the urate crystals on which body part?
c. prevents many adverse effects. a. Blood
d. prevents drug-resistant mutation of the cancer b. Joints
cells. c. Kidneys

‰

Copyright © 2016, Elsevier Inc. All Rights Reserved.


258 CHAPTER 102 Q
 
&

>
 

š
]<

 


  
- 2. The patient asks why people are more likely to
apeutic agent, which information is most impor- get cancer if they have a family history of cancer.
tant for the nurse to know? How are genetics linked to cancer?

#$

 


 



antiemetic

{
$ 
 
$
  
-
curs
c. The mechanism of action of the drug 3. The nurse is discussing strategies to minimize the
d. Whether gloves are needed during administra- adverse effects of chemotherapy, including mouth
tion 
=   ?
!
 


 

drugs cause mouth ulcers if they are given intrave-


nously. How should the nurse respond?
CASE STUDIES
Case Study 1
A 28-year-old female patient is admitted to the oncology
unit with a diagnosis of suspected Hodgkin’s disease. 4. What interventions can the nurse suggest to mini-
!
  


$

 
!
 

mize the effects of chemotherapy on the patient’s
started on treatment, which consists of external radiation \#
 ƒ
treatments and chemotherapy. The patient asks the nurse,
“How does cancer spread?”

1. What information can the nurse provide about the


characteristics of cancer cells that promote growth Case Study 2
and metastasis? A nurse has just lost his sister to breast cancer. Before she
died, his sister challenged him to use his nursing knowl-
edge and energy to prevent cancer suffering and death.
The nurse networks with fellow nurses to identify what

 



 
$

 ‚
  

5. What actions could the nurse take?

102
Anticancer Drugs I: Cytotoxic Agents

STUDY QUESTIONS
†

#

 
 
 % 




 



-
True or False table as prescribed.

‹

>%
 ­ 



   


  T   F equally effective during any phase of the cell
   cycle.
5. ___ Bifunctional alkylating agents are more ef-
1. ___ Cytotoxic cancer drugs kill cancer cells and fective than monofunctional agents.
healthy cells. 6. ___ Alkylating agents are considered cell-cycle

^

>


\0 are phase are most likely to  ­ 
be killed by cytotoxic drugs.

Copyright © 2016, Elsevier Inc. All Rights Reserved.


CHAPTER 102 Q
 
&

>
  259

~

€ 
 

€% ­  15. The nurse is teaching a patient who has been pre-
8. ___ Folic acid analogs are effective because folic scribed oral cyclophosphamide. Which statement,
acid is toxic to cancer cells. if made by the patient, suggests a need for further

‡

… 


$



teaching?
right time when administering high doses of 
’#


$


$
#


$

methotrexate coupled with leucovorin rescue higher than 100.4° F.”


can kill the patient. 
’#


 


]

$

^

10. ___ Antitumor antibiotics are used to treat infec- hours after meals.”
tions associated with neutropenia caused by 
’#



 
—

$


cancer drugs. every day, especially water.”



’#

$


$




changes.”
CRITICAL THINKING, PRIORITIZATION,
AND DELEGATION QUESTIONS
]—


 
=€ ?
  

 

mustard alkylating agents?


š
]]

 


 
 



#
 
  

 
common, serious adverse effect of every category 
#
 

$
@
 
 

of cytotoxic drugs? DNA.


a. “Change positions slowly to prevent dizziness 
#
 


or fainting.” 
#

  


 
 
b. “Night driving may not be safe.”

’#‚
 

$
 %  
- 6]~
#

$
  

$

 

 

cise.” with the oncologist before administering cisplatin



’ˆ 


 @



- if the patient reports which symptom since the last
self.” infusion?
a. Headache
612. Which diagnostic test result would be of greatest b. Nausea
priority when a patient is receiving cytotoxic drugs c. Ringing in his ears
for cancer? 
ˆ 
$"
a. Absolute neutrophil count 476/mm3
b. Platelets 130,000/mm3 18. What is most likely to worsen symptoms of
c. Reticulocytes 3.2% peripheral sensory neuropathy associated with
d. WBC 11,000/mm3   
=Z ?ƒ
a. Dehydration
13. Which administration strategy would be most 
Z 




 
 
 % 
 
c. Fatigue
drugs are present when cancer cells will be 
{ 
harmed by the drug?
a. Combined with other drugs š
]‡
*



$ 
 
$
  

b. Frequent intervals with prolonged infusion age be instructed to use two reliable forms of birth
c. Rapid infusion of large doses control when prescribed methotrexate?

{ 
 
 

 
‹

  a. During therapy
b. During therapy and through at least 1 month

]‹
>%
 ­ 

 


after therapy


\0 phase because the c. During therapy and through at least 6 weeks
a. cell has time to repair damage. after therapy




 
$



\0 phase. d. During therapy and through at least 6 months
c. cell is reproducing. after therapy
d. cell’s DNA is not harmed.
620. Which diagnostic test result, if it is a change from
a previous result, would be of greatest priority to
report to the prescriber of methotrexate?

‰!

  
 '‰

–ˆ+
^^
 '‰

> 
  
‹
‰'

*  
]`<
'‰

Copyright © 2016, Elsevier Inc. All Rights Reserved.


260 CHAPTER 102 Q
 
&

>
 

621. A patient with laryngeal cancer is receiving mas- 626. Which symptom, if occurring in a patient receiv-

 
$
 


=$

 
=|?


$



$ ?



$
 - greatest priority to the nurse?
est priority? a. Constipation

 

  
  
=¢- b. Diarrhea
$ ?
†`

$

 
$

- c. Hair loss
trexate infusion d. Petechiae
b. Administering the leucovorin exactly as pre-
scribed 27. A patient who is receiving an infusion of liposo-

#$ 

 

 

 
 
=&?


 


prescribed time interval chest tightness within minutes of the start of the
d. Monitoring for electrolyte imbalances because infusion. The nurse stops the infusion. The symp-
of the high risk of diarrhea or vomiting toms abate, and the patient’s vital signs are within
normal limits. What is the appropriate nursing
22. Which statement by a patient who has been taught action in this situation?
about prevention of adverse effects of methotrex- a. Administer an antidote.
ate therapy suggests understanding of the teach- 
> 


  
{!!
ing? c. Hold the drug until the next scheduled dose.
a. “A new cough should be reported to the doc- d. Restart the infusion at a slower rate and moni-
tor.” tor for return of symptoms.
b. “Drinking cranberry juice will help protect my
kidneys from damage by methotrexate.” 28. The nurse should warn patients that they may ex-
c. “Drinking grapefruit juice can prevent my liver perience a blue-green discoloration of their urine
from breaking down this drug.” and the whites of their eyes if they receive which

’#

 


 

‹
 
antitumor antibiotic?
each day.” 
Z 
=Z?

#  
=# ?
23. The nurse teaches a patient that dexamethasone is 
‰  
  
=& ?
prescribed for patients who are being treated with 
€ 
=+ ?

= ?



ƒ
a. Anemia 629. The nurse is preparing to administer bleomycin
b. Nausea 

 

  
 
#


$

c. Rash greatest priority to report which diagnostic test



{  result to the prescriber?

Z
 
 
=Z{?
]`
'
š
^‹


$
 
$$
$

 - hr
 
$ 
$
  
=  %>?


$

* 
=*?
†<œ
most concern to the nurse? 
| 
 

=|Z…?
‹œ
$
 
a. Anorexia, nausea, and bruising d. Platelets 180,000/mm3
b. Headache, fever, and vomiting

€
 
 
” 

   630. An adolescent patient is scheduled to receive
d. Nausea, vomiting, and fever  
=}?
$
* ‚
 

Developmentally, which adverse effect is most

^

 
$$
$
 
=?

likely to be a priority concern for this patient?
 

  
 
$

 ƒ
={

a. Alopecia
 
? b. Constipation
a. Blistering of the palms and soles c. Nausea
b. Dark spots on the skin d. Paresthesias
c. Diarrhea

‰
$
  631. Which nursing action would be of greatest prior-
e. Mouth ulcers ity when administering an infusion of paclitaxel
=! ?ƒ
a. Monitor urine output.
b. Monitor vital signs.
c. Prepare for possible vomiting.
d. Reposition patient every 2 hours.

Copyright © 2016, Elsevier Inc. All Rights Reserved.


CHAPTER 102 Q
 
&

>
  261

š
†^

 


 
  
$

CASE STUDIES
 


 
=! ?ƒ
a. Blood pressure 147/90 mm Hg Case Study 1

Z %

$
<`

$

 
-
ber urine A 41-year-old patient is admitted to the outpatient area

Z 
@ $

 
 
$
 
 
=?

d. Two-plus pitting edema of the ankles chemotherapy for metastatic breast cancer.

š
††
!
 
$

 


 @ 
1. What techniques should the nurse use when ad-
=   ?

 

 




ministering this drug to prevent personal harm?
night with diarrhea. The patient is currently in bed
and is too weak to stand up. The spouse asks the
nurse what to do. What is the most appropriate
response? 2. What administrative techniques should the nurse

 
]
 
$
 
 
=|% use to prevent patient injury from this drug?
– ?
$
 
$

b. Allow the patient to rest.
c. Have the patient take frequent sips of a sports
drink with sugar and electrolytes.

{
 
 

$

  Case Study 2
!
 

 

 
 
= -
DOSE CALCULATION QUESTIONS ?


†<% %
 

  
 

†‹
…  
=+ ?

  

 ' ' 
3. Developmentally, what effect of this drug would
The patient weighs 132 lb. What dose should be be a priority for the nurse to prepare this patient to
administered? experience?

35. What is the maximum recommended lifetime dose


$
 
= ?
$

 


4. What things should the nurse assess and teach the
72 inches tall and weighs 220 lb, when the cumu- patient to report relating to the risk of cardiotoxic-
lative lifetime dose should be kept below 550 mg/ ity?
m2?



 
= ?
^

 

  
 

should the nurse include in teaching about this


drug?

Copyright © 2016, Elsevier Inc. All Rights Reserved.


262 CHAPTER 103 Q
 
&
##

103
Anticancer Drugs II: Hormonal Agents, Targeted Drugs, and
Other Noncytotoxic Anticancer Drugs

STUDY QUESTIONS 6 8. A 58-year-old patient has been taking tamoxifen to


prevent breast cancer. Which question would be of
True or False greatest priority for the nurse to ask?
a. “Has anyone in your family ever had breast
   
  T   F cancer?”
   b. “Have you ever been told that your cholesterol
is high?”
1. ___ Hormonal agents lack serious adverse ef- c. “Have you experienced a broken wrist, hip, or
fects. backbone?”
2. ___ Chemotherapy and hormonal therapy are d. “Have you had any vaginal bleeding after the
drugs added to cancer treatment to improve drug was started?”
the response to other therapy.
3. ___ For hormonal agents to be effective, the 9. Which teaching would be appropriate for the most
tumor must have receptors for the hormone common adverse effect of tamoxifen?
being blocked. a. Adequate hydration
4. ___ Tamoxifen is not recommended for breast b. Dress in layers
cancer prevention for women older than 60 
{ $
 
$
}!>
   
years because they have a lower incidence of 
{ 
$"
 
breast cancer than women ages 40-59 years.
5. ___ Antidepressant drugs Prozac, Paxil, and 610. Which possible adverse effect would be of greatest
¢$

  
$
 $


priority to report to the prescriber of a selective
liver causing tamoxifen toxicity.   

 
={Z€?ƒ
6. ___ The likelihood of infusion reactions to a. Calf pain and swelling
 @
=*?
 

 

*
 

 
 
subsequent dose of the drug. 
‰ 
  

d. Nausea without vomiting
CRITICAL THINKING, PRIORITIZATION,
11. What is the expected effect on tamoxifen levels of
AND DELEGATION QUESTIONS  
 

>™|^&—
 




=|@ ?ƒ
 7. Which nursing action would be most likely to sup-

& 


port patient adherence to taking antiestrogen drug
b. Decreased tamoxifen levels
therapy for breast cancer?

# 


a. Demonstrating administration techniques

# 
 $


Z @

$
 

#$
 

612. Which laboratory test result would be of greatest
d. Teaching mechanism of action of the drugs
priority to report to the prescriber if a patient is
  
$
=…  ?ƒ

> 
<<
 '‰

€  
^^
Z"'‰

| 
^~
Z"'‰

{
]‹~
Z"'‰

> 
Ÿ
^`]—
Z 
#

 
 
CHAPTER 103 Q
 
&
## 263

13. A 38-year-old patient with breast cancer asks š


]‡


  
 
$

why she has been prescribed tamoxifen instead of metastasis of breast cancer to bone?
 @
=?

 


‹«



=&!?
be included in the explanation by the nurse? b. Bilateral weak hand grasps
a. Anastrozole blocks production of estrogen by 
+ 
–  ‚

the ovaries, causing severe menopausal symp- d. Positive Homans’ sign
toms.
b. Anastrozole can cause endometrial cancer.  20. A patient who is receiving leuprolide for advanced
c. Tamoxifen is more effective and has fewer prostate cancer is at risk for new-onset diabetes.
adverse effects. Which symptom suggests that the patient is expe-
d. Tamoxifen activates receptors increasing bone riencing hyperglycemia?
density and healthy lipid levels. a. Anorexia
b. Diaphoresis
 14. The nurse would consult the prescriber of trastu- c. Frequent urination
@
=*?
 
$

 


suddenly developed which symptom?

> 
 


 21. Which injection interval should the nurse question

\ @
  $

 

  
‰

”-
c. Headache with sensitivity to light tion?

! 
 
 
]``‹„
…
=†<„
>? a. Once a month
b. Once every 4 months

]




 
$
 
$

c. Once every 6 months
 




 
 
$
 - d. Once every 12 months
@
=*?
 
$

$

breath?  22. Which instruction should be included in teaching


a. Assess lung sounds. for a patient who has been prescribed nilutamide

Z 

 
$

 =+ ?ƒ
c. Notify the prescriber. a. Change positions slowly to prevent dizziness or

{


$  fainting.

‰


 







16. What would suggest possible cardiotoxicity from not need to get up to go to the bathroom.
 @
=*?ƒ
={

 
? c. Night driving may not be safe.

–+|
<`
   '‰ 
ˆ 


 @



-

Z   

”
$ 
$
self.
60%
c. Pulse 58 beats/min 23. Which new symptom since starting nilutamide

{$
$

$
  
 
=|€#?


=+ ?

 


 




left drug and consult the prescriber?



{ 
 

 


   a. Dyspnea

\ 
17. The prescriber of a potentially cardiotoxic drug 
# 
asks the nurse, “What is the patient’s weight?” d. Nausea
What information is the prescriber seeking?
a. Actual weight in kilograms 24. A patient who is prescribed high-dose ketocon-

–


=–€#? @
=+@ ?
$
  
 



c. Change in weight in the past 24 hours  


 
#


$
  
-
d. How weight relates to ideal body weight ity to monitor which laboratory tests?
a. Complete blood count and differential
18. Which herb has been known to increase metabo- 
Z
 
$
  
=! ?

  
 

‰
effectiveness? 
ˆ  

Z 
b. Feverfew

\ 

{
 ‚


> 
Ÿ
^`]—
Z 
#

 
 
264 CHAPTER 103 Q
 
&
##

25. Which drugs are commonly used for premedica-


†]
 


 


$

˜!


$
$ 
%!
=| ?ƒ
syndrome and should be reported to the prescriber
={

 
? $

=!   ?ƒ
a. Acetaminophen a. Dizziness with position changes
b. Aspirin 
|  

  

c. Diphenhydramine 
{
$
 

  


Z 
ˆ 
$ 

# $

†^

 



$
  

26. Which value should be reported to the pre- concern to the nurse caring for a male patient who
scriber immediately when a patient is prescribed 
  

={?
$
 
 

 
=! ?

 @ 
=   ?ƒ cell carcinoma?
a. ANC 875/mm3 a. 10 mm Hg increase in BP over baseline
b. Hb/Hct 13.2/37% 
‰  
 



c. Platelets 400,000/mm3 c. Dizziness with position changes
d. WBC 11,000/mm3 
*  
]†^
'‰

27. The teaching plan for patients receiving cetuximab  33. Which symptom, if occurring 12-24 hours after
should include the importance of reporting sudden 
 
$ 
$

= ?
 -
onset of which symptom to the oncologist? gests possible drug-induced tumor lysis syndrome
a. Fever =!‰{?ƒ

#  a. Constipation, epigastric pain, and cool, clammy
c. Nausea skin

&$
  b. Muscle spasms and tingling around the mouth
c. Anorexia, vomiting, altered mental status, and

^<

 

  
 

 
muscle cramps



 
=Š ?

- d. Weakness and thirst
riencing hypomagnesemia?
a. Constipation  †‹

 



= ?
!

b. Tremors nurse should teach the patient to report which


c. Fever symptom that suggests a critical adverse effect
d. Hypotension warranting immediate medical attention?
a. Dizziness and throat tightness

^‡

}!>

 
 

$
- b. Muscle aches and pains
 ƒ
={

 
? 
{ 

  
 

 
=!? d. Vomiting and diarrhea

> 
   
=!ˆ€{?

& 
=– ? 35. The nurse teaches a patient who is receiving ibri-

} @


   
=¢ 

 
=#!?


%‡`
 

}!>? the most severe neutropenia and thrombocytopenia



 
=¢  ? will occur at what point in therapy?

$

 
 
 30. The nurse notes a 2-lb weight gain in 24 hours b. At the time of the last dose
when assessing a hospitalized patient who has c. 7-9 weeks after treatment


 
=\?
$

d. 10-15 weeks after treatment


=>€‰?
#

$
  


36. Teaching should include prompt reporting of


for the nurse to assess which body system at this
abdominal pain, chest pain, shortness of breath,
time?
change in mental status, and blood in sputum
a. Cardiopulmonary status
when a patient is receiving treatment with which

\   
 
drug?
c. Musculoskeletal status

– @
= ?
d. Neurologic status

–@
=Š ?

\@
@ 
=€  ?


= ?

> 
Ÿ
^`]—
Z 
#

 
 
CHAPTER 103 Q
 
&
## 265

37. What is a symptom of the most common serious 43. What is the greatest nursing priority when a pa-
 
$$
$
  
=¢@ ?ƒ tient is experiencing neuropsychiatric effects from
a. Anorexia interferon alfa-2b?
b. Fatigue a. Completing drug therapy with interferon alfa-
c. Frequent watery stool 2b
d. Dyspnea b. Compliance with antipsychotic drugs

Z   
 

†<
#$

 

 
$
-
d. Preventing injury

=™?
 
 




 


ƒ
a. Weight gain, fatigue, and cold intolerance DOSE CALCULATION QUESTIONS
b. Weight loss, fatigue, and cold intolerance
c. Weight gain, fatigue, and heat intolerance 44. Fulvestrant 500 mg is to be administered as two
b. Weight loss, fatigue, and heat intolerance %‰
” 


 

 


]%^
 
!
%‰
 

 

^']`

39. To prevent some of the most common adverse


increments. How long would it take to administer
$$
$
$
$ %^
=#%?

 

 
”
$

 
 
^']`

=]

teaches the patient about administering which


?


 ƒ
#

 ƒ
drugs?

 
=!?

&  
=& ?

‹

 
 
$

=Z ?
<``




& 
=– ?

$ 

  
$

 


‚]`“


}  
=¢$ ?
and 180 pounds. The recommended dose is 400
mg/m2
#

  
 
$

 -
40. The oncology nurse has just completed infusing
tic?
 
=|?


 


becomes very short of breath. Which action should



 
$
 ƒ
a. Assess the patient’s vital signs. CASE STUDY

… 

#Š

Z 

 
$

   
=¢ ?

 

  
$

d. Notify the oncologist. patient with metastatic castration-resistant prostate cancer.

41. Nursing preparation for administration of bacille 1. The nurse should teach the patient to report what
> %\
=–>\?
 
 symptoms of possible electrolyte imbalances?
a. accessing a central venous line.
b. assessing the intravenous site for patency.
c. inserting a urethral catheter.

$

 
  
$

infusion.
2. What foods should the patient consume and avoid

‹^
!
 
 

 



–>\
to decrease the risk of these imbalances?
vaccine to use precautions to protect from contam-
ination with excreted urine for 6 hours after the
end of therapy, including cleaning the toilet with
what?
a. Alcohol
b. Ammonia 3. What are possible effects of a decrease in gluco-
c. Bleach corticoid production?
d. Peroxide

> 
Ÿ
^`]—
Z 
#

 
 
266 CHAPTER 104 Q
&
$

Z

4. What symptoms should the nurse teach the patient 5. What over-the-counter drug can rise to toxic levels
to report because they suggest possible liver dam- if taken when prescribed this therapy? What are
age from the abiraterone? the possible effects?

104
Drugs for the Eye

STUDY QUESTIONS a. Phenylephrine


b. Brimonidine
Matching c. Anticholinergic drugs
d. Beta-adrenergic blocker

         e. Carbonic anhydrase inhibitor
$
> 
=  ?
 
1. ___ Block muscarinic receptors preventing pupil g. Cholinesterase inhibitor
constriction h. Acute angle-closure glaucoma
2. ___ Progressive optic nerve damage with even- i. Cycloplegia
tual impairment of vision j. Miosis
3. ___ Contracts the ciliary muscle facilitating k. Mydriasis

$
"
 l. Osmotic agents
4. ___ Decrease production of aqueous humor 
| 
%  
 
=|}\?
5. ___ Dilate pupil by activating alpha1-adrenergic n. Prostaglandin analog
receptors

—

…  

$
"


 
Completion

 

 
 Œ
 
 

pigmentation of eyelid
]
 % 
  
  
=€&?
  

7. ___ Maximally effective doses reduce aqueous loss of _____________________ vision.




`œŒ


 
$
 

who have been refractory to preferred medi- 16. _____________________ are yellow deposits
cations under the retina that occur with ARMD.

<

# 

 
$


causing miosis, focusing of the lens for near 17. Vision loss occurs in advanced
 


$
#}| _____________________ and
9. ___ Paralysis of ciliary muscle _____________________ ARMD.
10. ___ Constriction of the pupil
11. ___ Widening of the pupil 18. Patients who take vitamins to prevent

]^

>  



$




ARMD should also ensure adequate in-
plasma, thereby causing a rapid and marked  
$







$
#}| _____________________.
13. ___ May delay optic nerve degeneration and
may protect retinal neurons from death

]‹

#}|
 
 


  

levels

Copyright © 2016, Elsevier Inc. All Rights Reserved.


CHAPTER 104 Q
&
$

Z 267

CRITICAL THINKING, PRIORITIZATION,  24. The drug handbook lists retinal detachment as an
AND DELEGATION QUESTIONS uncommon but serious adverse effect of pilo-
carpine eye drops. The nurse determines that the
619. What is the priority action for prevention of blind- patient needs more teaching if the patient includes

$
|}\ƒ which issue in the list of things to be reported im-
a. Administer eye medications as ordered. mediately to the ophthalmologist?

Z 

 
$
$
- 
… 



$
 
mendations for eye screening. b. Halos around lights

#$
 

 
 
$
   
‰ 
 
d. Teach the common symptoms of glaucoma. d. Painless loss of part of his vision

620. The nurse works in emergency department triage.


^

= ?
$



 


Which patient would be of greatest priority for 


  
 
=| 
#?

receiving immediate care? would suggest possible development of cataracts?


a. Patient who sees halos around lights. ={

 
?
b. Patient who experiences itching and watering 
&$
 
 

of eyes. 
Z
 
c. Patient who has sensitivity to light. 
… 



$
 
d. Patient who has severe eye pain that started 10 d. Halos around lights
minutes ago. 
#

   
$


 ^]
Z
 
 
$$
$
 

š
^—

 



  
 
 @-
=!?
 
=| 
#?

=& 
{" ?
$
$ 
|}\


and pilocarpine eye drops are uncommon, which experiences the adverse effects of severe vomiting




 
$

 

 
and diarrhea should be monitored by the nurse for
the prescriber before administering the drops? what?
={

 
? 
> 
 
 
]]
 '‰
a. Audible wheezing 
>  

 
 
 
]``
 '‰
b. Blood pressure 170/100 mm Hg 
| 

 
†
Z"'‰
c. Clouding of lens of eye 
{
 
 
]‹
Z"'‰
d. Keyhole appearance to pupil
e. Pulse 50 beats/min
^~
#

$
  



$
 
-



 

 


 

 22. What should the nurse do if a brown discoloration agent for acute angle-closure glaucoma?
of the iris is noted on a patient who has hazel eyes a. Bowel sounds



   
=   ?ƒ 
ˆ


 

 



  c. Pain in eyes
b. Assess for symptoms of a migraine headache. d. Vision changes
c. Teach the patient proper eye cleansing hygiene.
d. Withhold the medication and contact the pre- 28. Which adverse effects of mydriatic drugs are seri-
scriber. ous and warrant withholding the drug and consult-


  ƒ
={

 
?
 ^†
#

 
$

 

 
  
a. Blurred vision
blood pressure precautions when a patient is pre- b. Headache
 


$
|}\ƒ c. Palpitations

 
=#? 
{ 

 

–  
=‰ ? 
{

 

–
= ?

‰   
=   ? 29. The nurse can help decrease the progression of
ARMD by teaching patients at risk to
a. have preventive laser surgery.
b. use adequate light when reading.

 

$
 
>

Z


carotene, zinc, and copper.



 
 

ˆŠ%
  

in bright light.

Copyright © 2016, Elsevier Inc. All Rights Reserved.


268 CHAPTER 104 Q
&
$

Z

6†`
#

$
  

$

 

 

 - 1. What are mydriatic and cycloplegic effects, and
tient who had vitreous humor injection of pegap- what are their purposes in this situation?
 
=€  ?

 

 
 

if what occurs?
a. Blurred vision
b. Conjunctival redness
2. After further examination, the patient is diagnosed
c. Halos around lights

|}\
*





 



‰ 
 
disease, glaucoma, and not be aware of it? How
should the nurse respond?
31. Because of effects of the drug, patients who are
  



  

a. protect skin from sunlight.
b. assess for fever.
c. report eye discharge.
†
!
 

  
   
=   ?

d. expect some ocular pain. 1 drop to each eye at bedtime. The prescription
is denied by the patient’s insurer. The prescriber
32. A patient asks the nurse why the prescriber does changes the order to betaxolol 1 drop in each eye
not advise using the ocular decongestant tetrahy- twice a day. Why had the prescriber originally
@
=Š ?
$
 
” 
 
chosen latanoprost instead of betaxolol for this
would be included in the nurse’s explanation? patient?

–
 
 
 


b. Rebound congestion is likely.
c. The drug can cause cataracts.

!

 
 
#}|
4. What teaching should the nurse provide the patient
regarding the betaxolol prescription and her glau-
DOSE CALCULATION QUESTIONS coma?

33. Acetazolamide is available in 250-mg immediate-


release tablets. How many tablets should the nurse
administer if the dose is 0.5 g? 5. The patient returns to the ophthalmologist to have

#}|

*
#}|
 




* 
!
  


= ?

34. Mannitol is prescribed preoperatively for a 165-lb 1 drop instilled in each eye 3 times a day and
patient with acute closed-angle glaucoma. The @ 
=! ?
]

†


 
!

drug is prescribed 0.25g/kg as a 20% solution. The patient asks why the prescriber did not just order
drug is to be infused over 30 minutes one hour 
 

$
   
=   ?
*

prior to surgery. How many grams of mannitol should the nurse respond?
will the patient receive?

CASE STUDY 6. What teaching should the nurse provide about


therapy with brimonidine and dorzolamide?
A 62-year-old patient is seen by her ophthalmologist at


$

 
 
  
=|>|?
{

has a history of chronic obstructive pulmonary disease


=>}|&?
   

 
 
{

 

by her adult son and is reluctant to seek care because she






’“

 
 

 






   
 

#}|





* 
! 
=! ?




and the patient is sent to wait until mydriatic and cyclo-


plegic effects occur.

Copyright © 2016, Elsevier Inc. All Rights Reserved.


CHAPTER 105 Q
&
$

{ 269

105
Drugs for the Skin

STUDY QUESTIONS CRITICAL THINKING, PRIORITIZATION,


AND DELEGATION QUESTIONS
Completion
1. The _____________________ is the outermost 13. The nurse is teaching the parents of a 2½-year-old
layer of the skin. child about administration of topical glucocorti-
coid therapy prescribed for their son’s eczema.
2. All cells of the epidermis arise Which statement, if made by a parent, would sug-
from the _____________________ gest the need for further teaching?
_____________________. 
’#






 


cream on his chin.”


3. The cytoplasm of dead epidermal cells is con-

’#





$




verted to _____________________.
the cream was applied unless the doctor tells
4. The outer layer of the epidermis is called the stra- me to.”
tum _____________________. 
’#

 
 
#



 


areas with the rash.”


5. _____________________ determines skin color d. “The cream can help stop itching so my child




$
  
$
ˆŠ
   won’t scratch it and get infected.”

—
{
 

  



$

in the _____________________. 14. A patient who was admitted for a knee replace-
ment asks the nurse to get an order for “cortisone

~
{ 
 



 
cream” because she has an irritated area of skin
known as _____________________. 

 
#

 
 
$

 

to do what?

<
{  
 

 
 
$


Z 
 
 
 



 

_____________________.
in moist areas.
9. __________________ __________________ are 
#  
 

 


 
lesions of acne commonly called  . c. Keep the area covered with a dressing.

 



 



10. ___________________ ___________________ the prescriber.


develop when pores become blocked with sebum
and scales below the skin surface. 615. The nurse is teaching an adolescent about ex-

]]
ˆ


$

pected and adverse effects of applying prescribed
sebum production and turnover of follicular epi- salicylic acid. The priority teaching would be
thelial cells are increased, leading to the develop- instructing the patient to report which symptom?
ment of acne. a. Ringing in the ears
b. Flaking skin
12. Common misconceptions regarding non- c. Headache
drug treatment of acne include that vigorous 
#








_____________________ and avoiding certain
_____________________ can cure or prevent
acne.

Copyright © 2016, Elsevier Inc. All Rights Reserved.


270 CHAPTER 105 Q
&
$

{

16. A patient who has recently been prescribed ben- 621. Which effects should the nurse teach the patient to
zoyl peroxide calls the telephone triage nurse in a report to the prescriber when prescribed isotreti-
  
$
  



- 
$

ƒ
={

 
?
encing scaling and swelling at the site of applica- a. Back pain and muscle stiffness
tion. What would an expected protocol for the b. Burns easily when out in sun
nurse’s response normally include? c. Frequent nosebleeds
a. Continue to use the benzoyl peroxide and try d. Missed menstrual period
an oil-based moisturizer to relieve the severe e. No longer interested in normally pleasurable
dryness. activities
b. Continue to use the benzoyl peroxide as pre- f. Peeling of skin from palms and soles
scribed because this is an expected therapeutic 
{
  
$

  


effect. vision
c. Continue to use the benzoyl peroxide, but use it
less often. š
^^
!
 
 
Z  

  
$


{
 

@
 acne treatment for a 17-year-old female patient
who also desires contraception. Developmentally,
17. When taking a history from a new patient being which effect of the estrogen component of this
seen for acne, it is a priority to alert the prescriber drug is most likely to be a concern to this patient?
that the adolescent has a history of a. Anorexia
a. asthma. b. Nausea
b. ear infections. 
{
 
c. Hashimoto’s thyroiditis. d. Weight gain
d. tendonitis.
 23. A patient has been prescribed Ortho Tri-Cyclen
18. A 56-year-old patient asks what the nurse can 
 
= ?
$
 




 

=%?


and treatment of acne that has been resistant to
wrinkles. The nurse could share the research that  
  
#

 

 


suggests that tretinoin patient to report which symptom that suggests


a. causes the skin to feel softer. hyperkalemia related to this therapy?
b. eliminates deep wrinkles. a. Weakness and nausea
c. prevents sunburn. b. Bone pain and constipation
d. repairs sun-damaged skin. c. Cool, clammy skin

! 

 


]‡

 


  
  
=&$$?

$


 = ?


 
24. Why does the nurse teach patients to use sun-


  ƒ
={

 
? screens that contain avobenzone?
a. An increase in acne lesions is common early in 
#

$$



 


therapy. the nose.


b. Apply a sunscreen, such as zinc oxide, before 
#



 
 

ˆŠ]

applying the cream. rays.



Z

 

   
  

#


  


 
$ 

 
   
#

 
$



‚
  

{


$


 



of the skin occurs. 25. Recent research suggests that psoriasis is caused

ˆ 

 

 

  by what?


  
 

^`
#

 

 
 

 


Z 

$
 
  
@ 

=@?




Z 

$
 
 
={

 
? d. Poor hygiene
a. after washing.
b. evenly but avoiding the eyes, nose, and mouth.
c. only to active lesions.
d. to every area that is red.

Copyright © 2016, Elsevier Inc. All Rights Reserved.


CHAPTER 105 Q
&
$

{ 271

26. A patient asks the nurse why he should not use a 31. A history of which condition would be a contra-
high-potency glucocorticoid on his face. The basis indication for treatment of psoriasis with adalim-
of the nurse’s response is that the face is especially 
=* ?
 
=Z ?


prone to what effect of topical glucocorticoids? = ?



 
={  ?ƒ
a. Acne-like eruptions a. Bipolar disorder
b. Changes in pigmentation 
\    

  
=\Z&?

{ 

  c. Hypothyroidism
d. Thinning of the skin 

  
=!–?

 27. A patient is admitted with fatigue, nausea, vomit- 632. The priority nursing intervention to prevent squa-
 

  
‰
 
 

+ + mous cell carcinoma is teaching patients to
]†—
Z"'‰
+
†
Z"'‰

> ++
]^
 '‰
a. avoid chronic exposure of the skin to sunlight.
!
 


 


 
b. examine moles for irregular borders.
 


 ‚
 
  

 
#

c. report rough, scaly, red-brown papules to the
important for the nurse to determine the dose and primary care provider.
frequency of medication if this patient is being d. use sunscreen.
treated for psoriasis with which drug?
a. Anthralin 33. The nurse in a long-term care facility is admin-

> 
=&?  
 
 
=>  ?




c. Tars keratosis lesion. Which assessment suggests that



! @ 
=! @ ? therapy has achieved the desired effect and treat-
ment can be stopped?
 28. A 28-year-old patient is prescribed anthralin. a. Burning and vesicle formation
Which statement, if made by the patient, suggests b. Complete healing of lesion
a need for additional teaching? 
Z 
 

 

’#



  
 $

 


{
  

 
hands after applying the cream.”

’#

 
$
#


 


“ 34. What is an important nursing teaching for patients

’#




 

 



 
$ 

={  @?

"

stain my clothing.”  


=  ?ƒ

’#


 

  
$


a. Minimize exposure to sunlight.
around my psoriasis becomes red.” b. Cover the treated area with an occlusive dress-
ing.
 29. Which laboratory result for a 57-year-old female c. Discontinue treatment if redness or itching oc-
patient who is receiving methotrexate for psoriasis curs.
would be of most concern to the nurse? d. Be aware that the medications may stain cloth-
a. Alanine aminotransferase 35 international ing.
 '‰

*  
]^
'‰ 635. Which laboratory result for a 27-year-old patient,
c. Platelet count 2000/mm3 who has an appointment to receive podophyllum




=–>?

‡```'3 =|%^?
$
 
  


 

important for the nurse to report to the gynecolo-


 30. The nurse would be especially vigilant about gist administering the treatment?
monitoring the cardiovascular system of a patient a. Alanine aminotransferase 35 international
who has severe psoriasis being treated with  '‰

={  ?




 
$
 

>\
 
condition? 
*  

'‰

>
 
 
  
=>}|&? 



=–>?

```'3

& 

=&€?

\    

  
=\Z&? 36. What is the primary action of deodorant?

}  
=}? a. Cover odor with a pleasant fragrance

&  
”
  
=& &? 
& 

$




 

glands

# 
 
$


# 


$
$ 


Copyright © 2016, Elsevier Inc. All Rights Reserved.


272 CHAPTER 105 Q
&
$

{

37. The principle of treatment for seborrheic dermati- 5. The patient reports that she has always used a sun-
tis is suppression of 
$

 
{|…
†`

 

  

a. cellular reproduction. because she enjoys activities out in the sun. What
b. growth of yeasts. teaching about skin protection should the nurse
c. histamine release. provide to this patient?
d. scalp oil production.

DOSE CALCULATION QUESTIONS


Case Study 2

†<
#
= ?
]`


  
!





^`

$
`‡œ
+ >


An 18-year-old female high school senior is being seen
infused over 3 hours. What is the rate of infusion 

  
$
$
 

 -

‰'ƒ  
@


=%
€?


  
=&$$?


 
^
 
 
$ 


provide acceptable results.

†‡
ˆ 
={  ?




$

‹
 '`
‰
|  

‡`
 
 


6. What data needs to be collected when the pre-
correct dosage? scriber is considering treatment with isotretinoin?

CASE STUDIES
7. The dermatologist has decided to prescribe
Case Study 1  




$
 

-
!
 

 

 




force teaching regarding this drug. What are some
when collecting data from a 46-year-old female patient nursing diagnoses that should be included when
who is being seen in the dermatology practice. The patient planning teaching relating to this patient, diagno-

 
$
 

= ?



$
sis, and treatment?


{

$


 
 

1. What information can the nurse provide?

8. How should the nurse respond to the patient’s


question of how isotretinoin therapy differs from
tretinoin?
2. The patient is concerned about cost and asks if
tretinoin is better than face creams that you can
buy in the drugstore. What does research suggest
on this topic?
9. Because patients do not always remember what is
explained to them, what teaching methods besides
oral explanation should the nurse provide to this
patient?
3. The patient and prescriber agree that tretinoin
= ?

 
$

 
 

information can the nurse provide when the patient





$$
$


=%

€?
  
$

‚
ƒ 10. What teaching must the nurse provide this patient
 

|‰Z&\Z
  ƒ

4. What teaching should the nurse provide about use


of tretinoin products?

Copyright © 2016, Elsevier Inc. All Rights Reserved.


CHAPTER 106 Q
&
$

Z  273

106
Drugs for the Ear

STUDY QUESTIONS CRITICAL THINKING, PRIORITIZATION,


AND DELEGATION QUESTIONS
Completion
1. The visible outside part of the ear is 11. The nurse is caring for a 2-year-old child. When
called the ____________________ or bathing the child, the nurse notes sticky, crusting
____________________.  


 
 
 
  
!

nurse would use which term when documenting


2. The auditory canal and auricle make 
 ƒ
up the ____________________ a. Otalgia
____________________. b. Otitis media
c. Otorrhea
3. The malleus, incus, and stapes transmit sound
d. Otosclerosis
vibrations from the ____________________ to the
____________________ ear.

]^

 






4. The __________________ __________________ amoxicillin would be a reason for the parent to


allows air pressure within the middle ear to equal- contact the prescriber?
ize with air pressure in the environment. a. Allergy to clarithromycin
b. Diarrhea
5. The __________________ __________________ c. Facial rash that is maculopapular and does not
provide our sense of balance. itch
d. Multiple intensely itchy vesicles on the chest
True or False and back
   
  T   F
  
]†
 


 
  
 

-
lactic antibiotic therapy for recurrent otitis media

—

Z 
 
=?



$
is not currently recommended?
the auditory canal every time we bathe. 
#
 


$
 
  -
tance.

~




=}€?
 

  



#


 
$

 
 


blockage of the eustachian tube.

#
$"
  
  

<

| 
$
 
 
=   ?
  
- 
#


 
ous complications have occurred.

‡

#

 
 


}€

14. Recommendations for acute otitis externa include
is more important to treat the pain than the instilling into the ear canal a 2% solution of the
infecting microbe. acid found in
a. alcohol.
10. ___ AOM primarily occurs in children ages 5-12
b. oranges.
years.
c. peroxide.
d. vinegar.

Copyright © 2016, Elsevier Inc. All Rights Reserved.


274 CHAPTER 106 Q
&
$

Z 

 15. The nurse is doing telephone triage in a primary 2. The patient’s mother is upset and complains to the
 
$

 
 
 
$
 
 
$
 
 

 


 

Which information would warrant the patient be- in the past when her older child had an ear infec-


 ƒ
={

 
? tion. What should the nurse include in the explana-

Z
 
  tion of why antibiotics are not being prescribed?

# 
 
c. Pain when the outer ear is touched
d. Purulent otic discharge

ˆ 


  

 
 
  
3. What drug treatment should be provided for this
child?
16. Which vaccination has been shown to decrease the
risk of AOM?
a. DTaP
b. MMR
c. OPV 4. What symptoms should the nurse teach the pa-
d. PVC tient’s mother to report regarding the need for the
child to be brought back for reevaluation?
17. Amoxicillin-clavulanate is used for antibiotic-
resistant AOM because the clavulanate
a. allows for a lower dose of amoxicillin.
b. decreases incidence of adverse effects.
5. The nurse notes that this child has had multiple
c. increases antibacterial action against ) -
episodes of otitis media. What teaching can the
 
 
nurse provide that may decrease the incidence
d. prevents destruction of the antibiotic by bacte-
of otitis media for this child and the new sibling
rial enzymes.
when born?

DOSE CALCULATION QUESTIONS


18. A child who was born prematurely and weighs Case Study 2
10 lb at 3 months is prescribed amoxicillin 200
mg to be administered twice a day for AOM. The A female pediatric patient has been prescribed Cipro HC
pharmacy supplies an oral suspension of 250 mg/ drops for otitis externa.
‰
#

 
$
$

†%%
ƒ
*

much amoxicillin should be administered? 6. What information should the nurse include when
teaching about administration of these drops?

19. Amoxicillin 750 mg is prescribed for a 38-kg



#

 
 ƒ
*
 

should be administered if the drug is available as 7. What technique should the nurse use to evaluate
``
 ']`

ƒ the learning if the child’s parents do not appear to
understand oral instruction?

CASE STUDIES
Case Study 1 8. The patient’s parents ask if they can use old drops
A pregnant woman has brought her 2½-year-old son to from a previous infection. What should the nurse
the pediatrician because he was fussy all last night. The include in the response?
 
 
 
=|+|?
  
}€


does not prescribe an antibiotic.

1. What factors did the PNP consider when deciding 9. The nurse notes that the patient has had repeated
whether to treat with an antibiotic? episodes of otitis externa. What teaching can the
nurse provide that might decrease the number of
episodes?

Copyright © 2016, Elsevier Inc. All Rights Reserved.


CHAPTER 107 Q Miscellaneous Noteworthy Drugs 275

107
Miscellaneous Noteworthy Drugs

STUDY QUESTIONS  8. Which of these laboratory test results, if identi-



$

$ 
 



  

Matching =! ?




 



  

immediately?
    
    
‰!
—`
#ˆ'‰
  #!"*$  # $ 
> 
]‹
 '‰
c. Hbg 12.8/ Hct 37
1. ___ Derivative of prostacyclin that promotes 
>\
 
vascular relaxation, suppresses growth of
vascular smooth muscle cells, and inhibits 9. Which drug for type 2 diabetes mellitus is contra-
platelet aggregation indicated when a patient is prescribed bosentan
2. ___ Reduces pulmonary arterial pressure by =! ?ƒ
causing dilation of pulmonary blood vessels 
\

†

# 
 



  

\@
smooth muscle 
\ 
4. ___ Reduce pulmonary vascular resistance and d. Metformin
may have a favorable impact on vascular
remodeling  ]`
!
 

‰!
 
$

 
-
5. ___ Prevents thrombosis 
  
=! ?

‡
  
 '‰

—





 

What should the nurse do if it is time to administer
cardiac preload the drug?
a. Administer the drug as prescribed.
a. Ambrisentan and bosentan b. Assess for nausea, vomiting, fever, jaundice,
b. Amlodipine, felodipine, and nifedipine and fatigue.

Z 
 

  c. Withhold the drug and consult with the pre-
d. Furosemide and hydrochlorothiazide scriber regarding a dose decrease or interrup-

{ 

    tion.
f. Warfarin d. Withhold the drug and consult with the pre-
scriber regarding discontinuing the drug.
CRITICAL THINKING, PRIORITIZATION,
11. Which statement, if made by a patient who has
AND DELEGATION QUESTIONS ” 

  
 
= ?
$
|*

would warrant teaching to prevent a possible fatal


š
~
 



 
 


 
reaction?
6  
|*



 
=Š  ?ƒ

’#

#

 


#
$


a. Change positions slowly.


not have enough energy.”
b. Cough and deep-breathe.

’#
 


#


  
  
#

c. Do not cross your legs.


also heard that it is good for my heart.”

ˆ 
 %
  

’#
 


 
 
 



tongue that the doctor prescribed for my wife



#

 
 “
d. “My hearing is not as good as it used to be.”

Copyright © 2016, Elsevier Inc. All Rights Reserved.


276 CHAPTER 107 Q Miscellaneous Noteworthy Drugs

š
]^
!
 

 
   
=#€?
17. A patient has come to the outpatient oncology
6  dexamethasone to a pregnant patient who is at risk 
$

$ 
$
  
=Z?

for premature delivery of the fetus. The patient’s Which symptom suggests that the patient may be
skin is hot and dry, and the patient is voiding 200 experiencing methemoglobinemia?

†``




  
 
 
a. Cyanosis
is of greatest priority for the nurse to monitor in b. Fever
this situation? c. Nausea and vomiting

–ˆ+ d. Rash
b. Creatinine

\  
]<

 



  
=>&?


ˆ

  who receives hemodialysis is prescribed calcium
   
=| ‰?



  
!

š
]†
!
 
$
 

$
  

$

achieve maximum binding of phosphate and mini-
6   
 

  
={  ?ƒ mal absorption of the calcium, the drug should be

#$$
 
   administered in which way?

&

 a. First thing in the morning
c. Hypothermia b. At least 1 hour before meals

#$$
 %$ c. At least 2 hours after meals
d. With meals
 ]‹
…
  
 
  
 
$
=|-
@?


 

 

19. The nurse is caring for a patient who is prescribed
 
  


ƒ  
*>
= ?


 -
a. After chest physiotherapy phatemia of CKD. Which respiratory change
b. After meals suggests that the patient may be experiencing
c. Before chest physiotherapy metabolic acidosis?
d. Before meals a. Hyperventilation
b. Diminished lung sounds
 15. The nurse is caring for a 15-year-old female pa- 
‰
 
 
=  ?
tient who has been admitted in a sickle cell crisis. 
{
 
=?
!
 
 
$

”
 
=]`
$

]`?
!
 

 

 


"- 20. Which laboratory test result would be of most



 


  
# - concern to the nurse when a patient is prescribed
nous morphine 0.5 mg and oral ibuprofen 200 mg  %  
=\*–?
=?
$
  -
are ordered every 3 hours as needed. The patient plexy associated with narcolepsy?
has not had any analgesic medication in the past 3 
–+|
]~`
   '‰
hours. What should the nurse do? 
>\
]^
#ˆ'‰
a. Administer the ibuprofen. 
*&‰

 '‰
b. Administer the morphine. 
#+
]]
c. Assess for other physiologic indicators of pain.
d. Provide nonpharmacologic pain relief mea-
^]


 


@
=?
$

sures. 
  
 
=‰{?


 
-
portant for the nurse to assess for which symptom
 16. A 16-year-old patient with sickle cell anemia has of the underlying condition?

  

=& ?

a. Dizziness
nursing problem would be of most concern relat- b. Nausea and vomiting
ing to this drug therapy? c. Respiratory distress
a. Adherence to prescribed regimen d. Weakness
b. Body image

#$
d. Tissue perfusion

Copyright © 2016, Elsevier Inc. All Rights Reserved.


CHAPTER 107 Q Miscellaneous Noteworthy Drugs 277

22. Which question is of greatest priority when a CASE STUDY


 

  
  @
= @?
$

treatment of chorea associated with HD? A 22-month-old African-American pediatric patient is ad-
a. “Are you concerned about weight gain when mitted to a pediatric hospital because of fever, irritability,
you take medications?” ” 


   
‰  
 
 

b. “Are you having any thoughts about doing  




 
$
  
{


anything to harm yourself?” the diagnosis of sickle cell disease. The patient’s parent
c. “Do you experience dizziness when you stand asks the nurse why the patient’s eyes are so yellow.
up?”

’* 

 

$
 ƒ“ 1. What would be the logical explanation for jaun-
dice in sickle cell disease?
 23. Which data would warrant consulting the
prescriber before administering milnacipran
={  ?ƒ
a. BP 100/64 mm Hg

\…
‹^
‰' 2. The parent has heard of a new drug, hydroxyurea,

#+
`‡ and asks why it has not been prescribed for the
d. Pulse 62 beats/min child. What is the best explanation?
a. The drug has adverse effects.

^‹


$
 
 
=*Z?
b. The drug has only been approved for patients
have a possible adverse effect of anaphylaxis? who are age 18 and older.
={

 
? c. The drug is an orphan drug.
a. Berinert d. The drug is only used as a last resort.
b. Cinryze
c. Firazyr 3. The parent asks the nurse if anything can be done
d. Kalbitor to keep the child from experiencing sickle cell
e. Winstrol crises. How should the nurse respond?

DOSE CALCULATION QUESTIONS


25. The recommended dose of belimumab is 10 mg/
kg. What is the dose for a patient with systemic

  
={‰Z?

 
]†~
 ƒ

26. A patient is prescribed 0.2 mg/kg of rasburicase.


What is the dose for a patient weighing 165 lb?
!
 


$


`

0.9% normal saline to infuse over 1 hour. The


$ 


   

‰'
 
 
$

infusion should be programmed into the pump?

Copyright © 2016, Elsevier Inc. All Rights Reserved.


278 CHAPTER 108 Q
& 
{

108
Dietary Supplements

STUDY QUESTIONS 16. ___ Dietary supplements have been found to not
contain the stated ingredients in the amount
Matching listed on the label.
17. ___ Dietary supplement labels may claim to treat
     
  

  

" 
  

supporting the claim.


1. ___ Benign prostatic hyperplasia
]<

!



 
 

2. ___ Reduce triglycerides supplements are effective or safe.


3. ___ Depression

‹

# 
 %$
 
  
5. ___ Prophylaxis of migraine CRITICAL THINKING, PRIORITIZATION,
6. ___ Treat vertigo AND DELEGATION QUESTIONS
7. ___ Treat symptoms of menopause

<

{ 

$
]‡
>
\
€ $ 
| 
=>\€| ?

9. ___ Promote sleep are designed to ensure that dietary supplements


10. ___ Treat constipation a. list their active and inert ingredients.
b. meet the needs of all socioeconomic groups.
a. Black cohosh c. are generic formulations, not brand name.

Z  d. are used to promote health.
c. Feverfew

\  620. The priority teaching when a patient chooses to

\ 
 use a dietary supplement is that it is best to
$
\ 
 a. always buy the more expensive brand.
g. Flaxseed b. take the herb with at least 8 ounces of water.

{
 ‚
 c. use products that have the seal of approval from

{ 
  
ˆ{|
j. Valerian d. use the lowest dose possible.

True or False 621. What is a priority nursing action when a nurse


discovers that a patient self-prescribes herbal
   
  T   F
products?
  
a. Assess for drug interactions and adverse effects
of the product.
11. ___ Natural products are always better and safer
b. Be an advocate for the patient’s right to make
than synthetic products.
choices.
12. ___ Products marketed as dietary supplements
c. Convince the patient to stop taking the prod-
do not require rigorous evaluation before
ucts.
marketing.
d. Teach the patient the importance of informing
13. ___ The FDA cannot remove a dietary supple-
all health care providers of all products being
ment from the market even if there is evi-
used.
dence that it causes harm.
14. ___ Dietary supplement labels can insinuate that


 
 
15. ___ Dietary supplements have been found to
contain harmful ingredients such as arsenic,
mercury, and lead.

Copyright © 2016, Elsevier Inc. All Rights Reserved.


CHAPTER 108 Q
& 
{ 279

22. A patient who is taking metronidazole is at risk for  28. The nurse is assessing a new admission. The

  %
=  ?
 
$
 
 
patient lists echinacea, garlic, and kava among the
an herbal product in which form? 
 

 
  
}
 
  -
a. Decoctions tion, the nurse notes that the patient’s skin has

Z 


 
 a yellowish color. What is the most appropriate

{
  action for the nurse to take next?
d. Teas and infusions a. Continue to assess the patient.
b. Document jaundice in the nurses’ notes.
623. Which laboratory test would be a priority for the 
+$

 
 

$



nurse to monitor if a patient with heart failure was {!!


prescribed warfarin and self-prescribes coenzyme d. Tell the patient to stop taking the herbal prod-
˜%]`

 
  
 

ucts.
and/or ginkgo biloba?

‰! 29. A patient who regularly takes feverfew to prevent
b. CBC migraine headaches stopped taking the herb before

#+ elective surgery. The nurse should assess the pa-

{ tient for what?
a. Bleeding
24. Research suggests which possible effect of cran- b. Flatulence
berry juice? 
# 

# 

$
 d. Nausea
b. Making urine acidic, which kills bacteria
†`

 


  
@
=\?

c. Preventing bacteria from adhering to the blad- $



^
 



=>-
der wall ?
$
 
”
$

 
 

d. Preventing kidney stones she has started taking ginger because she read in

 @
 


 
#





š
^
– 

\ 
 


 
$

teach the patient to be alert for possible hypogly-
nurse to teach women prescribed antihyperten- cemia symptoms, including


 
 
 
$

 

a. dry, itchy skin.
insomnia to b. headache.
a. avoid prolonged exposure to the sun. c. loss of appetite.
b. change positions slowly. d. palpitations and sweating.


" 

$
 
d. stop taking the herb if headache occurs. 6†]
#




$
 


 -
der that ginkgo biloba can aggravate their condi-
26. Pregnant nurses should wear gloves if they are tion?
administering which herbal supplement? a. Diabetes mellitus
a. Aloe b. Hay fever
b. Feverfew c. Hypertension

\  
{@

{ 
 
32. What is an appropriate outcome for a 12-year-old

^~
 



  ƒ
={

 
- child relating to probiotics prescribed during a
? rotavirus infection?

#
 
  

   
{$
$


#
 


 
 

 
b. No bacterial growth in urine
 

 

 



  c. Temperature 98° F to 99° F

#
 



 

 % d. No adventitious lung sounds
term use.

#
 


$


 633. What is a priority concern for the nurse when

#


$


 a patient with multiple medical disorders self-
  
{
 ‚
ƒ

Z$$


  

b. Constipation
c. Photosensitivity
d. Potency of the preparation

Copyright © 2016, Elsevier Inc. All Rights Reserved.


280 CHAPTER 108 Q
& 
{

34. The nurse is aware that which herbal supplement 3. The patient insists on using raw garlic. What infor-
has been associated with liver failure? mation should the nurse provide about raw garlic
a. Black cohosh therapy?

\ 
c. Kava
d. Valerian

635. The nurse is caring for a patient who admits to us- 4. Why is it important for the patient to inform all

 


 
= ?
$
 
 
health care providers and his pharmacist that he is
#



$

 


 ƒ using garlic therapy?
a. Assess mental status.
b. Monitor vital signs.
c. Not administer any other sedatives.
d. Weigh the patient daily.
Case Study 2
CASE STUDIES 
<% %
 

 
  



 

 
=‰ ?

 $ 
=> ?

Case Study 1 At times, she is fatigued and forgetful. Her friend suggests
that she take ginkgo biloba to give her more energy.
A 52-year-old man is admitted with chest pain. Assess-



 
‚
<“
 
^^`
 
  
5. What possible complications can occur with this
circumference 42”, BP 145/86 mm Hg, and pulse 78 combination of drugs and herbs?
  '
‰
 

 
†<`
 '‰

‰&‰
^‹`
 '‰

*&‰
†‹
 '‰
*

  


   

  
=\Z&?

 -
bolic syndrome. He informs his health care provider that
he wants to try natural therapy and has heard that garlic 6. What teaching should the nurse provide this pa-
can help many things. The health care provider asks the tient regarding the use of herbal products?
nurse to discuss garlic therapy with this patient.

1. What are possible positive effects of garlic supple-


mentation for this patient?
7. The patient informs the nurse that she is not




 


  



purchased a bottle of echinacea. How should the


nurse respond?
2. The health care provider recommends enteric-
coated garlic supplements. What can the nurse
teach the patient to ensure that the product that he
purchases contains effective amounts of allicin?

Copyright © 2016, Elsevier Inc. All Rights Reserved.


CHAPTER 109 Q Management of Poisoning 281

109
Management of Poisoning

STUDY QUESTIONS 10. The nurse knows that the most accurate and ef-


$
$

 



Matching dose is what?


a. Analysis of urine, blood, and gastric contents
 
# $     
Z  
$
 

#)$ 
Z 

 
1. ___ Arsenic 
#

  
2. ___ Copper

†

Z

= $@? 11. What is the purpose of an antidote to a poison?

‹

\ 
# 
 

$

 



# b. Prevent absorption of the poison.

—

‰  c. Remove residual poison from the gastrointesti-
7. ___ Mercury  
=\#?
 
d. Counteract the effects of a poison.

&$ 
=& $ ?

& 
=–‰

? 12. Charcoal is most effective in binding with poisons

Z 
 
 
=> 


\#
 



$
-
Z&!? istered

…@
=@? a. after the patient has vomited.

| 
=&? b. for fat-soluble poisons.
$
{
=>? c. within 30 minutes of poison ingestion.
d. with milk.
CRITICAL THINKING, PRIORITIZATION,
 13. The nurse is assessing a 3-year-old patient who
AND DELEGATION QUESTIONS 

`
‰'
$



=\‰™!Z‰™?

  
=+\?
 
$

6 8. What is the initial priority of nursing care of a having ingested 60 tablets of adult iron supple-
patient with a suspected ingestion of a poison? 

 



$
 

a. Administering an antidote concern to the nurse?


b. Assessing airway, breathing, and circulation a. BP 80/52 mm Hg

#$

  b. Dark brown emesis
d. Preventing absorption of poison that has not c. Pulse 120 beats/min
been absorbed 
‹~

$
% 
"


 

hour
 9. A person with an unknown medical history arrives


 
 
=Z&?
$
  
š
]‹



  
 

$
‚



 
!
 

  
 

=  
?
!
Z&





Z&

 
=#Š?
resident informs the nurse that treatment will
line of normal saline is infusing, respirations are include enhancing renal excretion of the drug. The
unlabored at 17 breaths per minute, and BP is nurse would question orders for all substances
]]<'~^

* 

 

 - except
 
  ƒ
={

 
? a. acetic acid.

>  

^%†
 b. ammonium chloride.
b. Diaphoresis c. citric acid.
c. DTR 2+ d. sodium bicarbonate.
d. Dry mucous membranes
e. Pulse 108 beats/min

Copyright © 2016, Elsevier Inc. All Rights Reserved.


282 CHAPTER 109 Q Management of Poisoning

š
]

 


 
$
 

-  20. The nurse is caring for a 3-year-old child who
perfusion, which passes blood over an absorbent weighs 15 kg and is scheduled to receive an intra-
resin. The nurse should monitor the patient for   
 
$
 
 
Z&!
$
 

a. bleeding. poisoning. The child’s urinary output has averaged


b. dehydration. ]
‰'
$

 
‹
 
 



c. hyperglycemia. nurse do?


d. seizures. a. Administer the drug and continue nursing care.
b. Consult the prescriber regarding the need for
16. A patient is being treated for acute ferric iron poi-  
 


$ 
=& $ ?
!
 
c. Withhold the medication and consult the pre-
knows that this drug works by scriber.
a. binding with iron in the blood, causing excre- d. Withhold the medication and assess for renal
tion in the urine. failure.






\#
 


absorption.  ^]
!
 


 
Z&!



 

\#
  contact the prescriber if which laboratory test
d. inducing vomiting. result occurs?

 

'‰
š
]~

 
 

 
 
$

 


 
]‡`
  
 '‰


 
$
$

 



|
 
<†
'‰
$ 
=& $ ?


  

|


^<~
 '‰
that the administration rate of the drug might be
too rapid?  22. To detect serious adverse effects, it is most impor-
a. Blood pressure 150/85 mm Hg tant for the nurse to monitor the results of which
b. Pulse 112 beats/min laboratory test when caring for a patient with
c. Respirations 15/min Wilson’s disease who is receiving penicillamine

! 
]``‹„
…
=†<„
>? =&?ƒ
a. Alanine aminotransferase
18. The nurse is teaching a patient who has been b. Blood urea nitrogen
  
$ 
=Z” ?
$

 
c. Complete blood count and differential
associated with beta-thalassemia. Which state- d. Creatinine
ment, if made by the patient, suggests a need for
additional instruction? 23. A child who was admitted after ingesting anti-

’#


 





$@

 

$@
=@?


make sure that the drug dose and my kidneys symptom suggests that the child’s body is attempt-
are okay.” ing to compensate for metabolic acidosis?

’#


 

 





$
a. Decreased urine output
glass of juice.” 
# 



’#


 


$
#
 

$
c. Rapid, deep respirations
higher than 99° F.” 
{ 
 

’#$
#
 




$
  


won’t be absorbed.” 24. The nurse teaches parents that dialing 1-800-222-
1222 will connect them with what?
 19. Which would be an appropriate technique when a. A local poison center

 

 
 
=–‰

b. The national poison center
?ƒ c. A pharmacist
a. Deep injection into the dorsogluteal muscle d. A specially trained nurse

{  
”

 
$ 

{  
”


 
  -
lis DOSE CALCULATION QUESTIONS

¢% 
”


  
 

^
+ 
]


  






heroin overdose. Available is naloxone 0.4 mg/


‰
 



$
 


-
istered?

Copyright © 2016, Elsevier Inc. All Rights Reserved.


CHAPTER 110 Q Potential Weapons of Biologic, Radiologic, and Chemical Terrorism 283

^—
> 
Z&!
]




``

3. The parents are sure that the ingestion of the
$
 


#



$ 

 


 
†`


\ -
‡`
 
 
 

‰'


- tric lavage and aspiration is ordered. What is the
 



ƒ nurse’s role?

CASE STUDY
A 4-year-old girl, who has been playing outside in her 4. Why is the child positioned on the left side with
yard, comes in the house and says she doesn’t feel well. her head down?
Her parents notice a purple stain on their daughter’s face.
Knowing that they do not have any grape juice in the
house, the parents ask their daughter what she has eaten.
The child tells them about some berries she found in the
yard. The parents call the poison control center, and the


 


Z&
$

   5. The child is stabilized and observed overnight.
What instruction should the nurse provide this
1. When the nurse questions the parents, they family before discharge?
describe “weeds” growing at the back of the
yard with blueberry-size purple berries that their
daughter may have ingested. What should be done
to identify the ingested substance?

2. The stomach contents contain a noncaustic neuro-


toxin. What data can the nurse collect to assist the
emergency physician with deciding if charcoal,
gastric lavage, or whole bowel irrigation is the
best approach for this poisoning?

110
Potential Weapons of Biologic, Radiologic, and Chemical Terrorism

STUDY QUESTIONS 3. ___ Approximately 20% of patients who con-


tract cutaneous anthrax die despite antibiotic
True or False therapy.

‹

# 
 

 


   


  T   F and spores.
   5. ___ Research has proven that raxibacumab neu-
tralizes deadly anthrax toxins in humans.
1. ___ Anthrax spores can live outside of a host for 6. ___ Antibiotic treatment of cutaneous anthrax is
many years. not likely to prevent skin lesions and necro-
2. ___ Respiratory anthrax can cause hemorrhage sis.

  
$

 
$


brain.

Copyright © 2016, Elsevier Inc. All Rights Reserved.


284 CHAPTER 110 Q Potential Weapons of Biologic, Radiologic, and Chemical Terrorism

~

#  
 
$
 
 

š
]
#   

`



 


administered to immunocompromised pa- prescribed for a 150-lb man who has been diag-
tients. nosed with tularemia. Nursing assessments before
8. ___ Anthrax vaccination can be administered administration of the drug include BP 140/72 mm
before and immediately after exposure. Hg, pulse 88 beats/min, respirations 24/min, moist
9. ___ Bubonic plague is not transmitted person to cough, and end inspiratory crackles in upper lobes.
person. What should the nurse do?

]`

{ 
 




a. Administer the drug and consult the prescriber
with the live virus. regarding the low dose.

]]

Z 
 

 

$
$- b. Administer the drug as one injection in the
fective when treating smallpox. dorsogluteal site.

 

 %$
  
 


administering the drug.


CRITICAL THINKING, PRIORITIZATION, d. Withhold the drug and consult the prescriber.
AND DELEGATION QUESTIONS
16. A patient is admitted with a diagnosis of pneu-
12. Which symptom would be a priority to report to monic plague. Which precaution would be best to
the prescriber if a patient is suspected of being prevent spread of this infection?
exposed to ,    3 a. Contact precautions
a. Blurred vision b. Droplet precautions
b. Diarrhea 
# 
  
$

 
c. Hemoptysis d. Prophylactic vaccination of staff
d. Malaise

]~


 
 
  

13. Which statement, if made by a patient receiving smallpox infection?


 
$
 
 

- a. Headache, high fever, chills, and rigors
cate understanding of drug therapy teaching? b. Multiple red spots on the tongue and buccal

’#
 
 

$

 


mucosa
  
$
#





 
c. Painless ulcers with a necrotic core that de-
could occur.” velop black eschar

’#

 

 



$ 

!
  
 


off.”

’#$
#
 

 
 



18. The public health nurse is administering smallpox
 
 
#




 -  


$ 

 
$

cles.” made by a vaccination recipient, would indicate a



’€
 



  
$
#

need for further teaching?
pregnant.” 
’#







 -
ing the live vaccine.”
14. The community health nurse, who works in areas 
’#


 
 
$

  
-
known to have the presence of ,  , curs from the vaccination site.”
promotes routine vaccination with anthrax vaccine 
’#

 
 
#


  
$


=–! ?
$



 ƒ least 4 weeks after receiving this vaccination.”

|
$ 
’!
  


$$

$
#

b. Postal worker was exposed to smallpox yesterday.”


c. Receptionist

{
$ 
]‡
!
& 
$
* 

* 
{


 




 


 
=€Š?
  
$
>€^```

 
$

terrorist attack with smallpox. Why has MVA been


chosen?

#

$
$
 


#


 

 
$
  
 -
litis or cardiac problems.

#







…&

#


 


$$
 

Dryvax.

Copyright © 2016, Elsevier Inc. All Rights Reserved.


CHAPTER 110 Q Potential Weapons of Biologic, Radiologic, and Chemical Terrorism 285

 20. The nurse is preparing to vaccinate a patient who 25. The nurse knows that adverse effects of ferric
has been exposed to the smallpox virus. The   $ 
=|  
?
 

 

patient states that she is pregnant. What should the excretion of nonradioactive thallium can cause hy-
nurse do? pokalemia, increasing the risk of toxicity of which
a. Administer the vaccine. medication?
b. Administer cidofovir before administering the a. Acetaminophen
smallpox vaccine. b. Atenolol
c. Assess if the patient has been informed of ben- c. Digoxin


 
$
   d. Furosemide
d. Withhold the vaccine.

621. What is the highest nursing priority when caring DOSE CALCULATION QUESTIONS
for a patient with botulism poisoning?

Z$$
  26. A woman has potassium iodide 65-mg tablets. Her

# 
  2½-year-old daughter is exposed to radiation. How

{ 
 many tablets should she administer if the recom-
d. Visual acuity mended dose is 32 mg?

 22. The nurse is providing emergency care after a


disaster at a nuclear power plant that caused the
^~
> 
 
> %&|!

]




release of radioactive material. Which dose of po- 5% dextrose in water is to be administered over
tassium iodide should be administered to a patient †%‹
 
*
 



 -
who breast-feeds her infant? tered every 10 seconds if the drug is administered
a. None over 3 minutes and 20 seconds?
b. 65 mg daily
c. 130 mg daily
d. 195 mg daily CASE STUDIES
š
^†
!

  



Case Study 1
@
 
=¢%&!|?

 
 

 -
tient to include adequate amounts of which food? |


ˆ
{ 

$
  
$
-

Z thrax spores in October 2001. The nation was on alert for

‰ terrorist attacks.
c. Red meat
d. Whole grains 1. What are possible reasons for the infections not
being promptly diagnosed and treated in time to
24. Which statement, if made by a patient who has prevent all deaths?
been prescribed Ca-DPTA therapy for americium
poisoning, would suggest understanding of teach-
ing?

’#
 

 

$




not functioning properly.” 2. How can the nurse contribute to early recognition

’#


 



 
$

 
^
of infections caused by biologic agents?
months.”

’#


]``

$

  

water, each day.”



’#





 

 


every 8 hours.” 3. People who were possibly exposed to , -


cis were offered prophylactic drug therapy. What
teaching should be provided by the community
health nurses assisting with this mass prophylaxis?

Copyright © 2016, Elsevier Inc. All Rights Reserved.


286 CHAPTER 110 Q Potential Weapons of Biologic, Radiologic, and Chemical Terrorism

Case Study 2 9. What symptoms would the nurse expect to occur


 ƒ

 




Z&
  
$
 



unknown powder.

‹
#$

 

 

 



can the infection spread to other people? 10. What should the nurse assess to determine if the
most serious consequences could be occurring?

5. What precautions should the nurse take to prevent


the spread of anthrax infection? Case Study 4
Reports of ricin being used by terrorists cause a great deal
of fear.

Case Study 3 11. Why is ricin a logical weapon of bioterrorism?


The military nurse is participating in a drill imitating
exposure of soldiers to mustard gas.

—
 



 

 

 
12. What treatment would the nurse anticipate if a
should take? patient is admitted with inhalation of ricin?

7. How can the nurse prevent additional exposure to 13. What test would detect the early effects of ricin on
the agent for the soldiers and health care provid- the gastrointestinal tract?
ers?

8. What questions about the episode would the nurse


ask to determine which persons probably received
the most exposure to the gas?

Copyright © 2016, Elsevier Inc. All Rights Reserved.


Answer Key 287

Answer Key

Answers to the Case Studies 6. generic 30. c


can be reviewed online at http:// 7. brand 31. a (Most general anesthetics are
evolve.elsevier.com/Lehne/. 8. Generic excreted by exhalation.)
9. generic 32. b, c, d
CHAPTER 1 10. generic 33. b
1. b 11. d 34. d
2. d 12. b, c
3. c 13. a CHAPTER 5
4. a 14. b, d 1. T
5. Selectivity 2. F
6. Low cost CHAPTER 4 3. T
7. Reversible action 1. e 4. T
8. Freedom from interactions 2. c 5. T
9. Possession of a simple generic 3. b 6. T
name 4. f 7. F
10. Effectiveness 5. a 8. F
11. Ease of administration 6. d 9. T
12. Safety 7. c 10. T
13. Chemical stability 8. a 11. F
14. Predictability 9. b 12. T
15. c 10. T 13. T
16. a (The others are not within the 11. F 14. T
scope of nursing practice.) 12. T 15. a
13. T 16. a, d
CHAPTER 2 14. F 17. b, d, e
1. d 15. T 18. b
2. b 16. F 19. b (Patients must be closely
3. f 17. F monitored for recurrence of opi-
4. g 18. T oid toxicity when the antagonist
5. a 19. F effects of naloxone fade; the
6. e 20. T half-life of naloxone is 2 hours
7. c 21. F and effects persist for about an
8. b 22. T hour.)
9. d (Suggests possible anaphylac- 23. T 20. a
tic reaction.) 24. T
10. c 25. F CHAPTER 6
11. d 26. T 1. T
12. b (These assessments are a 27. d (Continued administration of 2. F
priority, as well as being needed morphine would lead to respira- 3. T
to determine if prescribed tory arrest.) 4. F
pain medication can safely be ^<

=!
 
 



5. F
administered.) much additional heparin is to be 6. T
13. d administered in order to prevent 7. a (Combining metronidazole
DVT; since the nurse has real- and alcohol leads to acetal-
CHAPTER 3 ized the error just after admin- dehyde syndrome manifested
istration, assessment for DVT by nausea, copious vomiting,
1. b
is not a priority; the incident   
   
  

2. a
report would be completed after sweating, thirst, chest pain,
3. c

 ?
weakness, blurred vision, and
4. generic
29. a hypotension; blood pressure
5. chemical

Copyright © 2016, Elsevier Inc. All Rights Reserved. 287


288 Answer Key

may ultimately decline to shock once the previous patch has CHAPTER 11
levels.) been removed.) 1. F
8. d ‡

=\—|&

$$
2. T
9. c the oxygen-carrying red blood 3. F
10. d (By inhibiting the effects of cells; when persons with this 4. F
clopidogrel, the use of omepra- 
 
 
 
5. T
zole can lead to reduced anti- they develop hemolytic anemia 6. T
platelet effects and increase the [red blood cells are destroyed 7. F
risk of recurrent MI.) faster than the body can replace 8. T
11. b them].) 9. T
12. c 10. a 10. c (Creatinine clearance is a
13. c 11. c (These are signs of more precise test used to help
14. a hypokalemia.) detect and diagnose kidney
dysfunction.)
CHAPTER 7 CHAPTER 9 11. a
1. c 1. T 12. d
2. f 2. F 13. b (Methyldopa can cause ortho-
3. b 3. T static hypotension and brady-
4. g 4. F cardia in older adults.)
5. a 5. T 14. c (Is an indicator of severe
6. e 6. F urinary retention requiring
7. physically dependent 7. T catheterization.)
8. allergic 8. T 15. b
9. Carcinogenic 9. F
10. teratogenic 10. T CHAPTER 12
11. side effects 11. d 1. i
12. idiosyncratic 12. f 2. f
13. Iatrogenic 13. a 3. b
14. Toxicity 14. g 4. c
15. d 15. b 5. d
16. a, d, e 16. c 6. a
17. c (Suggests possible anaphylac- 17. h 7. e
tic reaction.) 18. e 8. h
18. d 19. d 9. g
19. c (Statins can cause hepatotox- 20. a 10. T
icity; liver injury is evidenced 21. b 11. F
by elevations in serum trans- 22. c 12. T
aminase levels.) 23. c 13. F
20. a 14. F
21. d CHAPTER 10 15. T
22. b 1. F 16. a, b, c
23. c 2. T 17. b
24. c 3. T 18. d
25. a 4. F 19. a, c, d
26. d 5. T 20. b
6. F
CHAPTER 8 7. b CHAPTER 13
1. body surface area 8. a 1. e
2. increase 9. c (Normal respiratory rate for a 2. b
3. Bioavailability newborn is 30-60 respirations 3. d
4. Tachyphylaxis per minute; a respiratory rate of 4. c
5. bleeding 22/min is depressed.) 5. a
6. intense; longer 10. a 6. skeletal muscle
7. c 11. a, c 7. pupillary constriction; focuses
8. d (Tolerance to nitroglycerin- 12. b eye
induced vasodilation can 13. a 8. bronchodilation; respiratory rate
develop over the course of a 14. 227 mg is calculated safe dose. 9. slowing
single day; the prescriber must Yes, 225 mg is safe. 10. motility (peristalsis); digestion
be contacted to determine when 15. 4.5 mL 11. emptying (voiding)
the patch should be reapplied, 12. skeletal muscle

Copyright © 2016, Elsevier Inc. All Rights Reserved.


Answer Key 289

13. c CHAPTER 15 18. b


14. b 1. c 19. 45 mL/hr
15. a (Cholinesterase inhibitors can 2. g 20. Dilute the vial in at least 50 mL
cause bradycardia.) 3. f of diluent and infuse at a rate of
16. d 4. b 3 mL/hr.
17. b (Principal adverse effects of 5. h
doxazosin are hypotension, 6. a CHAPTER 18
fainting, dizziness, somnolence, 7. e 1. b
and nasal congestion.) 8. d 2. c
18. c 9. b 3. a
19. a (Flomax may increase the risk 10. a, e 4. T
of cataract surgery-related com- 11. a, b 5. T
plications due to relaxation of 12. a (Neuromuscular blockade 6. F
the smooth muscles of the iris.)  


7. F
20. b (Inhibiting MAO increases the [coughing, gagging]; in the 8. F
risk of triggering a hypertensive period after surgery, the loss of 9. T
crisis.) muscle strength resulting from 10. T
blockade can lead to aspiration.) 11. T
CHAPTER 14 13. c 12. c
1. a. administer 14. d 13. a
b. consult 15. 2.5 tablets per dose 14. b
c. consult 16. 2.8 mL of neostigmine 15. c (Terazosin can cause ortho-
d. consult static hypotension [a drop in
e. administer CHAPTER 16 {–|
$
³
^`

* ¤



f. consult 1. c tachycardia [increased heart


g. administer 2. b rate in response to stimulus
h. administer 3. e conveyed through the cardiac
i. consult 4. d nerves].)
j. consult 5. a ]—

={ 

 


2. c, e 6. a combination with alpha block-


3. b (Prominent symptoms of 7. d ers, may cause a severe drop in
muscarinic poisoning are 8. c, d blood pressure, lightheadedness,
profuse salivation, lacrimation 9. c dizziness, or fainting.)
[tearing], visual disturbances, 10. c 17. a
bronchospasm, diarrhea, brady- 11. a, d 18. b
cardia, and hypotension. Severe 12. d (Succinylcholine produces a 19. c
poisoning can produce cardio-  
$
 
    Œ
 - 20. b
vascular collapse.) ratory depression secondary to 21. a
4. d neuromuscular blockade is the 22. a (Betaxolol, although preg-
5. d major concern.) nancy category C, crosses
6. a 13. 250 mg the placenta and may lead to
7. d (Compared with other drugs bradycardia in the neonate; the
for OAB, trospium is notable CHAPTER 17 normal pulse for a newborn is
for its low bioavailability, lack 100-160 bpm.)
1. a
of CNS effects, and lack of 23. c (While metoprolol is approved
2. c
metabolism-related interactions for treating heart failure, it can
3. b
with other drugs.) also cause heart failure if used
4. d
8. c incautiously.)
5. T
9. c, d 24. a, b, c, d, e
6. F
10. a, c 25. b
7. T
11. a 26. d (Blockade of beta2 recep-
8. F
12. a, e tors in the lung can cause
9. T
13. c bronchoconstriction.)
10. T
14. b (Patients should wash their 27. c
11. b, c, d
hands immediately after appli- 28. d (Combining a beta blocker
12. b
cation of GELNIQUE.) with a calcium channel
13. c, d
15. 5-mL dose blocker can produce excessive
14. c, d
16 a. 0.1 mg is safe dose cardiosuppression.)
15. a
b. 0.2 mL dose 29. 6 seconds/push
16. b, d
30. 180 mL/hr
17. d, e

Copyright © 2016, Elsevier Inc. All Rights Reserved.


290 Answer Key

CHAPTER 19 18. d (Ergot derivatives have been 15. d (Interferon beta can suppress
1. peripheral adrenergic associated with valvular heart bone marrow function, thereby
2. Hypertension injury.) decreasing production of all
3. direct-acting adrenergic recep- 19. a blood cell types.)
tor blockers 20. c 16. d
4. drowsiness; xerostomia 21. b, d, e 17. d, e
5. vasodilation 22. 6 tablets 18. c, e
6. b 23. 2 orally disintegrating tablets 19. a, c
7. c (Clonidine is embryotoxic in 20. b
animals. Because of the possi- CHAPTER 22 21. d
bility of fetal harm, clonidine is 1. F 22. c
not recommended for pregnant 2. F 23. 0.2 mL/dose
women. Pregnancy should be 3. T 24. 115 mL/6 = 19 mL
ruled out before clonidine is 4. F
given.) 5. F CHAPTER 24
8. b 6. T 1. e
9. c (Reserpine produces sedation 7. T 2. m
and a state of indifference to the 8. F 3. a
environment. In addition, the 9. F 4. k
drug can cause severe depres- 10. T 5. g
sion. These effects are thought 11. F 6. n
to result from depletion of cer- 12. T 7. j
tain neurotransmitters [catechol- 13. T 8. c
amines, serotonin] from neurons 14. d 9. f
in the brain.) 15. a (With oral dosing, the most 10. h
10. d common cholinergic effects are 11. i
11. a, b nausea [47%], vomiting [31%], 12. b
12. a diarrhea [19%], abdominal pain 13. l
13. c [13%], and anorexia [17%].) 14. d
14. 2 tablets 16. c 15. a, b, c
15. 200 mL/hr 17. a, b, c 16. a
18. d 17. b
CHAPTER 20 19. c 18. c, e
1. a, d 20. b 19. d
2. b, c 21. e (Estrogen/progesterone 20. a
3. a, c therapy may actually increase 21. b
4. c risk.) 22. a (Between 2% and 5% of
22. None. The nurse needs to patients develop a morbilliform
CHAPTER 21 consult the prescriber and [measles-like] rash. Rarely,
the pharmacist because IR morbilliform rash progresses
1. Dopamine agonists
and ER formulas are not to toxic epidermal necrolysis
2. Catechol-O-methyltransferase
interchangeable. or Stevens-Johnson syndrome,
(COMT inhibitor)
23. 2.5 mL—use 3-mL syringe 
  

  

3. Anticholinergic agents
without needle characterized by red macules,
4. Inhibitors of monoamine
papules, and tubercles. If a rash
oxidase-B (MAOB inhibitor)
CHAPTER 23 develops, phenytoin should be
5. Levodopa
1. T stopped.)
6. Amantadine
2. F 23. b
7. c
3. F 24. b
8. b
4. T 25. a (To avoid transmitting infec-
9. b
5. T tions, wash hands frequently
10. c
6. T with soap and water.)
11. c
7. F 26. a (Carbamazepine can cause
12. b
8. T vertigo. These reactions are
13. a
9. T 


 


14. c, e
10. F of treatment. Fortunately, toler-
15. c
11. F ance usually develops with con-
16. a (Ropinirole may cause in-
12. c, e tinued use. These effects can be
creased hepatic enzymes.)
13. c minimized by initiating therapy
17. b
14. b at low doses and giving the
largest portion of the daily dose

Copyright © 2016, Elsevier Inc. All Rights Reserved.


Answer Key 291

at bedtime [safety measures to avoid alcohol and all other CNS 8. d


prevent falls].) depressants.) 9. c
27. c 12. c 10. b
28. d (A major risk factor for severe 13. a 11. a (Depression of respiratory and
skin reactions is a genetic varia- 14. b cardiac function is a concern
tion known as human leukocyte 15. 0.4 mL with virtually all inhalation
antigen [HLA]-B*1502, which 16. No. The safe dose is 168 mg. anesthetics.)
occurs primarily in people of 12. d
Asian descent.) CHAPTER 26 13. c
29. c 1. T 14. b
30. a (Valproic acid has been as- 2. T 15. c
sociated with fatal liver failure; 3. T 16. d (The risk of malignant hyper-
inform patients about signs 4. F thermia is greatest when an in-
and symptoms of liver injury 5. F halation anesthetic is combined
[reduced appetite, malaise, nau- 6. T with succinylcholine.)
sea, abdominal pain, jaundice] 7. F ]~

=#  
  
-
and instruct them to notify the 8. F sion; hypotension results from
prescriber if these develop.) 9. T vasodilation. Amlodipine
31. b 10. T lowers blood pressure through
32. a 11. F peripheral arterial dilation.)
33. a (Severe overdose produces 12. a 18. b
generalized CNS depression; 13. c 19. d
death results from depression 14. d (When spinal anesthesia is 20. a, b, d
of respiration. Assessing vital used, the prescriber should be 21. a
signs evaluates respiratory 
$

 
$ 

22. b (Propofol can cause profound
status.) void within 8 hours of the end respiratory depression [includ-
34. c, d of surgery; spinal anesthesia ing apnea].)
35. c frequently causes headache.
36. c These “spinal” headaches are CHAPTER 28
37. a posture-dependent and can be 1. T
38. d relieved by having the patient 2. F
39. b assume a supine position. Due 3. T
40. c to headaches, asking the patient 4. T
41. d to sit up to attempt to void 5. F
42. c or having him/her go to the 6. F
43. Yes. Calculated safe/effective bathroom to void may not be 7. T
dose is 19.88 mg possible.) 8. d
44. 0.16 mL 15. b, e 9. b (Safety issue—fall risk with
16. a (Systemic absorption of opioid analgesics. Do not keep
CHAPTER 25 epinephrine can result in patients in position or leave
1. T systemic toxicity [e.g., palpita- patients unattended in position
2. F tions, tachycardia, nervousness, from which they could fall.)
3. T hypertension].) 10. c
4. F 17. d 11. c (Signs of withdrawal include
5. T 18. b excessive crying, sneezing,
6. F 19. c (When the tourniquet is loos- 
 
$

7. F ened at the end of surgery, about and diarrhea; fever and diarrhea
8. F 15% to 30% of administered may result in hypovolemia.)
9. F anesthetic is released into the 12. d
10. T systemic circulation; it is at this 13. b (The half-life of naloxone is
11. d (Centrally acting muscle point systemic adverse events approximately 2 hours. Dosing
relaxants can produce general- can occur.) is repeated at 2- to 3-minute
ized depression of the CNS. intervals until a satisfactory
Drowsiness, dizziness, and CHAPTER 27 response has been achieved.)
lightheadedness are common. 1. f 14. d
Patients should be warned not 2. d 15. a
to participate in hazardous 3. b 16. c
activities [e.g., driving] if CNS 4. e 17. a
 

  
#
- 5. g 18. a (The dose of fentanyl in the
dition, they should be advised to 6. c lozenge on a stick is suf-
7. a 


 

Copyright © 2016, Elsevier Inc. All Rights Reserved.


292 Answer Key

individuals—especially 11. b (While all of these are a 18. a


children.) concern with a patient receiving 19. a
19. d meperidine, breathing [safety] is 20. c
20. d the priority.) 21. b
21. b 12. c (Ergotamine can cause periph- 22. c (Haloperidol should be used
22. a eral vasoconstriction.) with caution in patients with
23. b 13. a dysrhythmia risk factors, in-
24. a 14. a, c cluding long QT syndrome, hy-
25. c 15. d pokalemia or hyperkalemia, or
26. d 16. b (Coronary vasospasm is the a history of dysrhythmias, heart
27. a, c biggest concern in patients tak- attack, or severe heart failure.)
28. d ing sumatriptan; however, about 23. b, d
29. 0.6 mL. Push one line (0.2 mL) 50% of patients experience 24. c (Clozapine produces agranu-
every 10 seconds. Total liquid unpleasant chest symptoms, locytosis in 1% to 2% of
is 6 mL; 5 lines/mL = 30 lines usually described as “heavy patients. The overall risk of
(“pushes”) over 5 minutes (300 arms” or “chest pressure” rather death is about 1 in 5000. The
seconds). Push one line (0.2 than pain. These symptoms usual cause is gram-negative
mL) every 10 seconds. are transient and not related to septicemia; high fever would be
30. 0.08 mL ischemic heart disease.) an indicator of such infection.)
17. a 25. a, c, d
CHAPTER 29 18. c 26. 2 mL in 2 sites (right and left
1. c 19. b (Propranolol can exacerbate ventrogluteal muscles) because
2. a symptoms of asthma.) should not exceed 2-3 mL in
3. d 20. d this site
4. b 21. d 27. 0.08 mL every 10 seconds
5. d 22. 1 mL
6. c 23. 2 tablets CHAPTER 32
7. b 1. little interest; pleasure
8. c CHAPTER 31 2. dosage change
9. c 1. F 3. 1; 3
10. a 2. T 4. 4; 9
11. d 3. F 5. more adverse effects
12. c 4. T 6. stimulating; insomnia
13. b 5. F 7. heart
14. c (NSAIDs inhibit platelet 6. T 8. MAOI
aggregation.) 7. F 9. antihistamines; sleep aids
15. d (Severe respiratory depression 8. F 10. headache; jaw pain; dental
can be reversed with naloxone; 9. c problems
however, caution is required. 10. a 11. tyramine
Excessive dosing will reverse 11. a (Laryngeal dystonia can im- 12. b (Patients with depression
analgesia, thereby putting the pair respiration.) often think about or attempt
patient in great pain.) 12. d (Acute dystonia can be both suicide. During treatment with
16. a disturbing and dangerous. antidepressants, especially
17. d (Respiratory depression is Typically, the patient develops early on, the risk of suicide
increased by concurrent use of severe spasm of the muscles of may actually increase. If mood
other drugs that have CNS- the tongue, face, neck, or back.) deteriorates, or if thoughts of
depressant actions (e.g., alcohol, 13. b (Dantrolene can reduce rigid- suicide intensify, the patient
barbiturates, benzodiazepines.) ity and hyperthermia associated should see his or her prescriber
18. Yes, the rate will be safe. with neuroleptic malignant immediately.)
syndrome.) 13. d
CHAPTER 30 14. c 14. c (If mood deteriorates, or if
1. c 15. d (First-generation agents pro- thoughts of suicide intensify,
2. e duce varying degrees of mus- the patient should see his or
3. f carinic cholinergic blockade, her prescriber immediately; the
4. d thus eliciting the full spectrum answer “provide a safe environ-
5. a of anticholinergic responses. ment” assumes the patient is in
6. b Monitoring intake and output the hospital, which may not be
7. g is one means of assessing for the case.)
8. b, d, e urinary retention.) 15. c
9. d 16. c ]—

=–  


 

10. b 17. a, e bound to plasma proteins, it

Copyright © 2016, Elsevier Inc. All Rights Reserved.


Answer Key 293

can displace other highly bound 11. b 22. d


drugs. Displacement of warfarin 12. a, d 23. a
[an anticoagulant] is of particu- 13. c (Diuretics promote sodium 24. b
lar concern. Monitor responses loss, and can thereby increase 25. d (Severe barbiturate overdose
to warfarin closely.) the risk of lithium toxicity.) produces generalized CNS
17. a 14. a, b, d depression; death results from
18. c 15. a (Administer the drug; a serum depression of respiration.)
19. d level at 6 mcg/mL is within the 26. b, e
20. b (SNRIs can result in neonatal target range of 4-12 mcg/mL.) 27. c
withdrawal syndrome, charac- 16. a 28. a
terized by irritability, abnormal 17. d (Lamotrigine can cause life- 29. a
crying, tremor, respiratory threatening rashes, including 30. b, e
distress, and possibly seizures.) Stevens-Johnson syndrome and 31. 2 tablets
21. b toxic epidermal necrolysis; typi- 32. First dose 2 mL, second dose 3
22. a cal presentation includes blister- mL
23. b (The combination of a TCA ing of mucous membranes,
with an MAOI can lead to se- typically in the mouth.) CHAPTER 35
vere hypertension, owing to ex- 18. 2 capsules 1. T
cessive adrenergic stimulation 19. 7.5 mL 2. F
of the heart and blood vessels.) 3. T
24. a, c, e CHAPTER 34 4. T
25. a 1. a 5. F
26. b 2. c 6. F
27. c (Tranylcypromine is an MAOI 3. d 7. F
and symptoms of hypertensive 4. b 8. T
crisis include headache, tachy- 5. T 9. F
cardia, palpitations, nausea, and 6. T 10. T
vomiting.) 7. F 11. agoraphobia
28. c, e 8. F 12. generalized anxiety disorder
29. b 9. T (GAD)
30. d 10. T 13. social anxiety disorder
31. c (Patients should not take 11. T 14. obsessive compulsive disorder
Wellbutrin if they have seizures; 12. F (OCD)
the most serious adverse effect 13. T 15. posttraumatic stress disorder
of Wellbutrin is seizures.) 14. F (PTSD)
32. a 15. d, e 16. panic disorder
33. d 16. b 17. b (An intoxicated person is
34. a 17. b considered legally incapable of
35. 2 CR tablets of 25 mg each 18. b (Flumazenil can reverse the consent.)
36. 2 capsules sedative effects of benzodi- 18. c
azepines, but may not reverse 19. c, e
CHAPTER 33 respiratory depression.) 20. a (Alprazolam should be used
1. F 19. d (Safety is paramount as pa- with caution in those with pul-
2. T tients taking benzodiazepines in monary impairment.)
3. T sleep-inducing doses may carry 21. c
4. F out complex behaviors, and 22. d (Overdose with a TCA can be
5. F then have no memory of their life-threatening; the lethal dose
6. T actions. Reported behaviors is only 8 times the average daily
7. F include sleep driving, prepar- dose.)
8. F ing and eating meals, making 23. b (To reduce the risk of
9. c (In a mixed episode, patients phone calls, and having sexual tyramine-induced hypertensive
experience symptoms of mania intercourse. Additionally, when crisis, the following precautions
and depression simultane- employed to treat anxiety, must be taken: avoid foods rich
ously. Patients may be agitated benzodiazepines sometimes in tyramine—which include
and irritable (as in mania), but cause paradoxical responses, yeast extracts, most cheeses,
may also feel worthless and including insomnia, excitation, fermented sausages [e.g., sa-
depressed. The combination euphoria, heightened anxiety, lami, pepperoni, bologna], and
of high energy and depression and rage.) 
 

 ?



  
 
$
20. a 24. a
suicide.) 21. b, c, d 25. b
10. a 26. c

Copyright © 2016, Elsevier Inc. All Rights Reserved.


294 Answer Key

27. 1 1/2 (1.5) tablets 8. F 19. c


28. 1.1–3.3 mg 9. T 20. c
10. F 21. a
CHAPTER 36 11. F 22. a, b
1. c 12. T 23. a
2. f 13. b 24. d (Postmarketing reports indi-
3. b 14. e cate that varenicline can cause
4. e 15. a serious neuropsychiatric effects,
5. g 16. c including mood changes, erratic
6. a 17. d behavior, and suicidality.)
7. d 18. a, c, e 25. a, b, c, d
8. T 19. d 26. 3 SR tablets
9. F 20. b 27. 1 tablet
10. T 21. a (Approximately 35% of cases
11. T of acute pancreatitis can be CHAPTER 40
12. F attributed to alcohol, making al- 1. F
13. F cohol the second most common 2. T
14. T cause of the disorder. Symp- 3. T
15. a, d toms of pancreatitis include 4. T
16. a severe abdominal pain radiating 5. F
17. c to the back; amylase is used to 6. T
18. a diagnose pancreatitis. Normal 7. F
19. c amylase ranges from 25-85 8. T
20. b units/L.) 9. F
21. d 22. a, c, d 10. T
22. a 23. d 11. F
23. b 24. a 12. F
24. d 25. a 13. F
25. d (Guanfacine activates 26. c 14. T
brainstem alpha2-adrenergic 27. d 15. F
receptors, and thereby reduces 28. c 16. T
 



 
29. d (Acamprosate is used along 17. F
and blood vessels.) with counseling and social sup- 18. T
26. 60 mg is a safe dose. port to help people who have 19. F
27. 6 gtt/min stopped drinking large amounts 20. T
of alcohol to avoid drinking 21. F
CHAPTER 37 alcohol again.) 22. T
30. d 23. F
1. Tolerance 31. 1/2 tablet
2. physically dependent 24. T
32. 1 mL (2 mL of drug + 1 mL 25. c (The desire to avoid symp-
3. Psychological dependence diluent = 3 mL)
4. withdrawal syndrome toms of withdrawal may be
5. cross-tolerance $




CHAPTER 39 drug use.)


6. Cross-dependence
7. Dopamine 1. T 26. d (Following IV injection, ef-
8. Chronic; relapsing 2. F fects of naloxone begin almost
9. a, b, d 3. F immediately and persist about
10. a, b, d 4. T 1 hour; respiratory depression
11. d 5. T will return when the effects of
12. b 6. F (10 seconds) naloxone have worn off.)
13. c 7. T 27. b
14. a 8. F 28. a
9. F 29. d
CHAPTER 38 10. T 30. d
11. T 31. b (Barbiturates reduce ventila-
1. T 12. T tion; doses only 3 times greater
2. F 13. T than those needed to induce
3. F 14. F sleep can cause complete
4. F 15. T suppression of the neurogenic
5. T 16. F respiratory drive. With severe
6. T 17. T overdose, barbiturates can cause
7. F 18. c apnea and death.)

Copyright © 2016, Elsevier Inc. All Rights Reserved.


Answer Key 295

32. a 39. c (Some water-softening contraindicated for patients with


33. d systems replace calcium and AV heart block.)
34. c magnesium ions with sodium 24. a
35. c ions. The higher the concentra- 25. 10,800 mL/24 hr
36. a, b, c, d tion of calcium and magnesium, 26. 675 mL/hr
37. d the more sodium needed to
38. b soften the water. Patients on a CHAPTER 43
39. 2 sublingual tablets very low-sodium diet who are 1. F
40. 4 mL concerned about the amount 2. T
of sodium in softened water 3. T
CHAPTER 41 may want to consider a water- 4. T
1. b  
 
 
 
5. T
2. e potassium chloride instead.) 6. F
3. d 40. 0.5 (1/2) tablet 7. F
4. a 41. 30 seconds/line 8. T
5. c 9. T
6. T CHAPTER 42 10. F
7. F 1. volume expansion 11. F
8. F 2. volume contraction 12. f
9. F 3. hypertonic contraction 13. b
10. T 4. isotonic contraction 14. g
11. F 5. twice (two times) the osmolality 15. l
12. T of sodium 16. a
13. T 6. metabolic alkalosis 17. h
14. F 7. Respiratory alkalosis 18. e
15. 180,000 8. metabolic acidosis 19. k
16. 1800 9. d 20. i
17. sodium; chloride 10. b 21. j
18. thick segment of the ascending 11. c 22. d
limb of the loop of Henle 12. a 23. c
19. acids; bases; calcium; chloride; 13. a (In isotonic contraction, 24. b
glucose; lipids; magnesium; volume should be replenished 25. a, b, c
potassium; sodium; uric acid; slowly to avoid pulmonary 26. d
 
=? edema.) 27. a
20. c 14. d 28. a (A cough that produces frothy
21. c 15. b sputum that may be tinged with
22. a 16. c blood is an indication of pulmo-
23. a, b, d, e 17. a (With the exception of the nary edema.)
24. c sustained-release tablets, solid 29. a, b, c, f
25. c formulations of KCl can pro- 30. a
26. a (This selection has the 20 mm duce high local concentrations
Hg drop in SBP associated with of potassium, resulting in severe CHAPTER 44
orthostatic hypotension.) intestinal injury [ulcerative 1. d
27. a, b, c, d lesions, bleeding, perforation]; 2. e
28. c (Muscle weakness and cramp- death has occurred.) 3. g
ing are symptoms associated 18. b (Serum levels of potassium 4. f
with hypokalemia.) are regulated primarily by the 5. b
29. b kidneys.) 6. a
30. d 19. a 7. i
31. c 20. d, e 8. h
32. d 21. b (The most serious conse- 9. c
33. b quence of hyperkalemia is dis- 10. d
34. b ruption of the electrical activity 11. c
35. b of the heart.) 12. a
36. b (Adolescents and young 22. d 13. c
adults are particularly con- 23. b (In the heart, excessive mag- 14. b
cerned about body image, nesium can suppress impulse 15. a, c, d, e

  

 
conduction through the atrio- 16. d (A precipitous drop in blood
different from their peers.) ventricular (AV) node. Accord- pressure may occur follow-
37. d ingly, magnesium sulfate is 

 
 
$

>Z

38. c, d

Copyright © 2016, Elsevier Inc. All Rights Reserved.


296 Answer Key

inhibitor. This reaction is caused 16. c 14. f


by widespread vasodilation 17. d 15. e
secondary to abrupt lowering of 18. c 16. c
angiotensin II levels. First-dose 19. b 17. d
hypotension is most likely in 20. d (Caution is advised when us- 18. a
patients with severe hyperten- ing nifedipine in persons with 19. a, b, c
sion, in patients taking diuretics, kidney disease.) 20. b
and in patients who are sodium- 21. 11 mL/hr 21. c
depleted or volume-depleted.) 22. 20 mg is the safe dose for this 22. d
17. c patient. 23. a
18. a (Angioedema is a poten- 24. c (The most disturbing side
tially fatal reaction that devel- CHAPTER 46 effect of alpha blockers is ortho-
ops in up to 1% of patients. 1. T static hypotension. Hypotension
Symptoms, which result from 2. F can be especially severe with
increased capillary permeability, 3. F 
 
 
{  

include giant wheals and edema 4. T hypotension continues with


of the tongue, glottis, and phar- 5. F subsequent doses, but is less
ynx. Severe reactions should 6. T profound. Hypotension puts the
be treated with subcutaneous 7. F patient at risk for falls.)
epinephrine.) 8. T 25. c
19. d 9. T 26. a (The major cause of treatment
20. a 10. c (Principal adverse effects of failure in patients with chronic
21. d (Aspirin, ibuprofen, and hydralazine are hypotension, hypertension is lack of adher-
other NSAIDs may reduce the tachycardia; the risk of ortho- ence to the prescribed regimen.
antihypertensive effects of ACE static hypotension is low.) In this section, we consider the
inhibitors.) 11. b causes of nonadherence and
22. b 12. d discuss some solutions.)
23. d 13. a 27. b
24. a 14. c 28. d
25. b 15. d (If an SLE-like reaction oc- 29. c
26. d curs, hydralazine should be 30. b
27. c withdrawn. Approximately 50% 31. a
28. 2 tablets of patients have constitutional 32. a
29. 30 seconds symptoms of fever, weight loss, 33. c
and fatigue. Reporting fever as 34. b (Beta blockers can suppress
CHAPTER 45 a priority over fatigue is impor- glycogenolysis and mask early
1. vasodilation tant, as those with an SLE-like signs of hypoglycemia, and
2. veins (venous system) reaction may also develop therefore must be used with
3. arterioles pericarditis, of which low-grade caution.)
4. vasodilation; reduced arterial fever is a sign.)
pressure; increased coronary 16. a CHAPTER 48
perfusion 17. c 1. e
5. -ine 18. d 2. d
6. b (In the SA node, calcium 19. a 3. c
channel blockade can cause 20. 0.3 mL/10 sec 4. b
bradycardia; in the AV node, 21. 72 mL/hr 5. a
blockade can cause partial or 6. c
complete AV block; and in the CHAPTER 47 7. f
myocardium, blockade can 1. n 8. e
decrease contractility.) 2. m 9. b
7. a 3. j 10. a
8. a, c, e 4. d 11. h
9. b 5. b 12. d
10. b 6. k 13. g
11. d (Blockade of cardiac calcium 7. l 14. c
channels can cause bradycardia, 8. o 15. b
AV block, and heart failure.) 9. g 16. a, c, d
12. b 10. h 17. b
13. a 11. a 18. c (Weakness is a symptom of
14. b 12. i hyperkalemia.)
15. a 13. c

Copyright © 2016, Elsevier Inc. All Rights Reserved.


Answer Key 297

19. d (Dopamine is administered by 26. d (A postvoid residual > 100 CHAPTER 51


continuous infusion. Constant mL may necessitate urinary 1. oxygen; heart
monitoring of blood pressure, catheterization.) 2. preload; afterload
the electrocardiogram [ECG], 27. b 3. diastolic
and urine output is required.) 28. a 4. aspirin
20. b 29. d 5. c (Anginal pain is precipi-
21. a 30. b tated when the oxygen supply
22. a, c, d (All of these are signs 31. a 

 

 $


and symptoms of drug-induced 32. a meet oxygen demand. Cardiac


lupus-like syndrome.) 33. c oxygen supply is determined by
23. a 34. a   

?
24. b 35. c (Verapamil may produce a de- 6. b
25. a crease in blood pressure below 7. d
26. b normal levels, which may result 8. a
27. c in dizziness or symptomatic 9. c
28. d hypotension.) 10. b
29. c (Hypokalemia increases 36. b 11. a
the risk of digoxin-induced 37. c 12. d
dysrhythmias.) 38. 18.75 mg/dose 13. a
30. 4 mL/hr (calculates to 4.125 39. 30 mL/hr 14. a
mL/hr) 15. c
31. Calculates to recommended CHAPTER 50 16. b
dose of 52.5 mg—yes, 50 mg is 1. hormones 17. c
safe. 2. liver 18. d
3. saturated 19. a, d
CHAPTER 49 4. Lipoproteins 20. d
1. absence 5. water-loving; fear of water 21. 2 inches
2. abnormal 6. 100 22. 3 mL/hr
3. slower 7. 40
4. faster 8. 30; 60 CHAPTER 52
5. ventricular 9. c 1. T
6. c 10. b 2. F
7. a 11. a 3. T
8. b 12. d 4. T
9. c ]†

={ 

 

5. F
10. b Food and Drug Administration 6. T
11. a Pregnancy Risk Category X: the 7. d
12. a (Cardiac output affects all risks to the fetus outweigh any 8. c (In heparin-treated patients,
of the other diagnoses; there-  

$
 ? platelet aggregation is the major
fore, without adequate cardiac 14. c (Elevated CK level is an indi- remaining defense against hem-


 
 
cation of muscle damage. Rhab- orrhage. A depressed platelet
perfusion, and breathing cannot domyolysis is the breakdown of count means this defense is
be adequately maintained.) muscle tissue that leads to the weakened; hence, heparin must
13. a  
$
 
 

be employed with caution.)
14. b into the blood. These substances 9. a
15. b are harmful to the kidney and 10. b
16. a often cause kidney damage) 11. a
17. d 15. d 12. c
18. b 16. b 13. c
19. b 17. c 14. d
20. a 18. a 15. b
21. b (Cinchonism [overdose/toxic- 19. d 16. b
ity] is characterized by tinnitus 20. c 17. a, b, c, d, e
[ringing in the ears], headache, 21. b 18. d (Spinal hematomas may occur
nausea, vertigo, and disturbed 22. b in patients anticoagulated with
vision. These symptoms can 23. c drugs such as enoxaparin and
develop with just one dose.) 24. 3 tablets who receive spinal anesthesia;
22. a 25. 2 capsules these hematomas may result
23. b in long-term or permanent
24. a, b, c, e paralysis.)
25. c 19. a, b, c, d, e

Copyright © 2016, Elsevier Inc. All Rights Reserved.


298 Answer Key

20. c, d possible dose of metoprolol 9. f


21. b used. In this instance, the nurse 10. b
22. d should clarify the dose [which 11. e
23. b (Dabigatran dosing has not totals 200 mg/day] with the 12. m
been determined for patients prescriber.) 13. k
with an eGRF/[CrCl] < 15.) 15. a 14. a
24. a (12% of patients receiving 16. d 15. d
bivalirudin experience hypoten- 17. d 16. c
sion; patients are advised to 18. a 17. b
change positions slowly to min- 19. c 18. e
imize orthostatic hypotension.) 20. 0.8 mL 19. d
25. a 21. Adding 4.2 mL of diluent for a 20. b (Anemia is common in
26. d total of 5 mL and pushing 0.1 patients with chronic kidney
27. d mL every 5 seconds will result disease.)
28. b in completing the drug admin- 21. a
29. c istration in 4 minutes and 10 22. c, d
30. a seconds. 23. c
31. d (Neutropenia develops in 24. d
2.4% of patients receiving CHAPTER 54 25. c
ticlopidine, and is sometimes 1. F 26. b
severe. This reduces the body’s 2. T 27. a


 
$$
$ ? 3. F 28. c
32. b, c, d 4. F 29. a (To minimize anaphylactic
33. c 5. T reactions, intravenous iron
34. c 6. F dextran should be administered
35. c (If treatment begins soon after 7. T following a small test dose [25
symptoms start, heart attack 8. T mg over 5 minutes] and observe
deaths and heart damage can 9. b the patient for signs and symp-
often be avoided.) 10. c toms of anaphylaxis for at least
36. 5 mL 11. a 15 minutes.)
37. 12 mL/hr 12. d 30. d
13. c 31. c (Epoetin has been associated
CHAPTER 53 14. d with an increase in cardiovas-
1. myocardial infarction 15. b cular events. Among these are
2. STEMI 16. c cardiac arrest, hypertension, HF,
3. ST segment 17. d (Symptoms of anaphylaxis and thrombotic events, includ-
4. ST-elevation include wheezing, tightness in ing stroke and MI.)
5. advanced age; family history of the throat, shortness of breath, 32. b (Because of its unique
MI; obesity; high serum choles- and swelling of the face. The composition [ferumoxytol is a
terol; hypertension; smoking; treatment of choice is epineph- superparamagnetic form of iron
diabetes rine, injected subQ, after stop- oxide], the drug can interfere
6. atherosclerotic plaque ping administration of the factor with magnetic resonance imag-
7. angiotensin II concentrate!) ing [MRI] studies.)
8. troponins; creatine kinase 18. a 33. a
9. 90% 19. a, c 34. c (Early manifestations of
10. d 20. c pernicious anemia include
11. a, b, d, e 21. 1125 units of factor VIII for this paresthesias of the hands and
12. a, b child $Œ
$


 

13. b 22. 2.25 mL desmopressin will be neurologic damage can become


14. d (Patient with asthma or other injected into 50 mL 0.9% NS permanent.)
similar diseases should not and infused at 100 mL/hr 35. b
receive beta blockers, including 36. a, c, d
metoprolol. Because of its rela- CHAPTER 55 37. a (Hematopoietic growth factors
tive beta1 selectivity, however, have been associated with an in-
1. a
metoprolol may be used with crease in cardiovascular events.)
2. j
caution in patients with asthma 38. Yes, dose is safe and effective
3. l
who cannot tolerate other treat- (50 mg in divided doses).
4. i
ments. Since beta1 selectivity is 39. 0.2 mL
5. g
not absolute, a beta2-stimulating 6. h
agent should be administered 7. d
concomitantly, and the lowest 8. c

Copyright © 2016, Elsevier Inc. All Rights Reserved.


Answer Key 299

CHAPTER 56 10. T 11. a (By the second trimester,


1. brain; bone marrow; liver; 11. T the fetal thyroid gland is fully
heart; uterus; testes; blood 12. F functional; hence, the fetus can
vessels 13. F supply its own hormones from
2. cancer 14. a then on. Therefore, to help en-
3. cardiovascular events; death 15. a sure healthy fetal development,
4. 1 16. d maternal hypothyroidism must
5. two 17. b be diagnosed and treated very
6. yeast 18. d early.)
7. increase 19. c (Persons receiving insulin are 12. a (In thyrotoxicosis the heart-
8. clots at risk for hypoglycemia; sweat- beat is rapid and strong, and
9. a (Epoetin has been associated ing and confusion/altered level dysrhythmias and angina may
with an increase in cardiovas- of consciousness are signs of develop.)
cular events. Among these are hypoglycemia.) 13. c
cardiac arrest, hypertension, HF, 20. b 14. d
and thrombotic events, includ- 21. b 15. a, b, c, d, e
ing stroke and MI.) 22. a 16. a
10. b 23. a 17. b
11. d 24. a 18. b
12. a (Darbepoetin is generally 25. d 19. c
well-tolerated; the most com- 26. c 20. d (Thionamides can cause
mon problem is hypertension.) 27. a neonatal hypothyroidism and
13. b 28. c goiter. Accordingly, these drugs
14. c 29. b must be used judiciously during
15. c (High levels of uric acid in the 30. d pregnancy. To minimize effects
blood can cause solid crystals to 31. d on the fetus, dosage should be
form within joints. This causes †^

=#$

 


kept as low as possible.)
”
  ? 



 
21. d
16. c nothing should be administered 22. b
17. b by mouth.) 23. c
18. b 33. d 24. c
19. c 34. b 25. No. Safe range is 32.7-40 mcg
20. c 35. a per day.
21. b, c 36. a 26. 1/2 tablet
22. a (Oprelvekin causes retention 37. c
of sodium and water by the 38. b CHAPTER 59
kidney. As a result, about 48% 39. c 1. h
of patients experience dyspnea.) 40. d 2. a
23. d (Eltrombopag may cause bone 41. c 3. e
 
   
  
42. a 4. c
malignancy, and thrombotic/ 43. a 5. g
thromboembolic events, and 44. c, d 6. d
may pose a risk of bleeding; in 45. c (Glucagon promotes the 7. i
addition, the drug may cause breakdown of glycogen to 8. f
liver injury. Accordingly, safety glucose, reduces conversion of 9. b
is a priority in patients receiving glucose to glycogen, and stimu- 10. d
this drug.) lates biosynthesis of glucose.) 11. b
24. b, e 46. d 12. a (Mecasermin can cause a
25. 375 mcg = 38 mL/dose 47. 24 mL/hr variety of adverse effects. The
26. 10 mL/hr most common is hypoglycemia,
CHAPTER 58 which develops in nearly 50%
CHAPTER 57 1. T of patients, usually during the
1. T 2. F  

$
 
& -
2. T 3. T phoresis and tremor are signs of
3. T 4. F hypoglycemia.)
4. F 5. F 13. d
5. T 6. T 14. b, c , d
6. F 7. F 15. c
7. T 8. F 16. b
8. F 9. T 17. c
9. T 10. c

Copyright © 2016, Elsevier Inc. All Rights Reserved.


300 Answer Key

18. d (Tolvaptan blocks vasopressin 9. b 28. c (Combination OCs have been


V2 receptors in renal collecting 10. a, d associated with an increased
ducts, and thereby increases 11. b risk of venous thromboembo-
renal excretion of free water, 12. d (SERMs can increase the lism [VTE], arterial throm-
causing the sodium concentra- risk of endometrial cancer and boembolism, and pulmonary
tion in blood to rise.) thromboembolism.) embolism.)
19. b (In a few patients treated 13. a (High-dose therapy during 29. d (OCs may precipitate gall-
with pegvisomant, serum levels 
 
‹

$
  
bladder disease in women who
of hepatic transaminases rise, has been associated with an already have gallstones or a his-
indicating liver injury. Moni- increased incidence of birth tory of gallbladder disease; right
toring of hepatic function is defects.) upper quadrant pain is a symp-
recommended.) 14. d tom of gallbladder disease.)
20. 2 mg/dose 15. d 30. a
21. 0.08 mL 31. d
CHAPTER 62 32. c
CHAPTER 60 1. T 33. b
1. c 2. F 34. c
2. a 3. T 35. d
3. b 4. F 36. b, c, d
4. F 5. F 37. c
5. T 6. F 38. c
6. T 7. T 39. c
7. F 8. F 40. a, b, e
8. T 9. T
9. F 10. T CHAPTER 63
10. F 11. T 1. c
11. T 12. T 2. a
12. F 13. F 3. b
13. T 14. T 4. F
14. F 15. T 5. F
15. a 16. T 6. T
16. b 17. F 7. T
17. a (Cushing’s syndrome is 18. T 8. F
characterized by hyperglycemia 19. F 9. F
and glycosuria, among other 20. T 10. T
symptoms.) 21. b (The principal concern with 11. T
18. d drospirenone is venous throm- 12. F
19. c boembolism, which occurs 13. F
20. b (At times of stress, patients more often than with other pro- 14. F
must increase their glucocorti- gestins. The thromboembolism 15. T
coid dosage. Failure to increase can break apart, causing pieces 16. c (Very rarely, clomiphene can
the dosage can be fatal.) to travel through the blood- cause ovarian hyperstimulation.
^]

=
 


stream, resulting in pulmonary Symptoms include low abdomi-
water may retained in excess, embolism.) nal pain, pressure, weight gain,
resulting in expansion of blood 22. d and swelling.)
volume. Patients should be 23. c (Drospirenone is a structural 17. c
monitored for weight gain.) analog of spironolactone, a 18. d
22. a potassium-sparing diuretic that 19. b
23. a blocks receptors for aldoste- 20. b (Hyperstimulation syndrome
24. 0.5 (1/2) tablet rone; taken with NSAIDs, following hCG therapy may
25. Yes. The safe dose is 24-30 mg. there is an increased risk of cause pleural effusion.)
hyperkalemia.) 21. a
CHAPTER 61 24. d 22. b
1. d 25. b (By increasing levels of clot- 23. 0.5 (1/2) tablet
2. b ting factors, OCs can decrease 24. 2 mL in 2 sites (total 4 mL)
3. a the effectiveness of warfa-
4. e rin; therefore, it is important CHAPTER 64
5. c patients adhere to blood testing 1. F
6. a schedules.) 2. F
7. c 26. a 3. T
8. c 27. b 4. T

Copyright © 2016, Elsevier Inc. All Rights Reserved.


Answer Key 301

5. F periodic tests of liver function. 25. b (Terazosin may be dangerous


6. a, b, c, d Inform patients about signs of for men with reduced blood
7. d (Adverse effects of greatest liver dysfunction [jaundice, pressure.)
concern when administering ter- malaise, anorexia, fatigue, nau- 26. b (Doxazosin’s principal ad-
butaline are pulmonary edema; sea], and instruct them to notify verse effects are hypotension,
shortness of breath and foamy the prescriber if these occur.) fainting, and dizziness.)
sputum are signs and symptoms 14. c (Androgens can cause 27. c
of pulmonary edema.) cholestatic hepatitis and other
8. c (There is some concern that disorders of the liver. Since CHAPTER 67
nifedipine may compromise this patient does not report 1. d
  

? symptoms that are emergent in 2. c
9. a nature, it is a priority to gather 3. e
10. c all assessment data [history and 4. i
11. c physical] plus laboratory data, 5. b
12. a, c, e prior to calling the provider.) 6. g
13. a (Because it poses a risk of 15. 0.25 mL 7. a
severe hypertension, methy- 16. 4 pellets 8. f
lergonovine is considered a 9. j
second-line drug for controlling CHAPTER 66 10. h
postpartum hemorrhage.) 1. F 11. c
14. a, d, f, g 2. F 12. e
15. d, e 3. T 13. a
16. d (The pouch is removed when 4. F 14. d
active labor occurs.) 5. F 15. b
6. T 16. gamma globulins;
CHAPTER 65 7. F immunoglobulins
1. follicle-stimulating hormone 8. F 17. B lymphocytes (cells); T lym-
(FSH); luteinizing hormone 9. F phocytes (cells)
(LH) 10. T 18. d
2. anemia 11. T 19. b
3. ALT; AST; bilirubin 12. F 20. c
4. water; sodium (or salt) 13. F 21. d
5. irreversible 14. b, d 22. c
6. a (Androgens can lower plasma 15. c 23. a
levels of high-density lipopro- ]—

=#$
 

   
24. c
tein [HDL] cholesterol [“good are combined, life-threatening 25. b
cholesterol”] and elevate hypotension could result. There- 26. c
plasma levels of low-density $
 



lipoprotein [LDL] cholesterol contraindicated for men taking CHAPTER 68


[“bad cholesterol”]. These nitrates.) 1. f
actions may increase the risk ]~

= 
 

- 2. g
of atherosclerosis and related ally well-tolerated, it can be 3. c
cardiovascular events.) dangerous for men taking 4. d
7. c (Edema can result from  
   
- 5. l
androgen-induced retention of cally alpha-adrenergic blockers, 6. h
salt and water. This complica- nitroglycerin, and other nitrates 7. e
tion is a concern for patients used for angina pectoris.) 8. j
with heart failure and for 18. d 9. k
those with a predisposition to ]‡

=Š  
 


10. b
developing edema from other cardiac QT interval, and might 11. a
causes.) therefore pose a risk for serious 12. i
8. d dysrhythmias.) 13. a
9. d 20. a 14. d
10. b 21. a 15. b
11. d 22. a, b, c, e 16. c (Swelling of lips is symptom-
12. d 23. c atic of anaphylactic reaction.)
13. b (The 17-alpha-alkylated ^‹

=& 

 

17. b
androgens can cause choles- FDA pregnancy risk category 18. d, e
tatic hepatitis, jaundice, and X. It can be absorbed through 19. a
other liver disorders. Rarely, the skin, so pregnant women 20. b, d, e
liver cancer develops. Obtain should not handle the drug.)

Copyright © 2016, Elsevier Inc. All Rights Reserved.


302 Answer Key

21. c (Children with severe im- 16. d (Like tacrolimus, NSAIDs can 7. Acetylcysteine


+}!
injure the kidneys. Accordingly, 8. 2000 mg
be given MMR. Severe immu- NSAIDs should be avoided.) 9. b (The most common side

 
 
$
17. c effects are gastric distress,
immunosuppressive drugs [e.g., 18. a (Owing to the risk of infec- heartburn, and nausea. Occult
glucocorticoids, cytotoxic an- tion, patients taking sirolimus GI bleeding occurs often.)
ticancer drugs], certain cancers should avoid sources of conta- 10. d
[e.g., leukemia, lymphoma, gion. In addition, for 12 months 11. a
generalized malignancy], and after transplant surgery, patients 12. b
advanced HIV infection.) should take medicine to prevent 13. d
22. d Pneumocystis pneumonia.) 14. c (Smoking increases platelet
23. a, c, d 19. a aggregation; caution should be
24. a 20. b (The full range of glucocor- exercised when treating patients
25. a ticoid adverse effects can be who smoke cigarettes with aspi-
26. c expected in persons taking pred- rin alone—the preferred drug to
27. d nisone, including osteoporosis decrease platelet aggregation in
28. b with resultant fractures.) smokers is Plavix.)
29. a, e 21. b 15. a (The aspirin hypersensitivity
30. b, d 22. a, b, d reaction begins with profuse,
31. a 23. 2.7 mL watery rhinorrhea and may
32. d (Rotarix may carry a small 24. 14 mL/hr progress to generalized urti-
risk of intussusception, a rare, caria, bronchospasm, laryngeal
life-threatening form of bowel CHAPTER 70 edema, and shock. Maintaining
obstruction that occurs when 1. T a patient’s airway is the highest
the bowel folds in on itself, like 2. T priority.)
a collapsing telescope. Bloody, 3. T 16. c
mucus-like bowel movement, 4. F 17. a
sometimes called a “currant 5. T 18. d
jelly” stool, is a symptom.) 6. T 19. b
33. d 7. F 20. a (The principal risks to preg-
8. T nant women are [1] anemia
CHAPTER 69 9. T [from GI blood loss], and
1. hyperglycemia 10. F [2] postpartum hemorrhage.
2. azathioprine (Imuran) 11. b A uterus that has not con-
3. 6:00 PM (1800) 12. d tracted down well [boggy] is
4. 12 weeks 13. d the main cause of postpartum
5. fat 14. a hemorrhage.)
6. antacids 15. c 21. b
7. c (Immunosuppressive drugs 16. c 22. c
inhibit immune responses 17. d (Antihistamines should 23. c
which poses increased risk of be avoided late in the third 24. d
infection.) trimester, because newborns 25. b
8. d are particularly sensitive to the 26. b (Celecoxib can impair renal
9. d adverse actions of these drugs; function, thereby posing a
10. b in this case, the sedating effects. risk to patients with hyperten-
11. a (Renal damage occurs in The normal respiratory rate for sion, edema, heart failure, or
up to 75% of patients taking a newborn is 30-60 breaths per kidney disease. A daily weight
cyclosporine. Injury manifests minute.)  


 
$




 



18. c, d accumulation.)

 
 
19. d 27. c (There is strong evidence that
rate.) 20. 5 mL coxibs increase the risk of MI,
12. d (Signs of anaphylaxis include 21. No. 25 mg of the drug would be stroke, and other serious cardio-
  
  
  
infused in 50 seconds. vascular events.)
hypotension, and tachycardia. If 28. a
anaphylaxis develops, discon- CHAPTER 71 29. b
tinue the infusion and treat with 30. a (Acetylcysteine causes al-
1. 8 days
epinephrine and oxygen.) lergic reactions [rash, itching,
2. 1 week
13. c, d, e angioedema, bronchospasm,
3. 2
14. a, b, d hypotension], most often in
4. Reye’s syndrome
15. b   


 
 

5. vinegar
6. acetaminophen

Copyright © 2016, Elsevier Inc. All Rights Reserved.


Answer Key 303

Fortunately, these reactions tend 15. c 2. hypercalcemia


to be mild and self-limiting.) 16. d 3. clot
31. a (Use of aspirin in children ]~

=#
 
 

 
4. lethargy; depression
under 18 is associated with of heart failure. Exercise cau- 5. tetany, convulsions, spasm of
Reye’s syndrome; characteristic tion in patients with existing the pharynx, spasm of other
symptoms are encephalopathy heart failure, and monitor them muscles (or spasm of the phar-
and fatty liver degeneration.) closely for disease progression.) ynx and other muscles)
32. 400 mL/hr 18. b 6. increases
33. 1.2 mL/dose 19. d 7. whole grains (or bran); spinach
20. a (Rituximab can cause severe (or rhubarb, Swiss chard, beets)
CHAPTER 72 infusion-related hypersensitiv- 8. vitamin D
1. b ity reactions. The immediate ‡
 Œ
" Œ

2. c reaction and its sequelae include thyroid hormone; phenytoin;


3. e hypotension, bronchospasm, an- bisphosphonates
4. d gioedema, hypoxia, pulmonary 10. a (Calcium is critical to function
5. a   
  
$ - in skeletal, nervous, muscular,
6. c (Because of their mineralocor- tion, and cardiogenic shock.) and cardiovascular systems.
ticoid activity, glucocorticoids 21. b, d However, maintenance of
can cause sodium and water 22. No. The safe dose is 20 mg. normal levels is a priority in the
retention and potassium loss.) 23. 0.6 mL cardiovascular system, as cal-
7. d (By suppressing host defenses cium plays a role in myocardial
[immune responses and phago- CHAPTER 74 contraction, vascular contrac-
cytic activity of neutrophils and 1. F tion, and blood coagulation.)
macrophages], glucocorticoids 2. T 11. a
can increase susceptibility to 3. T 12. c
infection.) 4. F 13. b (For severe hypercalcemia,
8. b 5. T initial therapy consists of
9. a, b, c, d, e 6. T  
 



10. a, c, e 7. a saline.)
11. b 8. a 14. d (Hypocalcemia increases
12. c 9. b (The most serious toxicity is a neuromuscular excitability. As a
13. a rare but potentially fatal hyper- result, tetany, convulsions, and
14. a sensitivity syndrome, character- spasm of the pharynx and other
15. a, d, e ized by rash, fever, eosinophilia, muscles may occur.)
16. 0.2 mL/15 sec and dysfunction of the liver and 15. a
17. Yes. Safe dose is up to 11.25 mg kidneys. To prevent renal injury, 16. b
per day or 5.625 mg per dose. 
 


$
17. d

  



$

18. b, e
CHAPTER 73 least 2 L/day.) 19. d
10. c 20. d (Parenteral calcium may cause
1. b
11. c (Allopurinol can inhibit severe bradycardia in patients
2. c
hepatic drug-metabolizing taking digoxin.)
3. a
enzymes, thereby delaying the 21. b, c, e
4. c, e
inactivation of other drugs. 22. a, c
5. b
This interaction is of particu- 23. d
—

=

 

lar concern for patients taking 24. c, e


patients may be given 10 to
warfarin, whose dosage should 25. a (Esophagitis, sometimes
20 mg/day until symptoms are
be reduced.) resulting in ulceration, is the
controlled, followed by gradual
12. b most serious adverse effect of
drug withdrawal over 5 to 7
13. a (During premarketing trials, alendronate. The cause of injury
days.)
pegloticase triggered anaphy- is prolonged contact with the
7. a
laxis in 6.5% of patients. Symp- esophageal mucosa, which can
8. d
toms include wheezing, perioral occur if alendronate fails to pass
9. d
or lingual edema, hemodynamic completely through the esopha-
10. c
instability, and rash.) gus, as would be the case in
11. a
14. 1 1/2 tablets per dose dysphagia.)
12. a
15. 125 mL per hour 26. d
13. d (Retinal damage, which is
27. b
rare, is the most serious toxicity.
CHAPTER 75 28. b
Retinopathy may be irreversible
29. c, d
and can produce blindness.) 1. parathyroid hormone; vitamin
30. a
14. b D; calcitonin

Copyright © 2016, Elsevier Inc. All Rights Reserved.


304 Answer Key

31. c 20. a (The selectivity of the beta2- 3. a


32. b adrenergic agonists is only 4. f
33. d (Raloxifene increases the risk relative, not absolute. Accord- 5. d
of thromboembolic events. Be- ingly, these drugs are likely 6. b
cause inactivity promotes DVT, to produce some activation of 7. h
patients should discontinue ral- beta1 receptors in the heart. If 8. e
oxifene at least 72 hours before dosage is excessive, stimula- 9. e
prolonged immobilization and tion of cardiac beta1 receptors 10. b
should not resume the drug until can cause angina pectoris and 11. h
full mobility has been restored.) tachydysrhythmias.) 12. g
†‹

= $

 

21. d (Following withdrawal of 13. c
FDA pregnancy risk category oral glucocorticoids [or transfer 14. a
X—the potential for fetal harm to inhaled glucocorticoids], 15. d
 

  

several months are required 16. f
of use during pregnancy.) for recovery of adrenocorti- 17. a
35. b cal function. Throughout this 18. b, d, e
36. c time, all patients—including 19. b
37. b (Denosumab increases the risk those switched to inhaled 20. c
of serious infections; patients glucocorticoids—must be given 21. b
who develop signs of severe supplemental oral or IV glu- 22. c
infection should seek immediate cocorticoids at times of severe 23. a
medical attention. Flank pain stress. Failure to do so can 24. d
and fever may indicate UTI or prove fatal.) 25. b (By activating alpha1-
infection of the abdomen.) 22. b adrenergic receptors on
38. b, c, d, e 23. c systemic blood vessels, sym-
39. d 24. c pathomimetics can cause
40. b 25. a widespread vasoconstriction;
41. 2 to 10 minutes 26. b for individuals with cardiovas-
42. 1.7 mL 27. d cular disorders—hypertension,
28. a coronary artery disease, cardiac
CHAPTER 76 29. b dysrhythmias, cerebrovascular
1. l 30. c (The most severe adverse disease—widespread vasocon-
2. f effects of omalizumab are striction can be hazardous.)
3. k malignancy and anaphylaxis. 26. c
4. e Perioral edema is symptomatic 27. b
5. j of anaphylaxis.) 28. b
6. a 31. d (Smoking cigarettes [one to 29. b
7. c two packs a day] accelerates 30. d
8. i metabolism and decreases the 31. b (If benzonatate is sucked or
9. d half-life by about 50%.) chewed, rather than swallowed,
10. h 32. a (Patients with peanut allergy the drug can cause laryngo-
11. b should avoid two ipratropium spasm, bronchospasm, and
12. g products: Atrovent HFA [ip- circulatory collapse.)
13. c (Symptoms of asthma result ratropium alone] and Combi- 32. a, b
$

  
$
 - vent [ipratropium/albuterol]. 33. 2 capsules
mation and bronchoconstriction. Both products contain soya leci- 34. 8 capsules
Accordingly, treatment must thin as a carrier. Soya is in the
address both components.) same plant family as peanuts, CHAPTER 78
14. c and about 10% of people with 1. h
15. a peanut allergy are cross-allergic 2. a
16. d to soya.) 3. e
17. b, c, d 33. c 4. g
18. d (Like oral glucocorticoids, 34. c 5. b
inhaled glucocorticoids can 35. d 6. f
promote bone loss—at least 36. 1.6 mL/dose 7. c
in premenopausal women. 37. 1.2 mL at 2 subcutaneous sites 8. d
Fortunately, the amount of loss (total 2.4 mL) 9. T
is much lower than the amount 10. F
caused by oral glucocorticoids.) CHAPTER 77 11. T
19. b 1. c 12. F
2. g 13. T

Copyright © 2016, Elsevier Inc. All Rights Reserved.


Answer Key 305

14. F because increased peristalsis 16. c (Cannabinoids can cause


15. T could cause bowel perforation. tachycardia and hypotension.
16. T High-pitched, tinkling sounds Normal blood pressure is less
17. F are a sign of early intestinal than 120/80 mm Hg. Symptoms
18. c (Smoking delays ulcer heal- obstruction.) of hypotension are not typically
ing and increases the risk of 11. b present unless BP is less than
recurrence.) 12. c 90/60 mm Hg. Tachycardia
19. d 13. d is a resting heart rate greater
20. b 14. a (Lactulose can enhance than100 bpm. Therefore, the
21. c intestinal excretion of am- priority for this patient is a
22. a (Bismuth can impart a monia. This property has been pulse increase.)
harmless black coloration to exploited to lower blood am- 17. b
the tongue and stool. Patients monia content in patients with 18. c
should be forewarned. How- portal hypertension and hepatic 19. c
ever, stool discoloration may encephalopathy secondary to 20. a
confound interpretation of chronic liver disease. Normal 21. d
gastric bleeding. Therefore, ammonia levels range from 15- 22. d
an abdominal assessment is a 110 mcg/dL [this number may 23. d
priority.) vary dependent upon lab].) 24. c
23. a 15. b 25. d (Postmarketing data indicate
24. b 16. d that tegaserod can cause serious
25. b, d, e 17. b cardiovascular events, namely,
26. c 18. d myocardial infarction (MI),
27. b 19. a (Easy bruising and mucosal unstable angina, and stroke.
28. a (PPIs have been associated bleeding can occur when medi- Because of the potential for
with a dose-related increase cations interfere with vitamin K cardiovascular harm, access to
in the risk of infection with C. absorption.) tegaserod is restricted.)
  , a bacterium that can 20. b 26. b, f
cause severe diarrhea. Patients 21. 30 mL 27. a
experiencing diarrhea while 22. 1 capsule 28. c (The most common adverse
taking omeprazole or other PPIs effect of olsalazine is watery
should report immediately to CHAPTER 80 diarrhea, which occurs in 17%
their health care provider for 1. medulla oblongata of patients. The electrolyte
testing.) ^

=
?   



-
29. d 3. chemoreceptor trigger zone ated with diarrhea, combined
30. b, e 4. vagus with diuretic use, makes Fluid
31. a, b, c, e 5. before (or prior to) 



 -
32. a chemotherapy ing diagnosis.)
33. c 6. serotonin (or 5-HT3) 29. a, b
34. d 7. dexamethasone 30. b
35. b 8. doxylamine; vitamin B6 31. d (Tuberculosis and opportunis-
36. 10 mL 9. prolong tic infections are of particular
37. 16-17 drops per 15 seconds 10. Irritable bowel syndrome concern in patients receiving
11. b (A patient with prolonged  
!$

-
CHAPTER 79 vomiting and diarrhea will tive cough would be a priority
1. c show a decrease in sodium and nursing concern.)
2. d potassium [electrolytes] and an 32. a
3. a elevated BUN.) 33. d
4. b 12. a 34. b
5. d 13. b, d, e 35. c
6. c (The Rome III criteria deter- 14. d (Patients should be advised 36. a
mines constipation more by to report local burning or pain 37. Yes. The recommended dose
stool consistency [degree of immediately when receiving calculates to 5.5 mg, so a 5-mg
hardness] than by how often promethazine IV. Extravasa- capsule is safe and effective.
bowel movements occur.) tion of IV promethazine can 38. 377 mg
7. a cause abscess formation, tissue
8. a necrosis, and gangrene that CHAPTER 81
9. d requires amputation. Therefore, 1. d
10. c (Laxatives should not be used the priority action is to stop the 2. a
in patients with fecal impaction IV infusion.) 3. b
or obstruction of the bowel, 15. c 4. e

Copyright © 2016, Elsevier Inc. All Rights Reserved.


306 Answer Key

5. c 14. T severe hypotension] is an imme-


6. b 15. F diate hypersensitivity reaction,
7. a 16. F mediated by IgE. Anaphylactic
8. d, e 17. T reactions occur more frequently
9. a (In high doses, vitamin A can 18. F with penicillins than with any
cause liver injury.) 19. F other drugs.)
10. b 20. b (Unasyn is a broad-spectrum 19. a
11. a antibiotic. Narrow-spectrum 20. c
12. c antibiotics are active against 21. a, e
13. a (Vitamin A has been used for only a few species of micro- 22. d
skin conditions, including acne; organisms. In contrast, broad- 23. d
it has also been used to treat spectrum antibiotics are 24. c (Ticarcillin interferes with
heavy menstrual periods and active against a wide variety platelet function and can
premenstrual syndrome.) of microbes. Narrow-spectrum thereby promote bleeding.)
14. b drugs are generally preferred to 25. c
15. d broad-spectrum drugs.) 26. No. Dose is 500 mg/day. 7.5 kg
16. d 21. c × 20 mg = 150 mg/day; 7.55 kg
17. b 22. a × 40 mg = 300 mg/day.
18. c 23. c, d 27. 200 mL/hr
19. b 24. c (As a rule, patients with a
20. a history of allergy to penicillins CHAPTER 85
21. 1 1/2 tablets should not receive them again. 1. beta-lactamases
22. 0.25 mL The exception is treatment of 2. positive
a life-threatening infection for 3. Methicillin-resistant )-
CHAPTER 82 which no suitable alternative is coccus aureus (MRSA)
1. c available. For emergency treat- 4. increased
2. b ment, people typically receive 5. negative
3. a epinephrine; antihistamines —
>$ 
=! ?
4. b and corticosteroids may also be 7. good
5. a ordered.) 8. c (In patients with renal insuf-
6. d 25. b (Tetracycline has been as- 
 
$
 
 -
7. d signed to pregnancy category D losporins must be reduced [to
8. c (By reducing fat absorption, by the FDA.) prevent accumulation to toxic
orlistat can reduce absorption of 26. d levels].)
fat-soluble vitamins [vitamins 27. c 9. c (Ceftriaxone can cause bleed-

&
Z

¤Š 

- 28. a, c, d ing tendencies.)
ciency can intensify the effects 29. a 10. b (Hypersensitivity reactions
of warfarin, an anticoagulant. are the most frequent adverse
In patients taking warfarin, CHAPTER 84 events. Maculopapular rash that
anticoagulant effects should be 1. F develops several days after the
monitored closely.) 2. T onset of treatment is most com-
9. a 3. F mon. Prior to any other action,
10. a 4. T since a rash is not a severe reac-
11. a, d 5. F tion requiring administration
12. 2 capsules 6. F of epinephrine, the assessment
13. 0.5 (1/2) tablet 7. T must be completed in order to
8. T have all relevant information
CHAPTER 83 9. T available for the prescriber.)
1. T 10. F 11. c, d
2. F 11. T 12. b
3. T 12. F 13. a
4. F 13. T 14. c (Because of structural simi-
5. T 14. a (All patients who are can- larities between penicillins and
6. T didates for penicillin therapy cephalosporins, a few patients
7. F should be asked if they have allergic to one type of drug
8. F penicillin allergy.) may experience cross-reactivity
9. T 15. c with the other. For patients
10. T 16. b with mild penicillin allergy,
11. F 17. d cephalosporins can be used with
12. T 18. c (Anaphylaxis [laryngeal minimal concern. However,
13. F edema, bronchoconstriction, because of the potential for fatal

Copyright © 2016, Elsevier Inc. All Rights Reserved.


Answer Key 307

anaphylaxis, cephalosporins 27. c (Linezolid can cause revers- occurred following intraperito-
should not be given to patients ible myelosuppression, mani- neal or intrapleural instillation
with a history of severe reac- festing as anemia, leukopenia, of aminoglycosides. However,
tions to penicillins.) thrombocytopenia, or even neuromuscular blockade has
15. a pancytopenia. Complete blood also occurred with IV, IM, and
16. a counts should be done weekly.) oral dosing.)
17. c 28. a 17. b
18. a 29. c 18. d (Comparing the eGFR to
19. b 30. d (Telithromycin can cause the dosing recommendations
20. b severe liver injury [fulminant provided in the package insert
21. c (Imipenem can reduce blood hepatitis, hepatic necrosis] and or IV drug book, in concert with
levels of valproate, a drug used acute hepatic failure. If liver trough drug level, will enable
to control seizures. Break- injury is diagnosed, telithromy- the nurse to determine if the
through seizures have occurred. cin should be discontinued and dosing is appropriate.)
The patient is a safety risk due never used again.) 19. c
to the possibility of seizures.) 31. a 20. b
22. d 32. c (Chloramphenicol is a broad- 21. a (In patients with normal
23. b spectrum antibiotic with the renal function, half-lives of the
24. a potential for causing fatal aminoglycosides range from
25. d aplastic anemia and other blood 2-3 hours. However, because
26. No. 750 mg is higher than the dyscrasias. Anemia is one of the elimination is almost exclu-
10 mg/kg recommended for per- most common causes of pallor.) sively renal, half-lives increase
sons with normal renal function. 33. b dramatically in patients with
27. 254 mg. Yes, this is a safe dose. 34. a renal impairment. Accordingly,
35. 4 mL to avoid serious toxicity, dosage
CHAPTER 86 36. No. The safe dose is 837.5 mg must be reduced or the dosing
1. F every 6 hours. interval increased in patients
2. T with kidney disease.)
3. T CHAPTER 87 22. c
4. F 1. bactericidal 23. 100-166 mg every 8 hours. Cal-
5. F ^
>{…
=   
? culating 1.7 mg/kg equals 170
6. T 3. kidneys mg per dose, but that exceeds 5
7. F 4. narrow mg/day.
8. F 5. negative 24. An appropriate dose is 325 mg
9. T 6. kidneys; inner ear every 8 hours; the prescribed
10. F 7. hours dose is too low.
11. b, c, e 8. concentration-dependent
12. d 9. enzymes; aminoglycosides CHAPTER 88
13. a 10. anaerobes 1. T
14. c 11. a, c, f 2. F
15. c 12. b 3. T
16. d (Tetracyclines irritate the GI 13. a (Patients on aminoglycoside 4. T
tract. As a result, oral therapy is therapy should be monitored 5. F
frequently associated with epi- $

!
 
 
$
6. F
gastric burning, cramps, nausea, impending vestibular damage is 7. T
vomiting, and diarrhea. Because headache, which may last for 1 8. F
diarrhea may result from super- or 2 days. A complete assess- 9. T
infection of the bowel [in addi- ment must be performed prior 10. F


 
 ¤
to notifying the prescriber, in 11. T
it is important that the cause of order for him/her to determine 12. T
diarrhea be determined.) whether the headache is related 13. d
17. a to tobramycin. Therefore, as- 14. b (To minimize the risk of renal
18. a, c, d sessment is the priority action.) damage, adults should maintain
19. c 14. d a daily urine output of 1200
20. c 15. d mL. This can be accomplished
21. b 16. c (Aminoglycosides can inhibit by consuming 8 to 10 glasses
22. d neuromuscular transmission, of water each day. Because the
23. c   
 
   

solubility of sulfonamides is
24. a potentially fatal respiratory highest at elevated pH, al-
25. c depression. Most episodes of kalinization of the urine can
26. d neuromuscular blockade have

Copyright © 2016, Elsevier Inc. All Rights Reserved.


308 Answer Key

further decrease the chances of the drug to rise, thereby posing 32. Yes. Safe dose range is 400-800
crystalluria.) a risk of systemic toxicity.) mg/day = 200-400 mg/dose.
15. c (In addition to hemolytic 16. a
anemia, sulfonamides can cause 17. 2 capsules CHAPTER 91
agranulocytosis, leukopenia, 18. 0.5 tablet 1. tendon
thrombocytopenia, and very 2. Neisseria gonorrhoeae
rarely, aplastic anemia; red cell CHAPTER 90 3. anaerobes
distribution width [RDW] may 1. T 4. phototoxicity (or severe
be elevated in persons with 2. F sunburn)
hemolytic anemia.) 3. F 5. Achilles tendon
16. a 4. F 6. a (If treatment is to succeed and
17. b 5. T drug resistance reduced, the
18. a 6. F identity and drug sensitivity of
19. c (Local application of 7. F the causative organism must be
mafenide is frequently painful.) 8. T determined.)
20. c 9. F ~

=> 
 


21. a, d 10. T variety of mild adverse effects,


22. b (People with hyperkalemia 11. T including GI reactions [nausea,
may report symptoms such as 12. b, c, d vomiting, diarrhea, abdominal
muscle weakness, tiredness, 13. a pain]; vomiting and diarrhea
tingling sensations, or nausea.) 14. d put the child at increased risk of
23. a 15. b electrolyte imbalance.)
24. c 16. d (Isoniazid can cause hepato- 8. a, d, e
25. a cellular injury and multilobular 9. d
26. d necrosis. Patients should be 10. c
27. 9 mL/dose informed about signs of hepati- 11. c
tis [anorexia, malaise, fatigue, 12. d
CHAPTER 89 nausea, yellowing of the skin 13. b
1. c or eyes] and instructed to notify 14. b, c
2. f the prescriber immediately if 15. c
3. b these develop.) 16. a
4. a 17. b 17. c
5. e 18. a, c, e 18. a
6. d 19. c 19. d
7. b 20. b 20. a, b, c
8. b (Prevention of repeat UTIs in 21. a 21. 494 mg is the recommended
women includes wiping front to 22. b dose. This is safe.
back.) 23. c 22. Yes, it is safe. 1.2 g is the rec-
9. a, b, c, d 24. b ommended dose.
10. d (If treatment is to succeed, the 25. a, d
identity and drug sensitivity of 26. b (Ethambutol can produce CHAPTER 92
the causative organism must be dose-related optic neuritis, 1. mycosis
determined. To do so, urine for resulting in blurred vision, 2. external (or outside)
microbiologic testing should constriction of the visual 3. throughout the body
be obtained before giving any 

  
$

4. T
antibiotics.) discrimination.) 5. T
11. c 27. a 6. F
12. b 28. c 7. T
13. a 29. a 8. F
14. a, b, c, e 30. c (GI symptoms [nausea, vomit- 9. T
15. c (For two reasons, nitrofuran- ing, cramping, diarrhea] are 10. T
toin should not be administered common but mild. The drug 11. F
to individuals with renal impair- frequently imparts a harmless 12. T
ment [creatinine clearance less red color to feces, urine, sweat, 13. b
than 40 mL/min]. First, in the tears, and saliva. Deposition of 14. d
absence of good renal function, clofazimine in the small intes- 15. c
levels of nitrofurantoin in the tine produces the most serious 16. a
urine are too low to be effec- effects: intestinal obstruction, 17. c
tive. Second, renal impairment pain, and bleeding.) 18. c
reduces nitrofurantoin excre- 31. 507 mL divided by 3 = 169 mL/ 19. b
tion, causing plasma levels of hr 20. d

Copyright © 2016, Elsevier Inc. All Rights Reserved.


Answer Key 309

21. a (Amphotericin is toxic to cells is increased by dehydration and 12. b (Rilpivirine can cause depres-
of the kidney. Renal impair- use of other nephrotoxic drugs. sion. Instruct patients to contact
ment occurs in practically all Dry mucous membranes are the provider immediately if they
patients.) indicative of dehydration. To start feeling sad, hopeless, or
22. c avoid nephrotoxicity, hydrating suicidal.)
23. a the patient is priority.) 13. a
24. b 15. a (For herpes simplex genitalis, 14. c
25. c valacyclovir is indicated for 15. d (The most common adverse
26. b treatment of initial and recur- effect is rash; for most patients,
27. d rent episodes for immunocom- the rash is benign. However, if
28. c petent patients; however, for the patient experiences severe
^‡

=! 

 @
suppressive therapy, this drug rash or rash associated with
has been associated with hepatic is approved for management in fever, blistering, oral lesions,
necrosis, Stevens-Johnson immunocompetent and HIV- conjunctivitis, muscle pain, or
syndrome, and anaphylaxis. infected adults with a CD4+ cell joint pain, nevirapine should
Fever, along with mucocutane- count of at least 100 cells/mm3.) be withdrawn, because these
ous lesions, are early symptoms 16. a symptoms may indicate devel-
of Stevens-Johnson syndrome.) 17. b opment of erythema multiforme
30. d 18. d or Stevens-Johnson syndrome.)
31. b 19. c 16. c
32. b (Posaconazole inhibits 20. c 17. d
CYP3A4, and can thereby 21. d 18. d (Use of PIs has been associ-
increase levels of many other 22. c ated with redistribution of body
drugs. Combined use of 23. a fat, sometimes referred to as
posaconazole with ergot alka- 24. a (Lactic acidosis, pancreatitis,      
 or
loids is contraindicated, because and severe hepatomegaly are pseudo-Cushing’s syndrome.
raising their levels can lead to rare but dangerous complica- Fat accumulates in the abdo-
ergotism.) tions. If one of these conditions men [“protease paunch”] in the
33. a develops, lamivudine should breasts of men and women, and
34. c be discontinued. A sign of between the shoulder blades
35. d lactic acidosis is deep, rapid [“buffalo hump”]. Fat is lost
36. d breathing.) from the face, arms, buttocks,
37. a 25. a and legs. Leg and arm veins
38. c 26. c become prominent.)
39. 86 or 87 mL/hr; total volume is 27. b 19. d
130 mL over 90 minutes. 28. a, b, c, d, e 20. a
40. 12.5 mg; yes, 12 mg is safe. 29. c 21. c
30. d 22. d
CHAPTER 93 31. 300-400 mg in three divided 23. d
1. T doses (100-133 mg) 24. a, b, d
2. F 32. 600 mL 25. d (Enfuvirtide has also been
3. T associated with Guillain-Barré
4. T CHAPTER 94 syndrome.)
5. T 1. d 26. a
6. F 2. b 27. d
7. T 3. a 28. a
8. T 4. e 29. a, b
9. F 5. f 30. d
10. F 6. c 31. c
11. d 7. c 32. c
12. c 8. a 33. 30 mL
13. a (Intravenous acyclovir is gen- 9. b, c (Trimethoprim- 34. 40 mg
erally well-tolerated. The most sulfamethoxazole has been
common reactions are phlebitis associated with thrombo- CHAPTER 95

  


$ 
cytopenia, leukopenia, and 1. T
site.) neutropenia; it has also been 2. T
14. c (Reversible nephrotoxic- associated with kidney failure, 3. a
ity, indicated by elevations in and elevated BUN/creatinine.) 4. d (About half of the infants born
serum creatinine and blood 10. b to women with cervical C. tra-
urea nitrogen, occurs in some 11. d chomatis acquire the infection
patients. The risk of renal injury

Copyright © 2016, Elsevier Inc. All Rights Reserved.


310 Answer Key

during delivery, putting them at removed by rinsing with water 17. a (Symptoms of hypoglycemia
risk for pneumonia.) and 70% alcohol prior to BAC include sweating, chills, and
5. a application.) clamminess.)
6. c 26. b 18. d
7. b 27. c 19. b
8. c (Among preadolescent chil- 20. a (The most serious and fre-
dren, the most common cause of CHAPTER 97 quent effect from primaquine is
gonococcal infection is sexual 1. T hemolysis, which can develop
abuse. Vaginal, anorectal, and 2. T in patients with glucose-
pharyngeal infections are most 3. F 6-phosphate dehydrogenase
common.) 4. T £\—|&¤

&

9. a 5. F primaquine therapy, the urine


10. b 6. T should be monitored [darken-
11. d (Neonatal gonococcal infec- 7. T ing indicates the presence of
tion is acquired through contact 8. T hemoglobin].)
with infected cervical exudates 9. F 21. c (Quinine has quinidine-like
during delivery. Infection can 10. b effects on the heart and must be
be limited to the eyes or it may 11. d used cautiously in patients with
be disseminated. Gonococcal 12. c  
  
–
 -
neonatal ophthalmia is a serious 13. b, c, d, f, g ing atrioventricular conduction,
infection. The initial symptom 14. c (Limited experience with quinine can increase passage of
is conjunctivitis.) mebendazole in pregnant atrial impulses to the ventricles,
12. b women has shown no increase thereby causing a dangerous
13. d in spontaneous abortion or fetal increase in ventricular rate.)
14. a malformation. Nonetheless, 22. b
15. c pregnant women should avoid 23. b
16. b this drug, especially during the 24. a (Intravenous quinidine is
17. 40 mg every 6 hours; 160 mg  
 ? more cardiotoxic than quinine.
total per day. 15. a Accordingly, patients require
18. 100 mL/hr 16. a continuous electrocardiographic
17. d (Indirect effects of diethyl- monitoring and frequent moni-
CHAPTER 96 carbamazine, occurring second- toring of blood pressure.)
1. e ary to death of the parasites, 25. c
2. b can be more serious. These 26. a
3. c include rashes, intense itching, 27. c
4. d encephalitis, fever, tachycardia, 28. 5.6 mL/hr
5. f lymphadenitis, leucocytosis, 29. 166 (or 165) mg every 8 hours
6. a and proteinuria.)
7. g 18. 5 chewable tablets CHAPTER 99
8. F 19. 202.5 mg (200-mg tablet) 1. c
9. F 2. b
10. T CHAPTER 98 3. f
11. T 1. young 4. e
12. F 2. resistant 5. a
13. T 3. " mosquito 6. d
14. F 4. tuberculosis 7. c (Cryptosporidiosis is charac-
15. T 5. hepatocytes (or liver cells) terized by diarrhea, abdominal
16. F 6. erythrocytes cramps, anorexia, low-grade
17. a 7. Vivax fever, nausea, and vomiting. For
18. a 8. relapse immunocompromised individu-
19. b 9. Black water fever als the disease can be prolonged
20. a (Isopropanol promotes local 10. c and life-threatening, with diar-
vasodilation and can thereby 11. b rhea volume up to 20 L/day.
increase bleeding from needle 12. a For this person on prednisone
punctures and incisions.) 13. c $

 



21. a, b 14. c 







22. b 15. b he/she is immunocompromised


23. a, c 16. a (Falciparum malaria can and could potentially have life-
24. b produce serious complications, threatening diarrhea.)
25. a (Because BAC is inactivated including pulmonary edema.) 8. b
by soap, all soap must be 9. d

Copyright © 2016, Elsevier Inc. All Rights Reserved.


Answer Key 311

10. a 17. c (The major concern with be devastating; nurses must as-
11. c hyperuricemia is injury to the  

 


 

12. a kidneys secondary to deposi- to cope with this problem.)


13. a (Suramin concentrates in the tion of uric acid crystals in renal 31. b (Severe hypersensitivity reac-
kidneys and can cause local tubules.) tions [hypotension, dyspnea,
damage, resulting in the ap- 18. b angioedema, urticaria] have
pearance of protein, blood cells, occurred during infusion of
and casts in the urine. If urinary CHAPTER 102 Taxol.)
casts are observed, treatment 1. T 32. c
should cease.) 2. F 33. d
14. b 3. T 34. 300 micrograms
15. b 4. F 35. 1237.5 mg
]—

=#




5. T
of glucose-6-phosphate dehy- 6. T CHAPTER 103
drogenase, nifurtimox can cause 7. F 1. F
hemolysis.) 8. F 2. T
17. 254.5 mg; dose is safe and 9. T 3. T
therapeutic. 10. F 4. F
18. Yes, the dose is safe. It falls 11. d 5. F
between the 30-40 mg/kg/day in 12. a (Bone marrow suppression 6. F
3 divided doses recommended [neutropenia, thrombocytope- 7. c
for children (295 mg to 394 mg nia] is the usual dose-limiting 8. d (Perhaps the biggest concern
every 8 hours). toxicity for patients receiving with tamoxifen is endometrial
cytotoxic drugs.) cancer. In postmenopausal
CHAPTER 100 13. b women, endometrial cancer
1. P 14. a is usually caught early, due to
2. S 15. b abnormal menstrual bleeding.)
3. S 16. d 9. b
4. B 17. c (Other adverse effects of 10. a (Because of its estrogen
5. P cisplatin include ototoxicity, agonist actions, tamoxifen poses
6. S which manifests as tinnitus and a small risk of thromboembolic
7. B high-frequency hearing loss.) events, including deep vein
8. B 18. b thrombosis, pulmonary embo-
9. P 19. d lism, and stroke.)
10. B 20. c (High doses can directly injure 11. b
11. c the kidneys. To promote drug 12. c (Toremifene prolongs the QT
12. d excretion, and thereby minimize interval, and thereby poses a
13. a renal damage, the urine should risk of potentially fatal dys-
14. c, d be alkalinized and adequate rhythmias. To reduce risk,
15. 30-lb child: None, because the hydration maintained.) toremifene should be avoided
child weighs 13.6 kg (less than 21. b (It should be noted that in patients with hypokalemia or
15 kg). 44-lb child: 4-8 mg. leucovorin rescue is potentially hypomagnesemia. A potassium
16. 3 tablets dangerous: Failure to administer level of 2.7 is low; a magne-
leucovorin in the right dose at sium level of 2.2 is acceptable.)
CHAPTER 101 the right time can be fatal.) 13. d
1. heart disease 22. a 14. a
2. 30; 74 23. c 15. d
3. Prostate 24. b 16. a, d
4. surgery 25. a, c, e 17. c
5. Cytotoxic 26. d (Bone marrow suppression 18. d
—

 [neutropenia, thrombocytope- 19. b
7. d nia, anemia] is the principal 20. c
8. b, d dose-limiting toxicity. Petechiae 21. d
9. c often occur in the setting of 22. c
10. d thrombocytopenia.) 23. a
11. a 27. d 24. b (Side effects of ketoconazole
12. b 28. d include nausea and vomiting.
13. a 29. c (Bleomycin can cause severe These may lead to electrolyte
14. c injury to the lungs.) imbalance.)
15. a 30. a (For adolescents, hair loss 25. a, c
16. b secondary to chemotherapy can 26. a

Copyright © 2016, Elsevier Inc. All Rights Reserved.


312 Answer Key

27. d for early POAG is important can result when large amounts
28. b among individuals at high risk. are used for a prolonged period.
29. b, d, e With early detection and treat- Symptoms of salicylism include
30. a (Fluid retention occurs in 52% ment, blindness can usually be tinnitus.)
to 68% of patients taking ima- prevented.) 16. c
tinib, and may lead to pleural 20. d (Angle-closure glaucoma 17. a (Some formulations of ben-
effusion, pericardial effusion, develops suddenly and is ex- @

 
 

pulmonary edema, or ascites.) tremely painful. In the absence which can cause potentially
31. d of treatment, irreversible loss of serious allergic reactions. The
32. b vision occurs in 1 to 2 days.) incidence of reactions is highest
33. c 21. a, e in patients with asthma.)
34. a 22. a 18. a
35. c 23. c 19. a, c, e
36. a 24. b 20. a, b, c
37. d 25. a, d 21. d, e, f, g
38. a 26. c 22. d
39. a ^~

=} 

  

23. a
40. c to move from the eye into the 24. b
41. c plasma, thereby causing a rapid 25. a
42. c and marked reduction of IOP. 26. d
43. d (In patients taking interferon However, osmotic agents must 27. b
alfa-2b, neuropsychiatric ef- be used with caution in persons 28. d
fects—especially depression— with CKD and compromised 29. c
are a serious concern, owing cardiac function. Urine output 30. b
to a risk of death by suicide. must be monitored to ensure 31. d
Therefore, preventing injury is 


 ? 32. a (Sunscreens alone cannot
the priority.) 28. c, e completely protect against sun
44. 125 seconds = 2 minutes plus 5 29. c damage. Accordingly, to further
seconds. It is acceptable. Even 30. d (The biggest concern is reduce risk, sunglasses, pro-
though it is taking 5 seconds   

  - tective clothing, and a wide-
longer than 2 minutes, it is safe. tion inside the eye caused by brimmed hat should be worn. In
Because of the setup of a clock bacterial, viral, or fungal infec- addition, avoid sun exposure in
(watch), it is easier to time a tion. Patients who experience the middle of the day, especially
drug every 5 seconds than every symptoms [e.g., redness, light between 10:00 AM and 4:00 PM.
3 or 4 seconds. sensitivity, pain] should seek If you must be outside at these
45. Yes. The patient’s BSA = 2 m2. immediate medical attention.) times, try to stay in the shade
2 × 400 mg = 800 mg. 31. a as much as possible. All of
32. b these work together to reduce
CHAPTER 104 33. 2 tablets chronic exposure of the skin to
1. c 34. 18.75 grams sunlight.)
2. m 33. c
3. f CHAPTER 105 34. a
4. d 1. epidermis 35. b
5. a 2. stratum germinativum or basal 36. d
6. n layer 37. b
7. e 3. keratin 38. 83 to 84 mL/hr
8. g 4. corneum 39. 1 mL
9. i 5. Melanin
10. j 6. dermis CHAPTER 106
11. k 7. sebum 1. auricle; pinna
12. l 8. fat 2. external ear
13. b 9. Open comedones 3. eardrum; inner
14. h 10. Closed comedones or 4. auditory (eustachian) tube
15. central whiteheads 5. semicircular canals
16. Drusen 11. androgens 6. F
17. dry; wet 12. scrubbing; foods 7. T
18. anemia 13. a 8. F
19. b (Since POAG has no symp- 14. d 9. T

£
  

15. a (Salicylic acid is readily 10. F
irreversible optic nerve injury absorbed through the skin, so 11. c
has occurred], regular testing systemic toxicity [salicylism] 12. d

Copyright © 2016, Elsevier Inc. All Rights Reserved.


Answer Key 313

13. a 26. 15 mg; 100 mL/hr Accordingly, the herb should


14. d be avoided by patients at risk
15. a, b, d CHAPTER 108 for seizures, including those
16. d 1. i taking drugs that can lower the
17. d 2. d seizure threshold, including
18. Yes, the dose is safe; it falls 3. h antipsychotics, antidepres-
between 80-90 mg/kg/d in 2 4. f sants, cholinesterase inhibitors,
divided doses. 0.8 mL 5. c     
 %  

19. No, the dose is too low; should 6. e antihistamines, and systemic
administer 1000 mg per dose 7. a glucocorticoids.)
(the maximum dose); 15 mL 8. b 32. a
9. j 33. a (St. John’s wort is known to
CHAPTER 107 10. g interact adversely with many
1. c 11. F drugs—and the list continues
2. e 12. T to grow. Three mechanisms
3. b 13. F are involved: induction of
4. a 14. T cytochrome P450 enzymes,
5. f 15. T induction of P-glycoprotein,
6. d 16. T 
  
$


7. a (More importantly, in persons 17. F effects.)


taking iloprost, peripheral vaso- 18. T 34. c
dilation may result in orthostatic 19. a 35. b (Ma huang [ephedra] contains
hypotension and syncope.) 20. c (Rather than wait for imple- ephedrine, a compound that
8. d mentation of CGMPs, four can elevate blood pressure and
9. c private organizations—the U.S. stimulate the heart and CNS.)
10. b Pharmacopeia [USP], Consu-
11. c merLab, the Natural Products CHAPTER 109
12. c (These are symptoms of Association, and NSF Inter- 1. b, e, f
hyperglycemia; glucose level is national—have already begun 2. e, f
the priority.) testing dietary supplements for 3. d
13. a (Surfactant therapy lowers the quality. A “seal of approval” 4. b, e
surface tension forces that cause is given to products that meet 5. a, e
alveolar collapse, and thereby their standards, which are very 6. b, c, e, f
rapidly improves oxygenation similar to the CGMPs described 7. b, e, f
and lung compliance and re- previously. The USP standards 8. b (Supportive care is the most
duces the need for supplemental are enforceable by the FDA.) important element in manag-
oxygen and mechanical ventila- 21. a (Herbal products and other ing acute poisoning. Support is
tion. Based on these factors, dietary supplements can interact based on the clinical status and
Ineffective airway clearance is with conventional drugs, some- "

 


the highest priority in a neonate 



  
 $
to the poison involved. Mainte-
receiving beractant.) results. The principal concerns nance of respiration and circula-
14. c are increased toxicity and de- tion are primary concerns.)
15. b creased therapeutic effects. Cli- 9. b, e
16. c nicians and consumers should 10. a
17. a be alert to these possibilities.) 11. d
18. d 22. b 12. c
19. a 23. c (CoQ-10 is structurally 13. b
20. a similar to vitamin K2, and hence 14. d
21. c may antagonize the effects of 15. a
22. b (Recall that both depression warfarin.) 16. a
and suicidality are common in 24. c 17. b
HD. Tetrabenazine increases the 25. b 18. c
risk of both. Accordingly, all 26. d 19. d
patients using the drug should 27. a, b, c 20. a
be watched closely for new or 28. a 21. d
worsening depression, and for 29. c 22. c
expression of suicidal thoughts 30. d (Symptoms of hypoglycemia 23. c
or behavior.) include pounding heart, sweat- 24. a
23. b ing, chills, and clamminess.) 25. 2 1/2 mL
24. a, b, d 31. d (There is concern that 26. 333 mL/hr
25. 625 mg ginkgo may promote seizures.

Copyright © 2016, Elsevier Inc. All Rights Reserved.


314 Answer Key

CHAPTER 110 13. a 22. c


1. T 14. d 23. d
2. T 15. a 24. c
3. F 16. b 25. c
4. F 17. b 26. 1/2 tablet
5. F 18. b 27. 0.25 mL every 10 seconds ×
6. T 19. a 200 seconds = 5 mL
7. F 20. c
8. T 21. a (Supportive care, which may
9. T be needed for several months,
10. T includes mechanical assistance
11. F of ventilation; hence, effective
12. d (Initial symptoms of anthrax breathing is the highest nursing
include fever, cough, malaise, priority.)
and weakness. They may be
relatively mild.)

Copyright © 2016, Elsevier Inc. All Rights Reserved.

Вам также может понравиться